Vous êtes sur la page 1sur 356

Mastering

!!!!!!Psychiatry:!
A"core"textbook"for"undergraduates!
!!!!!!!!!!!!!!!!!!!!!!!!!!!!!!!!!!!!!!!!!!!!!!!!
!

Melvyn"WB"Zhang"
Cyrus"SH"Ho"
Roger"CM"Ho"
"
Sanjeev"Sockalingam"
Raed"Hawa"
"
Disclaimer:
"All the MCQs, SAQs and sample OSCEs were written by Dr. Roger Ho only. The objective of this companion book is to
help medical students to revise and understand psychiatry through practice and active learning. This allows medical
students to have a better understanding the exam format and expectations. The authors allow medical students to print
and the authors do not receive any financial incentives. The questions will not be used in the future end of posting tests
or exams."

MCQ exam (Paper 1) Questions

Questions

History, mental state exam and psychopathology!

1. A 40-year-old man admitted to the medical ward for upper gastrointestinal tract bleeding is a
known patient dependent on alcohol. His amylase level is high. During the interview, he
mentions that, I dont have a problem with alcohol. What is the defence mechanism?

A. Acting out
B. Denial
C. Projection
D. Rationalization
E. Reaction formation.

2. A 50-year-old woman is referred by her family doctor because she suffers from depressive
disorder. During the interview, she has difficulty in verbalizing her emotions. The
phenomenon is BEST described as:

A. Ambivalence
B. Affective flattening
C. Alexithymia
D. Alogia
E. Anhedonia.

3. Visual hallucinations are LEAST likely to be found in which of the following?


!

A. Alcohol!withdrawal!delirium!!
B. Central!anticholinergic!delirium!!
C. Lewy!body!dementia!
D. Parkinsons!disease!
E. Schizophrenia.!

4. Which of the following is NOT a common sign or symptom of panic attack?

A. Hypotension
B. Palpitation
C. Shortness of breath
D. Sweating
E. Trembling.

Cognitive assessment!

5. Which of the following is NOT a prominent cognitive feature of cortical dementia such as
Alzheimers disease?

!
!
1
A. Amnesia
B. Anhedonia
C. Apathy
D. Agnosia
E. Apraxia.

Psychiatric epidemiology

6. In the community, the MOST common psychiatric diagnosis over age 65 is:

A. Alzheimers disease

B. Late-onset schizophrenia

C. Mood disorder

D. Delirium

E. Vascular dementia.

Psychiatric aetiology, diagnosis and classification

7. Which!of!the!following!is!LEAST!LIKELY!to!be!a!risk!factor!for!delusional!disorder?!!
!

A. Anxiety!
B. Increased!age!!
C. Immigration!
D. Sensory!impairment!!
E. Social!isolation.!
!

8. A!female!adolescent!was!raised!by!an!abusive!father!when!she!was!young.!Although!he!
has!changed!and!is!no!longer!abusive!towards!her,!she!still!becomes!anxious!as!soon!as!
she!sees!him.!What!is!the!BEST!explanation!based!on!psychological!theory?!
!
A. Classical!conditioning!
B. Operant!conditioning!
C. Free!association!
D. Reaction!formation!
E. Transference.!
!

9. Which of the following factors is the MOST IMPORTANT predictor of the development of Post-
Traumatic Stress Disorder (PTSD) in a 30-year-old man who has just been involved in a road traffic
accident?

!
!
2
a. Death of a friend in the accident
b. Duration of medical leave
c. His age and gender
d. History of panic disorder
e. History of substance abuse.

10. Paul, a 20-year-old man, suffers from schizophrenia. He has a monozygotic twin brother
called Peter. Based on the findings from genetic studies, what is the risk (in %) that Peter will
develop schizophrenia?

A. 17%
B. 27%
C. 37%
D. 47%
E. 57%.

11. The MOST important predisposing factor for Attention Deficit and Hyperactivity Disorder
(ADHD) is:

A. Adverse social economic status


B. Development of autism before the onset of ADHD.
C. Female gender
D. Prenatal exposure to alcohol and nicotine by mother.
E. Presence of antisocial personality disorder in father.

12. A 40-year-old widow, said the following, I cant stop thinking about my husband. The pain is
unbearable. Its been eight months and its like it happened yesterday. I still cant believe it. I
cant look at his picture, its too painful. I feel numb. And my friends dont understand. They
still have their husbands. And, I want to kill his doctor. Its his fault my husband died. My life
is completely empty. Its just not fair. Based on her description, what is the MOST correct
diagnosis?

A. Adjustment disorder
B. Antisocial personality disorder
C. Delusional disorder
D. Intense grief reaction
E. Schizophrenia.

General adult psychiatry

13. All!of!the!following!statements!regarding!inpatient!suicide!are!true!EXCEPT:
A. Hanging is a common method used.
B. Inpatient suicide most often occurs during home leave.
C. Male!inpatients!are!at!higher!risk!of!committing!suicide!during!hospitalization.!
D. The!first!week!of!admission!is!a!high!risk!period.!
E. The!risk!for!women!increases!as!they!get!older.!!

!
!
3
14. Which of the following is the MOST important component of maintenance treatment for
schizophrenia?

A. Cognitive behaviour therapy


B. Monitored compliance in antipsychotic treatment
C. Occupational rehabilitation
D. Psychosocial rehabilitation
E. Family therapy.

15. Which!of!the!following!statements!regarding!schizophrenia!is!FALSE?!
A. Eye!movement!dysfunction!may!be!a!trait!marker.!
B. Hallucination!is!pathognomonic!for!schizophrenia.!
C. Intelligence!continues!to!deteriorate!with!the!progression!of!the!disorder.!!
D. Post!schizophrenia!depression!occurs!in!25%!of!patients.!
E. Prodromal!signs!and!symptoms!can!be!nonNspecific.!
!
16. Which!of!the!following!is!NOT!a!sign!or!symptom!of!atypical!depression?!!
!

A. Excessive!guilt!
B. Increased!appetite!
C. Interpersonal!rejection!sensitivity!!
D. Leaden!paralysis!
E. Mood!reactivity.!
!

17. Which of the following change is LEAST likely to occur in anorexia nervosa, binge-
purging type?
a. Hypoestrogenemia
b. Hypomagnesemia
c. Hypokalaemia
d. Hyperamylasemia
e. Hyperphosphatemia.
!

18. Which of the following is classified as circadian rhythm sleep disorder?

A. Delayed sleep phase syndrome


B. Obstructive sleep apnoea
C. Kleine Levin syndrome
D. Late-onset insomnia
E. Narcolepsy.

19. A!23TyearTold!medical!student!comes!to!the!clinic!with!a!complaint!of!feeling!worried.!!
He!states!that!he!may!fail!his!clinical!exam!in!Medicine!because!he!is!nervous!about!
presenting!a!case!and!performing!a!physical!examination!in!front!of!examiners.!!When!
he!thinks!about!the!exam,!he!feels!nervous.!!He!skipped!a!lot!of!bedside!teaching!when!

!
!
4
he!knew!it!would!be!his!turn!to!present!a!case!in!front!of!his!classmates!and!tutors.!He!
worries!about!criticism!from!his!classmates!and!tutor.!!The!MOST!likely!diagnosis!is:
A. Agoraphobia
B. Generalized anxiety disorder
C. Normal shyness
D. Panic disorder
E. Social phobia.
Old age psychiatry

20. The!MOST!COMMON!cause!of!psychotic!symptoms!in!elderly!outpatients!in!Singapore!is:!
!

A. Alcohol!abuse!
B. Anxiety!
C. Dementia!
D. Depression!
E. Delirium.!

21. Which!of!the!following!statements!is!TRUE!about!a!76TyearTold!woman!suffering!from!an!
acute!confusional!state!after!a!total!hip!replacement?!!
!

A. Anticholinergic!drug!is!the!treatment!of!choice.!
B. Her!age!and!gender!are!not!a!risk!factor.!
C. She!should!be!allowed!to!leave!the!ward!against!doctor!advice!should!she!wish!to.!
D. Fast,!spike!waves!are!often!seen!on!the!EEG.!!
E. There!is!a!mortality!of!up!to!15%.!!

22. A 65-year-old person is referred for early dementia. Which of the following is classified as
basic activities of daily living?

A. Bathing
B. Doing household chores and laundry
C. Managing medication
D. Preparing food
E. Shopping and managing finances.

23. Which of the following drugs/substances is LEAST likely to be abused by old people?

A. Alcohol
B. Amphetamine
C. Analgesics
D. Anticholinergics
E. Cough mixture.

24. You!are!the!resident!working!at!the!Accident!and!Emergency!Department.!A!70TyearTold!

!
!
5
woman!with!a!history!of!Alzheimers!disease!presents!to!you!after!a!fall.!!She!had!been!
experiencing! difficulty! sleeping! at! night! for! several! months! and! was! increasingly!
suspicious!of!her!husband!for!harming!her.!!You!have!decided!to!start!an!antipsychotic.!
Which!of!the!following!practices!is!MOST!appropriate?

A. Adjust doses of antipsychotics slowly with long intervals between dose increments.
B. Augmentation with benzodiazepine is highly recommended.
C. Prescribe small doses of several different antipsychotics rather than using one
antipsychotic.
D. Use a long-acting injectable antipsychotic to aid adherence.
E. Use a rapid loading dose of antipsychotic to speed up the response.
Child and adolescent psychiatry

25. Which!of!the!following!is!the!MOST!COMMON!disorder!for!children!and!adolescents!to!be!
referred!to!see!a!child!and!adolescent!psychiatrist?!!
!

A. ChildNonset!schizophrenia!
B. Conduct!disorder!!
C. Down!syndrome!
D. Separation!anxiety!disorder!
E. Somatization!disorder.!

26. An 8-year-old boy presents with stereotypies, pronoun reversal, word substitution, social
problems, and echolalia, the most likely diagnosis is:!
!

A. Autism!
B. Attention!deficit!and!hyperactivity!disorder!
C. Conduct!disorder!
D. Retts!syndrome!
E. Separation!anxiety!disorder.!
!

27. A 14-year-old adolescent continues to be significantly depressed despite actively


participating in psychotherapy for 3 months. Which of the following is the best treatment
approach?

A. Change psychotherapy approach


B. Start amitriptyline
C. Start electroconvulsive therapy
D. Start fluoxetine
E. Start olanzapine.

28. Which of the listed disorders is the MOST common co-morbidity with ADHD in children?
A. Autism
B. Learning disorders in mathematics
C. Learning disorders in expressive language
D. Oppositional defiant disorder
E. Gender identity disorder of childhood.

!
!
6
Substance abuse

29. Based on clinical severity, men from which of the following ethnicities are MOST vulnerable
to alcohol dependence in Singapore?

A. Chinese

B. Eurasian

C. Indian

D. Malay
E. Other ethnic groups.

30. The!aspartate!aminotransferase!(AST)!/alanine!aminotransferase!(ALT)!ratio!in!alcohol!hepatitis!
is:!
!

1. <!0.5!
2. 0.6!!0.9!
3. 1!!1.4!
4. 1.5!!1.9!
5. >!2.!
!

31. A 20-year-old man is suspected of acute stimulant intoxication. His symptoms are
MOST LIKELY TO resemble which of the following conditions?

A. Alcohol intoxication
B. Antisocial personality disorder
C. Mania
D. Obsessive-compulsive disorder
E. Panic disorder.

32. A 30-year-old man is dependent on 3 mg alprazolam (Xanax) on a daily basis. He


wants to stop his dependence on alprazolam. Which of the following is the BEST
management?
A. Change to 15 mg diazepam and then taper off gradually
B. Change to 100mg hydroxyzine and then taper off gradually
C. Change to 400mg lithium and then taper off gradually
D. Change to 10 mg olanzapine and then taper off gradually
E. Change to 200mg quetiapine and then taper off gradually.

33. A 30-year-old man drinks alcohol and develops flushing and tachycardia. He took disulfiram 2
hours ago. Accumulation of which of the following is accountable for the above
phenomenon?

!
!
7
A. Acetaldehyde
B. Acetylcholine
C. Adrenaline
D. Alanine
E. Aspartic acid.

Psychopharmacology

34. Which of the following tricyclic antidepressants is MOST effective in the treatment of
obsessive compulsive disorder?

A. Amitriptyline
B. Clomipramine
C. Desipramine
D. Doxepin
E. Imipramine.

35. A depressed patient does not want to take medication on a daily basis. The selective
serotonin reuptake inhibitor (SSRI) with the LONGEST half-life is?

A. Escitalporam
B. Fluoxetine
C. Fluvoxamine
D. Paroxetine
E. Sertraline.

36. The antidepressant, mirtazapine is less likely to cause nausea because of its effects on which
of the follow receptors?

A. Serotonin 5HT1 receptors


B. Serotonin 5HT2 receptors
C. Serotonin 5HT3 receptors
D. Serotonin 5HT4 receptors
E. Serotonin 5HT5 receptors.

37. Acetylcholinesterase inhibitors (AChEIs) are BEST conceptualized as:

A. Contraindicated in the treatment of Lewy body dementia.


B. Only indicated for severe stages of Alzheimer's disease.
C. Predominately associated with the side effect of metabolic syndrome.
D. Improving cognitive function significantly from baseline.
E. Stabilizing cognition, activities of daily living, and behavioural function.

38. Which of the following is FALSE regarding the first generation antipsychotics

!
!
8
(FGAs)?

A. FGAs have high D2 receptor blocking effects.


B. FGAs have increased risk of extrapyramidal side effects as compared with the second
generation antipsychotics.
C. FGAs have increased risk of tardive dyskinesia as compared with the second generation
antipsychotics.
D. FGAs have increased risk of causing metabolic syndrome as compared with the second
generation antipsychotics.
E. FGAs have proven efficacy as pharmacological treatment for schizophrenia.

39. All of the following are true of a patient on risperidone 6mg daily who gets parkinsonism side
effects, EXCEPT:

A. 75% occupancy at D2 receptors is associated with parkinsonism.


B. This patient is at higher risk for developing secondary negative symptoms.
C. The dose is above the antipsychotic threshold in this case.
D. Raising the dose of risperidone leads to a paradoxical reduction in parkinsonism.
E. Anticholinergic drug can reduce parkinsonism in this patient.

40. Which of the following herbs has been MOST commonly used by patients to treat
mild to moderate depression in developed countries?
!

A. Gingko!
B. Ginseng!
C. Kava!kava!
D. Passion!flower!
E. St.!Johns!Wort.!
!

41. A!40TyearTold!female!patient!with!schizophrenia!has!been!treated!with!haloperidol!and!
trifluoperazine!for!many!years!in!the!past.!!Currently,!she!exhibits!a!nonTrhythmical!
hyperkinetic!movement!disorder!of!the!lips,!jaw!and!tongue.!!The!movement!disorder!
MOST!likely!consistent!with!this!finding!is:!

A. Acute dystonia
B. Akathisia
C. Aphasia
D. Pseudoparkinsonism
E. Tardive dyskinesia.

42. A 35-year-old accountant presents to a psychiatrist 8 months after a motor vehicle


accident. He has difficulty sleeping because he has frequent nightmares about the
accident. He has not been able to drive since the accident, and his wife usually
drives for him. Even then, he finds it very difficult to be in a car such as a taxi, after

!
!
9
panicking if another car is near them on the road. Which of the following
medications would be the MOST appropriate for him?

A. Amitriptyline
B. Clonazepam
C. Paroxetine
D. Propranolol
E. Quetiapine.

Psychotherapy

43. A!66TyearTold!man!who!worked!as!an!executive!accountant!previously.!He!has!retired!
recently!and!presents!with!a!major!depressive!disorder.!!What!is!the!best!choice!of!
psychotherapy?!
!

A. Biofeedback!
B. Eye!movement!desensitization!and!reprocessing!
C. Hypnosis!
D. Interpersonal!psychotherapy!
E. Systemic!desensitization.!

44. A 40-year-old man with obsessive-compulsive disorder presents for treatment. He would
prefer not using a medication due to possible side effects. Which of the following
psychological interventions would be the treatment of choice for his disorder?

A. Exposure and response prevention


B. Hypnotherapy
C. Interpersonal psychotherapy
D. Psychoanalysis
E. Supportive psychotherapy.

45. Effective psychotherapy techniques for Borderline Personality Disorder include all of
the following features EXCEPT:
A. A treatment contract should be established in the beginning phase of
psychotherapy.
B. Adverse effects of self-laceration are identified.
C. Focus of treatment is to establish connection between actions and feelings.
D. Therapist is quiet, distant and does not get too involved.
E. Therapist pays careful attention to his or her own feelings in order to manage
countertransference.

Ethics and laws

46. A woman with a history of depression that responded well to antidepressant


medication is now depressed again. She does not want to take medications due to

!
!
10
concerns about side effects and risks. She would like to try psychotherapy instead.
Her husband disagrees with her. Which of the following ethical principles is MOST
relevant in this situation?
A. Autonomy
B. Beneficence
C. Fiduciary duty
D. Non-maleficence
E. Justice.

Liaison and neuropsychiatry

47. You!are!the!resident!working!in!the!oncology!ward.!Which!of!the!following!factors!is!
MOST!predictive!of!high!suicide!risk!in!a!palliative!cancer!patient?!
!

A. Alopecia!associated!with!chemotherapy!
B. Low!energy!level!
C. Poor!appetite!
D. Financial!difficulty!
E. Hopelessness.!
!

48. A 40-year-old woman presents with chronic headache and blurred vision. She wants
symptomatic relief but is not concerned about the underlying condition. Which of the
following psychiatric diagnoses is LEAST relevant?
!

A. Conversion!disorder!
B. Moderate!depressive!episode!with!somatic!complaints!
C. Hypochondriasis!
D. Pain!disorder!
E. Somatization!disorder.!
!

MCQ exam (Paper 1) Answer Key

Questions Answers!

History, mental state exam and !


psychopathology!

49. A 40-year-old man admitted to the The answer is B.


medical ward for upper gastrointestinal
tract bleeding is a known patient
dependent on alcohol. His amylase level
is high. During the interview, he Explanation: He exhibits denial and denies the
mentions that, I dont have a problem reality that he is dependent on alcohol and

!
!
11
with alcohol. What is the defence leading to medical complication.
mechanism?

F. Acting out
G. Denial
H. Projection Year: 2013.
I. Rationalization
J. Reaction formation.

50. A 50-year-old woman is referred by her The!answer!is!C.!


family doctor because she suffers from
depressive disorder. During the !
interview, she has difficulty in
verbalizing her emotions. The
Explanation:!This!patient!has!difficulty!to!
phenomenon is BEST described as:
express!her!emotion!and!this!phenomenon!is!
known!as!alexithymia.!!
F. Ambivalence
G. Affective flattening !
H. Alexithymia
I. Alogia !
J. Anhedonia.
Year:!2013!

51. Visual hallucinations are LEAST The!answer!is!E.!


likely to be found in which of the
following? !
! Explanation:!!The!other!options!A!to!D!are!
associated!with!visual!hallucination.!Patients!
F. Alcohol!withdrawal!delirium!!
suffering!from!Parkinsons!disease!may!
G. Central!anticholinergic!delirium!!
H. Lewy!body!dementia! present!with!visual!hallucinations!as!a!result!
I. Parkinsons!disease! of!side!effect!of!madopar.!As!most!patients!
J. Schizophrenia.! suffering!from!schizophrenia!in!Singapore!do!
not!use!illicit!drugs,!visual!hallucinations!are!
relatively!less!common.!

Year:!2013!

52. Which of the following is NOT a The!answer!is!A.!


common sign or symptom of panic
attack? !

Explanation:!!As!a!result!of!sympathetic!
F. Hypotension

!
!
12
G. Palpitation drive,!the!patient!may!experience!
H. Shortness of breath hypertension!rather!than!hypotension.!!
I. Sweating
J. Trembling.
!

Year:!2013.!

Cognitive assessment! !

53. Which of the following is NOT a The!answer!is!B.!


prominent cognitive feature of cortical
dementia such as Alzheimers disease? !

Explanation:!Anhedonia!(loss!of!interest)!
F. Amnesia
G. Anhedonia and!depression!are!more!common!in!
H. Apathy subcortical!dementia.!The!other!4!As:!
I. Agnosia amnesia,!apathy,!agnosia!and!apraxia!are!
J. Apraxia.
common!in!cortical!dementia.!

Year:!2013!

This!is!an!advanced!level!question.!

Psychiatric epidemiology !

54. In the community, the MOST common The answer is C.


psychiatric diagnosis over age 65 is:

A. Alzheimers disease Explanation: The National Institute of Mental


Health's Epidemiologic Catchment Area (ECA)
B. Late-onset schizophrenia program has found that the most common mental
disorder of old age is depressive disorder.
C. Mood disorder

D. Delirium

E. Vascular dementia.
Year: 2013

Psychiatric aetiology, diagnosis and !


classification

55. Which!of!the!following!is!LEAST! The!answer!is!A.!


LIKELY!to!be!a!risk!factor!for!

!
!
13
delusional!disorder?!! !
!
Explanation:!The!risk!of!delusional!disorder!
F. Anxiety! increases!with!age.!Hearing!loss!is!a!risk!
G. Increased!age!! factor.!Low!socioeconomic!status!and!severe!
H. Immigration!
stress!are!also!risk!factors.!
I. Sensory!impairment!!
J. Social!isolation.! !
!
!

Year:!2013.!

! !

56. A!female!adolescent!was!raised!by!an! The!answer!is!A.!


abusive!father!when!she!was!young.!
Although!he!has!changed!and!is!no! !
longer!abusive!towards!her,!she!still!
becomes!anxious!as!soon!as!she!sees! Explanation:!The!father!is!a!conditioned!
him.!What!is!the!BEST!explanation! stimuli!and!abuse!is!an!unconditioned!
based!on!psychological!theory?! stimuli.!The!fear!is!the!conditioned!response.!
! Even!without!the!abuse,!the!patient!is!fearful!
F. Classical!conditioning! of!the!father.!This!is!known!as!classical!
G. Operant!conditioning!
conditioning.!
H. Free!association!
I. Reaction!formation! !
J. Transference.!
! !

Year:!2013.!

This!is!an!advancedN!level!question.!

57. Which of the following factors is the MOST The!answer!is!A.!


IMPORTANT predictor of the development
of Post-Traumatic Stress Disorder (PTSD) in !
a 30-year-old man who has just been
involved in a road traffic accident? Explanation:!Severity of event or accident is best
predictor. Death of a friend in the accident indicates
f. Death of a friend in the accident severe accident.
g. Duration of medical leave
h. His age and gender
i. History of panic disorder

!
!
14
j. History of substance abuse.

Year: 2013.!

58. Paul, a 20-year-old man, suffers from The answer is D.


schizophrenia. He has a monozygotic
twin brother called Peter. Based on the
findings from genetic studies, what is
the risk (in %) that Peter will develop Explanation: Based on the genetic studies
schizophrenia? focusing on familial transmission of
schizophrenia, the risk is 47% for monozygotic
F.17% twins of schizophrenia patients developing
G. 27%
schizophrenia.
H. 37%
I. 47%
J. 57%.
Risk of schizophrenia in Specific Populations

Population Risk (%)


General Population 1
Nontwin sibling of a 8
schizophrenic patient
Child with one schizophrenic 12
parent
Dizygotic twin of a schizophrenic 12
patient
Child of two schizophrenic 40
parents
Monozygotic twin of a 47
schizophrenic patient

Year: 2013.

59. The MOST important predisposing The!answer!is!D.!


factor for Attention Deficit and
Hyperactivity Disorder (ADHD) is: !

Explanation:!Foetal!alcohol!syndrome!and!
F. Adverse social economic status
antenatal!exposure!to!nicotine!is!the!most!
G. Development of autism before the
onset of ADHD. important!predisposing!factor!for!ADHD.!
H. Female gender Option!A!is!nonNspecific.!Male!gender,!rather!
I. Prenatal exposure to alcohol and than!female!gender!is!more!common!for!
nicotine by mother. ADHD.!Option!E!is!an!important!predisposing!
J. Presence of antisocial personality factor!for!conduct!disorder.!!
disorder in father.
!

!
!
15
Year: 2013.!

60. A 40-year-old widow, said the following, The answer is D.


I cant stop thinking about my husband.
The pain is unbearable. Its been eight
months and its like it happened
yesterday. I still cant believe it. I cant Explanation: She suffers from intense grief
look at his picture, its too painful. I feel which persists for longer than 6 months
numb. And my friends dont after the loss, traumatic distress, sense of
understand. They still have their disbelief, anger and bitterness, distressing,
husbands. And, I want to kill his doctor.
intrusive thoughts related to the death,
Its his fault my husband died. My life is
completely empty. Its just not fair. avoidance of reminders of deceased and
Based on her description, what is the separation distress.
MOST correct diagnosis?

F. Adjustment disorder Year: 2013.


G. Antisocial personality disorder
H. Delusional disorder
I. Intense grief reaction
J. Schizophrenia.

General adult psychiatry !

61. All! of! the! following! statements! The!answer!is!E.!


regarding! inpatient! suicide! are! true!
EXCEPT: !
F. Hanging is a common method used.
G. Inpatient suicide most often occurs during Explanation:!Option!A,!B,!C,!D!are!correct.!
home leave. Hanging!in!bathroom!is!common!and!
H. Male!inpatients!are!at!higher!risk!of!
bathroom!needs!to!be!specially!designed.!It!
committing!suicide!during!
hospitalization.! is!important!for!nurses!or!ward!staff!to!give!
I. The!first!week!of!admission!is!a!high!risk! patients!or!family!a!call!during!home!leave!or!
period.! immediately!after!discharge!as!these!are!
J. The!risk!for!women!increases!as!they!get! high!risk!periods.!Option!D!is!incorrect.!The!
older.!! risk!for!women!plateaus!or!decreases!with!
age.!The!risk!for!men!shows!two!peaks:!
young!age!and!old!age.!

!
Year:!2013.!

62. Which of the following is the MOST The answer is B.


important component of maintenance
treatment for schizophrenia?

Explanation: Without maintenance


F. Cognitive behaviour therapy pharmacological treatment, 60-70% of
G. Monitored compliance in antipsychotic schizophrenia patients relapse within 1 year,

!
!
16
treatment and almost 90% relapse in 2 years.
H. Occupational rehabilitation Maintenance on antipsychotic therapy is the
I. Psychosocial rehabilitation single most important factor in preventing
J. Family therapy.
rehospitalisation.

Year: 2013.

63. Which!of!the!following!statements! The!answer!is!B.!


regarding!schizophrenia!is!FALSE?!
F. Eye!movement!dysfunction!may!be!a! !
trait!marker.!
G. Hallucination!is!pathognomonic!for! Explanation:!Hallucination!can!occur!in!other!
schizophrenia.! psychiatric!disorders.!Eye!movement!
H. Intelligence!continues!to!deteriorate! dysfunction!occurs!in!50N85%!of!
with!the!progression!of!the!disorder.!! schizophrenia!patients.!!!
I. Post!schizophrenia!depression!occurs!in! !
25%!of!patients.!
J. Prodromal!signs!and!symptoms!can!be! !
nonNspecific.!
! Year:!2013.!

64. Which!of!the!following!is!NOT!a!sign! The!answer!is!A.!


or!symptom!of!atypical!depression?!!
! !

F. Excessive!guilt! Explanation:!Excessive!guilt!is!part!of!
G. Increased!appetite! melancholia.!
H. Interpersonal!rejection!sensitivity!!
I. Leaden!paralysis! !
J. Mood!reactivity.!
! !

Year:!2013.!

65. Which of the following change is Answer!is!E.!


LEAST likely to occur in anorexia
nervosa, binge-purging type? !
f. Hypoestrogenemia
g. Hypomagnesemia Explanation:!It!should!be!hypophosphatemia!
h. Hypokalaemia and!hypocalcaemia.!As!bingeNpuruging!type!
i. Hyperamylasemia
j. Hyperphosphatemia. is!associated!with!recurrent!vomiting,!it!will!
! lead!to!hypokalaemia!and!inflammation!in!
salivary!gland,!i.e.!hyperamylasemia.!
Anorexia!nervosa!in!general!is!associated!
with!hypoestrogenemia!and!!

!
!
17
hypomagnesemia.!

Year:!2013!

This!is!an!advanced!!level!question.!

66. Which of the following is classified as The!answer!is!A.!


circadian rhythm sleep disorder?
!
F. Delayed sleep phase syndrome
Explanation:!Delayed!sleep!phase!syndrome!
G. Obstructive sleep apnoea is!classified!as!circadian!rhythm!sleep!
H. Kleine Levin syndrome disorder.!
I. Late-onset insomnia
J. Narcolepsy. !

Year:!2013!

67. A!23TyearTold!medical!student!comes! The!answer!is!E.!


to!the!clinic!with!a!complaint!of!
feeling!worried.!!He!states!that!he! !
may!fail!his!clinical!exam!in!Medicine!
because!he!is!nervous!about! Explanation:!He!suffers!from!social!
presenting!a!case!and!performing!a! phobia!because!he!develops!marked!fear!
physical!examination!in!front!of! which!is!brought!by!being!the!focus!of!
examiners.!!When!he!thinks!about!the! attention.!
exam,!he!feels!nervous.!!He!skipped!a!
lot!of!bedside!teaching!when!he!knew! !
it!would!be!his!turn!to!present!a!case!
in!front!of!his!classmates!and!tutors.! !
He!worries!about!criticism!from!his!
classmates!and!tutor.!!The!MOST! Year:!2013!
likely!diagnosis!is:
F. Agoraphobia
G. Generalized anxiety disorder
H. Normal shyness
I. Panic disorder
J. Social phobia.
Old age psychiatry !

68. The!MOST!COMMON!cause!of! The!answer!is!C.!


psychotic!symptoms!in!elderly!
outpatients!in!Singapore!is:! !

!
!
18
! Explanation:!Dementia!is!the!most!common!
cause!for!psychotic!symptoms!in!elderly!
F. Alcohol!abuse! outpatients.!
G. Anxiety!
H. Dementia! !
I. Depression!
J. Delirium.! !

Year:!2013.!

69. Which!of!the!following!statements!is! The!answer!is!E.!


TRUE!about!a!76TyearTold!woman!
suffering!from!an!acute!confusional! !
state!after!a!total!hip!replacement?!!
! Explanation:!!In!elderly,!the!mortality!of!
delirium!and!acute!confusional!state!is!
F. Anticholinergic!drug!is!the!treatment!of! between!6!to!18%.!Generalized!slowing!is!
choice.! the!most!common!finding!on!EEG.!Her!age!is!
G. Her!age!and!gender!are!not!a!risk!factor.!
a!risk!factor.!
H. She!should!be!allowed!to!leave!the!ward!
against!doctor!advice!should!she!wish!to.!
!
I. Fast,!spike!waves!are!often!seen!on!the!
EEG.!! !
J. There!is!a!mortality!of!up!to!15%.!!
Year:!2013!

70. A 65-year-old person is referred for The!answer!is!A.!


early dementia. Which of the following
is classified as basic activities of daily !
living?
Explanation:!A!is!considered!to!be!basic!
F. Bathing activities!of!daily!living!while!the!others!are!
G. Doing household chores and laundry classified!as!instrumental!activities!of!daily!
H. Managing medication living.!
I. Preparing food
J. Shopping and managing finances.
!

Year:!2013!

!
!
19
71. Which of the following The!answer!is!B.!
drugs/substances is LEAST likely to be
abused by old people? !

Explanation:!Old!people!are!most!likely!to!
F. Alcohol
G. Amphetamine misuse!alcohol!or!prescribed!drugs!than!
H. Analgesics street/illicit!drugs!such!as!stimulant.!
I. Anticholinergics
J. Cough mixture. !

!
Year:!2013.!

72. You! are! the! resident! working! at! the! The!answer!is!A.!


Accident!and!Emergency!Department.!
A! 70TyearTold! woman! with! a! history! !
of! Alzheimers! disease! presents! to!
you! after! a! fall.! ! She! had! been! Explanation:!Elderly!with!dementia!are!
experiencing! difficulty! sleeping! at! sensitive!to!side!effects.!As!a!result,!the!
night! for! several! months! and! was! doctor!should!adjust!the!dose!slowly!with!
increasingly! suspicious! of! her! long!intervals!between!increments.!Routine!
husband! for! harming! her.! ! You! have!
prescription!of!benzodiazepine!to!elderly!
decided! to! start! an! antipsychotic.!
Which! of! the! following! practices! is! with!dementia!is!not!recommended!as!it!will!
MOST!appropriate? lead!to!confusion.!

F. Adjust doses of antipsychotics !


slowly with long intervals
between dose increments. !
G. Augmentation with
benzodiazepine is highly Year:!2013.!
recommended.
H. Prescribe small doses of
several different antipsychotics
rather than using one
antipsychotic.
I. Use a long-acting injectable
antipsychotic to aid adherence.
J. Use a rapid loading dose of
antipsychotic to speed up the
response.
Child and adolescent psychiatry !

73. Which!of!the!following!is!the!MOST! The!answer!is!B.!


COMMON!disorder!for!children!and!
adolescents!to!be!referred!to!see!a! !
child!and!adolescent!psychiatrist?!!
! Explanation:! Children! with! externalizing!
disorders! (e.g.! conduct! disorder)! are! more!
F. ChildNonset!schizophrenia! likely! to! be! referred! and! treated.! 10%! of!

!
!
20
G. Conduct!disorder!! children! and! adolescents! seeing! a!
H. Down!syndrome! psychiatrist! have! conduct! problem.!
I. Separation!anxiety!disorder! Separation! anxiety! disorder! and! Down!
J. Somatization!disorder.!
Syndrome!may!not!need!to!see!a!psychiatrist.!
0.1%! have! psychotic! disorder! and! 1%! have!
somatization!disorder.!

Year:!2013!

74. An 8-year-old boy presents with The!answer!is!A.!


stereotypies, pronoun reversal, word
substitution, social problems, and !
echolalia, the most likely diagnosis is:!
! Explanation:!Repetitive!movement,!
communication!problem!and!social!problems!
F. Autism!
indicate!this!boy!suffers!from!autism.!
G. Attention!deficit!and!hyperactivity!
disorder!
!
H. Conduct!disorder!
I. Retts!syndrome! !
J. Separation!anxiety!disorder.!
! Year:!2013.!

75. A 14-year-old adolescent continues to The!answer!is!D.!


be significantly depressed despite
actively participating in psychotherapy !
for 3 months. Which of the following is
the best treatment approach? Explanation:!Starting!an!antidepressant!as!
F. Change psychotherapy approach
an!adjunct!to!psychotherapy!after!the!
G. Start amitriptyline adolescent!has!failed!to!respond!to!
H. Start electroconvulsive therapy psychotherapy!alone!is!a!standard!approach!
I. Start fluoxetine to!the!treatment!of!adolescent!depression.!!
J. Start olanzapine.
!

Year:!2013!

76. Which of the listed disorders is the The!answer!is!D.!


MOST common co-morbidity with ADHD
in children? !
F. Autism
G. Learning disorders in mathematics
Explanation:!sleep!disturbance,!conduct!

!
!
21
H. Learning disorders in expressive disorder,!oppositional!defiant!disorder,!
language depression,!anxiety!and!substance!abuse!are!
I. Oppositional defiant disorder
J. Gender identity disorder of common!comorbidity!with!ADHD.!
childhood.
!

Year:!2013.!

Substance abuse !

77. Based on clinical severity, men from The!answer!is!C.!


which of the following ethnicities are
MOST vulnerable to alcohol !
dependence in Singapore?
Explanation:!Indian!men!seem!to!be!most!
A. Chinese
vulnerable!to!alcohol!dependence!in!
B. Eurasian Singapore.!Malay!men!have!the!lowest!risk.!

C. Indian !

D. Malay !
E. Other ethnic groups.
Year:!2013.!

78. The!aspartate!aminotransferase!(AST)! The!answer!is!E.!


/alanine!aminotransferase!(ALT)!ratio!in!
alcohol!hepatitis!is:! !
!
Explanation:!When!greater!than!1.0!but!less!
6. <!0.5!
7. 0.6!!0.9! than!2.0,!it!is!likely!to!be!associated!with!
8. 1!!1.4! cirrhosis.!If!<1,!it!indicates!viral!hepatitis.!
9. 1.5!!1.9!
10. >!2.! !
!
!

Year:!2013.!!

This!is!an!advanced!level!question.!!

79. A 20-year-old man is suspected of The answer is C.


acute stimulant intoxication. His
symptoms are MOST LIKELY TO
resemble which of the following

!
!
22
conditions? The syndrome picture of acute cocaine or
amphetamine intoxication can include
hyperawareness, hypersexuality,
F. Alcohol intoxication
hypervigilance, agitation, paranoia, and
G. Antisocial personality disorder
H. Mania delusions. It resembles mania or psychosis.
I. Obsessive-compulsive disorder
J. Panic disorder.

Year: 2013.

80. A 30-year-old man is dependent on 3 Answer: A


mg alprazolam (Xanax) on a daily
basis. He wants to stop his
dependence on alprazolam. Which
of the following is the BEST Explanation: Diazepam is the longest
management? acting benzodiazepine and this will prevent
F. Change to 15 mg diazepam and withdrawal symptoms such as withdrawal
then taper off gradually
G. Change to 100mg hydroxyzine and
fit.
then taper off gradually
H. Change to 400mg lithium and then
taper off gradually
I. Change to 10 mg olanzapine and
then taper off gradually
J. Change to 200mg quetiapine and Year: 2013
then taper off gradually.

81. A 30-year-old man drinks alcohol and The!answer!is!A.!


develops flushing and tachycardia. He
took disulfiram 2 hours ago. !
Accumulation of which of the following
is accountable for the above Explanation:!The!mechanism!of!actions!of!
phenomenon?
disulfiram!is!to!inhibit!aldehyde!
dehydrogenase!and!lead!to!acetaldehyde!
F. Acetaldehyde

!
!
23
G. Acetylcholine accumulation.!
H. Adrenaline
I. Alanine !
J. Aspartic acid.
!

Year:!2013!

This!is!an!advancedNlevel!question.!

Psychopharmacology !

82. Which of the following tricyclic The!answer!is!B.!


antidepressants is MOST effective in
the treatment of obsessive compulsive !
disorder?
Explanation:!Among!all!TCAs,!clomipramine!
F. Amitriptyline
G. Clomipramine
is!most!potent!is!blocking!5HT!reuptake.!
H. Desipramine
I. Doxepin !
J. Imipramine.
!

Year:!2013!

83. A depressed patient does not want to The!answer!is!B.!


take medication on a daily basis. The
selective serotonin reuptake inhibitor !
(SSRI) with the LONGEST half-life is?
The!have!life!of!fluoxetine!is!longer!than!72!
F. Escitalporam hours!and!patient!can!take!the!medication!
G. Fluoxetine every!other!day.!
H. Fluvoxamine
I. Paroxetine !
J. Sertraline.
!

Year:!2013!

84. The antidepressant, mirtazapine is less The!answer!is!C.!


likely to cause nausea because of its
effects on which of the follow !
receptors?
Explanation:!Its!effect!on!5HT3 receptors is
F. Serotonin 5HT1 receptors less likely to cause nausea side effect as
G. Serotonin 5HT2 receptors compared to SSRIs.
H. Serotonin 5HT3 receptors
I. Serotonin 5HT4 receptors
J. Serotonin 5HT5 receptors.

!
!
24
Year: 2013.

This is an advanced level question.

85. Acetylcholinesterase inhibitors The answer is E.


(AChEIs) are BEST conceptualized
as:

F. Contraindicated in the treatment of Explanation: AChEIs are best


Lewy body dementia. conceptualized as drugs that stabilize
G. Only indicated for severe stages of cognition, activities of daily living, and
Alzheimer's disease. behavioural function and slow clinical
H. Predominately associated with the side deterioration in Alzheimer's disease but not
effect of metabolic syndrome. significant improvement from baseline.
I. Improving cognitive function AChEI e.g. rivastigmine can be used for
significantly from baseline. dementia with Lewy bodies.
J. Stabilizing cognition, activities of daily Gastrointestinal side effects e.g. nausea,
living, and behavioural function. vomiting and diarrhoea are the most
common adverse effects.

Year: 2013.

86. Which of the following is FALSE The!answer!is!D.!


regarding the first generation
antipsychotics (FGAs)? !

Explanation:!The!second!generation!
F. FGAs have high D2 receptor blocking antipsychotics!have!higher!risk!for!metabolic!
effects. syndrome!as!compared!to!the!FGAs.!
G. FGAs have increased risk of
extrapyramidal side effects as !
compared with the second generation
antipsychotics. !
H. FGAs have increased risk of tardive
dyskinesia as compared with the Year:!2013.!
second generation antipsychotics.
I. FGAs have increased risk of causing
metabolic syndrome as compared with
the second generation antipsychotics.
J. FGAs have proven efficacy as
pharmacological treatment for

!
!
25
schizophrenia.

87. All of the following are true of a patient The!answer!is!D.!


on risperidone 6mg daily who gets
parkinsonism side effects, EXCEPT: !

A. 75% occupancy at D2 receptors is Explanation:!Increase!the!dose!will!worsen!


associated with parkinsonism. parkinsonism.!
B. This patient is at higher risk for developing
secondary negative symptoms. !
C. The dose is above the antipsychotic
threshold in this case. !
D. Raising the dose of risperidone leads to a
paradoxical reduction in parkinsonism. Year:!2013.!
E. Anticholinergic drug can reduce
parkinsonism in this patient.

88. Which of the following herbs has The!answer!is!E.!


been MOST commonly used by
patients to treat mild to moderate !
depression in developed countries?
! Explanation:!St!Johns!Wort!is!the!most!
commonly!used!herb!to!treat!depression!in!
F. Gingko!
G. Ginseng! western!countries.!It!shares!similar!
H. Kava!kava! properties!as!other!SSRIs!such!as!increase!in!
I. Passion!flower! bleeding.!
J. St.!Johns!Wort.!
! !

Year:!2013.!

89. A!40TyearTold!female!patient!with! The!answer!is!E.!


schizophrenia!has!been!treated!with!
haloperidol!and!trifluoperazine!for! !
many!years!in!the!past.!!Currently,!she!
exhibits!a!nonTrhythmical! Explanation:!Tardive!dyskinesia!is!
hyperkinetic!movement!disorder!of! characterised!by!lip!smacking,!chewing!and!
the!lips,!jaw!and!tongue.!!The! fly!catching!tongue!protrusion!after!longN
movement!disorder!MOST!likely! term!treatment!of!antipsychotics.!
consistent!with!this!finding!is:!
!
F. Acute dystonia
G. Akathisia !
H. Aphasia
I. Pseudoparkinsonism Year:!2013.!!
J. Tardive dyskinesia.

!
!
26
90. A 35-year-old accountant presents The!answer!is!C.!
to a psychiatrist 8 months after a
motor vehicle accident. He has !
difficulty sleeping because he has
frequent nightmares about the Explanation:!SSRIs!are!generally!most!
accident. He has not been able to appropriate!medication!of!choice!for!post!
drive since the accident, and his traumatic!stress!disorder.!Examples!
wife usually drives for him. Even
include!paroxetine,!sertraline!and!
then, he finds it very difficult to be in
a car such as a taxi, after panicking fluoxetine.!
if another car is near them on the
road. Which of the following !
medications would be the MOST
!
appropriate for him?
Year:!2013!
F. Amitriptyline
G. Clonazepam
H. Paroxetine
I. Propranolol
J. Quetiapine.

Psychotherapy !

91. A!66TyearTold!man!who!worked!as!an! The!answer!is!D.!


executive!accountant!previously.!He!
has!retired!recently!and!presents! !
with!a!major!depressive!disorder.!!
What!is!the!best!choice!of! Explanation:!IPT!and!CBT!are!the!evidenceN
psychotherapy?! based!psychotherapy!indicated!for!major!
! depressive!disorder.!IPT!can!help!him!to!
work!on!role!transition,!grief!and!loss.!
F. Biofeedback!
G. Eye!movement!desensitization!and! !
reprocessing!
H. Hypnosis! !
I. Interpersonal!psychotherapy!
J. Systemic!desensitization.! Year:!2013!

92. A 40-year-old man with obsessive- The!answer!is!A.!


compulsive disorder presents for
treatment. He would prefer not using a !
medication due to possible side effects.
Which of the following psychological Explanation:!Exposure!with!response!
interventions would be the treatment of
choice for his disorder? prevention!(ERP)!is!part!of!CBT,!the!
psychotherapeutic!treatment!of!choice!for!
F. Exposure and response prevention obsessiveNcompulsive!disorder.!In!ERP,!
G. Hypnotherapy

!
!
27
H. Interpersonal psychotherapy patients!are!exposed!to!the!feared!stimuli!
I. Psychoanalysis and!obsessions!while!rituals!that!typically!
J. Supportive psychotherapy.
serve!to!reduce!anxiety!are!prevented.!!

Year:!2013!

93. Effective psychotherapy techniques The answer is D.


for Borderline Personality Disorder
include all of the following features
EXCEPT:
F. A treatment contract should be Explanation: Distant therapist may have
established in the beginning difficulty to establish trust and therapeutic
phase of psychotherapy. alliance with patient suffering from
G. Adverse effects of self- borderline personality disorder.
laceration are identified.
H. Focus of treatment is to
establish connection between
actions and feelings.
I. Therapist is quiet, distant and
does not get too involved. Year: 2013.
J. Therapist pays careful attention
to his or her own feelings in
order to manage
countertransference.

Ethics and laws !

94. A woman with a history of Answer:!The!answer!is!A.!


depression that responded well to
antidepressant medication is now !
depressed again. She does not want Explanation:!Autonomy!refers!to!the!
to take medications due to concerns obligation!of!a!doctor!to!respect!patients!
about side effects and risks. She
right!to!make!her!own!choice!of!treatment!
would like to try psychotherapy
instead. Her husband disagrees (i.e.!psychotherapy!instead!of!medication!in!
with her. Which of the following this!case).!
ethical principles is MOST relevant
in this situation? !
F. Autonomy Fiduciary!duty!refers!to!the!duty!that!a!
G. Beneficence doctor!should!act!as!the!best!interest!of!the!
H. Fiduciary duty patient.!In!this!scenario,!it!is!less!relevant!as!
I. Non-maleficence
compared!to!autonomy.!
J. Justice.

!
!
28
!

!
Year:!2013!

Liaison and neuropsychiatry !

95. You!are!the!resident!working!in!the! The!answer!is!E.!


oncology!ward.!Which!of!the!
following!factors!is!MOST!predictive! !
of!high!suicide!risk!in!a!palliative!
cancer!patient?! Explanation:!!Hopelessness!or!cognitive!
! symptoms!are!most!predictive!of!high!
suicide!risk!in!a!palliative!cancer!patient.!
F. Alopecia!associated!with!chemotherapy!
G. Low!energy!level! !
H. Poor!appetite!
I. Financial!difficulty! !
J. Hopelessness.!
! Year:!2013.!

96. A 40-year-old woman presents with Answer!is!C.!


chronic headache and blurred vision.
She wants symptomatic relief but is not !
concerned about the underlying
condition. Which of the following Explanation:!Patients!suffering!from!
psychiatric diagnoses is LEAST
relevant? hypochondriasis!are!more!concerned!about!
! the!diagnosis!rather!than!symptomatic!relief.!
The!other!options!are!associated!with!
F. Conversion!disorder! concern!of!symptoms!as!stated!in!this!case.!
G. Moderate!depressive!episode!with!
somatic!complaints! !
H. Hypochondriasis!
I. Pain!disorder! !
J. Somatization!disorder.!
! Year:!2013.!

!
!
29
MCQ!exam!(Paper!2):!Questions!

History, mental state exam and psychopathology!

1. Which!of!the!following!is!the!BEST!example!of!inattention?!
A. The!patient!interrupts!the!conversation!to!ask!when!he!will!be!discharged.!
B. The!patient!is!oriented!and!aware!of!his!recent!medical!problems!but!falls!asleep!during!the!
conversation.!
C. The!patient!suddenly!bursts!into!tears!when!you!are!discussing!his!recent!amputation.!
D. The!patient!watches!a!fly!buzzing!on!the!ceiling!while!you!are!discussing!the!prognosis!for!
his!lung!cancer,!then!falls!asleep.!
E. The!patient!cannot!remember!what!you!have!told!him!3!minutes!ago.!Then!he!tries!to!make!
up!the!answer.!
!

2. A!30TyearTold!man!firmly!believes!that!the!alien!has!put!an!implant!in!his!body!and!he!
feels!there!is!a!pushing!sensation!on!his!aorta.!Which!types!of!hallucinations!BEST!
suited!his!description?!
!

A. Auditory!
B. Cenesthetic!
C. Gustatory!
D. Kinesthetic!!
E. Visual.!
!

3. Perseveration!is!MOST!commonly!seen!in!which!of!the!following!disorders?!!!
!

A. Autism!
B. Attention!deficit!and!hyperactivity!disorder!
C. Bipolar!disorder!!
D. Frontal!lobe!dementia!!
E. Obsessive!compulsive!disorder.!
!

4. A patient presents with persistence, perfectionism and body image distortion. Which of the
following disorders is MOST likely to be associated with the above clinical features?

A. Anorexia nervosa
B. Borderline personality disorder
C. Delusional disorder
D. Hypochondriasis
E. Somatization disorder.
Cognitive assessment!

5. Which of the following statement is FALSE about Mini-mental state examination (MMSE)?

A. A total score of 5 indicates severe dementia.

!
!
30
B. Assessment of orientation to time and place is part of the MMSE.
C. The MMSE can be administered in Chinese or English.
D. The MMSE can be used to monitor treatment progress after initiation of acetylcholinesterase
inhibitors (AChEIs).
E. The MMSE is the gold standard in establishing the diagnosis of dementia.

Psychiatric epidemiology

6. Which!of!the!following!settings!have!the!HIGHEST!prevalence!of!delirium?!
!

A. Accident!and!emergency!department!
B. Cardiac!surgery!ward!
C. General!surgery!ward!
D. Hospice!with!palliative!advanced!cancer!patients!
E. Nursing!home!looking!after!dementia!patients.!
!!

7. The worldwide point prevalence of schizophrenia is BEST estimated as:

A. 1%
B. 2.5%
C. 5%
D. 7.5%
E. 10%.

Psychiatric aetiology, diagnosis and classification

8. Advanced paternal age is a well- established risk factor for which of the following
psychiatry illnesses?

A. Alcoholism
B. Major depressive disorder
C. Obsessive compulsive disorder
D. Generalized anxiety disorder
E. Schizophrenia.

9. Which of the following social factors is MOST LIKELY to be associated with


relapse of schizophrenia after hospitalization?
A. Discrimination
B. Financial problems
C. Homelessness
D. Increased expressed emotion
E. Isolation and absence of family member.

10. Individuals!with!which!one!of!the!following!genes!have!the!HIGHEST!risk!of!

!
!
31
developing!Alzheimers!disease?

A. Homozygous apolipoproteins!(APO) E2/E2


B. Homozygous APO E4/E4
C. Homozygous APO E6/E6
D. Heterozygous APO E2/E4
E. Heterozygous APO E2/E6.

General adult psychiatry

11. A!40TyearTold!man!suffered!from!a!heart!attack!under!general!anaesthesia.!The!
doctors!in!the!operation!theatre!successfully!resuscitated!him.!After!recovery,!he!
wants!to!sue!the!hospital!for!causing!him!postTtraumatic!stress!disorder!(PTSD).!He!
claims!the!resuscitation!process!was!!!extremely!traumatic.!Which!of!the!following!is!
the!MOST!IMPORTANT!feature!to!exclude!the!diagnosis!of!PTSD?!!
!

A. He does not have past history of PTSD.


B. He has hidden agenda of suing the hospital and tries to seek compensation.
C. He was under general anaesthesia and he was not conscious during resuscitation.
D. The process of resuscitation is not considered to be traumatic in general and doctor
resuscitated him in his best interest.
E. He did not seek treatment from a psychiatrist after the operation.

12. Which of the following schizophrenia patients has the HIGHEST suicide risk?
!
A.!An!adolescent!with!prodromal!symptoms!and!vague!paranoid!idea.!

B.!An!elderly!retired!man!with!late!onset!schizophrenia.!

C.!A!middleNaged!man!with!negative!symptoms.!

D.!A!middleNaged!woman!who!suffers!from!simple!schizophrenia.!

E.!A!young!male!university!student!who!once!had!high!expectations.!

13. Which of the following is NOT a recognized alteration of sleep disturbance associated with
major depressive disorder?

A. Difficulty in falling asleep


B. Increase in nocturnal awakening
C. Increase in density of Rapid Eye Movement (REM) sleep
D. Increase in the latency of Rapid Eye Movement (REM) sleep
E. Reduction of total sleep time.

14. A 40-year-old woman is staying with her god-brother. She was rejected by her family and

!
!
32
she felt helpless in the past. She needs her god-brother to make decision for her and her
god-brother asks her to be the housekeeper in return. One day, her god-brother needs to
go to Vietnam for 1 week and she feels extremely uncomfortable. Which of the following
personality traits BEST describes this person?

A. Borderline personality
B. Dependent personality
C. Histrionic personality
D. Schizoid personality
E. Schizotypal personality.

15. A 30-year-old man comes to see you and he worries about premature ejaculation. The
MOST appropriate time frame to meet the diagnosis of premature ejaculation is:
A. Ejaculation occurs within 60 seconds following vaginal penetration.
B. Ejaculation occurs within 90 seconds following vaginal penetration.
C. Ejaculation occurs within 120 seconds following vaginal penetration.
D. Ejaculation occurs within 150 seconds following vaginal penetration.
E. Ejaculation occurs within 180 seconds following vaginal penetration.

16. A!30TyearTold!man!suffered!from!eight!episodes!of!mood!disturbances!within!a!single!
year.!These!mood!episodes!met!the!diagnostic!criteria!for!major!depression,!mania!
and!hypomania.!These!episodes!were!separated!by!remission.!Which!of!the!following!
is!the!MOST!likely!diagnosis?
!

A. Cyclothymic!disorder!
B. Emotionally!unstable!personality!disorder!
C. Mixed!affective!disorder!
D. Rapid!cycling!bipolar!disorder!
E. Schizoaffective!disorder.!

Old age psychiatry

17. Which!of!the!following!is!NOT!a!predisposing!factor!for!depression!in!the!elderly?

A. Cerebrovascular accident
B. Poly-pharmacy
C. Looking after a spouse with chronic illness
D. Male gender
E. Widowhood.

18. Dementia!in!general!is!BEST!described!as:!!
!

A. Behavioural!disturbance!
B. Global!impairment!
C. Impaired!memory!for!personal!events!(e.g.!marriage,!past!occupation)!
D. Personality!change!

!
!
33
E. Progressive!deterioration.!

19. A!65TyearTold!man!presents!with!memory!loss,!which!of!the!following!is!the!
LEAST!important!indication!for!a!computerized!tomography!(CT)!scan!for!his!brain:

A. A sudden decrease in cognitive function over a one-month period


B. Gait abnormalities
C. His age
D. Urinary incontinence
E. Use of anticoagulants.

Child and adolescent psychiatry

20. Which!of!the!following!is!THE!MOST!IMPORTANT!predictor!of!bipolar!disorder!in!
adolescent!depression?!
!

A. Diurnal!variation!of!mood!
B. Hypomanic!symptoms!after!taking!antidepressants!
C. Hypersomnia!
D. Increased!weight!
E. Multiple!suicide!attempts.!

21. The!MOST!COMMON!reason!for!children!with!autism!are!brought!to!medical!attention!
by!their!parents!is: !
!

A. Language!delays!!
B. Lack!of!toilet!control!
C. Odd!play!
D. School!difficulties!
E. Stereotyped!behaviour.!

22. Which of the following statements is FALSE regarding separation anxiety disorder?

A. It involves recurrent distress when separate from attachment figures.


B. Separation anxiety disorder does not occur in adolescents.
C. Separation anxiety disorder is associated with school refusal.
D. The child has difficulty falling asleep at night.
E. The child may complain of somatic symptoms.

23. Which of the following symptoms is MOST LIKELY to persist when a young person suffers
from Attention Deficit and Hyperactivity Disorder (ADHD) becoming an adult?

A. Accident risk
B. Hyperactivity

!
!
34
C. Impulsivity
D. Inattention
E. Learning difficulties.

24. A baby is born with low birth weight, microcephaly, small eyes, upturned nose and a smooth,
undeveloped philtrum. He fails to thrive and develops seizures. Which of the following was he MOST
likely exposed to during pregnancy?

A. Alcohol
B. Cocaine
C. LSD
D. Methamphetamine
E. Inhalants.

Substance abuse

25. The!term!which!BEST!describes!!an!altered!physiological!state!and!neuroT!adaptation!
caused!by!repeated!administration!of!a!drug!is:!
!

A. Addiction!
B. Dependence!
C. Misuse!
D. Withdrawal!
E. Tolerance.!
!

26. While!driving,!a!40TyearTold!man!with!no!previous!history!refused!to!stop!for!the!
traffic!police.!He!was!subsequently!brought!into!the!accident!and!emergency!
department!(AED)!by!them.!You!are!the!resident!working!in!the!AED.!!When!you!
assess!him,!he!states!that!he!was!hearing!voices!in!clear!consciousness.!He!admits!to!a!
history!of!alcohol!use.!What!is!the!most!appropriate!diagnosis?!
!

A. Alcoholic!hallucinosis!!
B. Delirium!tremens!
C. Korsakoff!psychosis!
D. Schizophrenia!
E. Wernickes!encephalopathy.!
!

27. Which of the following BEST describes the mechanism of action with naltrexone in the treatment of
alcohol dependence?

A. Naltrexone acts on the benzodiazepine receptor and prevents alcohol withdrawal.


B. Naltrexone blocks opioid receptors to decrease alcohol craving.
C. Naltrexone inhibits aldehyde dehydrogenase to decrease alcohol craving.
D. Naltrexone is a GABA agonist and decreases alcohol craving.
E. Naltrexone is a glutamate antagonist and decreases alcohol craving.

!
!
35
28. Which of the following complications is LEAST likely to occur in cocaine abusers?
A. Arrhythmia
B. Hyperthermia
C. Myocardial infarction
D. Phlebitis
E. Seizure.

29. The primary neurotransmitter involved with Lysergic!acid!diethylamide!(LSD)!is:


a. Acetylcholine
b. Dopamine
c. NAminobutyric!acid!
d. Norepinephrine
e. Serotonin.

Psychopharmacology

30. Monoamine oxidase inhibitor (MAOI) is LEAST useful in treating which of the following
disorders?

A. Atypical depression
B. Obsessive compulsive disorder
C. Panic disorder
D. Severe depressive disorder not responding to selective serotonin reuptake inhibitor
E. Severe depressive disorder with hyponatraemia

31. Which! of! the! following! antidepressants! is! LEAST! likely! to! contribute! to! the!
development!of!serotonin!syndrome?!!$
!

A. Bupropion!
B. Fluoxetine!!
C. Moclobemide!
D. Paroxetine!
E. Venlafaxine.!

32. Which of the following antipsychotics exhibit a novel mechanism as a partial antagonist?

A. Aripiprazole
B. Clozapine
C. Olanzapine
D. Quetiapine
E. Ziprasidone.

33. Regarding!lamotrigine,!which!of!the!following!statements!is!FALSE?!!!
A. It!acts!at!voltageNsensitive!sodium!channels.!

!
!
36
B. It!inhibits!the!release!of!excitatory!amino!acid!neurotransmitters.!
C. It!is!not!effective!in!the!treatment!of!bipolar!depression.!!
D. Its!use!can!lead!to!toxic!epidermal!necrolysis.!
E. There!is!no!need!to!monitor!lamotrigine!blood!levels!during!treatment.!
!

34. Anticholinergic side effects include all of the following EXCEPT:

A. Bradycardia
B. Constipation
C. Dry mouth
D. Exacerbation of open-angle glaucoma
E. Urinary retention.

35. In a schizophrenia patient without history of asthma, the BEST treatment for chronic
akathisia is:

A. Alprazolam
B. Benzhexol
C. Clonazepam
D. Propranolol
E. Vitamin E.

36. Which of the following psychotropic medications is MOST EFFECTIVE in preventing


relapse in rapid cycling bipolar disorder?

A. Haloperidol
B. Lithium
C. Olanzapine
D. Valproate
E. Risperidone.

Psychotherapy

37. In! conducting! psychotherapy! with! individuals! who! have! experienced! a! traumatic!
event! and! suffer! from! postTtraumatic! stress! disorder! (PTSD),! the! following! are! all!
recommended!techniques!EXCEPT:!!
!

A. Encouraging!avoidance!of!emotion!!
B. Examining!feelings!of!guilt!
C. Examining!the!patients!response!to!the!trauma!
D. Offering!consolation!

!
!
37
E. Overcoming!avoidance!or!phobia!related!to!the!trauma.!

38. A 30-year-old woman with panic disorder does not respond to an initial treatment with an
selective serotonin reuptake inhibitor (SSRI). Which of the following treatment is
considered the best approach?

A. Benzodiazepine
B. Cognitive behaviour therapy
C. Serotonin-noradrenaline reuptake inhibitor
D. Olanzapine
E. Psychodynamic psychotherapy.

39. Which of the following statements is INCORRECT about Interpersonal therapy?


!

A. Has!been!shown!to!be!efficacious!in!adult!outpatients!suffering!from!depression.!
B. Is!more!efficacious!than!psychodynamic!psychotherapy!
C. Works!as!quickly!as!antidepressant!medication.!
D. Works!best!in!mild!to!moderate!depression.!
E. Works!best!in!people!facing!problems!in!role!transition.!
!

40. A! 23TyearTold! student! is! seen! in! the! University! counselling! centre! because! of!
relationship!problems.!!She!has!had!one!serious!relationship!with!her!boyfriend!that!
lasted!about!six!months.!!She!terminated!the!relationship!because!she!discovered!that!
her!boyfriend!was!seeing!someone!else.!!During!psychotherapy,!she!mentions!that!All!
men!just!cant!be!trusted.!!Which!of!the!following!BEST!describes!this!thinking!error?

A. Arbitrary!inference!
B. Dichotomous!thinking!
C. Magnification!
D. Overgeneralization!
E. Selective!abstraction.!

Ethics and laws

41. All! medical! students! are! required! to! study! psychiatry! so! that! they! are! qualified! to!
assess! and! manage! common! psychiatric! disorders! after! graduation.! The! compulsory!
psychiatric!training!is!BEST!described!by!which!of!the!following!ethical!principles?!!!
!

A. Autonomy!
B. Beneficence!
C. Morality!
D. Justice!
E. Fiduciary!duty.!

Liaison and neuropsychiatry

!
!
38
42. You! are! a! medical! resident.! A! 40TyearTold! schizophrenia! patient! was! admitted! as! a!
result! of! polydipsia.! Which! of! the! following! statements! regarding! polydipsia! in!
schizophrenia!is!NOT!true?!!
!

A. It!is!associated!with!low!urine!osmolality.!
B. It!is!associated!with!high!sodium!in!urine.!
C. It!is!associated!with!low!serum!sodium.!!
D. Patients!should!be!investigated!for!Syndrome!of!Inappropriate!Secretion!of!AntiNDiuretic!
Hormone!(SIADH).!!
E. Polydipsia!occurs!in!between!5!to!20%!of!patients!suffering!from!chronic!schizophrenia.!!
!

43. A!40TyearTold!British!man!is!admitted!to!the!medical!ward.!He!tries!to!run!away!from!
the!ward.!He!is!very!aggressive!and!the!security!guards!have!difficulty!to!stop!him.!He!
claims!that!he!is!currently!working!in!the!office.!The!medical!records!show!that!he!has!
been!drinking!half!of!a!bottle!of!whisky!every!night.!You!need!to!order!a!medication!to!
sedate! him.! He! refuses! to! take! any! oral! medication.! ! Which! of! the! following!
medications!is!the!BEST!alternative!in!Singapore?!
!

A. Intramuscular!diazepam!
B. Intramuscular!lorazepam!
C. Intramuscular!olanzapine!
D. Intramuscular!quetiapine!
E. Intramuscular!risperidone.!
!

44. You! are! the! resident! working! for! the! liver! transplant! team.! The! MOST! common!
psychiatric! disorder! in! patients! with! liver! cancer! in! the! week! before! and! after!
transplantation!is:!
!

A. Adjustment!disorder!
B. Delirium!
C. Generalized!anxiety!disorder!!
D. Major!depressive!disorder!
E. Somatoform!pain!disorder.!
!

45. You!are!the!resident!working!in!the!obstetrics!department.!Which!of!the!following!is!
the!BEST!treatment!strategies!to!treat!hyperemesis!during!pregnancy?!
!

A. Benzodiazepines!e.g.!alprazolam!
B. Relaxation!therapy!
C. Psychoanalysis!
D. Selective!serotonin!reuptake!inhibitor!e.g.!fluvoxamine!
E. Stimulus!deprivation.!

!
!
39
!

MCQ!exam!(Paper!2):!Answer!Key!

History, mental state exam and psychopathology! !

46. Which!of!the!following!is!the!BEST!example!of! The!best!answer!is!D.!!


inattention?!
F. The!patient!interrupts!the!conversation!to!ask! A. Impulsive!interruptions!may!
when!he!will!be!discharged.! or!may!not!indicate!
G. The!patient!is!oriented!and!aware!of!his!recent! inattentiveness.!
medical!problems!but!falls!asleep!during!the! B. Falling!asleep!may!indicate!
conversation.! inattentiveness,!but!further!
H. The!patient!suddenly!bursts!into!tears!when!you! information!would!be!
are!discussing!his!recent!amputation.! needed!to!rule!out!other!
I. The!patient!watches!a!fly!buzzing!on!the!ceiling! explanations!such!as!recent!
while!you!are!discussing!the!prognosis!for!his! administration!of!a!sedating!
lung!cancer,!then!falls!asleep.! medication.!!
J. The!patient!cannot!remember!what!you!have! C. Sudden!bursts!of!affect!have!
told!him!3!minutes!ago.!Then!he!tries!to!make!up! a!significant!differential!
the!answer.! beyond!inattention.!!
! D. This!patient!seems!
distracted!despite!discussion!
of!an!issue!of!vital!personal!
importance!to!him.!!In!the!
context!also!of!apparent!
drowsiness,!the!clinician!
should!suspect!the!presence!
of!delirium.!
E. The!patient!may!suffer!from!
dementia!and!cannot!
register!the!information.!
!
!

Year:!2013.!

47. A!30TyearTold!man!firmly!believes!that!the! The!answer!is!B.!


alien!has!put!an!implant!in!his!body!and!he!
feels!there!is!a!pushing!sensation!on!his! !
aorta.!Which!types!of!hallucinations!BEST!
suited!his!description?! Explanation:!Cenesthetic!
! hallucinations!are!unfounded!
sensations!of!altered!states!in!bodily!
F. Auditory! organs.!!Examples!of!cenesthetic!
G. Cenesthetic! hallucinations!are!a!burning!
H. Gustatory! sensation!in!the!brain,!a!pushing!
I. Kinesthetic!! sensation!in!the!blood!vessels,!and!a!

!
!
40
J. Visual.! cutting!sensation!in!the!bone!
! marrow.!!Bodily!distortion!may!also!
occur.!
!

Year:!2013.!

This!is!an!advancedNlevel!question.!

48. Perseveration!is!MOST!commonly!seen!in! The!answer!is!D.!


which!of!the!following!disorders?!!!
! !

F. Autism! Explanation:!Perseveration!is!a!
G. Attention!deficit!and!hyperactivity!disorder! persisting!response!to!a!previous!
H. Bipolar!disorder!! stimulus!after!a!new!stimulus!has!
I. Frontal!lobe!dementia!! been!presented;!it!is!often!
J. Obsessive!compulsive!disorder.!
associated!with!frontal!lobe!lesion.!
!
!

Year:!2013!

49. A patient presents with persistence, The!answer!is!A.!


perfectionism and body image distortion. Which
of the following disorders is MOST likely to be !
associated with the above clinical features?
Explanation:!Persistence!and!
F. Anorexia nervosa perfectionism!are!associated!with!
G. Borderline personality disorder obsessiveNcompulsive!personality!
H. Delusional disorder traits!and!disorders,!along!with!
I. Hypochondriasis
J. Somatization disorder. body!image!distortion!occurring!in!
many!patients!with!anorexia!
nervosa.!!

Year:!2013!

!
!
41
Cognitive assessment! !

50. Which of the following statement is FALSE The!answer!is!E.!


about Mini-mental state examination (MMSE)?
!
F. A total score of 5 indicates severe dementia. Explanation:!MMSE!is!a!screening!
G. Assessment of orientation to time and place is part
of the MMSE. test!and!there!is!no!gold!standard!to!
H. The MMSE can be administered in Chinese or establish!the!diagnosis!of!dementia!
English. in!clinical!practice.!Clinical!
I. The MMSE can be used to monitor treatment
progress after initiation of acetylcholinesterase
assessment!by!a!psychiatrist!or!
inhibitors (AChEIs). geriatrician!is!more!important!than!
J. The MMSE is the gold standard in establishing the MMSE!in!establishing!diagnosis!of!
diagnosis of dementia.
dementia.!

Year:!2013!

Psychiatric epidemiology !

51. Which!of!the!following!settings!have!the! The!answer!is!D.!!


HIGHEST!prevalence!of!delirium?! !
! Explanation:!General!surgical!
patients:!!10N15%;!Cardiac!surgery!
F. Accident!and!emergency!department! patients:!!30%;!Hip!fractures:!!50%;!
G. Cardiac!surgery!ward! Age!>65!admitted!to!ICU:!!70%;!
H. General!surgery!ward! Palliative!advanced!cancer!patients:!!
I. Hospice!with!palliative!advanced!cancer!patients! 88%.!!
J. Nursing!home!looking!after!dementia!patients.! !
!!
!
Year:!2013.!
52. The worldwide point prevalence of The!answer!is!A.!
schizophrenia is BEST estimated as:
!

Explanation:!The!worldwide!point!
F. 1%
G. 2.5%
prevalence!is!1%!and!Singapore!is!
H. 5% 0.6%.!
I. 7.5%
J. 10%. !

Year:!2013!

!
!
42
Psychiatric aetiology, diagnosis and classification !

53. Advanced paternal age is a well- established The!answer!is!E.!


risk factor for which of the following psychiatry
illnesses? !

Explanation:!Advanced!paternal!age!
F. Alcoholism
G. Major depressive disorder is!a!risk!factor!for!schizophrenia.!
H. Obsessive compulsive disorder
I. Generalized anxiety disorder !
J. Schizophrenia.
!

Year:!2013.!

54. Which of the following social factors is The answer is D.


MOST LIKELY to be associated with relapse
of schizophrenia after hospitalization?
A. Discrimination
B. Financial problems High levels of expressed emotion
C. Homelessness (EE) in the families of patients with
D. Increased expressed emotion schizophrenia predicted relapse
E. Isolation and absence of family following hospital discharge.
member.
Psychoeducational family therapy
aims at helping the family reduce
the factors that constitute
expressed emotion. The therapist
also educates the family about
schizophrenia and the need to
continue antipsychotic medication
indefinitely.

Year:!2013.

55. Individuals!with!which!one!of!the!following! The!answer!is!B.!


genes!have!the!HIGHEST!risk!of!developing!
Alzheimers!disease? !

Explanation:!Homozygous!APO!
F. Homozygous apolipoproteins!(APO) E2/E2 E4/E4!increases!the!risk!of!
G. Homozygous APO E4/E4
developing!Alzhimers!disease!by!8!
H. Homozygous APO E6/E6
I. Heterozygous APO E2/E4 times.!
J. Heterozygous APO E2/E6.

!
!
43
!

Year:!2013.!

General adult psychiatry !

56. A!40TyearTold!man!suffered!from!a!heart! The!answer!is!D.!


attack!under!general!anaesthesia.!The!
doctors!in!the!operation!theatre!successfully! !
resuscitated!him.!After!recovery,!he!wants!to!
sue!the!hospital!for!causing!him!postT Explanation:!In!the!new!diagnostic!
traumatic!stress!disorder!(PTSD).!He!claims! criteria,!it!is!possible!for!a!PTSD!
the!resuscitation!process!was!!!extremely! patient!not!to!directly!witness!a!
traumatic.!Which!of!the!following!is!the!MOST! traumatic!event!and!learn!about!it!
IMPORTANT!feature!to!exclude!the!diagnosis!
from!others!after!the!accident.!The!
of!PTSD?!!
! most!crucial!feature!is!whether!
resuscitation!itself!is!not!a!traumatic!
F. He does not have past history of PTSD. process!and!!it!is!necessary!for!
G. He has hidden agenda of suing the hospital doctors!to!save!patients!life!by!
and tries to seek compensation. resuscitation.!
H. He was under general anaesthesia and he was
not conscious during resuscitation. !
I. The process of resuscitation is not considered
to be traumatic in general and doctor !
resuscitated him in his best interest.
J. He did not seek treatment from a psychiatrist Year:!2013!
after the operation.
!

This!is!an!advanced!!level!question.!

57. Which of the following schizophrenia patients The!answer!is!E.!


has the HIGHEST suicide risk?
! !
A.!An!adolescent!with!prodromal!symptoms!and!vague!
paranoid!idea.! Explanation:!The!profile!of!the!
patient!at!greatest!risk!is!a!young!
B.!An!elderly!retired!man!with!late!onset!schizophrenia.! man!who!once!had!high!
expectations,!declined!from!a!higher!
C.!A!middleNaged!man!with!negative!symptoms.!
level!of!functioning,!realizes!that!his!
D.!A!middleNaged!woman!who!suffers!from!simple! dreams!are!not!likely!to!come!true,!
schizophrenia.! and!has!lost!faith!in!the!effectiveness!
of!treatment.!
E.!A!young!male!university!student!who!once!had!high!

!
!
44
expectations.! !

! !

Year:!2013.!!!

58. Which of the following is NOT a recognized The!answer!is!D.!


alteration of sleep disturbance associated with
major depressive disorder? !

Explanation:!It!should!be!reduction!
F. Difficulty in falling asleep
G. Increase in nocturnal awakening of!latency!of!REM!sleep.!
H. Increase in density of Rapid Eye Movement (REM)
sleep !
I. Increase in the latency of Rapid Eye Movement
(REM) sleep !
J. Reduction of total sleep time.
Year:!2013.!

This!is!an!advanced!level!question.!

59. A 40-year-old woman is staying with her god- The!answer!is!B.!


brother. She was rejected by her family and she
felt helpless in the past. She needs her god- !
brother to make decision for her and her god-
brother asks her to be the housekeeper in Explanation:!She!is!best!described!to!
return. One day, her god-brother needs to go to
Vietnam for 1 week and she feels extremely suffer!from!dependent!personality!
uncomfortable. Which of the following because!she!cannot!make!decision!
personality traits BEST describes this person? on!her!own,!uncomfortable!when!
left!alone!and!subordination!to!her!
F. Borderline personality
G. Dependent personality godNbrothers!need!(e.g.!to!be!a!
H. Histrionic personality housekeeper).!
I. Schizoid personality
J. Schizotypal personality. !

Year:!2013!

60. A 30-year-old man comes to see you and he The!answer!is!A.!


worries about premature ejaculation. The MOST
appropriate time frame to meet the diagnosis of !
premature ejaculation is:
F. Ejaculation occurs within 60 seconds following Explanation:!Premature!ejaculation!
vaginal penetration.
G. Ejaculation occurs within 90 seconds following is!defined!as!ejaculation!occurs!
vaginal penetration. within!60!second!following!vaginal!
H. Ejaculation occurs within 120 seconds following penetration!and!the!patient!wishes!
vaginal penetration.

!
!
45
I. Ejaculation occurs within 150 seconds following it.!
vaginal penetration.
J. Ejaculation occurs within 180 seconds following !
vaginal penetration.
!

Year:!2013!

61. A!30TyearTold!man!suffered!from!eight! The!answer!is!D.!


episodes!of!mood!disturbances!within!a!
single!year.!These!mood!episodes!met!the! !
diagnostic!criteria!for!major!depression,!
mania!and!hypomania.!These!episodes!were! Explanation:!This!patient!has!more!
separated!by!remission.!Which!of!the! than!4!episodes!of!mania!and!
following!is!the!MOST!likely!diagnosis? depressive!disorder!in!1!year.!He!is!
! too!severe!for!cyclothymic!disorder.!
F. Cyclothymic!disorder! !
G. Emotionally!unstable!personality!disorder!
H. Mixed!affective!disorder! !
I. Rapid!cycling!bipolar!disorder!
J. Schizoaffective!disorder.! Year:!2013!

Old age psychiatry !

62. Which!of!the!following!is!NOT!a!predisposing! The!answer!is!D.!


factor!for!depression!in!the!elderly?
!

F. Cerebrovascular accident Explanation:!Women!have!longer!


G. Poly-pharmacy
life!span!compared!to!men.!As!a!
H. Looking after a spouse with chronic illness
I. Male gender result,!women!are!more!likely!to!be!
J. Widowhood. a!widow!and!correspond!to!option!E.!

Year:!2013.!

63. Dementia!in!general!is!BEST!described!as:!! The!answer!is!E.!


!
!
F. Behavioural!disturbance!
G. Global!impairment! Explanation:! dementia! is! a!
H. Impaired!memory!for!personal!events!(e.g.! progressive! ! deterioration! of!
marriage,!past!occupation)! cognition,!including!memory!and/or!
I. Personality!change!

!
!
46
J. Progressive!deterioration.! one! of! four! other! cognitive!
symptoms! (aphasia,! apraxia,!
agnosia,! impaired! executive!
functioning)! but! it! may! not! be! a!
global! impairment.! Impairment!
memory! for! recent! events! is! more!
common!than!personal!events.!

Year:!2013.!

64. A!65TyearTold!man!presents!with! The!answer!is!C.!


memory!loss,!which!of!the!following!is!the!
LEAST!important!indication!for!a! !
computerized!tomography!(CT)!scan!for!his!
brain: Explanation:!!The!other!factors!
suggest!an!organic!cause.!

F. A sudden decrease in cognitive function over a one- !


month period
G. Gait abnormalities !
H. His age
I. Urinary incontinence Year:!2013.!
J. Use of anticoagulants.

Child and adolescent psychiatry !

65. Which!of!the!following!is!THE!MOST! The!answer!is!B.!


IMPORTANT!predictor!of!bipolar!disorder!in!
adolescent!depression?! !
!
Explanation:!!Antidepressant!is!the!
F. Diurnal!variation!of!mood! most!common!cause!of!hypomania!
G. Hypomanic!symptoms!after!taking! and!hypomania!is!part!of!the!bipolar!
antidepressants! disorder.!
H. Hypersomnia!
I. Increased!weight! !
J. Multiple!suicide!attempts.!
!

Year:!2013.!

66. The!MOST!COMMON!reason!for!children!with! The!answer!is!A.!


autism!are!brought!to!medical!attention!by!
their!parents!is: ! !

!
!
47
! Explanation:!Language!delay!is!the!
most!obvious!sign!and!most!
F. Language!delays!! disturbing!to!parents!when!they!
G. Lack!of!toilet!control!
discover!their!children!are!lacking!
H. Odd!play!
I. School!difficulties! behind!in!language!development!
J. Stereotyped!behaviour.! compared!to!other!children.!

Year:!2013.!

67. Which of the following statements is FALSE The!answer!is!B.!


regarding separation anxiety disorder?
!
F. It involves recurrent distress when separate from Explanation:!Separation anxiety disorder
attachment figures.
G. Separation anxiety disorder does not occur in is more common in children and it can
adolescents. occur in adolescents.
H. Separation anxiety disorder is associated with school
refusal. !
I. The child has difficulty falling asleep at night.
J. The child may complain of somatic symptoms. !
Year:!2013.!

68. Which of the following symptoms is MOST !


LIKELY to persist when a young person suffers
from Attention Deficit and Hyperactivity The!answer!is!D.!
Disorder (ADHD) becoming an adult?
!
F. Accident risk
G. Hyperactivity Answer:!Although!symptoms!of!
H. Impulsivity
I. Inattention hyperactivity!and!impulsivity!often!
J. Learning difficulties. improve!as!the!child!grows!older,!
inattentive!symptoms!are!likely!to!
persist.!

Year:!2013!

69. A baby is born with low birth weight, microcephaly, The!answer!is!A.!


small eyes, upturned nose and a smooth, undeveloped
philtrum. He fails to thrive and develops seizures. !
Which of the following was he MOST likely exposed
to during pregnancy? Explanation:!This!is!a!classical!

!
!
48
F. Alcohol description!of!the!facial!features!of!
G. Cocaine foetal!alcohol!syndrome!
H. LSD
I. Methamphetamine
J. Inhalants.
!

Year:!2013.!

Substance abuse !

70. The!term!which!BEST!describes!!an!altered! The!answer!is!B.!


physiological!state!and!neuroT!adaptation!
caused!by!repeated!administration!of!a!drug! !
is:!
! Explanation:!The!term!dependence!
indicates!physiological!changes.!
F. Addiction! Misuse!refers!to!use!of!any!drug,!
G. Dependence! usually!by!selfNadministration,!in!a!
H. Misuse! manner!that!deviates!from!approved!
I. Withdrawal! social!or!medical!patterns.!Addiction!
J. Tolerance.! is!nonNspecific!and!ignores!the!
! concept!of!substance!dependence!is!
a!medical!condition.!!
!
!

Year:!2013.!
71. While!driving,!a!40TyearTold!man!with!no! The!answer!is!A.!
previous!history!refused!to!stop!for!the!traffic!
police.!He!was!subsequently!brought!into!the! !
accident!and!emergency!department!(AED)!
by!them.!You!are!the!resident!working!in!the! Explanation:!!Auditory!hallucinosis!
AED.!!When!you!assess!him,!he!states!that!he! is!usually!composed!of!auditory!
was!hearing!voices!in!clear!consciousness.!He! hallucination.!Sensorium!is!clear!and!
admits!to!a!history!of!alcohol!use.!What!is!the! it!is!different!from!delirium!tremens.!
most!appropriate!diagnosis?!
There!are!not!enough!symptoms!to!
!
conclude!that!he!suffers!from!
F. Alcoholic!hallucinosis!! schizophrenia.!
G. Delirium!tremens!
H. Korsakoff!psychosis! !
I. Schizophrenia!
J. Wernickes!encephalopathy.! Year:!2013.!
!

72. Which of the following BEST describes the The!answer!is!B.!


mechanism of action with naltrexone in the treatment
of alcohol dependence? !

!
!
49
F. Naltrexone acts on the benzodiazepine receptor and Explanation:!Option!A!refers!to!
prevents alcohol withdrawal. benzodiazepine.!Option!C!refers!to!
G. Naltrexone blocks opioid receptors to decrease alcohol
craving. the!action!of!disulfiram.!Option!D!
H. Naltrexone inhibits aldehyde dehydrogenase to decrease and!E!refer!to!the!action!of!
alcohol craving. acamprosate.!!
I. Naltrexone is a GABA agonist and decreases alcohol
craving. !
J. Naltrexone is a glutamate antagonist and decreases
alcohol craving.
!

Year:!2013.!

73. Which of the following complications is LEAST The!answer!is!D.!


likely to occur in cocaine abusers?
F. Arrhythmia !
G. Hyperthermia
H. Myocardial infarction Explanation:!Cocaine!is!least!likely!
I. Phlebitis
J. Seizure. to!be!injected!via!IV!route.!As!a!
result,!phlebitis!is!relatively!less!
likely!to!occur!in!patients!misusing!
cocaine.!

Year:!2013!

This!is!an!advancedNlevel!question.!

!!

74. The primary neurotransmitter involved with The!answer!is!E.!


Lysergic!acid!diethylamide!(LSD)!is:
f. Acetylcholine !
g. Dopamine
h. NAminobutyric!acid! Explanation:!LSD!increases!the!level!
i. Norepinephrine
of!serotonin!and!overdose!of!LSD!is!
j. Serotonin.
associated!with!serotonin!
syndrome.!

Year:!2013.!

!
!
50
!

Psychopharmacology !

75. Monoamine oxidase inhibitor (MAOI) is LEAST The!answer!is!B.!


useful in treating which of the following
disorders? !

Explanation:!MAOI!is!least!useful!to!
F. Atypical depression
G. Obsessive compulsive disorder treat!OCD.!
H. Panic disorder
I. Severe depressive disorder not responding to !
selective serotonin reuptake inhibitor
J. Severe depressive disorder with hyponatraemia !

Year:!2013.!

This!is!an!advancedNlevel!question.!

76. Which! of! the! following! antidepressants! is! The!answer!is!A.!


LEAST! likely! to! contribute! to! the!
development!of!serotonin!syndrome?!!$ !
!
Explanation:!Bupoprion!works!on!
F. Bupropion! dopamine!and!norepinephrine.!As!a!
G. Fluoxetine!! result,!it!is!least!likely!to!cause!
H. Moclobemide! serotonin!syndrome.!
I. Paroxetine!
J. Venlafaxine.! !

Year:!2013.!

!
This!is!an!advancedNlevel!question.!

77. Which of the following antipsychotics exhibit a The!answer!is!A.!


novel mechanism as a partial antagonist?
!
F. Aripiprazole Explanation:!Aripiprazole!exhibits!a!
G. Clozapine
H. Olanzapine novel!mechanism!by!definition!
I. Quetiapine while!other!antipsychotics!listed!do!
J. Ziprasidone. not!have!this!property.!

!
!
51
!

Year:!2013.!

78. Regarding!lamotrigine,!which!of!the!following! The!answer!is!C.!


statements!is!FALSE?!!!
F. It!acts!at!voltageNsensitive!sodium!channels.! !
G. It!inhibits!the!release!of!excitatory!amino!acid!
neurotransmitters.! Explanation:!DoubleNblind,!placeboN
H. It!is!not!effective!in!the!treatment!of!bipolar! controlled!trials!have!shown!that!it!
depression.!! has!acute!and!prophylactic!antiN
I. Its!use!can!lead!to!toxic!epidermal!necrolysis.! depressant!effects.!It!can!lead!to!
J. There!is!no!need!to!monitor!lamotrigine!blood!
severe!dermatological!condition!
levels!during!treatment.!
! such!as!Steven!Johnson!syndrome/!
toxic!epidermal!necrolysis.!In!
Singapore,!there!is!no!blood!test!to!
check!its!level.!

Year:!2013!

This!is!an!advancedNlevel!question.!

79. Anticholinergic side effects include all of The!answer!is!A.!


the following EXCEPT:
!
Explanation:!Anticholinergic!side!
F. Bradycardia effect!include!tachycardia.!
G. Constipation
H. Dry mouth !
I. Exacerbation of open-angle glaucoma
J. Urinary retention. !

Year:!2013.!

80. In a schizophrenia patient without history of The!answer!is!D.!


asthma, the BEST treatment for chronic
akathisia is: !

!
!
52
Explanation:!Propranolol!(beta!
blocker)!is!the!best!treatment!in!this!
F. Alprazolam
G. Benzhexol patient!without!history!of!asthma!as!
H. Clonazepam it!does!not!cause!dependence!as!in!
I. Propranolol Option!A!and!C.!Beta!blockers!can!
J. Vitamin E.
reduce!restlessness!and!tremor.!!
Option!B!is!more!useful!for!
pseudoparkinsonism.!

Year:!2013.!

81. Which of the following psychotropic The!answer!is!D.!


medications is MOST EFFECTIVE in preventing
relapse in rapid cycling bipolar disorder? !

Explanation:!Lithium!is!less!effective!
F. Haloperidol
G. Lithium in!preventing!relapse!in!rapid!
H. Olanzapine cycling!bipolar!disorder.!Valproate!
I. Valproate and!carbamazepine!are!preferred!
J. Risperidone.
over!lithium.!

2013!

Psychotherapy !

82. In!conducting!psychotherapy!with!individuals! The!answer!is!A.!


who!have!experienced!a!traumatic!event!and!
suffer! from! postTtraumatic! stress! disorder! !
(PTSD),! the! following! are! all! recommended!
techniques!EXCEPT:!! Explanation:!A!is!the!answer.!Affect!
! or!emotion!should!not!be!avoided!in!
psychotherapy.!
F. Encouraging!avoidance!of!emotion!!
G. Examining!feelings!of!guilt! !
H. Examining!the!patients!response!to!the!trauma!
I. Offering!consolation! !
J. Overcoming!avoidance!or!phobia!related!to!the!
trauma.! Year!2013.!

!
!
53
83. A 30-year-old woman with panic disorder does The!answer!is!B.!
not respond to an initial treatment with an
selective serotonin reuptake inhibitor (SSRI). !
Which of the following treatment is considered
the best approach?
Explanation:!The!effect!of!CBT!is!as!
F. Benzodiazepine efficacious!as!SSRI.!
G. Cognitive behaviour therapy
H. Serotonin-noradrenaline reuptake inhibitor !
I. Olanzapine
J. Psychodynamic psychotherapy. !

Year:!2013.!

84. Which of the following statements is The!answer!is!C.!


INCORRECT about Interpersonal therapy?
! !
F. Has!been!shown!to!be!efficacious!in!adult!outpatients! Explanation:!Interpersonal!therapy!
suffering!from!depression.!
G. Is!more!efficacious!than!psychodynamic! is!slower!than!the!effect!of!
psychotherapy! antidepressant!because!the!
H. Works!as!quickly!as!antidepressant!medication.! therapist!needs!time!to!assess!the!
I. Works!best!in!mild!to!moderate!depression.!
patient!and!formulate!strategies!(e.g.!
J. Works!best!in!people!facing!problems!in!role!
transition.! interpersonal!inventory!and!role!
! play).!

!
!

Year:!2013.!

85. A! 23TyearTold! student! is! seen! in! the! The!answer!is!D.!


University! counselling! centre! because! of!
relationship! problems.! ! She! has! had! one! !
serious! relationship! with! her! boyfriend! that! Explanation:!This!patient!
lasted! about! six! months.! ! She! terminated! the! demonstrates!overgenearlization!
relationship! because! she! discovered! that! her! because!she!believes!in!and!follows!
boyfriend! was! seeing! someone! else.! ! During!
a!general!rule!on!the!basis!of!limited!
psychotherapy,! she! mentions! that! All! men!
just! cant! be! trusted.! ! Which! of! the! following! examples.!
BEST!describes!this!thinking!error?
!
F. Arbitrary!inference!
!
G. Dichotomous!thinking!
H. Magnification!
Year:!2013.!
I. Overgeneralization!
J. Selective!abstraction.!

!
!
54
Ethics and laws !

86. All! medical! students! are! required! to! study! The!answer!is!B.!


psychiatry!so!that!they!are!qualified!to!assess!
and! manage! common! psychiatric! disorders! !
after!graduation.!The!compulsory!psychiatric!
training! is! BEST! described! by! which! of! the! Explanation:!With!competent!
following!ethical!principles?!!! psychiatric!knowledge,!the!future!
! doctors!can!provide!benefits!to!
patients!when!making!decision!on!
F. Autonomy!
diagnosis!and!management.!The!
G. Beneficence!
H. Morality! other!concept!that!is!relevant!is!
I. Justice! dereliction.!Fiduciary!duty!refers!to!
J. Fiduciary!duty.! the!duty!that!the!doctor!must!act!in!
the!patients!best!interest!and!it!is!
not!as!relevant!as!beneficence!in!this!
case.!!

Year:!2013!

Liaison and neuropsychiatry !

87. You! are! a! medical! resident.! A! 40TyearTold! The!answer!is!B.!


schizophrenia! patient! was! admitted! as! a!
result! of! polydipsia.! Which! of! the! following! !
statements! regarding! polydipsia! in!
schizophrenia!is!NOT!true?!! Explanation:!Psychogenic!
! polydispsia!is!a!common!condition!
for!chronic!schizophrenia.!The!
F. It!is!associated!with!low!urine!osmolality.! estimates!of!prevalence!range!from!
G. It!is!associated!with!high!sodium!in!urine.!
5!to!20%.!The!patient!drinks!excess!
H. It!is!associated!with!low!serum!sodium.!!
I. Patients!should!be!investigated!for!Syndrome!of! water!and!results!in!diluted!urine.!
Inappropriate!Secretion!of!AntiNDiuretic! The!body!tries!to!preserve!sodium!
Hormone!(SIADH).!! and!results!in!low!urine!sodium.!The!
J. Polydipsia!occurs!in!between!5!to!20%!of! serum!is!diluted.!SIADH!should!be!
patients!suffering!from!chronic!schizophrenia.!! considered!for!DDX.!
!
!
!
!

!
!
55
Year:!2013.!

This!is!an!advancedNlevel!question.!

88. A! 40TyearTold! British! man! is! admitted! to! the! The!answer!is!B.!


medical! ward.! He! tries! to! run! away! from! the!
ward.! He! is! very! aggressive! and! the! security! !
guards! have! difficulty! to! stop! him.! He! claims!
that!he!is!currently!working!in!the!office.!The! Explanation:!He!suffers!from!
medical! records! show! that! he! has! been! delirium!tremens.!Intramuscular!
drinking! half! of! a! bottle! of! whisky! every! diazepam!has!poor!absorption.!
night.! You! need! to! order! a! medication! to! Option!C!and!D!are!not!available!in!
sedate! him.! He! refuses! to! take! any! oral!
Singapore.!Option!E!is!a!depot!
medication.! ! Which! of! the! following!
medications! is! the! BEST! alternative! in! antipsychotics!and!it!is!not!useful!in!
Singapore?! acute!setting.!
!
!
F. Intramuscular!diazepam!
G. Intramuscular!lorazepam! !
H. Intramuscular!olanzapine!
I. Intramuscular!quetiapine! Year:!2013!
J. Intramuscular!risperidone.!
!
!
!

! !

89. You! are! the! resident! working! for! the! liver! The!answer!is!B.!
transplant! team.! The! MOST! common!
psychiatric! disorder! in! patients! with! liver! !
cancer! in! the! week! before! and! after! Explanation:!Delirium!is!the!most!
transplantation!is:! common!disorder!and!it!can!be!
! caused!by!systemic!infection,!side!
F. Adjustment!disorder! effect!of!immunosuppressant,!
G. Delirium! impairment!of!liver!function,!hepatic!
H. Generalized!anxiety!disorder!! encephalopathy.!
I. Major!depressive!disorder!
J. Somatoform!pain!disorder.! !
!
!

Year:!2013.!

90. You!are!the!resident!working!in!the!obstetrics! The!answer!is!B.!

!
!
56
department.! Which! of! the! following! is! the! !
BEST! treatment! strategies! to! treat! Explanation:!Benzodiazepine!may!
hyperemesis!during!pregnancy?! cause!cleft!palate!in!the!first!
!
trimester.!Psychoanalysis!is!not!for!
F. Benzodiazepines!e.g.!alprazolam! symptomatic!relief.!SSRI!like!
G. Relaxation!therapy! fluvoxamine!is!associated!with!
H. Psychoanalysis! nausea!side!effect.!Hyperemesis!is!
I. Selective! serotonin! reuptake! inhibitor! e.g.! not!associated!with!stimulus.!
fluvoxamine! Relaxation!therapy!is!the!best!
J. Stimulus!deprivation.!
treatment.!!

Year:!2013.!

MCQ!exam!(Paper!3)!:!Questions!

History and mental state exam!

1. A!40TyearTold!woman!is!brought!by!her!husband!brought!to!see!you!as!a!result!of!
abnormal!behaviour.!During!the!interview,!she!laughs!and!cries!with!a!very!short!
period!of!time.!In!your!record,!her!affect!is!BEST!described!as:!

!
!
57
!

A. Apathy!
B. Blunted!affect!
C. Euthymic!affect!
D. Labile!affect!
E. Flat!affect.!
Cognitive assessment!

2. Which of the following is the presentation of cortical dementia?

A. A 40-year-old man presents with memory loss and writhing!movements.!His!father!suffered!


from!the!same!condition.!
B. A!50NyearNold!man!presents!with!change!in!personality,!poor!judgment,!disinhibition!and!
perservation.!He!made!a!lot!of!mistakes!in!Lurias!hand!test.!
C. A 60-year-old woman presents with memory loss and depression after a cerebrovascular
accident.!
D. A 70-year-old man presents with memory loss, rigidity, slowness!of!movement!and!difficulty!
with!walking!and!gait.!
E. A 75-year-old woman presents with visual hallucination. The GP gave her haloperidol and she
develops severe extrapyramidal side effects.!

Psychiatric epidemiology

3. After the age of 65, the prevalence of dementia increases by how many times every 5
years?
A. 2 times
B. 4 times
C. 6 times
D. 8 times
E. 10 times.
Psychiatric aetiology, diagnosis and classification

4. An 18-year-old woman presents with questions about her risk of developing


schizophrenia. She is worried because her 23-year-old brother has recently been
diagnosed with schizophrenia and required hospitalization for psychosis. There are no
other family members who have a history of schizophrenia. Having a sibling with
schizophrenia increases her risk of developing schizophrenia by?

A. 2 times
B. 4 times
C. 6 times
D. 8 times
E. 10 times.

5. Which of the following neuroanatomical structures is the critical region for fear
conditioning?

A. Amgydala

!
!
58
B. Basal nucleus of Meynert

C. Mesocortical region

D. Nucleus accumbens

E. Raphe nucleus.

6. The most likely neuroanatomical substrate of Tourette's syndrome is


A. The thalamus.

B. The pituitary.

C. The prefrontal cortex.

D. The basal ganglia.

E. The amygdala.

Psychopathology

7. Agoraphobia!without!panic!disorder!can!be!described!as:!!!
!

A. A!fear!of!enclosed!spaces!!
B. A!fear!of!speaking!in!public!
C. A!fear!of!height!
D. Avoidance!of!authority!figures!
E. Worrying!excessively!about!physical!health.!
!

8. Uncomplicated!bereavement!is!NOT!characterized!by:!!
A. Anger!
B. Depressive!mood!for!one!month!
C. Feelings!of!worthlessness!!
D. Sensations!of!seeing!or!hearing!the!deceased!
E. Transient!guilt.!

9. The common clinical symptoms for pre-menstrual dysphoric disorder includes


all of the following EXCEPT:
a. Bloating
b. Difficulty concentrating
c. Guilt

!
!
59
d. Sleep disturbance
e. Lethargy and fatigue

10. A!40TyearTold!airTconditioner!technician!is!referred!to!you!because!of!change!
in!behaviour!and!deterioration!in!functioning.!When!he!feels the gust of cold
air coming from the air-conditioner, he interprets the cold air as poison gas
from Mars. This phenomenon is BEST described as:!

A. Delusional memory
B. Delusional perception
C. Tactile hallucination
D. Primary delusion
E. Secondary delusion.
!

11. Which of the following behaviour is NOT considered to be purging?

A. Self-induced vomiting
B. Excessive exercise
C. Excessive use of laxative
D. Excessive use of diuretic
E. Excessive use of enema.

General adult psychiatry

12. In!schizophrenia,!which!of!the!following!factors!is!LEAST!Likely!to!restrict!full!
rehabilitation!potential?!
!

A. Florid!delusions!and!hallucinations!!
B. Lack!of!pleasure!from!social!intervention!and!physical!activity!
C. Loss!of!interest!in!rehabilitation!
D. Poor!motivation!
E. Restricted!affect.!
!

13. Somnambulism!(Sleepwalking!disorder)!occurs!in!
!

A. Active!sleep!
B. Earliest!phase!of!sleep!
C. Paradoxical!sleep!
D. Rapid!eye!movement!(REM)!sleep!
E. SlowNwave!sleep.!

!
!
60
!

14. Which!of!the!following!is!a!poor!prognostic!factor!for!schizophrenia?!
!
A. Family!history!of!mood!disorder!
B. Female!gender!
C. Late!onset!
D. Precipitating!factor!
E. Presence!of!neurological!soft!sign.!
!

15. All of the following medical disorders are more common in schizophrenia patients as
compared to the general population EXCEPT?

A. Chronic obstructive pulmonary disease


B. Diabetes mellitus
C. Human immunodeficiency virus infection
D. Metabolic syndrome
E. Rheumatoid arthritis.

16. A!40TyearTold!executive!is!afraid!of!flying!by!aeroplane.!She!has!this!problem!since!young.!!As!
a!result,!she!has!to!quit!her!job!because!it!requires!frequent!travelling.!What!is!the!MOST!
LIKELY!diagnosis?!
!

A.!Agoraphobia!

B.!Generalized!anxiety!disorder!

C.!Panic!disorder!

D.!Social!phobia!

E.!Specific!phobia.!

Old age psychiatry

17. Which!is!the!MOST!preferred!antidepressant!in!elderly?!
A. Amitriptyline!
B. Imipramine!
C. Moclobemide!
D. Sertraline!
E. Venlafaxine.!

!
!
61
!

18. Which!of!the!following!statements!is!TRUE!regarding!risk!factors!for!Alzheimers!
disease? !
!

A. APO!E2!genotype!increases!the!risk!of!Alzheimers!disease.!
B. APO!E4!genotype!increases!the!risk!of!Alzheimers!disease.!
C. APO!E6!genotype!increases!the!risk!of!Alzheimers!disease.!!
D. Male!gender!is!a!risk!factor!!
E. Smoking!is!protective!against!Alzheimers!disease.

19. Which!of!the!following!is!the!MOST!common!type!of!hallucination!in!people!suffering!
from!dementia? !
A. Functional!
B. Gustatory!
C. Olfactory!
D. Tactile!
E. Visual.!!!
!

20. Late-onset psychotic disorder such as schizophrenia or delusional disorder is associated


with:

A. Less paranoia
B. More likely to have sensory deficits
C. More loosening of associations
D. More thought disorders
E. More violent behaviour.

21. Which of the following statements about sleep disorders in old people is FALSE?

A. Excessing napping is a common cause of insomnia.


B. Insomnia is common in old people in Singapore.
C. Insomnia is associated with increased irks of depression.
D. If there is a need for benzodiazepine, diazepam is the preferred sedative for old people.
E. The first line treatment is sleep hygiene.

22. An 80-year-old man presents with terminal insomnia, weight loss, decreased
appetite and pervasive anhedonia. Physical examination, laboratory
investigations and imaging reveal normal findings. The MOST specific symptom
to his likely diagnosis is:

A. Anxiety
B. Decreased concentration
C. Decreased energy
D. Psychomotor changes

!
!
62
E. Suicidal ideation.

Child and adolescent psychiatry

23. Which!of!the!following!childhood!psychiatric!disorders!has!the!EARLIEST!mean!age!
of!onset?!
!

A. Attention!deficit!hyperactivity!disorder!
B. Conduct!disorder!
C. Oppositional!defiant!disorder!
D. PostNtraumatic!stress!disorder!
E. Secondary!encopresis.!!

24. Which!of!the!following!is!MOST!CORRECT!about!Aspergers!disorder?!!!
!

A. They!are!good!at!sport.!
B. They!have!a!normal!IQ.!
C. They!have!impaired!nonNverbal!communication.!
D. They!have!delay!in!speech.!
E. They!have!high!intelligence!in!specific!areas.!

25. A 15-year-old girl presents with fatigue and dehydration. On physical


examination she is noted to have swollen salivary glands and calluses on her
knuckles. She is also hypokalemic. These findings are MOST consistent with
which of the following diagnoses?
A. Anorexia nervosa
B. Bulimia nervosa
C. Chronic fatigue syndrome
D. Obsessive-compulsive disorder
E. Rumination disorder
26. Which!of!the!following!antidepressants!has!the!GREATEST!body!of!evidence!for!
efficacy!in!treating!major!depressive!disorder!in!children!and!adolescents?
A. Bupoprion
B. Fluvoxamine
C. Fluoxetine
D. Paroxetine
E. Sertraline.

Substance abuse

27. A 55-year-old man who has been drinking Chinese wine every night says, I dont
think it is a problem. Which Prochaskas and Diclementes stage of change
BEST describes his current status?
A. Action
B. Contemplation
C. Precontemplation

!
!
63
D. Maintenance
E. Relapse.

28. A!23TyearTold!medical!student!is!brought!to!the!Accident!and!Emergency!
Department!by!his!parents.!He!admits!to!having!recently!used!a!recreational!drug!
during!his!overseas!elective!in!the!United!States.!He!returned!two!days!ago.!He!is!
suspicious!and!sensitive!to!what!others!are!saying.!Physical!examination!reveals!
dilated!pupils!and!electrocardiogram!(ECG)!shows!cardiac!arrhythmias.!Which!drug!
is!MOST!likely!to!be!responsible?!!
!

a. Alcohol!
b. Cannabis!
c. Cocaine!!
d. Mescaline!
e. Heroin.!
!

29. A 24-year-old man presents to the Accident and Emergency Department with
fever, hypotension, agitation, tachycardia and odour of alcohol on his breath. His
friend with him stated that he was prescribed a medication by a psychiatrist 4
weeks ago. The most likely medication responsible is:

A. Diazepam
B. Disulfiram
C. Fluoxetine
D. Haloperidol
E. Moclobemide.

30. A 40-year-old heroin addict reports continued carving. He is on methadone100mg daily.


His methadone dose is increased to 110mg daily. After one week, he has a syncope and
is taken to the Accident and Emergency Department. The most likely finding to explain
his syncope is:
A. Electroencephalogram (EEG) shows evidence of seizure activity.
B. Liver function tests reveal marked transaminitis.
C. Prolonged!QTc!interval
D. Ultrasound Doppler shows deep vein thrombosis.
E. Urine toxicology screen is positive for opioid.

31. A!50TyearTold!with!history!of!major!depressive!disorder!and!ischaemic!heart!
disease.!He!has!not!received!any!psychiatric!treatment.!Which!of!the!following!
medications!is!the!BEST!option!to!help!him!to!quit!smoking?!
!

A. Bupropion!
B. Naltrexone!

!
!
64
C. Nicotine!replacement!treatment!
D. Sertraline!
E. Varenicline.!

Psychopharmacology

32. The WEAKEST evidence for efficacy for Post-traumatic Stress Disorder (PTSD) is for
which class of pharmacological agents?
a. Amitriptyline
b. Diazepam
c. Fluoxetine
d. Fluvoxamine
e. Paroxetine.

33. Regarding!escitalopram,!all!of!the!following!are!true!EXCEPT: !
A. Absorption!is!not!affected!by!food.!
B. It!has!better!tolerability!than!other!SSRIs.!
C. It!shows!linear!pharmacokinetics!at!clinically!relevant!doses.!!
D. It!causes!a!lot!of!drug!interaction.!!
E. Peak!plasma!levels!occur!2N4!hours!after!a!single!dose.!
!

34. The MOST common side effect of selective serotonin reuptake inhibitor (SSRI) is:

A. Acne
B. Erectile dysfunction
C. Gastrointestinal disturbances
D. Metabolic syndrome
E. Tinnitus.

35. Which of the following antipsychotic drugs is MOST LIKELY to be associated with
adverse haematological side effects?

A. Clozapine
B. Olanzapine
C. Quetiapine
D. Risperidone
E. Ziprasidone.

36. Which!of!the!following!is!the!LEAST!common!side!effects!of!acetylcholinesterase!
inhibitors?!!!
!

A. Dizziness!
B. Nausea!
C. Nightmares!

!
!
65
D. Seizures!
E. Tachycardia.!
!

37. Priapism is MOST likely to be associated with which of the following medications?
A. Bupoprion
B. Duloxetine
C. Moclobemide
D. Phenelzine
E. Trazodone.

38. A patient presents with involuntary frowning, blinking grimacing, and


choreoathetoid movements of the upper extremities after several years of
antipsychotic medication treatment. Which one of the following is the MOST
likely diagnosis?

A. Akinesia
B. Akathisia
C. Acute dystonia
D. Parkinsonism
E. Tardive dyskinesia

Psychotherapy

39. At!the!one!year!followTup!of!panic!disorder,!which!of!the!following!treatments!
results!with!the!BEST!outcome!and!the!LEAST!functional!impairment?!!
!

A. Benzodiazepines!
B. BetaNblockers!
C. Brief!dynamic!psychotherapy!
D. Cognitive!behaviour!therapy!$
E. Supportive!psychotherapy.!

40. A 20-year-old with patient suffers from agoraphobia and she has phobia of using
MRT train. Which is the BEST psychological treatment for this patient?

A. No exposure to MRT train and encourage alternative transportation mode

B. Short exposure to MRT train with coping avoidance

C. Short exposure to MRT train without coping avoidance

D. Graded exposure to MRT train with coping avoidance

E. Graded exposure to MRT train without coping avoidance.

!
!
66
41. Defining!highTrisk!situations,!covert!antecedents,!and!stimulus!control!techniques!
are!a!focus!of!which!of!the!following!therapeutic!modalities?
A. Brief!psychodynamic!psychotherapy
B. Cognitive!behaviour!therapy
C. Supportive!therapy
D. Interpersonal!therapy
E. Relapse!prevention!therapy.

42. A!security!guard!attacks!a!government!building,!killing!3!individuals!and!
significantly!injuring!another!10!before!he!takes!his!own!life.!!Psychologists!from!a!
general!hospital!are!dispatched!to!the!scene!to!provide!immediate!psychological!
debriefing!to!civil!servants!working!in!the!building!to!prevent!postTtraumatic!stress!
disorder!(PTSD)!symptoms.!!Using!an!evidenceTbased!framework,!what!is!the!most!
likely!impact!of!this!intervention!on!direct!survivors?
!
A. It!will!be!ineffective!or!harmful!to!the!direct!survivors.
B. It!will!decrease!the!probability!of!developing!PTSD!symptoms.
C. It!will!decrease!the!severity!of!PTSD!symptoms.
D. It!will!decrease!the!time!period!for!recovery!from!PTSD!symptoms.
E. It!will!delay!the!onset!of!PTSD!symptoms.
Ethics and laws

43. When!managing!a!chronically!suicidal!patient,!respecting!the!patients!preference!to!
remain!at!home!and!not!be!admitted!to!the!psychiatric!unit!for!additional!care!
reflects!which!of!the!following!ethical!principles?

A. Autonomy
B. Beneficence
C. NonNmaleficence
D. Egalitarianism
E. Fiduciary!duty.!

Liaison and neuropsychiatry

44. Features of neuroleptic malignant syndrome include all of the following


EXCEPT?
a. Diaphoresis
b. Elevated temperature
c. Extrapyramidal side-effects
d. Labile blood pressure
e. Flaccidity.

45. You are working as a resident in the weight management programme. A 40-year-old
schizophrenia patient is referred to you for weight management issue. Which of the
following medications has the BEST evidence as an adjunct for reducing weight gain

!
!
67
associated with the second generation antipsychotics:
A. Fluoxetine
B. Metformin
C. Sibutramine
D. Simvastatin
E. Topiramate.

46. In medically ill patients who suffer from psychiatric illness, which of the following
pharmacological properties is MOST appropriate?

A. Long half-life psychotropic drug.


B. Psychotropic drugs with active metabolites.
C. Psychotropic drug with wide therapeutic index.
D. Psychotropic drug which inhibits metabolic enzymes.
E. Psychotropic drug which induces metabolic enzymes.

47. A!47TyearTold!woman!is!brought!for!a!psychiatric!evaluation!to!assess!recent!
changes!in!her!behaviour.!!Over!the!past!six!months,!she!has!become!increasingly!
disinhibited!and!impulsive!in!her!behaviour.!!Physical!examination!reveals!mild!
dysarthria,!dysphagia,!and!drooling.!!Slit!lamp!examination!of!the!eyes!indicates!the!
presence!of!Kayser!Fleischer!rings.!!Which!of!the!following!BEST!describes!the!
genetic!basis!for!her!illness?
A. Autosomal!recessive!
B. CoNdominant!
C. Mitochondrial!
D. Polygenetic!
E. XNlinked!dominant.!
!

MCQ!exam!(Paper!3)!

History and mental state exam! !

48. A!40TyearTold!woman!is!brought!by!her! The!answer!is!D.!


husband!brought!to!see!you!as!a!result!of!
abnormal!behaviour.!During!the! !
interview,!she!laughs!and!cries!with!a!
very!short!period!of!time.!In!your!record,! Explanation:!Labile!affect!refers!to!rapid!
her!affect!is!BEST!described!as:! fluctuation!of!affect!(e.g.!from!tearfulness!and!
! laughter!in!a!10!!minute!interview,!seen!in!
patients!with!bipolar!disorder).!
F. Apathy!
G. Blunted!affect! !
H. Euthymic!affect!
I. Labile!affect!

!
!
68
J. Flat!affect.! !

Year:!2013.!

Cognitive assessment! !

49. Which of the following is the presentation The!answer!is!B.!


of cortical dementia?
!
F. A 40-year-old man presents with memory loss
and writhing!movements.!His!father! Explanation:!Option!B!refers!to!frontal!lobe!
suffered!from!the!same!condition.! dementia!which!is!cortical!dementia.!
G. A!50NyearNold!man!presents!with!change!in!
personality,!poor!judgment,!disinhibition! !
and!perservation.!He!made!a!lot!of!mistakes!
in!Lurias!hand!test.! Option!A,!C,!D!and!E!are!considered!to!be!
H. A 60-year-old woman presents with memory
subcortical!dementia.!
loss and depression after a cerebrovascular
accident.!
I. A 70-year-old man presents with memory loss, !
rigidity, slowness!of!movement!and!difficulty!
with!walking!and!gait.! Option!A!=!Hungtingtons disease
J. A 75-year-old woman presents with visual
hallucination. The GP gave her haloperidol Option C = Vascular dementia!
and she develops severe extrapyramidal side
effects.! Option!D!=!Parkinsons!disease!

Option!E!=!Lewy!body!dementia!

Year:!2013.!

This!is!an!advancedNlevel!question.!

Psychiatric epidemiology !

50. After the age of 65, the prevalence of The!answer!is!A.!


dementia increases by how many times
every 5 years? !
F. 2 times
G. 4 times !
H. 6 times
I. 8 times
J. 10 times. Year:!2013!

Psychiatric aetiology, diagnosis and classification !

!
!
69
51. An 18-year-old woman presents with The!answer!is!E!
questions about her risk of developing
schizophrenia. She is worried because her !
23-year-old brother has recently been
diagnosed with schizophrenia and required
Explanation:!The!risk!of!general!population!is!
hospitalization for psychosis. There are no
other family members who have a history 1%.!The!risk!of!a!sibling!is!10%.!Hence,!the!
of schizophrenia. Having a sibling with risk!is!increased!by!10!times.!
schizophrenia increases her risk of
developing schizophrenia by? !

!
F. 2 times
G. 4 times
Year:!2013.!
H. 6 times
I. 8 times
J. 10 times.

52. Which of the following neuroanatomical The!answer!is!A.!


structures is the critical region for fear
conditioning? !

A. Amgydala Explanation:!Option!B!for!Alzheimers!
B. Basal nucleus of Meynert
disease,!Option!C!for!schizophrenia,!Option!D!
for!addiction!and!Option!E!for!depression.!
C. Mesocortical region
!
D. Nucleus accumbens
!
E. Raphe nucleus.
Year:!2013.!

53. The most likely neuroanatomical The answer is D.


substrate of Tourette's syndrome is
A. The thalamus.

B. The pituitary. Explanation: Lesions in basal ganglia can


cause motor tic and compulsive behaviour.
C. The prefrontal cortex.

D. The basal ganglia.

E. The amygdala.
Year: 2013

This is an advanced-level question.

!
!
70
Psychopathology !

54. Agoraphobia!without!panic!disorder!can! Answer!is!A.!


be!described!as:!!!
! !

F. A!fear!of!enclosed!spaces!! Explanation:!A!fear!of!enclosed!space!is!one!of!
G. A!fear!of!speaking!in!public! the!most!common!complaints!among!patients!
H. A!fear!of!height! in!Singapore!(e.g.!crowded!MRT!trains).!
I. Avoidance!of!authority!figures! Option!B!is!social!phobia.!
J. Worrying!excessively!about!physical!health.!
! !

Year:!2013.!

55. Uncomplicated!bereavement!is!NOT! The!answer!is!C.!


characterized!by:!!
F. Anger! !
G. Depressive!mood!for!one!month!
H. Feelings!of!worthlessness!! Indicators!that!bereavement!may!have!
I. Sensations!of!seeing!or!hearing!the! progressed!into!depression!include:!
deceased!
J. Transient!guilt.! 2. symptoms!still!present!two!months!
after!the!loss!
3. guilt!about!things!other!than!actions!
taken!or!not!taken!by!the!survivor!at!
the!time!of!death!
4. thoughts!of!death!other!than!the!
survivor!feeling!that!he!or!she!would!
be!better!off!dead!or!should!have!died!
with!the!deceased!person!
5. morbid!preoccupation!with!
worthlessness!
6. marked!psychomotor!retardation!
7. prolonged!and!marked!functional!
impairment!
8. hallucinatory!experiences!other!than!
thinking!that!he!or!she!hears!the!voice!
of,!or!transiently!sees!the!image!of,!
the!deceased!person!
!

56. The common clinical symptoms for The answer is C.


pre-menstrual dysphoric disorder
includes all of the following EXCEPT:
f. Bloating
g. Difficulty concentrating Explanation: Guilt is not considered to be a
h. Guilt common clinical symptom of premenstrual

!
!
71
i. Sleep disturbance dysmorphic disorder.
j. Lethargy and fatigue

Year: 2013

57. A!40TyearTold!airTconditioner! The!answer!is!B.!


technician!is!referred!to!you!because!
of!change!in!behaviour!and! !
deterioration!in!functioning.!When!
Explanation:!As!this!person!works!as!an!airN
he!feels the gust of cold air coming
conditioner!technician,!he!is!used!to!feel!gust!
from the air-conditioner, he
interprets the cold air as poison gas of!cold!air!from!air!conditioner.!This!is!a!
from Mars. This phenomenon is BEST normal!perception.!He!adds!a!delusional!
described as:! interpretation!and!claims!that!the!cold!air!is!a!
poison!gas!from!another!planet.!This!is!
illogical!and!impossible.!As!a!result,!there!is!
A. Delusional memory delusional!interpretation!of!a!real!perception.!
B. Delusional perception
This!phenomenon!is!known!as!delusional!
C. Tactile hallucination
perception.!
D. Primary delusion
E. Secondary delusion.
!
!

Year:!2013!

58. Which of the following behaviour is The answer is B.


NOT considered to be purging?

F. Self-induced vomiting Explanation: In medicine, purging is defined


G. Excessive exercise as to cause evacuation of the
H. Excessive use of laxative gastrointestinal or urinary system. Option B
I. Excessive use of diuretic is not related to the gastrointestinal or
J. Excessive use of enema.
urinary system.

Year:!2013!

!
!
72
General adult psychiatry !

59. In!schizophrenia,!which!of!the!following! The!answer!is!A.!


factors!is!LEAST!Likely!to!restrict!full!
rehabilitation!potential?! !
!
Explanation:!Option!A!is!positive!symptom!of!
F. Florid!delusions!and!hallucinations!! schizophrenia!and!is!least!likely!to!restrict!full!
G. Lack!of!pleasure!from!social!intervention! rehabilitation!potential!as!compared!to!
and!physical!activity! negative!symptoms.!
H. Loss!of!interest!in!rehabilitation!
I. Poor!motivation! !
J. Restricted!affect.!
! !!

Year:!2013!

60. Somnambulism!(Sleepwalking!disorder)! The!answer!is!E.!


occurs!in!
! !

F. Active!sleep! Explanation:!Parasomnia!is!divided!into!slowN
G. Earliest!phase!of!sleep! wave!sleep!disorders!(e.g.!sleep!terrors,!
H. Paradoxical!sleep! sleepwalking!disorder)!and!rapid!eye!
I. Rapid!eye!movement!(REM)!sleep! movement!sleep!disorders!(e.g.!nightmare!
J. SlowNwave!sleep.!
disorder,!REM!behaviour!disorder!such!as!
!
kicking!in!the!middle!of!the!night).!!
!
!

Somnabulism!or!sleepwalking!disorder!is!
associated!with!slow!wave!sleep.!Because!
sleepwalking!disorder!arises!from!SWS!!

sleep,!the!patient!is!difficult!to!be!!

awaken!during!sleepwalking.!

Year:!2013.!

61. Which!of!the!following!is!a!poor! The!answer!is!E.!


prognostic!factor!for!schizophrenia?!

!
!
73
! !

F. Family!history!of!mood!disorder! Explanation:!Option!E!suggests!that!the!aetiology!of!
G. Female!gender! schizophrenia!is!biological!and!resulted!from!a!neurological!
H. Late!onset! lesion.!Hence,!the!prognosis!poor.!Good!prognostic!and!poor!
I. Precipitating!factor! prognostic!factors!are!summarized!as!follows:!

J. Presence!of!neurological!soft!sign.! !
!
Good!Prognosis! ! Poor!Prognosis!
! Late!onset! ! ! Young!onset!

! Obvious!precipitating!factors!No!precipitating!factors!

Acute!onset! ! Insidious!onset!

Good!premorbid!function!! Poor!premorbid!function!

Married! ! !!!!!!!!!!!!!!!!!!!!Single,!divorced,!or!widowed!

Family!history!of!mood!disorders!Family!history!of!
schizophrenia! !

Good!support!systems! Poor!support!systems!

Positive!symptoms! ! Negative!symptoms!

!!

Year:!2013.! ! ! ! !

62. All of the following medical disorders are The!answer!is!E.!


more common in schizophrenia patients as
compared to the general population !
EXCEPT?
Explanation:!The!risk!for!schizophrenia!
F. Chronic obstructive pulmonary disease patients!developing!RA!is!oneNthird!the!risk!of!
G. Diabetes mellitus general!population.!The!reason!is!due!to!
H. Human immunodeficiency virus infection reduction!in!immune!responses!in!
I. Metabolic syndrome
J. Rheumatoid arthritis. schizophrenia!patients.!

Year:!2013.!

63. A!40TyearTold!executive!is!afraid!of!flying!by! The!answer!is!E.!


aeroplane.!She!has!this!problem!since!young.!!
As!a!result,!she!has!to!quit!her!job!because!it! !
requires!frequent!travelling.!What!is!the!
MOST!LIKELY!diagnosis?!

!
!
74
! Explanation:!Fear!of!shadow!is!specific!phobia.!

A.!Agoraphobia! !

B.!Generalized!anxiety!disorder! !

C.!Panic!disorder! Year:!2013.!

D.!Social!phobia!

E.!Specific!phobia.!

Old age psychiatry !

64. Which!is!the!MOST!preferred! The!answer!is!D.!


antidepressant!in!elderly?!
F. Amitriptyline! !
G. Imipramine!
H. Moclobemide! Explanation:!Sertraline,!a!SSRI,!is!the!most!
I. Sertraline! preferred!antidepressants!as!compared!to!
J. Venlafaxine.! imipramine!and!amitriptyline!which!are!TCA.!
! Venlafaxine!is!associated!with!doseNrelated!
hypertension.!

Year:!2013!

This!is!an!advancedNlevel!question.!

65. Which!of!the!following!statements!is! The!answer!is!B.!


TRUE!regarding!risk!factors!for!
Alzheimers!disease? ! !
!
Explanation:!Studies!have!suggested!that!
F. APO!E2!genotype!increases!the!risk!of! APOE4!genotype!increases!risk!of!Alzheimers!
Alzheimers!disease.! disease,!and!especially!in!women.!APOE2!
G. APO!E4!genotype!increases!the!risk!of! genotype!is!protective.!Female!gender!is!a!
Alzheimers!disease.!
risk!factor!and!smoking!is!not!protective.!
H. APO!E6!genotype!increases!the!risk!of!
Alzheimers!disease.!!
!
I. Male!gender!is!a!risk!factor!!
J. Smoking!is!protective!against!Alzheimers!

!
!
75
disease. !

Year:!2013!

66. Which!of!the!following!is!the!MOST! The!answer!is!E.!


common!type!of!hallucination!in!people!
suffering!from!dementia? ! !
F. Functional!
G. Gustatory! Explanation:!Visual!hallucinations!are!the!
H. Olfactory! most!common!type!of!hallucination!in!
I. Tactile! dementia.!It!is!because!one!type!of!dementia,!
J. Visual.!!! lewy!body!dementia!is!associated!with!visual!
!
hallucinations.!

Year:!2013!

67. Late-onset psychotic disorder such as The!answer!is!B.!


schizophrenia or delusional disorder is
associated with: !

Explanation:!LateNonset!psychotic!disorder!is!
F. Less paranoia
G. More likely to have sensory deficits more!likely!to!be!associated!with!sensory!
H. More loosening of associations deficit.!
I. More thought disorders
J. More violent behaviour. !

Year:!2013.!

68. Which of the following statements about The!answer!is!D.!


sleep disorders in old people is FALSE?
!
F. Excessing napping is a common cause of Explanation:!Diazepam!is!longNacting!and!
insomnia.
G. Insomnia is common in old people in cause!oversedation!or!hangNover!effect.!This!
Singapore. will!increase!fall!risk!among!the!elderly.!
H. Insomnia is associated with increased irks of
depression. !
I. If there is a need for benzodiazepine,
diazepam is the preferred sedative for old !
people.
J. The first line treatment is sleep hygiene.
Year:!2013!

69. An 80-year-old man presents with The answer is E.


terminal insomnia, weight loss,

!
!
76
decreased appetite and pervasive
anhedonia. Physical examination,
laboratory investigations and imaging Explanation: This elderly man suffers from
reveal normal findings. The MOST depressive disorder based on the history
specific symptom to his likely provided. Hence, suicidal ideation is the
diagnosis is: most specific symptom.

F. Anxiety
G. Decreased concentration
H. Decreased energy
I. Psychomotor changes Year: 2013.
J. Suicidal ideation.

Child and adolescent psychiatry !

70. Which!of!the!following!childhood! The!answer!is!E.!


psychiatric!disorders!has!the!EARLIEST!
mean!age!of!onset?! !
!
Explanation:!Encopresis:!4!years!by!
F. Attention!deficit!hyperactivity!disorder! definition.!ADHD:!12!years,!CD:!10!years,!
G. Conduct!disorder! ODD:!8!years,!PTSD:!can!be!childhood!or!
H. Oppositional!defiant!disorder! adult.!
I. PostNtraumatic!stress!disorder!
J. Secondary!encopresis.!! !

Year:!2013!

71. Which!of!the!following!is!MOST! The!answer!is!C.!


CORRECT!about!Aspergers!disorder?!!!
! !

F. They!are!good!at!sport.! Explanation:!They!have!better!verbal!
G. They!have!a!normal!IQ.! communications!as!compared!to!autism!and!
H. They!have!impaired!nonNverbal! no!delay!in!speech.!They!may!have!lower!IQ.!
communication.! Some!have!high!intelligence!in!specific!areas!
I. They!have!delay!in!speech.!
but!not!all.!
J. They!have!high!intelligence!in!specific!
areas.!
!

Year:!2013.!

!
!
77
72. A 15-year-old girl presents with fatigue The!answer!is!B.!
and dehydration. On physical
examination she is noted to have !
swollen salivary glands and calluses
on her knuckles. She is also Explanation:!This!patient!suffers!from!bulimia!
hypokalemic. These findings are nervosa.!She!has!selfNinduced!vomiting!and!
MOST consistent with which of the the!gastric!acid!causes!inflammation!in!the!
following diagnoses?
salivary!glands.!She!uses!her!fingers!to!induce!
F. Anorexia nervosa
G. Bulimia nervosa vomiting!and!leave!calluses!on!her!knuckles,!
H. Chronic fatigue syndrome which!is!known!as!Russells!sign.!Repeated!
I. Obsessive-compulsive disorder selfNinduced!vomiting!is!associated!with!
J. Rumination disorder hypokaelemia.!

Year:!2013.!

73. Which!of!the!following!antidepressants! The!answer!C!


has!the!GREATEST!body!of!evidence!for!
efficacy!in!treating!major!depressive! !
disorder!in!children!and!adolescents?
F. Bupoprion Explanation:!Fluoxetine!and!escitalopram!
G. Fluvoxamine have!the!greatest!body!of!evidence!for!
H. Fluoxetine efficacy!in!treating!major!depressive!disorder!
I. Paroxetine in!children!and!adolescents.!
J. Sertraline.
!

Year:!2013!

Substance abuse !

74. A 55-year-old man who has been The answer is C.


drinking Chinese wine every night
says, I dont think it is a problem.
Which Prochaskas and Diclementes
stage of change BEST describes his Explanation: He is unaware that his alcohol
current status? dependence is a problem.
F. Action
G. Contemplation
H. Precontemplation
I. Maintenance
J. Relapse. Year: 2013.

!
!
78
75. A!23TyearTold!medical!student!is!brought! The!answer!is!C.!
to!the!Accident!and!Emergency!
Department!by!his!parents.!He!admits!to! !
having!recently!used!a!recreational!drug!
during!his!overseas!elective!in!the! Explanation:!Dilated!pupil,!arrhythmia,!
United!States.!He!returned!two!days!ago.! nausea!and!suspiciousness!are!common!signs!
He!is!suspicious!and!sensitive!to!what! associated!with!cocaine!intoxication.!
others!are!saying.!Physical!examination!
reveals!dilated!pupils!and! !
electrocardiogram!(ECG)!shows!cardiac!
arrhythmias.!Which!drug!is!MOST!likely! !
to!be!responsible?!!
! Year:!2013.!

f. Alcohol! !
g. Cannabis!
h. Cocaine!! This!is!an!advancedNlevel!question.!
i. Mescaline!
j. Heroin.!
!

76. A 24-year-old man presents to the The!answer!is!B.!


Accident and Emergency Department
with fever, hypotension, agitation, !
tachycardia and odour of alcohol on
his breath. His friend with him stated Explanation:!Disulfiram!blocks!this!reaction!
that he was prescribed a medication by at!the!acetaldehyde!dehydrogenase.!After!
a psychiatrist 4 weeks ago. The most alcohol!intake!under!the!influence!of!
likely medication responsible is:
disulfiram,!the!concentration!of!acetaldehyde!
in!the!blood!may!be!five!to!10!times!higher!
F. Diazepam than!that!found!during!metabolism!of!the!
G. Disulfiram same!amount!of!alcohol!alone.!As!
H. Fluoxetine
I. Haloperidol acetaldehyde!is!one!of!the!major!causes!of!the!
J. Moclobemide. symptoms!experienced!by!this!man.!

!
Year:!2013.!

77. A 40-year-old heroin addict reports The!answer!is!C.!


continued carving. He is on
methadone100mg daily. His methadone !
dose is increased to 110mg daily. After one
week, he has a syncope and is taken to the Explanation:!Methodone!is!associated!with!
Accident and Emergency Department. The
most likely finding to explain his syncope prolonged!QTc.!This!occurs!after!an!increase!
is: in!the!dose!of!methadone.!This!may!lead!to!

!
!
79
F. Electroencephalogram (EEG) shows evidence cardiac!arrhythmia!and!results!in!syncope.!
of seizure activity.
G. Liver function tests reveal marked !
transaminitis.
H. Prolonged!QTc!interval !
I. Ultrasound Doppler shows deep vein
thrombosis. Year:!2013!
J. Urine toxicology screen is positive for opioid.
!

This!is!an!advanced!level!question.!

78. A!50TyearTold!with!history!of!major! The!answer!is!A.!


depressive!disorder!and!ischaemic!heart!
disease.!He!has!not!received!any! !
psychiatric!treatment.!Which!of!the!
following!medications!is!the!BEST!option! Explanation:!Bupoprion!is!the!best!option!in!
to!help!him!to!quit!smoking?! this!case!as!it!can!overcome!depression!and!
! smoking.!Option!C!and!E!can!tackle!smoking!
but!not!depression.!
F. Bupropion!
G. Naltrexone! !
H. Nicotine!replacement!treatment!
I. Sertraline! !
J. Varenicline.!
Year:!2013.!

Psychopharmacology !

79. The WEAKEST evidence for efficacy for The!answer!is!B.!


Post-traumatic Stress Disorder (PTSD) is
for which class of pharmacological !
agents?
f. Amitriptyline Explanation:!Diazepam,!a!long!term!
g. Diazepam
h. Fluoxetine benzodiazepine!may!cause!dependence!and!
i. Fluvoxamine does!not!help!to!relieve!anxiety!symptoms!in!
j. Paroxetine. long!run!as!it!may!cause!withdrawal.!

Year:!2013.!

80. Regarding!escitalopram,!all!of!the! The!answer!is!D.!


following!are!true!EXCEPT: !
F. Absorption!is!not!affected!by!food.! !
G. It!has!better!tolerability!than!other!SSRIs.!

!
!
80
H. It!shows!linear!pharmacokinetics!at! Explanation:!Escitalopram!causes!the!least!
clinically!relevant!doses.!! drug!interaction!among!all!SSRIs.!Sertralines!!
I. It!causes!a!lot!of!drug!interaction.!! absorption!is!significantly!affected!by!food.!
J. Peak!plasma!levels!occur!2N4!hours!after!a!
Escitalopram!is!better!tolerated!than!other!
single!dose.!
! SSRIs.!Fluoxetine!and!paroxetine!are!the!only!
SSRIs!that!do!not!show!linear!
pharmacokinetics!at!clinically!relevant!doses.!

Year:!2013!

81. The MOST common side effect of selective The!answer!is!C.!


serotonin reuptake inhibitor (SSRI) is:
!
F. Acne Explanation:!GI!disturbances!such!as!nausea!
G. Erectile dysfunction
H. Gastrointestinal disturbances and!vomiting!are!the!most!common!side!
I. Metabolic syndrome effects.!
J. Tinnitus.
!

Year:!2013.!

82. Which of the following antipsychotic drugs The!answer!is!A.!


is MOST LIKELY to be associated with
adverse haematological side effects? !
Explanation:!Clozapine!is!the!second!
F. Clozapine generation!antipsychotics!which!is!most!
G. Olanzapine likely!to!be!associated!with!agranulocytosis.!
H. Quetiapine
I. Risperidone !
J. Ziprasidone.
!

Year:!2013!

83. Which!of!the!following!is!the!LEAST! The!answer!is!E.!


common!side!effects!of!
acetylcholinesterase!inhibitors?!!! !
!
Explanation:!!The!AChEI!causes!bradycardia!
F. Dizziness! as!a!result!of!increased!vagal!tone.!
G. Nausea!
H. Nightmares! !

!
!
81
I. Seizures! !
J. Tachycardia.!
! Year:!2013.!

This!is!an!advancedNlevel!question!

84. Priapism is MOST likely to be associated with The!answer!is!E.!


which of the following medications?
F. Bupoprion !
G. Duloxetine
H. Moclobemide
Explanation:!Trazodone!is!a!serotonin!
I. Phenelzine
J. Trazodone. antagonist!and!reuptake!inhibitor.!It!is!good!
for!depression!and!insomnia.!It!is!available!in!
SGH!pharmacy!but!not!NUH!pharmacy.!It!is!
associated!with!priapism!in!men.!!

Year:!2013!

This!is!an!advancedNlevel!question.!

85. A patient presents with involuntary The!answer!is!E.!


frowning, blinking grimacing, and
choreoathetoid movements of the !
upper extremities after several years of Explanation:!This!patient!develops!
antipsychotic medication treatment. involuntary!movements!after!taking!
Which one of the following is the MOST
medication!for!several!years.!The!best!
likely diagnosis?
description!is!tardive!dyskinesia.!
F. Akinesia
G. Akathisia !
H. Acute dystonia
I. Parkinsonism !
J. Tardive dyskinesia
Year:!2013.!

Psychotherapy !

86. At!the!one!year!followTup!of!panic! The!answer!is!D.!


disorder,!which!of!the!following!
treatments!results!with!the!BEST! !
outcome!and!the!LEAST!functional!

!
!
82
impairment?!! Explanation:!CBT!has!better!outcome!than!
! brief!dynamic!psychotherapy!and!supportive!
psychotherapy.!Benzodiazepines!and!betaN
F. Benzodiazepines!
blockers!only!provide!symptomatic!relief.!
G. BetaNblockers!
H. Brief!dynamic!psychotherapy!
!
I. Cognitive!behaviour!therapy!$
J. Supportive!psychotherapy.! !

Year:!2013.!

87. A 20-year-old with patient suffers from The answer is E.


agoraphobia and she has phobia of
using MRT train. Which is the BEST
psychological treatment for this
patient? Explanation: The main principle of treatment
is to not have any form of avoidance but to
extinguish anxiety in response to the feared
A. No exposure to MRT train and encourage stimulus, i.e. the MRT train.
alternative transportation mode

B. Short exposure to MRT train with coping


avoidance

C. Short exposure to MRT train without coping Year: 2013.


avoidance

D. Graded exposure to MRT train with coping


avoidance

E. Graded exposure to MRT train without coping


avoidance.

88. Defining!highTrisk!situations,!covert! The!answer!is!B!


antecedents,!and!stimulus!control!
techniques!are!a!focus!of!which!of!the! !
following!therapeutic!modalities?
F. Brief!psychodynamic!psychotherapy Explanation:!Defining!high!risk!situations,!
G. Cognitive!behaviour!therapy covert!antecedents!and!stimulus!control!
H. Supportive!therapy techniques!are!part!of!the!dysfunctional!
I. Interpersonal!therapy thought!diary!used!in!CBT.!
J. Relapse!prevention!therapy.
!

Year:!2013.!

!
!
83
!

89. A!security!guard!attacks!a!government! The!answer!A.!


building,!killing!3!individuals!and!
significantly!injuring!another!10!before! !
he!takes!his!own!life.!!Psychologists!from!
a!general!hospital!are!dispatched!to!the! Explanation:!Debriefing!is!shown!to!be!
scene!to!provide!immediate! harmful!to!individuals!who!are!the!witness!of!
psychological!debriefing!to!civil!servants! a!traumatic!event.!The!individuals!should!be!
working!in!the!building!to!prevent!postT given!access!to!psychological!help!and!they!
traumatic!stress!disorder!(PTSD)!
can!consult!a!psychologist!on!a!voluntary!
symptoms.!!Using!an!evidenceTbased!
framework,!what!is!the!most!likely! basis.!!
impact!of!this!intervention!on!direct!
!
survivors?
!
!
F. It!will!be!ineffective!or!harmful!to!the!
direct!survivors. Year:!2013!
G. It!will!decrease!the!probability!of!
developing!PTSD!symptoms. !
H. It!will!decrease!the!severity!of!PTSD!
symptoms. This!is!an!advancedNlevel!question.!!
I. It!will!decrease!the!time!period!for!
recovery!from!PTSD!symptoms.
J. It!will!delay!the!onset!of!PTSD!
symptoms.
Ethics and laws !

90. When!managing!a!chronically!suicidal! The!answer!is!A.!


patient,!respecting!the!patients!
preference!to!remain!at!home!and!not!be! !
admitted!to!the!psychiatric!unit!for!
additional!care!reflects!which!of!the! Explanation:!As!this!!question!emphasizes!on!
following!ethical!principles? respecting!the!patients!preference,!the!best!
answer!is!respecting!patients!autonomy.!
F. Autonomy !
G. Beneficence
H. NonNmaleficence !
I. Egalitarianism
J. Fiduciary!duty.! Year:!2013.!

Liaison and neuropsychiatry !

91. Features of neuroleptic malignant The answer is E.


syndrome include all of the following
EXCEPT?
f. Diaphoresis
g. Elevated temperature Explanation: It should be rigidity and not

!
!
84
h. Extrapyramidal side-effects flaccidity.
i. Labile blood pressure
j. Flaccidity.

Year: 2013.

92. You are working as a resident in the weight The answer is B.


management programme. A 40-year-old
schizophrenia patient is referred to you for
weight management issue. Which of the Explanation: Meformin has the best evidence in
following medications has the BEST reducing weight gain associated with the second
evidence as an adjunct for reducing weight
generation antipsychotics.
gain associated with the second
generation antipsychotics:
F. Fluoxetine
G. Metformin
H. Sibutramine
I. Simvastatin
Year 2013
J. Topiramate.

This is an advanced-level question.

93. In medically ill patients who suffer from The answer is C.


psychiatric illness, which of the following
pharmacological properties is MOST
appropriate?
Explanation: Narrow therapeutic index like
F. Long half-life psychotropic drug. lithium is dangerous to medically ill patients.
G. Psychotropic drugs with active metabolites.
H. Psychotropic drug with wide therapeutic index.
I. Psychotropic drug which inhibits metabolic
enzymes.
J. Psychotropic drug which induces metabolic
enzymes. Year 2013

94. A!47TyearTold!woman!is!brought!for!a! The!answer!is!A.!


psychiatric!evaluation!to!assess!recent!
changes!in!her!behaviour.!!Over!the!past! !
six!months,!she!has!become!increasingly!
disinhibited!and!impulsive!in!her! Explanation:!This!patient!suffers!from!
behaviour.!!Physical!examination! Wilsons!disease!which!is!autosomal!
reveals!mild!dysarthria,!dysphagia,!and! recessive.!
drooling.!!Slit!lamp!examination!of!the!
eyes!indicates!the!presence!of!Kayser! !
Fleischer!rings.!!Which!of!the!following!
BEST!describes!the!genetic!basis!for!her! !
illness?

!
!
85
F. Autosomal!recessive! Year:!2013.!
G. CoNdominant!
H. Mitochondrial! !
I. Polygenetic!
J. XNlinked!dominant.! This!is!an!advancedNlevel!question.!

MCQ exam (Paper 4): Questions

History and mental state exam

1. You are a medical resident. The nurse informs you that an 80-year-old man who
was admitted for cellulitis seems to be depressed and mentions about passive

!
!
86
suicidal thought. When you assess him, he sees ghost in the ward and expresses
paranoid ideation against nursing staff. Which of the following assessment is
MOST important to establish the diagnosis?

A. Assess orientation
B. Assess judgement
C. Assess recognition
D. Assess registration and short-term recall
E. Assess somatic symptoms.

Cognitive assessment

2. Which of the following cognitive tasks is NOT an assessment of short term or


long term memory?

A. Ask the patient to name the current Prime Minister of Singapore.


B. Ask the patient to name as many animals as possible that can be found in the
Singapore Zoo.
C. Ask the patient to tell you his or her address and later, you check the answer with
patients medical record.
D. Inform the patient 3 objects (e.g. Apple, Newspaper and Train) and ask the patient to
name the 3 objects immediately.
E. Inform the patient 3 objects (e.g. Apple, Newspaper and Train) and ask the patient to
name the 3 objects after 5 minutes.

Psychiatric, epidemiology, etiology, diagnosis and classification

3. A 20-year-old man with an identical twin is diagnosed with major depressive


disorder. His twin brother asks you to comment on the chance he will develop
depressive disorder. Which of the following is the CORRECT response to his twin
brother?
A. Major depressive disorder is a strongly inheritable disease. You will definitely
develop it as well.
B. The likelihood of you developing depressive disorder is higher than other siblings.
C. The likelihood of you developing depressive disorder is same as non-identical twins.
D. There is only a slightly increased risk that you will become depressed as compared to
the general population.
E. You do not need to worry because environmental factor is an important factor to
determine whether you will develop depressive disorder.

4. A 35-year-old woman suffers from low mood, low energy, poor sleep, poor
appetite and recurrent suicidal thoughts. She firmly believes that she deserves
the death sentence for minor mistakes made in the past. Which of the following
statements is FALSE?

!
!
87
A. For severe depressive episode with psychotic features, psychotic symptoms usually
occur after manifestation of depressive symptoms.
B. Her psychotic symptoms are considered to be mood congruent.
C. Based on the case scenario, there is enough evidence to suggest that she suffers
from schizoaffective disorder.
D. The prescription of antipsychotic drug should be considered.
E. Electroconvulsive therapy (ECT) is a recognised treatment for his condition.

5. Which of the following disorders has been shown to have the GREATEST degree
of heritability?
A. Alcoholism
B. Attention-deficit/hyperactivity disorder (ADHD)
C. Autism
D. Major depressive disorder
E. Schizophrenia.

6. Which of the following is the MOST common cause of insomnia among


psychiatric patients in Singapore?

A. Sleep apnea
B. Shift work
C. Stimulant use
D. Underlying psychiatric illness
E. Use of sleeping pills.

7. Regarding the age of onset in schizophrenia, which of the following is CORRECT?

A. European women have earlier age of onset than Asian women.


B. Current research findings remain inconclusive about the age of onset in
schizophrenia.
C. There is no difference in the age of onset between men and women.
D. Men usually have earlier age of onset as compared to women.
E. Women usually have earlier age of onset as compared to men.

Psychopathology

8. Agoraphobia without panic attack is BEST referred to:

A. Concern about physical appearance


B. Constant worried about physical health
C. Fear of blood and needle
D. Fear of collapse in an enclosed space
E. Fear of authority figures.

!
!
88
9. Which of the following is MOST suggestive of the diagnosis of schizophrenia?
A. Acute onset of psychosis
B. Apathy
C. Cognitive impairment
D. Hearing own thoughts
E. Persistent deterioration of personality.

General adult psychiatry

10. Peter is 27-year-old and lives with his parents. He has been employed as a
delivery man for most of the time since leaving school, but has recently left his
job as a salesman. He has never taken any illicit drugs. His parents state that in
the last three weeks, he has been extremely active, requiring less sleep and not
appearing tired, being over-talkative and disinhibited and on occasions quite
irritable. He claimed to have invented a machine for curing cancer and wished to
go to the U.S. to sell it. When stopped by his parents, he became violent, and they
called the police. Which of the following diagnosis is MOST appropriate for this
patient?

A. Bipolar I disorder with manic features


B. Bipolar II disorder with hypomanic features
C. Cyclothymia
D. Mixed affective disorder
E. Schizoaffective disorder.

11. Which of the following personality disorders is LEAST likely to be considered as a


differential diagnosis to schizophrenia?

A. Borderline personality disorder


B. Obsessive compulsive personality disorder
C. Paranoid personality disorder
D. Schizotypal personality disorder
E. Schizoid personality disorder.

12. Which of the following is LEAST likely to be found in research in the relationship
between depressive disorder and hormones?

A. Blunted adrenocorticotropic hormone (ACTH) response to the corticotropin-releasing


hormone (CRH)
B. Blunted thyroid stimulating hormone (TSH) response to thyrotropin-releasing hormone
(TRH)
C. Cortisol suppression with dexamethasone

!
!
89
D. There is an increased cortisol.
E. There is an increased adrenal sensitivity to ACTH.

13. Which of the following disorders is LEAST likely to be considered as a differential


diagnosis of a 35-year-old driver develops nightmare, flashback and
hypervigilance after a road traffic accident?
A. Factitious disorder
B. Generalized anxiety disorder
C. Major depressive disorder
D. Obsessive compulsive disorder
E. Phobic disorder.

14. Rapid eye movement (REM) sleep is associated with all of the following EXCEPT:

A. Increased parasympathetic activity


B. Increased cerebral blood flow
C. Increased complexity of dreams
D. Maximal loss of muscle tone
E. Transient runs of conjugate eye movements.

15. A 17-year-old male comes to see you because he is experiencing tremendous


stress in his new job. He has finished his O level examination and waiting to
start the National Service in 6 months. He got an offer to work in a shoe shop
which specializes in selling female shoes. He realizes that he feels sexually
aroused by female shoes. He spends a lot of time fantasizing about female shoes.
He stole multiple pairs of shoes from the shop and used them for masturbation.
He used to be a normal person. He was a hardworking student whose main
interests were chemistry and physics. What is the MOST likely diagnosis?

A. Adjustment disorder
B. Fetishism
C. Kleptomania
D. Sadomasochism
E. Voyeurism.

16. Which of the following psychiatric disorders is MOST common among patients
with bulimia nervosa?

A. Alcohol use disorder


B. Major depressive disorder
C. Obsessive compulsive disorder
D. Panic disorder
E. Intermittent explosive disorder.

Old age psychiatry

!
!
90
17. Manic episodes in old people are associated with:

A. Less euphoria
B. Less mixed presentation with depression
C. Less paranoid delusions
D. More hyperactivity
E. More flight of ideas.

18. All of the following statements are true regarding post-operative delirium
EXCEPT:

A. Delirium cannot be diagnosed if all laboratory results are normal.


B. It can occur in an 80-year-old man after gastrectomy.
C. It can occur in a 16-year-old girl after renal transplant.
D. Haloperidol is one of the treatments of choice.
E. Physical restraint should be avoided if possible.

19. A 70-year-old man with a history of drinking Chinese wine, hepatitis, chronic renal
failure and hypertension was brought by his wife to the hospital for treatment of
an acute cellulitis. He was noted to have tachycardia in the Accident and
Emergency Department. You are the on-call medical resident and this man was
admitted to the medical ward. What is the MOST appropriate first approach?

A. Assess short term recall to rule out dementia on delirium.


B. Obtain history from collaterals whether he has in fact been drinking Chinese wine
recently.
C. Put him on propranolol 10mg three times per day to control tachycardia.
D. Start diazepam 5mg three times per day to prevent delirium tremens.
E. Start lorazepam 1mg three times per day to prevent delirium tremens.

20. A 70-year-old woman with severe depressive symptoms and strong suicidal
ideation which is not responded to amitriptyline 150mg daily and fluvoxamine
200mg daily. Each medication was tried for 6 months with good adherence. She
has good past health. Her children consult you for further management. The
MOST appropriate treatment which you would recommend is:

A. Cognitive behaviour therapy

!
!
91
B. Electroconvulsive therapy

C. Donepezil

D. Olanzapine

E. Repetitive transcranial magnetic stimulation.

Child and adolescent psychiatry and intellectual disability

21. You are helping a mother to develop behavioural program to deal with her son
who has oppositional defiant disorder. Which of the following is the MOST
significant component of this program?

A. Extinction
B. Positive reinforcement
C. Punishment
D. School suspension
E. Strict parenting.

22. Which of the following factors are LEAST likely to be the aetiological factors in
attention deficit and hyperactivity disorder (ADHD)?

A. Antenatal exposure to alcohol


B. Antenatal exposure to nicotine
C. Delivery complications
D. High birth weight
E. Traumatic brain injury during infancy and early childhood.
23. You are a general practitioner. A 40-year-old woman consults you as a result of
high!blood!pressure.!General!inspection!shows!ptosis,!a!broad!neck!and!indistinct!
hairline.!Her!chest!appears!to!be!broad.!Auscultation!reveals!cardiac!murmurs.!She!is!
single!and!stays!with!her!parents.!She!completed!her!education!in!the!Institute!of!
Technological!Education!(ITE)!and!works!as!a!store!room!officer.!If!chromosome!
analysis!is!performed,!the!MOST!likely!finding!is:
!
A. XO syndrome
B. XXY syndrome
C. XXXY syndrome
D. XYY syndrome
E. YO syndrome.

24. You are a paediatric resident. A 7-year-old boy is suspected to suffer from autism.
You referred this boy to see an education psychologist for assessment. The

!
!
92
education psychologist sends a report to you and you need to explain the
findings to her mother. His strength is MOST likely found in which of the following
areas?

A. Abstract thinking
B. Block design
C. Explain similarities
D. Oral presentation skills
E. Verbal concept formation

Substance abuse

25. Which of the following neuroanatomical structures is MOST implicated in


substance misuse and dependence?

A. Amygdala
B. Basal nucleus of Meynert
C. Nucleus accumbens
D. Hippocampus
E. Raphe nucleus.

26. A 50-year-old man is admitted to the hepatobillary ward as a result of pancreatitis.


You are the resident on call. His wife tells you that she has seen him intoxicated
with alcohol in the past few days. The patient denies. Which of the following is the
BEST method available in a general hospital in Singapore to confirm that this
patient has been drinking recently?

A. Carbohydrate-deficient transferrin (CDT)


B. Gamma-glutamyl transferase (GGT)
C. Mean corpuscular volume (MCV)
D. Serum alcohol level
E. Serum amylase.

27. A common and safe pharmacological treatment for reducing relapse in alcohol
dependence has direct actions upon which receptors?

A. Benzodiazapine
B. Cannabinoid
C. Dopamine
D. -Aminobutyric acid (GABA)
E. Opioid.

!
!
93
28. Which of the following is LEAST consistent with the objective of motivational
interviewing?

A. Allow the patient give their inputs without interruption.


B. Establishing a collaborative patient-therapist relationship.
C. Identifying appropriate reinforcements.
D. Providing empirical advice.
E. Usage of open-ended questions.

Psychopharmacology

29. Which of the following antidepressants is LEAST likely to have sexual side-
effects?

A. Amitriptyline
B. Moclobemide
C. Mirtazapine
D. Fluoxetine
E. Venlafaxine.

30. The benzodiazepine with the LONGEST half-life is:


A. Clonazepam
B. Diazepam
C. Flurazepam
D. Lorazepam
E. Triazolam.

31. Which of the following is NOT an acetylcholinesterase inhibitor?


A. Buprenorphine
B. Donepezil
C. Galantamine
D. Rivastigmine
E. Tacrine.

32. Which of the following regarding side effects of risperidone is FALSE?

A. Akathisia is common with high dose of risperidone.


B. Prolactin elevation, probably greater than that seen with other second generation
antipsychotics.
C. Insomnia, headache and nausea are common.
D. Rhinitis is a possible side effect.
E. The risk for weight gain and the metabolic syndrome is higher than olanzapine.

!
!
94
33. Tricyclic antidepressants should be avoided with all of the following EXCEPT
A. Patients hospitalized for severe melancholic depression
B. Recent myocardial infarction
C. Right bundle branch block
D. Untreated glaucoma
E. Urinary retention.

34. Which of the following is CORRECT regarding galactorrhoea induced by


antipsychotic drugs?

A. Antipsychotic drugs modulate the hypothalamic function and lead to galactorrhoea.


B. Antipsychotic drugs cause pituitary adenoma and galactorrhoea.
C. Bromocriptine can treat galactorrhoea induced by antipsychotic drugs.
D. Galactorrhea is caused by antipsychotics acting directly on the breast tissue.
E. Galactorrhoea is caused by the concurrent use of anticholinergic medication.

35. Which of the following drugs is LEAST likely to increase lithium toxicity?
A. Angiotensin-converting enzyme (ACE) inhibitors
B. Calcium channel blockers
C. Non-steroidal anti-inflammatory drugs (NSAIDS)
D. Sodium valproate
E. Thiazide diuretics.

36. Which of the following is FALSE about clozapine?

A. It can cause agranulocytosis.


B. It has high affinity for D2 receptors.
C. It has low risk of extrapyramidal side effects.
D. It is associated with metabolic syndrome.
E. It is good for treatment resistant schizophrenia.

37. A 30-year-old woman with 3 episodes of major depression in the past 5 years
responds to fluoxetine 40mg every morning. Her last episode was 6 months ago.
In order to minimize the risk of relapse, which of the following treatment
strategies is MOST effective?

A. Continue fluoxetine 40mg every morning for five years.


B. Continue fluoxetine 40 mg for six months and then stop.
C. Provide intermittent maintenance electroconvulsive therapy.
D. Stop fluoxetine and restart it at the first sign of relapse.

!
!
95
E. Stop fluoxetine and start cognitive behaviour therapy.

Psychotherapy

38. Which of the following treatment strategies has the MOST evidence for treating
obsessive compulsive disorder?
A. Fluoxetine and brief dynamic psychotherapy
B. Fluoxetine and exposure and response prevention
C. Fluoxetine and eye movement desensitization and reprocessing
D. Fluoxetine and hypnotherapy
E. Fluoxetine and interpersonal psychotherapy.

39. You have to study anaesthesia, emergency medicine, obstetrics and


gynaecology, otolaryngology, ophthalmology and psychiatry in less than two
weeks. Your first thought is, I will never be able to study all six subjects in such
a short time! The upcoming examination is very stressful. You ignore the fact
that you were successful in the previous professional MBBS examinations and
you could handle multiple subjects in one examination in the past. From a
cognitive therapy perspective, what is the BEST description of this type of
thinking?

A. Catastrophic thinking
B. Magnification
C. Minimization
D. Personalization
E. Selective abstraction.

Ethics and laws

40. In Singapore, a person who is disturbed and aggressive, with poor insight into his
illness, refusing treatment and threatening family members can be admitted
involuntarily under the:

A. Advanced Care Directive


B. Mental Health (Care and Treatment) Act
C. Mental Capacity Act
D. Mental Health Act
E. Psychiatric Disorders and Treatment Act.

Liaison and neuropsychiatry

41. Which of the following medical conditions is LEAST likely to present with panic
attacks?

!
!
96
A. Asthma

B. Cushing syndrome

C. Insulinoma

D. Phaeochromocytoma

E. Thyrotoxicosis.

42. A 50-year-old man suffering from schizophrenia and he has been taken
haloperidol for the past 20 years. His QTc is 550 ms. The medical resident wants
to find the potential medical complication if he continues to take haloperidol.
Which of the following complications is LEAST LIKELY:

A. Myocardial infarction
B. Palpitation
C. Ventricular fibrillation
D. Sudden cardiac death
E. Torsade de pointes.

43. A 40-year-old schizophrenia patient presents with high urine volume, low urine
osmolality, low serum sodium and low urine sodium. Which of the following is the
MOST likely diagnosis?

A. Diabetes mellitus

B. Nephrogenic diabetes insipidus

C. Psychogenic polydipsia

D. Simple hyponatremia

E. Syndrome of inappropriate antidiuretic hormone secretion.

44. You are a resident working in the gynaecology ward. A 35-year-old woman was
admitted for ovarian cystectomy. She suffers from major depressive disorder and
insomnia. She consults a psychiatrist at the Institute of Mental Health (IMH). She
is prescribed with venlafaxine, zolpidem, lorazepam and hydroxyzine. She is due
for discharge today and she wants to go home. She expresses concern that she
cannot see her psychiatrist in the coming weeks due to pain associated with the
operation. She requests to obtain 3-month supply of her psychiatric medications
from you. She also requests a pain killer called tramadol. She threatens that if you
do not give her medication, she will commit suicide. What is the BEST approach
to handle this situation?

!
!
97
A. Do not give her any psychotropic medication and tramadol. Discharge her the same
day.
B. Negotiate with her for shorter duration of psychotropic medications and tramadol.
Obtain earlier appointment at IMH. Discharge her today.
C. Postpone discharge and use this to motivate her not to request for more
psychotropic medications.
D. Give her 3-month supply of psychotropic medications and tramadol. Discharge her
today.
E. Send her to IMH for assessment because she threatens that she will commit suicide.

MCQ exam (Paper 4)

History and mental state exam

45. You are a medical resident. The nurse informs The answer is A.
you that an 80-year-old man who was admitted
for cellulitis seems to be depressed and
mentions about passive suicidal thought.
When you assess him, he sees ghost in the Explanation: This man may suffer
ward and expresses paranoid ideation against from delirium and acute
nursing staff. Which of the following confusional state. It is most
assessment is MOST important to establish important to assess orientation.
the diagnosis?

F. Assess orientation
G. Assess judgement Assessing somatic symptoms is
H. Assess recognition not useful as it is caused by
I. Assess registration and short-term recall medical conditions.
J. Assess somatic symptoms.

Year: 2013.

Cognitive assessment

46. Which of the following cognitive tasks is NOT The answer is B.


an assessment of short term or long term
memory?

Explanation: Option B refers to


F. Ask the patient to name the current Prime verbal fluency which is part of the
Minister of Singapore. frontal lobe assessment. Option D
G. Ask the patient to name as many animals as refers to registration and it is an
possible that can be found in the Singapore
important component before
Zoo.
H. Ask the patient to tell you his or her address assessing the short-term recall
and later, you check the answer with patients (Option E). To be fair to the

!
!
98
medical record. patient, the patient should be given
I. Inform the patient 3 objects (e.g. Apple, multiple attempts to register the 3
Newspaper and Train) and ask the patient to items before testing short-term
name the 3 objects immediately.
recall in Option E.
J. Inform the patient 3 objects (e.g. Apple,
Newspaper and Train) and ask the patient to
name the 3 objects after 5 minutes.
Year: 2013.

Psychiatric, epidemiology, etiology, diagnosis and


classification

47. A 20-year-old man with an identical twin is The answer is B.


diagnosed with major depressive disorder.
His twin brother asks you to comment on the
chance he will develop depressive disorder.
Which of the following is the CORRECT Explanation:
response to his twin brother?
F. Major depressive disorder is a strongly
inheritable disease. You will definitely develop
it as well. Family studies show that a person
G. The likelihood of you developing depressive has 40-70% chance to develop
disorder is higher than other siblings. depressive episode if a first degree
H. The likelihood of you developing depressive relative suffer from depressive
disorder is same as non-identical twins. episode.
I. There is only a slightly increased risk that you
will become depressed as compared to the
general population. Twin studies show that the
J. You do not need to worry because concordance rate for monozygotic
environmental factor is an important factor to twins is 40 50% and for dizygotic
determine whether you will develop depressive twins is 20%.
disorder.

Year: 2013.

48. A 35-year-old woman suffers from low mood, The answer is C.


low energy, poor sleep, poor appetite and
recurrent suicidal thoughts. She firmly
believes that she deserves the death sentence
for minor mistakes made in the past. Which of Explanation: This man suffers from
the following statements is FALSE? severe depressive disorder with
psychotic features. Option A, B, D
and E are correct.
F. For severe depressive episode with psychotic
features, psychotic symptoms usually occur
after manifestation of depressive symptoms.
G. Her psychotic symptoms are considered to be

!
!
99
mood congruent. There is not enough evidence to
H. Based on the case scenario, there is enough suggest the diagnosis of
evidence to suggest that she suffers from schizoaffective disorder because
schizoaffective disorder.
her symptoms do not suggest that
I. The prescription of antipsychotic drug should
be considered. she suffers from schizophrenia.
J. Electroconvulsive therapy (ECT) is a
recognised treatment for his condition.
Year: 2013.

49. Which of the following disorders has been The answer is C.


shown to have the GREATEST degree of
heritability?
F. Alcoholism
G. Attention-deficit/hyperactivity disorder (ADHD) Explanation:
H. Autism
I. Major depressive disorder Heritability of autism to be more
J. Schizophrenia. than 90%. 90% of the differences
between autistic and non-autistic
individuals is due to genetic
effects.

Year: 2013.

50. Which of the following is the MOST common The answer is D.


cause of insomnia among psychiatric patients
in Singapore?

Explanation: Underlying psychiatric


F. Sleep apnea disorder is the most common
G. Shift work cause of insomnia among
H. Stimulant use psychiatric patients. Students
I. Underlying psychiatric illness
should observe this during their
J. Use of sleeping pills.
clinical attachment. Depressive
disorder, bipolar disorder and
generalized anxiety disorder are
associated with insomnia.

51. Regarding the age of onset in schizophrenia, The answer is D.


which of the following is CORRECT?

!
!
100
F. European women have earlier age of onset Explanation: Current research
than Asian women. findings in schizophrenia show that
G. Current research findings remain men have earlier onset as
inconclusive about the age of onset in
compared to women.
schizophrenia.
H. There is no difference in the age of onset
between men and women.
I. Men usually have earlier age of onset as
compared to women.
J. Women usually have earlier age of onset as
compared to men.

Psychopathology

52. Agoraphobia without panic attack is BEST The answer is D.


referred to:

F. Concern about physical appearance Explanation: Patients suffering


G. Constant worried about physical health from agoraphobia has fear that
H. Fear of blood and needle they cannot escape from the
I. Fear of collapse in an enclosed space enclosed space.
J. Fear of authority figures.

Year: 2013.

53. Which of the following is MOST suggestive of The answer is D.


the diagnosis of schizophrenia?
F. Acute onset of psychosis
G. Apathy
H. Cognitive impairment Explanation: Hearing own thoughts
I. Hearing own thoughts or echo de la pense only occurs in
J. Persistent deterioration of personality. schizophrenia and is most
suggestive of such diagnosis.

Option A can occur in brief


psychosis or acute/transient
psychosis, not necessarily
schizophrenia.

Option B, D and E occur in both


schizophrenia and dementia.

!
!
101
Year: 2013.

General adult psychiatry

54. Peter is 27-year-old and lives with his parents. The answer is A.
He has been employed as a delivery man for
most of the time since leaving school, but has
recently left his job as a salesman. He has
never taken any illicit drugs. His parents state Explanation: Peter develops
that in the last three weeks, he has been grandiose delusion because he
extremely active, requiring less sleep and not does not have the capacity to
appearing tired, being over-talkative and make a machine to cure cancer
disinhibited and on occasions quite
based on his background. The
irritable. He claimed to have invented a
machine for curing cancer and wished to go to most appropriate diagnosis is
the U.S. to sell it. When stopped by his bipolar disorder with manic
parents, he became violent, and they called features.
the police. Which of the following diagnosis is
MOST appropriate for this patient?

F. Bipolar I disorder with manic features Year: 2013.


G. Bipolar II disorder with hypomanic features
H. Cyclothymia
I. Mixed affective disorder
J. Schizoaffective disorder.

55. Which of the following personality disorders The answer is B.


is LEAST likely to be considered as a
differential diagnosis to schizophrenia?
Explanation: Obsessive
compulsive personality disorder is
F. Borderline personality disorder
least likely to be associated with
G. Obsessive compulsive personality disorder
H. Paranoid personality disorder psychotic features.
I. Schizotypal personality disorder
J. Schizoid personality disorder.
Year: 2013

56. Which of the following is LEAST likely to be The answer is C.


found in research in the relationship between
depressive disorder and hormones?

Explanation: There is no
F. Blunted adrenocorticotropic hormone (ACTH) suppression of cortisol with
response to the corticotropin-releasing dexamethasone in research finding
hormone (CRH) in depressive disorder. Option A
G. Blunted thyroid stimulating hormone (TSH)
indicates the failure of negative
response to thyrotropin-releasing hormone

!
!
102
(TRH) feedback and causes an increase
H. Cortisol suppression with dexamethasone in cortisol (Option D and E). Option
I. There is an increased cortisol. B is associated with low T3 or T4
J. There is an increased adrenal sensitivity to
level.
ACTH.

Year: 2013.

This is an advanced-level
question.

57. Which of the following disorders is LEAST The answer is D.


likely to be considered as a differential
diagnosis of a 35-year-old driver develops
nightmare, flashback and hypervigilance after
a road traffic accident? Explanation: OCD is least likely to
F. Factitious disorder share symptoms with post-
G. Generalized anxiety disorder traumatic stress disorder (PTSD).
H. Major depressive disorder
I. Obsessive compulsive disorder As a result, OCD is least like to be
J. Phobic disorder. a DDX of PTSD.

Year: 2013.

58. Rapid eye movement (REM) sleep is The answer is A.


associated with all of the following EXCEPT:

F. Increased parasympathetic activity Explanation: REM sleep is


G. Increased cerebral blood flow associated with an increase in
H. Increased complexity of dreams sympathetic activity.
I. Maximal loss of muscle tone
J. Transient runs of conjugate eye
movements.
Year: 2013.

This is an advanced level question.

59. A 17-year-old male comes to see you because The answer is B.


he is experiencing tremendous stress in his
new job. He has finished his O level
examination and waiting to start the National
Service in 6 months. He got an offer to work in Explanation:
a shoe shop which specializes in selling
female shoes. He realizes that he feels

!
!
103
sexually aroused by female shoes. He spends
a lot of time fantasizing about female shoes.
He stole multiple pairs of shoes from the shop This man suffers from fetishism
and used them for masturbation. He used to because he derives sexual
be a normal person. He was a hardworking stimulation from an inanimate
student whose main interests were chemistry object (i.e. female shoes) and his
and physics. What is the MOST likely
fetishism causes social and
diagnosis?
occupational dysfunction because
F. Adjustment disorder he spends inordinate time
G. Fetishism fantasizing about female shoes.
H. Kleptomania
I. Sadomasochism
J. Voyeurism.
He stole shoes to satisfy his
fetishism and this is not a case of
kleptomania.

Year: 2013.

60. Which of the following psychiatric disorders is The answer is B.


MOST common among patients with bulimia
nervosa?

F. Alcohol use disorder Explanation: Major depressive


G. Major depressive disorder disorder is the most common
H. Obsessive compulsive disorder psychiatric comorbidity in bulimia
I. Panic disorder nervosa (around 50- 85%)
J. Intermittent explosive disorder.

Year: 2013.

Old age psychiatry

61. Manic episodes in old people are associated The answer is A.


with:

F. Less euphoria Explanation: Mania in old people is


G. Less mixed presentation with depression associated with less euphoria, less
H. Less paranoid delusions hyperactivity, less flight of ideas
I. More hyperactivity but more mixed presentation and
J. More flight of ideas.
more paranoid ideation.

!
!
104
Year: 2013.

This is an advanced level question.

The answer is A.

62. All of the following statements are true


regarding post-operative delirium EXCEPT:
Explanation: 30% of patients
suffering from delirium may have
F. Delirium cannot be diagnosed if all laboratory normal laboratory results and no
results are normal.
abnormality was found. Delirium
G. It can occur in an 80-year-old man after
gastrectomy. can occur in young people who
H. It can occur in a 16-year-old girl after renal undergo a complicated operation
transplant. with immunosuppressant.
I. Haloperidol is one of the treatments of choice.
J. Physical restraint should be avoided if possible.

Year: 2013.

63. A 70-year-old man with a history of drinking The answer is B.


Chinese wine, hepatitis, chronic renal failure
and hypertension was brought by his wife to
the hospital for treatment of an acute cellulitis. Explanation:
He was noted to have tachycardia in the
Accident and Emergency Department. You are This patient is already at a high
the on-call medical resident and this man was risk for delirium based on his age
admitted to the medical ward. What is the and medical comorbidities,
MOST appropriate first approach?
including chronic renal failure.
F. Assess short term recall to rule out
dementia on delirium.
G. Obtain history from collaterals whether he Without adequate collateral
has in fact been drinking Chinese wine history, providing a high dose
recently.
regimen of potentially unnecessary
H. Put him on propranolol 10mg three times
per day to control tachycardia. benzodiazepine puts the patient at
I. Start diazepam 5mg three times per day to risk of worsened delirium. His
prevent delirium tremens. elevated heart rate may reflect
J. Start lorazepam 1mg three times per day to agitation or pain due to acute
prevent delirium tremens. cellulitis.

Beta blockers most often mask the


sympathetic outflow signs of

!
!
105
withdrawal of alcohol and the
prescription of propranolol is not
necessary.

Year: 2013.

64. A 70-year-old woman with severe depressive The answer is B.


symptoms and strong suicidal ideation which
is not responded to amitriptyline 150mg daily
and fluvoxamine 200mg daily. Each
medication was tried for 6 months with good Explanation: This elderly woman
adherence. She has good past health. Her suffers from treatment resistant
children consult you for further management. depression and electroconvulsive
The MOST appropriate treatment which you therapy is the treatment of choice
would recommend is:
to treat suicidal ideation.
A. Cognitive behaviour therapy

B. Electroconvulsive therapy Old age is not a contraindication


for electroconvulsive therapy.
C. Donepezil

D. Olanzapine
Year: 2013.
E. Repetitive transcranial magnetic stimulation.

Child and adolescent psychiatry and intellectual


disability

65. You are helping a mother to develop The answer is B.


behavioural program to deal with her son who
has oppositional defiant disorder. Which of
the following is the MOST significant
component of this program? Explanation: Positive
reinforcement of promoting
desirable behaviours through
F. Extinction rewards is the most significant
G. Positive reinforcement
component of the program.
H. Punishment
I. School suspension
J. Strict parenting.
Year: 2013.

66. Which of the following factors are LEAST The answer is D.


likely to be the aetiological factors in attention

!
!
106
deficit and hyperactivity disorder (ADHD)?
Explanation: Low birth weight, not
high birth weight is associated with
A. Antenatal exposure to alcohol
the risk of developing ADHD.
B. Antenatal exposure to nicotine
C. Delivery complications
D. High birth weight
E. Traumatic brain injury during infancy Year: 2013.
and early childhood.

67. You are a general practitioner. A 40-year-old The!answer!is!A.!


woman consults you as a result of high!blood!
pressure.!General!inspection!shows!ptosis,!a! !
broad!neck!and!indistinct!hairline.!Her!chest!
appears!to!be!broad.!Auscultation!reveals! Explanation:!She!suffers!from!
cardiac!murmurs.!She!is!single!and!stays!with! Turners!syndrome!(XO!syndrome)!
her!parents.!She!completed!her!education!in!the! as!evidenced!by!webbed!neck!(a!
Institute!of!Technological!Education!(ITE)!and! broad!neck!and!a!low!or!indistinct!
works!as!a!store!room!officer.!If!chromosome!
hairline),!coarctation!of!aorta!
analysis!is!performed,!the!MOST!likely!finding!
is: (cardiac!murmurs,!hypertension)!
! and!normal!/borderline!intelligence.!
A. XO syndrome She!may!have!infertility.!Her!single!
B. XXY syndrome status!cannot!confirm!the!infertility!
C. XXXY syndrome but!it!remains!a!possibility.!
D. XYY syndrome
E. YO syndrome. !

Year: 2013.

This is an advanced-level
question.

68. You are a paediatric resident. A 7-year-old boy The answer is B.


is suspected to suffer from autism. You
referred this boy to see an education
psychologist for assessment. The education
psychologist sends a report to you and you Explanation: In autism,
need to explain the findings to her mother. His performance IQ (Block design) is
strength is MOST likely found in which of the higher than verbal IQ.
following areas?

A. Abstract thinking Year: 2013.


B. Block design
C. Explain similarities
D. Oral presentation skills

!
!
107
E. Verbal concept formation

Substance abuse

69. Which of the following neuroanatomical The answer is C.


structures is MOST implicated in substance
misuse and dependence?

Explanation: Substance abuse and


F. Amygdala dependence produce additive
G. Basal nucleus of Meynert effects on dopamine release in the
H. Nucleus accumbens nucleus accumbens.
I. Hippocampus
J. Raphe nucleus.
Year: 2013

70. A 50-year-old man is admitted to the The answer is B.


hepatobillary ward as a result of pancreatitis.
You are the resident on call. His wife tells you
that she has seen him intoxicated with alcohol
in the past few days. The patient denies. Explanation: GGT is an early
Which of the following is the BEST method indicator of alcohol relapse,
available in a general hospital in Singapore to SENSITIVITY is 40-60% and
confirm that this patient has been drinking SPECIFICITY is 80%. 4 drinks per
recently? day for the past 2 weeks will
increase GGT. CDT is a very
sensitive and specific test but it is
F. Carbohydrate-deficient transferrin (CDT) not readily available in a general
G. Gamma-glutamyl transferase (GGT) hospital in Singapore. The cost
H. Mean corpuscular volume (MCV) CDT is much higher than GGT.
I. Serum alcohol level
J. Serum amylase. Year: 2013

71. A common and safe pharmacological The answer is E.


treatment for reducing relapse in alcohol
dependence has direct actions upon which
receptors?
Explanation: This question refers
to naltrexone, a common and safe
F. Benzodiazapine pharmacological treatment for
G. Cannabinoid reducing relapse in alcohol
H. Dopamine
dependence.
I. -Aminobutyric acid (GABA)
J. Opioid.

Year: 2013

!
!
108
72. Which of the following is LEAST consistent The answer is D.
with the objective of motivational
interviewing?

Explanation: Motivational
F. Allow the patient give their inputs without interviewing emphasizes on
interruption. individuals patients needs and
G. Establishing a collaborative patient-therapist avoid empirical advice.
relationship.
H. Identifying appropriate reinforcements.
I. Providing empirical advice.
J. Usage of open-ended questions. Year: 2013

Psychopharmacology

73. Which of the following antidepressants is The answer is C.


LEAST likely to have sexual side-effects?

F. Amitriptyline
G. Moclobemide Explanation: Mirtazapine and
H. Mirtazapine bupoprion have relatively low risk
I. Fluoxetine of causing sexual side effects as
J. Venlafaxine. compared to other
antidepressants.

Year: 2013

74. The benzodiazepine with the LONGEST half- The answer is B.


life is:
F. Clonazepam
G. Diazepam
H. Flurazepam Explanation: Diazepam has the
I. Lorazepam longest half life which lasts for
J. Triazolam. more than one day.

Year: 2013

75. Which of the following is NOT an The answer is A.


acetylcholinesterase inhibitor?
F. Buprenorphine
G. Donepezil
H. Galantamine Explanation: Buprenorphine is a
I. Rivastigmine partial opioid agonist which is used
J. Tacrine. in the treatment of opioid
dependence but this medication

!
!
109
was banned in Singapore due to
diversion and misuse.

Year: 2013.

76. Which of the following regarding side effects The answer is E.


of risperidone is FALSE?

F. Akathisia is common with high dose of Explanation: The risk for weight
risperidone. gain and metabolic syndrome is
G. Prolactin elevation, probably greater than that lower than olanzapine.
seen with other second generation
antipsychotics.
H. Insomnia, headache and nausea are common.
I. Rhinitis is a possible side effect.
J. The risk for weight gain and the metabolic Year: 2013.
syndrome is higher than olanzapine.

77. Tricyclic antidepressants should be avoided The answer is A.


with all of the following EXCEPT
F. Patients hospitalized for severe melancholic
depression
G. Recent myocardial infarction Explanation: TCA may be helpful
H. Right bundle branch block in severe depression. The other
I. Untreated glaucoma are relative contraindications.
J. Urinary retention.

Year: 2013.

78. Which of the following is CORRECT regarding The answer is C.


galactorrhoea induced by antipsychotic
drugs?

Explanation: Galactorrhoea can be


F. Antipsychotic drugs modulate the hypothalamic treated with bromocriptine.
function and lead to galactorrhoea.
G. Antipsychotic drugs cause pituitary adenoma
and galactorrhoea.
H. Bromocriptine can treat galactorrhoea induced Galactorrhoea is due to a direct
by antipsychotic drugs. effect on the anterior pituitary by
I. Galactorrhea is caused by antipsychotics acting dopamine secreting

!
!
110
directly on the breast tissue. tuberoinfundibular neurons, which
J. Galactorrhoea is caused by the concurrent use normally inhibit prolactin release.
of anticholinergic medication.

Antipsychotic drugs do not cause


pituitary adenoma.

Year: 2013

This is an advanced-level
question.

79. Which of the following drugs is LEAST likely The answer is D.


to increase lithium toxicity?
F. Angiotensin-converting enzyme (ACE) inhibitors
G. Calcium channel blockers
H. Non-steroidal anti-inflammatory drugs (NSAIDS) Co-administration of valproate and
I. Sodium valproate lithium have no effect on the steady-
J. Thiazide diuretics. state kinetics of lithium.

Year: 2013

80. Which of the following is FALSE about The answer is B.


clozapine?

F. It can cause agranulocytosis. Explanation: It has low affinity for


G. It has high affinity for D2 receptors. D2 receptors.
H. It has low risk of extrapyramidal side effects.
I. It is associated with metabolic syndrome.
J. It is good for treatment resistant schizophrenia.
Year: 2013

81. A 30-year-old woman with 3 episodes of major The answer is A.


depression in the past 5 years responds to
fluoxetine 40mg every morning. Her last
episode was 6 months ago. In order to
minimize the risk of relapse, which of the Explanation: This patient has 3
following treatment strategies is MOST episodes of major depression in
effective? the past 5 years and the number of

!
!
111
episodes is considered to be
frequent.
F. Continue fluoxetine 40mg every morning for
five years.
G. Continue fluoxetine 40 mg for six months and
then stop. It is dangerous to reduce the dose
H. Provide intermittent maintenance or stop antidepressant soon.
electroconvulsive therapy.
I. Stop fluoxetine and restart it at the first sign of
relapse.
J. Stop fluoxetine and start cognitive behaviour Year: 2013
therapy.

Psychotherapy

82. Which of the following treatment strategies The answer is B.


has the MOST evidence for treating obsessive
compulsive disorder?
A. Fluoxetine and brief dynamic
psychotherapy Explanation: Exposure and
B. Fluoxetine and exposure and response response prevention is part of the
prevention cognitive behaviour therapy and
C. Fluoxetine and eye movement has the most evidence for treating
desensitization and reprocessing
obsessive compulsive disorder.
D. Fluoxetine and hypnotherapy
E. Fluoxetine and interpersonal
psychotherapy.
Year: 2013

83. You have to study anaesthesia, emergency The answer is E.


medicine, obstetrics and gynaecology,
otolaryngology, ophthalmology and
psychiatry in less than two weeks. Your first
thought is, I will never be able to study all six Explanation: This student
subjects in such a short time! The upcoming demonstrate selective abstraction:
examination is very stressful. You ignore the focus on current negative aspect
fact that you were successful in the previous but ignore the positive aspects in
professional MBBS examinations and you
the past.
could handle multiple subjects in one
examination in the past. From a cognitive
therapy perspective, what is the BEST
description of this type of thinking? Year: 2013
F. Catastrophic thinking

!
!
112
G. Magnification
H. Minimization
I. Personalization
J. Selective abstraction.

Ethics and laws

84. In Singapore, a person who is disturbed and The answer is B.


aggressive, with poor insight into his illness,
refusing treatment and threatening family
members can be admitted involuntarily under
the: Explanation: Mental Health (Care
and Treatment) Act was passed in
2008 and this is the most correct
F. Advanced Care Directive answer.
G. Mental Health (Care and Treatment) Act
H. Mental Capacity Act
I. Mental Health Act
J. Psychiatric Disorders and Treatment Act. Year: 2013.

Liaison and neuropsychiatry

85. Which of the following medical conditions is The answer is B.


LEAST likely to present with panic attacks?

A. Asthma Explanation: Cushing syndrome is


more likely to be associated with
B. Cushing syndrome depression rather than panic
attacks.
C. Insulinoma

D. Phaeochromocytoma
Year: 2013.
E. Thyrotoxicosis.

86. A 50-year-old man suffering from The answer is A.


schizophrenia and he has been taken
haloperidol for the past 20 years. His QTc is
550 ms. The medical resident wants to find the
potential medical complication if he continues Explanation: A prolonged QTc
to take haloperidol. Which of the following mainly affects the ventricles,
complications is LEAST LIKELY: leading to ventricular fibrillation,
torsade de pointes and sudden
cardiac death. Palpitation is a
F. Myocardial infarction
G. Palpitation recognized complication.
H. Ventricular fibrillation Myocardial infarction is unlikely to

!
!
113
I. Sudden cardiac death be associated with prolonged QTc.
J. Torsade de pointes.

Year: 2013.

This is an advanced-level
question.

87. A 40-year-old schizophrenia patient presents The answer is C.


with high urine volume, low urine osmolality,
low serum sodium and low urine sodium.
Which of the following is the MOST likely
diagnosis? Explanation: He suffers from
psychogenic polydipsia because
there is a dilution in urine volume,
A. Diabetes mellitus urine osmolality, serum sodium
B. Nephrogenic diabetes insipidus and urine sodium due to fluid
overload and a result of excessive
C. Psychogenic polydipsia drinking.

D. Simple hyponatremia

E. Syndrome of inappropriate antidiuretic hormone Year: 2013.


secretion.

88. You are a resident working in the gynaecology The answer is B.


ward. A 35-year-old woman was admitted for
ovarian cystectomy. She suffers from major
depressive disorder and insomnia. She Explanation: It is reasonable that
consults a psychiatrist at the Institute of patient may not be able to see her
Mental Health (IMH). She is prescribed with psychiatrist shortly after operation.
venlafaxine, zolpidem, lorazepam and
Her suicide risk may not be
hydroxyzine. She is due for discharge today
and she wants to go home. She expresses genuine in this case because it is
concern that she cannot see her psychiatrist after a condition (if no medication
in the coming weeks due to pain associated is given).
with the operation. She requests to obtain 3-
month supply of her psychiatric medications
from you. She also requests a pain killer
called tramadol. She threatens that if you do In real situation, the best answer is
not give her medication, she will commit to assess her suicide risk by
suicide. What is the BEST approach to handle yourself or your hospital
this situation? psychiatrist. This option is not
available and you need to choose
F. Do not give her any psychotropic medication

!
!
114
and tramadol. Discharge her the same day. the next best answer.
G. Negotiate with her for shorter duration of
psychotropic medications and tramadol.
Obtain earlier appointment at IMH. Discharge
her today. Option A may cause a relapse of
H. Postpone discharge and use this to motivate her psychiatric illness by not giving
her not to request for more psychotropic her any psychotropic medications.
medications.
I. Give her 3-month supply of psychotropic
medications and tramadol. Discharge her Option C will intensify her
today. disappointment and postpone
J. Send her to IMH for assessment because she
threatens that she will commit suicide. discharge does not offer solution.

Option D: 3-month supply


medication is too long and it may
pose risk to patients.

Option E will affect therapeutic


alliance and it is important to
further assess her suicide risk. She
mentions about suicidal thought as
a sign of anger.

Year: 2013

This is an advanced-level
question.

Paper!5:!Questions!

History, mental state exam and psychopathology!

1. A! 50TyearTold! woman! presents! with! severe! depressive! episode! with! psychotic!


features.!Which!of!the!following!delusions!is!LEAST!likely!to!occur!in!this!patient?!
!
A. Deserved!punishment!

!
!
115
B. Guilt!
C. Incurable!illnesses!
D. Jealousy!
E. SelfNdepreciation.!

2. A!30TyearTold!prisoner!gives!approximate!answers!to!questions.!For!example,!if!it!
is!Thursday,!he!will!say!it!is!Friday!and!claims!a!dog!has!three!legs.!This patient is
MOST likely suffering from:

A. Capgras syndrome
B. Ekboms syndrome
C. Fregolis syndrome
D. Gansers syndrome
E. Othellos syndrome.

3. A psychiatric patient suddenly realises that he is an interpreter for deaf people and tries
to interpret others speech in sign language. He demonstrates strange repetitive
movements. His signs appear to come in threes or fours, occasionally swinging his
shoulders, as if he is conveying a message. Which of the following is the BEST term to
describe his movements?
!

A. Ambitendency!
B. Mannerism!
C. Negativism!
D. Stereotypies!
E. Waxy!flexibility.!
!
!

Cognitive assessment!

4. A 33-year-old man with childhood developmental delay and epilepsy received special
education and now works as a cleaner. He has worked in a tofu factory for three years.
He can only perform simple routine work in the factory. His supervisor reports his work
performance is slow and poor. His family says he cannot live independently and
demonstrated delay in achievement in self-care since young. How would you rate the
level of mental retardation?

A. Very mild mental retardation


B. Mild mental retardation
C. Moderate mental retardation
D. Severe mental retardation
E. Profound mental retardation.

Psychiatric epidemiology

5. A 65-year-old retired teacher is concerned about dementia and has assessed online
resources for information. She asks which type of dementia is more common in Asians
as compared to Caucasians. Your answer is:

!
!
116
A. Alzheimers disease
B. Lewy body dementia
C. Pseudodementia
D. Vascular dementia
E. Fronto-temporal lobe dementia.
Psychiatric aetiology, diagnosis and classification

6. A 20-year-old national serviceman was diagnosed to suffer from adjustment


disorder with depressed mood. He read the internet and found another disorder
known as dysthymic disorder. He wants to know the differences between these
two disorders. The following statements about differences between adjustment
disorder with depressed mood and dysthymia are true EXCEPT:

K. Adjustment disorder with depressed mood results from a known stressor, whereas
dysthymic disorder does not.
L. Adjustment disorder with depressed mood shows a genetic pattern, whereas
dysthymic disorder does not.
M. The duration of symptoms is shorter for adjustment disorder with depressed mood
than it is for dysthymic disorder.
N. The co-existence of a major depressive disorder with dysthymic disorder is called
double depression. There is no term for a major depressive disorder complicating
adjustment disorder with depressed mood.
O. Treatment is usually shorter and does not require the use of medication for
adjustment disorder with depressed mood as compared to dysthymic disorder.

7. You are a family doctor. A male and a female schizophrenia patient who stay in the
community have decided to get married and have a child. What is the risk for their child
to develop schizophrenia in the future?

A. 16%
B. 26%
C. 36%
D. 46%
E. 56%.

8. You!are!an!oncology!resident.!!A!60TyearTold!oncology!patient!developed!severe!nausea!as!a!
sideTeffect!during!chemotherapy.!However,!after!completing!treatment!she!continues!to!
experience!nausea!whenever!she!returns!to!the!hospital!for!followTup.!Which!of!the!
following!psychological!theories!BEST!describes!her!experience?

A. Biofeedback
B. Classical conditioning
C. Cognitive learning

!
!
117
D. Operant conditioning
E. Social learning.

General adult psychiatry

9. A 25-year-old woman experiences episodes of depersonalization, hyperventilation,


dizziness, and diaphoresis lasting for 15 minutes whenever she arrives at work. She
has been overwhelmed by work and conflicts with her supervisor for the past 3 months.
There are no such episodes at home or outside the office. What is the MOST likely
diagnosis?
!

A.!Acute!stress!disorder!

B.!Generalized!anxiety!disorder!

C.!Panic!disorder!!

D.!PostNtraumatic!stress!disorder!

E.!Somatization!disorder.!

10. You are an orthopaedic resident managing a 24-year-old motorcyclist who was involved
in a road traffic accident 3 days ago in which he sustained multiple fractures and his
pillion rider died. He is irritable, sleeps poorly and has nightmares and refuses to talk
about the accident. The MOST likely psychiatric diagnosis is:
!

A.!Acute!stress!disorder!!

B.!Adjustment!disorder!with!anxiety!

C.!Depressive!disorder!

D.!Generalized!anxiety!disorder!

E.!PostNtraumatic!stress!disorder.!

11. Based on the current research findings, which of the following statements is CORRECT
regarding the prognosis for a 35-year-old Indonesian man with schizophrenia who lives
in a rural village and treated by risperidone?

A. He has a worse prognosis as compared to patients staying in an urban city.


B. His prognosis is poor because he will seek treatment from traditional healer in the village.
C. He has a better prognosis as compared to patients staying in an urban city.
D. His prognosis is the same as patients staying in in an urban city
E. His prognosis is not related to the urban development of the society where he resides.

!
!
118
!

12. You!are!a!resident!working!in!the!Accident!and!Emergency!Department.!!A!39TyearTold!
English!man!was!married!to!a!Chinese!Singaporean!is!brought!to!the!Emergency!
Department!after!he!attempted!to!hang!himself.!He!is!actively!suicidal!and!has!marital!
problems.!He!has!not!spoken!to!his!wife!for!three!days.!The!hospital!does!not!have!a!
psychiatric!ward!nor!a!stayTin!psychiatric!team.!What!is!the!MOST!appropriate!
management!plan?!
A. Admit Peter to the medical ward of your hospital.
B. Discharge Peter from Emergency Department with follow-up in the polyclinic to reduce the
stigma.
C. Discharge Peter from Emergency Department with an early follow-up in the psychiatric
department.
D. Refer the couple for marital counselling by on-call medical social worker.
E. Transfer Peter to IMH for assessment and consider admission under Mental Health (Care and
Treatment) Act.
13. A 40-year-old single man consulting you in a general practice clinic reveals that since
adolescence he enjoys observing naked people and those involved in sexual activities
as it helps him achieve sexual arousal. The psychiatric condition associated with this
behaviour is:

A. Exhibitionism

B. Fetishism

C. Sadomasochism

D. Transvestism

E. Voyeurism.

14. A 65-year-old retired man consulting you in a general practice clinic reveals that he has
marital problems. His wife complains that he is very fuzzy about cooking. He does not
eat outside nowadays. If he goes to a high class restaurant, he expects that the steak
must be properly done and he is entitled to change the steak if it is poorly done. Now
he finds most steak restaurants in Singapore are not up to his standard. He claims it is
better to cook by himself. If the food is not up to his standard, he has no one to blame.
He likes to go to high-class club in town. He claims to be a special member in the club
due to his unlimited success and donation. Which of the following personality traits
BEST describes this man?

A. Antisocial
B. Avoidant
C. Borderline
D. Histrionic
E. Narcissistic.

!
!
119
15. A!30TyearTold!executive!complains!that!he!cannot!fall!asleep!at!night.!!Two!months!ago,!his!
life! was! interrupted! by! a! work! trip! to! the! United! States.! ! After! returning! to! Singapore,! he!
can!only!falls!asleep!at!2!to!3!a.m.!and!he!cannot!wake!up!on!time!the!next!morning.!!This!
leads!to!negative!impact!on!his!work.!!What!is!the!MOST!likely!diagnosis?!!
A. Circadian rhythm sleep disorder advanced sleep phase
B. Circadian rhythm sleep disorder delayed sleep phase
C. Excessive daytime sleepiness
D. Kleine-Levin syndrome
E. Random eye movement sleep disorder.
Old age psychiatry

16. A!65TyearTold!lady!with!bipolar!disorder!and!severe!functional!decline!presents!with!threeT
day!history!of!acute!agitation,!suicidal!ideation!and!refusing!to!eat!and!drink.!Her!MiniT
Mental!Status!Examination!(MMSE)!score!was!29/30.!auditory!hallucinations.!She!has!not!
responded!to!sodium!valproate!and!olanzapine.!!The!MOST!appropriate!treatment!at!this!
stage!is:!!!
A. Acetylcholinesterase!inhibitor!
B. Augmentation!with!antidepressant!
C. Augmentation!with!anxiolytics!
D. Change!valproate!to!lithium!
E. Electroconvulsive!therapy.!

17. You!are!public!health!doctor!and!work!in!the!Ministry!of!Health.!The!Ministry!
wants!to!identify!protective!factors!against!the!development!of!Alzheimers!
disease.!Which!of!the!following!is!least!associated!with!the!development!of!
Alzheimers!disease?!!!
A. Consuming!red!meat!on!a!frequent!basis!
B. Female!gender!
C. High!education!level!!
D. High!homocysteine!level!
E. Never!married.!

18. You!are!a!member!of!the!Hospital!Quality!Improvement!Committee.!The!hospital!wants!to!
improve! the! management! of! delirium.! Which! of! the! following! statements! regarding!
delirium!is!FALSE?!
!

A. Around!30%!of!the!elderly!admitted!to!the!medical!ward!will!develop!delirium!during!
hospitalisation.!
B. Delirium!is!often!missed!or!overlooked!by!health!care!professionals!in!the!elderly.!
C. Delirium!occurs!in!10%!of!children!undergoing!day!surgery.!
D. Delirium!usually!clears!within!3!days!of!correcting!underlying!medical!cause.!
E. The!incidence!of!delirium!increases!with!age.!
!

19. You are a geriatric doctor. The manager of a nursing home wants to consult you the
behavior which is most likely correlated with delusions in Alzheimers disease. Your
answer is:

A. Aggression and agitation


B. Disrobing (i.e.state of wearing of no clothes)

!
!
120
C. Shadowing
D. Suicide
E. Wandering.

20. You!are!a!paediatric!resident.!You!are!concerned!about!the!mental!health!of!a!50T
yearTold! mother! whose! 12TyearTold! daughter! died! of! leukaemia.! You! are! not!
certain!whether!she!suffers!from!normal!grief!or!pathological!grief.!Normal!grief!is!
NOT!characterized!by:!!
!

A. Denial!
B. Recurrent!suicidal!thoughts!!
C. Searching!for!the!deceased!
D. Sensations!of!seeing!or!hearing!the!deceased!
E. Transient!guilt.!
!

21. You are a geriatric resident reviewing a 75-year-old woman with Alzheimers disease.
Her family is keen for her to take acetylcholinesterase inhibitor. Which of the following
medical conditions is MOST contraindicated?

A. Anaemia
B. Asthma
C. Diabetes
D. Hyperthyroidism
E. Rheumatoid arthritis.

Child and adolescent psychiatry

22. Which of the following is the COMMONEST cardiac complication in a 14-year-


old adolescent suffering from anorexia nervosa on admission?

A. Atrial fibrillation and cardiac arrhythmia.


B. Bradycardia and hypertension
C. Bradycardia and hypotension
D. Tachycardia and hypotension
E. Tachycardia and hypertension.

Substance abuse

23. A! 30TyearTold! man! admitted! to! the! medical! ward! has! no! access! to! the! substance!
which! he! used! prior! to! admission.! He! complains! of! nausea! and! muscle! pain.!
Physical!examination!reveals!pupillary!dilation.!Which!of!the!following!substances!
is!MOST!likely!to!cause!the!above!withdrawal!symptoms?!
!
A. Alcohol!

!
!
121
B. Cocaine!
C. Midazolam!
D. Nictoine!
E. Opiate.

24. A female student has returned from Amsterdam after an exchange programme and
admits using cocaine for 1 year. She developed psychotic symptoms. Which of the
following is the MOST IMPORTANT risk factor for the development of psychosis while
using cocaine?

A. An!elevated!body!mass!index!
B. Combination!with!benzodiazepine!
C. Being!a!first!time!user!of!cocaine!
D. Being!female!
E. NonNintravenous!use.!

25. A!30TyearTold!man!who!is!dependent!on!alcohol!enquires!of!his!family!physician!if!
disulfiram!will!help!him!to!reduce!his!alcohol!intake!while!he!still!drinks!in!a!controlled!
manner.!!Which!of!the!following!recommendations!is!CORRECT?!
!
A. Disulfiram!is!recommended!because!it!has!more!evidence!than!other!medications!in!
helping!patients!to!quit!alcohol.!
B. Disulfiram!is!recommended!because!it!will!discourage!him!from!drinking!alcohol.!
C. Disulfiram!is!recommended!because!he!has!motivation!to!reduce!his!drinking.!
D. Disulfiram!is!not!recommended!because!it!may!cause!severe!side!effects!when!it!mixes!
with!alcohol.!
E. Disulfiram!is!not!recommended!because!it!is!indicated!for!opioid!dependence.!
!

Psychopharmacology

26. A!50TyearTold!man!with!a!major!depressive!disorder!did!not!respond!well!to!an!adequate!
trial!of!sertraline!200mg!(a!selective!serotonin!reuptake!inhibitor).!!Neither!did!he!fully!
respond!to!a!subsequent!8Tweek!trial!of!venlafaxine!150mg!(serotonin!noradrenaline!
reuptake!inhibitor).!!Which!of!the!following!is!NOT!RECOMMENDED!for!this!man?!!!
!
A. Augment!SSRI!with!lithium!(a!mood!stabilizer)!
B. Augment!SSRI!with!phenelzine!(a!monoaxmine!oxidase!inhibitor)!
C. Consider!electroconvulsive!therapy!
D. Consider!psychotherapy!
E. Increase!the!dose!of!venlafaxine.!!

27. Bupoprion differs from SSRI in which of the following ways?

F. Bupoprion has a higher incidence of sexual side effects and a lower incidence of

!
!
122
sedation and weight gain.
G. Bupoprion has a lower incidence of sexual side effects and a lower incidence of
sedation and weight gain.
H. Bupoprion has a lower incidence of sexual side effects and higher incidence of
sedation and weight gain
I. Bupoprion has lower incidence of sexual side effects and weight gain and a higher
incidence of sedation.
J. Bupoprion has a higher incidence of sexual side effects and sedation and a lower
incidence of weight gain.
28. Besides treatment resistant schizophrenia, another appropriate indication for clozapine
is?

A. Severe agranulocytosis
B. Severe embolism
C. Severe hypersalivation
D. Severe metabolic syndrome
E. Severe tardive dyskinesia.
.

29. A!50TyearTold!cardiac!patient!suffers!from!depression!and!the!psychiatrist!
prescribed!fluoxetine.!Which!of!the!following!statements!about!fluoxetine!is!TRUE?!
!

A. !Fluoxetine!decreases!warfarin!levels.!
B.!!!!!Fluoxetine!increases!metabolism!of!warfarin.!
C. !!!!!Fluoxetine!increases!bruising!while!patient!is!taking!
!!!!!!!!!!!!!!!!warfarin.!

D. He!has!no!increased!risk!of!gastrointestinal!bleeding!with!fluoxetine.!
E. !!!!He!should!not!change!to!paroxetine!because!paroxetine!decreases!anticoagulant!effect!of!
warfarin.!
!

30. Which of the following hypnotic agents causes the LEAST daytime sedation is:

A. Diphenhydramine
B. Diazepam
C. Lorazepam
D. Zopiclone
E. Zolpidem.

31. Which of the following medications has the most convincing evidence for reducing
suicidal behavior in bipolar patients?

A. Clozapine
B. Lamotrigine

!
!
123
C. Lithium
D. Olanzapine
E. Sodium valproate.

32. An otherwise healthy 50-year-old male patient receiving clozapine 300mg at night for
treatment resistant schizophrenia suddenly dies. The MOST likely cause of death is?

A. Myocarditis

B. Neuroleptic malignant syndrome

C. Pneumonia

D. Serotonin syndrome

E. Tardive dyskinesia.

33. A 28-year-old woman has a borderline personality disorder, primarily manifested by an


inability to control her urges to hurt herself. She frequently cuts her wrists when
frustrated. Which of the following medications has been found to be MOST helpful and
safest in curbing her self-harm behaviour?

A. Aripiprazole
B. Clonazepam
C. Fluoxetine
D. Lithium
E. Olanzapine.

Psychotherapy

34. A 30-year-old woman suffers from obsessive compulsive disorder and washes her hand
50 times a day. Which of the following techniques is the MOST essential in the
psychological treatment of her compulsive hand washing behaviour?

A. Cognitive restructuring
B. Exposure and response prevention
C. Social skill training
D. Systemic desensitization
E. Thought stopping.

35. A!25TyearTold!woman!with!a!panic!disorder!seeks!an!opinion!whether!pharmacotherapy!or!
psychotherapy! is! the! best! treatment! option! for! her.! Which! of! the! following! statements! is!
CORRECT?
A. Adding alprazolam on p.r.n. basis will make exposure therapy ineffective.
B. Adding alprazolam in the first week will lead to dependence and does not improve patients
outcome.
C. Antidepressants such as selective serotonin reuptake inhibitors are not useful to treat panic
disorder.
D. Relaxation therapy is more effective than cognitive behaviour therapy in treating panic

!
!
124
disorder.
E. Psychotherapy such as cognitive behaviour therapy requires more motivation than taking
medication.

36. A!70TyearTold!man!lodged!a!complaint!against!the!hospital.!!His!wife!died!suddenly!one!
year!ago.!!She!suffered!from!diabetes!and!had!frequent!episodes!hypoglycaemia.!!The!
husband!gave!her!higher!than!prescribed!dose!of!hypoglycaemic!agent!because!he!thought!
this!could!control!her!diabetes.!!His!children!disagreed!with!his!management!but!he!
refused!to!listen.!!The!coroners!report!stated!that!his!wife!died!of!hypoglycaemia.!!The!70T
yearTold!man!is!angry!with!the!endocrinologist!and!thinks!that!it!was!his!fault!to!prescribe!
the!wrong!hypoglycamic!agent!leading!to!his!wifes!death.!!He!demanded!a!written!apology!
from!the!endocrinologist.!!Which!of!the!following!defence!mechanisms!is!demonstrated!by!
this!man?
A. Altruism
B. Displacement
C. Reaction formation
D. Splitting
E. Undoing.

37. For post-traumatic stress disorder, which of the following interventions is LEAST
helpful to patients?

A. Cognitive behaviour therapy


B. Eye movement desensitization and reprocessing
C. Immediate group debriefing
D. Relaxation training
E. Supportive psychotherapy

38. You are a resident in plastic surgery. A 24-year-old woman believes her nose is
malformed and always checks her nose in the mirror. She was seen by multiple
doctors and they confirmed that her nose is normal. Which of the following
psychological treatment is the treatment of choice for this patient?

A. Exposure and response prevention


B. Family therapy
C. Problem solving therapy
D. Psychodynamic psychotherapy
E. Supportive psychotherapy.

Ethics and laws

39. A! 40TyearTold! woman! complained! of! headache,! backache,! nausea,! numbness! and! visual!
problems.! ! She! consulted! a! neurologist! whom! she! trusted! to! be! a! good! doctor.! ! The!
neurologist! diagnosed! the! patient! suffering! from! migraine! and! somatisation! disorder.!!
When! the! patient! requested! for! Magnetic! Resonance! Imaging! (MRI),! the! neurologist!
claimed!that!the!patient!had!exaggerated!her!somatic!symptoms!and!insisted!that!the!MRI!

!
!
125
scan! should! be! reserved! for! more! severe! patients.! ! Six! months! later,! the! patient! died! of!
brain! tumour.! ! In! addition! to! negligence,! which! of! the! following! ethical! principles! was!
violated!by!the!neurologist?!
!
A. Autonomy!
B. Confidentiality!
C. Fiduciary!duty!
D. NonNmaleficience!
E. Justice.!
40. 40-year-old man arrested for shoplifting is referred for a psychiatric assessment after
he tells the judge he suffers from kleptomania. Which of the following criteria is MOST
important in establishing the diagnosis of kleptomania?

A. Acts reus.
B. Patient informs you that he was diagnosed with kleptomania by a private psychiatrist before
but cannot provide further clinical details.
C. Patient informs you that he cannot control his impulse to steal and he does not want to steal.
D. Patient informs you that he has started stealing since Primary 6.
E. Past history of criminal record of stealing.

41. A medical student has applied for a 3-month research elective and he follows a
professor to conduct research. The professor asks him to conduct a cross-sectional
questionnaire study to assess mood symptoms of cancer patients. He is expected to
recruit 300 patients in 3 months. This project was approved by the ethics committee.
He attempts to obtain consent from an inpatient who has received chemotherapy to
participate in this study. Patient refuses because he feels very lethargic. Thirty minutes
later, he asks the patient to re-consider participating in this study again. The patient
lodges a complaint to the professor about this medical student. Which ethical
principles has the medical student violated?

A. Autonomy!
B. Beneficence!
C. Confidentiality!
D. NonNmaleficence!
E. Justice.!

Liaison and neuropsychiatry

42. A 50-year-old man who is dependent on alcohol is admitted for observation after a fall when
intoxicated. The ward team is concerned about complications associated with alcohol withdrawal.
Which of the following statements is INCORRECT?
A. Delirium tremens usually occur 6-12 hours after the last drink.
B. Tremors usually occur 6-12 hours after the last drink.
C. Anxiety usually occurs 12-18 hours after the last drink.
D. Convulsion usually occurs 12-18 hours after the last drink.
E. Sleep disturbance usually occurs 12-18 hours after the last drink.

!
!
126
43. You are a resident in general surgery. A 50-year-old man was admitted last night
after he cut his head, arms and hands during a suicide attempt. He is married
with a 10-year-old child. He works in the field of informational technology.
According to his wife, his elder brother died of gastric cancer 6 months ago. He
believed that he is genetically vulnerable for gastric cancer. He saw multiple
gastroenterologists and gastric surgeons. He went through multiple endoscopies
and biopsies with normal findings. Despite normal findings, he still worries that
he has undiagnosed gastric cancer. He argued with his wife tonight and wanted to
commit suicide. What of the following is the MOST appropriate psychiatric
diagnosis?

A. Acute stress reaction/marital problems


B. Depression/abnormal grief
C. Generalised anxiety disorder / phobia
D. Hypochondriasis/illness anxiety
E. Schizophrenia/psychosis.

44. You have become a consultant physician. A resident called you and informed you
that the parameter of a 20-year-old male psychiatric patient over the phone, His
temperature is 41 degree Celsius. His heart rate is 105 per minutes and blood
pressure fluctuates from 120/70 to 180/100. His breathing is normal and chest is
clear. His abdomen is soft. His creatinine kinase is 800 IU/L (normal <100 IU/L)
and leukocyte count is 10 x109/L (normal range: 1.5-5 x109/L). His limbs are very
rigid and he demonstrates difficulty in swallowing. He was given intravenous
injection of haloperidol 20mg three days ago. What is the MOST likely
diagnosis?

A. Catatonia
B. Meningitis
C. Neuroleptic malignant syndrome
D. Serotonin syndrome
E. Status epilepticus.

45. You are a doctor working in the polyclinic. A 40-year-old man suffers from
bipolar disorder and he takes lithium on a daily basis. His blood pressure is
170/100mmHg. Which of the following medications is the BEST to lower his blood
pressure?

A. Angiotensin-converting-enzyme inhibitor
B. Alpha agonist
C. Benzodiazapine
D. Beta blocker
E. Thiazide diuretic.
46. In medically ill patients with psychiatric illness, which pharmacological properties of a

!
!
127
psychotropic drug is MOST appropriate?

A. Long half-life psychotropic drug.


B. Psychotropic drugs with active metabolites.
C. Psychotropic drug with wide therapeutic index.
D. Psychotropic drug which inhibits metabolic enzymes.
E. Psychotropic drug which induces metabolic enzymes.

2014!First!Rotation!MCQ!

History, mental state exam and psychopathology! !

47. A! 50TyearTold! woman! presents! with! severe! The!answer!is!D.!


depressive! episode! with! psychotic! features.!
Which! of! the! following! delusions! is! LEAST! !
likely!to!occur!in!this!patient?!
! Explanation:!Delusion!of!jealousy!is!
F. Deserved!punishment! the!least!mood!congruent!as!
G. Guilt! compared!to!other!options.!
H. Incurable!illnesses!
I. Jealousy! !
J. SelfNdepreciation.!
Year:!2014!

48. A! 30TyearTold! prisoner! gives! approximate! The!answer!is!D.!


answers! to! questions.! For! example,! if! it! is!
Thursday,!he!will!say!it!is!Friday!and!claims!a! !
dog! has! three! legs.! This patient is MOST
likely suffering from: Explanation:!The!patient!provides!
an!approximate!answer!which!is!a!
clinical!feature!of!Gansers!
F. Capgras syndrome
G. Ekboms syndrome syndrome.!
H. Fregolis syndrome
I. Gansers syndrome !
J. Othellos syndrome.
Option!A!and!C!are!misidentification!
syndrome.!Option!B!refers!to!
delusion!of!infestation.!Option!E!
refers!to!morbid!jealousy.!

Year:2014!

!
!
128
49. A psychiatric patient suddenly realises that he is The!answer!is!B.!
an interpreter for deaf people and tries to
interpret others speech in sign language. He !
demonstrates strange repetitive movements.
His signs appear to come in threes or fours,
Explanation:!Mannerism!is!a!goal!!
occasionally swinging his shoulders, as if he is
conveying a message. Which of the following is directed!repetitive!movement!(e.g.!a!
the BEST term to describe his movements? speaker!tries!to!move!his!hands!
! repetitively!to!convey!his!
messages).!
F. Ambitendency!
G. Mannerism! !
H. Negativism!
This!mans!movements!seemed!to!
I. Stereotypies!
J. Waxy!flexibility.! be!goal!directed!because!he!
! attempted!to!show!sign!language!
! but!not!understandable!to!deaf!
people.!

Year!2014!

Cognitive assessment! !

50. A 33-year-old man with childhood developmental Answer:!C!


delay and epilepsy received special education
and now works as a cleaner. He has worked in a !
tofu factory for three years. He can only perform
simple routine work in the factory. His
Explanation:! This! case! is! more!
supervisor reports his work performance is slow
and poor. His family says he cannot live severe! than! mild! retardation! which!
independently and demonstrated delay in demonstrates! more! competency! in!
achievement in self-care since young. How selfNcare!and!independence.!
would you rate the level of mental retardation?
People! with! severe! or! profound!
F. Very mild mental retardation
G. Mild mental retardation mental! retardation! are! not! able! to!
H. Moderate mental retardation work!in!the!factory.!
I. Severe mental retardation
J. Profound mental retardation. !

Year!2014!

Psychiatric epidemiology !

51. A 65-year-old retired teacher is concerned about The!answer!is!D.!


dementia and has assessed online resources for
information. She asks which type of dementia is

!
!
129
more common in Asians as compared to !
Caucasians. Your answer is:
Explanation:!Vascular!dementia!
F. Alzheimers disease
(VaD)!is!found!to!be!more!common!
G. Lewy body dementia than!Alzheimers!disease!(AD)!in!
H. Pseudodementia some!Asian!countries.!In!a!study!
I. Vascular dementia conducted!by!National!
J. Fronto-temporal lobe dementia.
Neuroscience!Institute!(Singapore),!
53%!suffered!from!VaD!versus!47%!
from!AD.!The!ethnic!variation!is!due!
to!genetic!variation,!differences!in!
vascular!risk!factors!and!lifestyles.!!

Year:!2014!

Psychiatric aetiology, diagnosis and classification !

52. A 20-year-old national serviceman was The answer is B.


diagnosed to suffer from adjustment
disorder with depressed mood. He read the
internet and found another disorder known
as dysthymic disorder. He wants to know Explanation:
the differences between these two
disorders. The following statements about Option A is correct and relevant for
differences between adjustment disorder with this case.
depressed mood and dysthymia are true
EXCEPT: Option B is incorrect because
dysthymic disorder is more likely
to show a genetic pattern.
P. Adjustment disorder with depressed mood
results from a known stressor, whereas Option C is true because
dysthymic disorder does not.
adjustment disorder begins within
Q. Adjustment disorder with depressed mood
shows a genetic pattern, whereas three months of a stressor, and
dysthymic disorder does not. ends within six months of the
R. The duration of symptoms is shorter for stressor ending. Dysthymic
adjustment disorder with depressed mood disorder has a duration of two
than it is for dysthymic disorder. years in adults and one year in
S. The co-existence of a major depressive children.
disorder with dysthymic disorder is called
double depression. There is no term for a Option D is correct
major depressive disorder complicating
adjustment disorder with depressed mood. Option E is correct.
T. Treatment is usually shorter and does not
require the use of medication for adjustment
disorder with depressed mood as compared
to dysthymic disorder. Year: 2014

!
!
130
53. You are a family doctor. A male and a female The correct answer is D.
schizophrenia patient who stay in the
community have decided to get married and
have a child. What is the risk for their child to
develop schizophrenia in the future? Prevalence of Schizophrenia in
Specific Populations
F.16%
G. 26%
H. 36%
I. 46% Population Prevalence
J. 56%. (%)
General Population 1
Non-twin sibling of a 9
schizophrenic patient
Child with one 13
schizophrenic parent
Dizygotic twin of a 14
schizophrenic patient
Child of two 46
schizophrenic parents
Monozygotic twin of a 46
schizophrenic patient

!
Year: 2014
54. You!are!an!oncology!resident.!!A!60TyearTold! The!answer!is!B.!
oncology!patient!developed!severe!nausea!as!a!
sideTeffect!during!chemotherapy.!However,!after! Explanation:! ! This! phenomenon! is!
completing!treatment!she!continues!to!experience!
known! as! classical! conditioning.!
nausea!whenever!she!returns!to!the!hospital!for!
followTup.!Which!of!the!following!psychological! Chemotherapy! is! an! unconditional!
theories!BEST!describes!her!experience? stimulus! (UCS)! and! the! hospital!
building! is! conditional! stimulus!
F. Biofeedback (CS).! The! conditional! (CR)! and!
G. Classical conditioning
H. Cognitive learning unconditional! response! (UCR)!
I. Operant conditioning include! nausea.! The! pairing! of! CS!
J. Social learning. and! UCR! will! persist! even! UCS! (i.e.!
chemotherapy)!disappears.!

Year:!2014!

General adult psychiatry !

55. A 25-year-old woman experiences episodes of The!answer!is!C.!


depersonalization, hyperventilation, dizziness,
and diaphoresis lasting for 15 minutes whenever !
she arrives at work. She has been overwhelmed
by work and conflicts with her supervisor for the Explanation:!the!history!and!
past 3 months. There are no such episodes at
home or outside the office. What is the MOST symptoms!suggest!the!diagnosis!of!

!
!
131
likely diagnosis? panic!disorder!because!the!anxiety!
! symptoms!are!episodic!with!a!clear!
trigger.!
A.!Acute!stress!disorder!
!
B.!Generalized!anxiety!disorder!
Year:!2014.!
C.!Panic!disorder!!

D.!PostNtraumatic!stress!disorder!

E.!Somatization!disorder.!

56. You are an orthopaedic resident managing a 24- The!answer!is!A.!!


year-old motorcyclist who was involved in a
road traffic accident 3 days ago in which he !
sustained multiple fractures and his pillion rider
died. He is irritable, sleeps poorly and has
Explanation:!The!duration!of!
nightmares and refuses to talk about the symptoms!is!only!2!days.!It!is!too!
accident. The MOST likely psychiatric diagnosis early!to!conclude!whether!he!will!
is: develop!PostNTraumatic!Stress!
!
Disorder!(PTSD).!
A.!Acute!stress!disorder!!
!
B.!Adjustment!disorder!with!anxiety!
Year:!2014!
C.!Depressive!disorder!

D.!Generalized!anxiety!disorder!

E.!PostNtraumatic!stress!disorder.!

57. Based on the current research findings, which of Answer!is!C!


the following statements is CORRECT regarding
the prognosis for a 35-year-old Indonesian man !
with schizophrenia who lives in a rural village
and treated by risperidone? Explanation:! ! Schizophrenia! patients!
residing! in! the! rural! area! have! a! more!
F. He has a worse prognosis as compared to favourable! prognosis! as! compared! to!
patients staying in an urban city. patients! staying! in! urban! or!
G. His prognosis is poor because he will seek industrialized! areas.! In! rural! areas,!
treatment from traditional healer in the village. there! is! less! likelihood! of! social!
H. He has a better prognosis as compared to patients isolation! and! a! greater! degree! of! social!
staying in an urban city.
support.! ! There! is! more! emphasis! on!
I. His prognosis is the same as patients staying in in
an urban city social! and! familial! bonds! that! protect!
J. His prognosis is not related to the urban patients! from! social! isolation.!
development of the society where he resides. Furthermore,!it!is!easier!for!patients!to!

!
!
132
cope! with! workNrelated! tasks! in!
agricultural! or! village! settings.! There! is!
less!stigma!in!rural!areas.!!

Year:!2014.

58. You!are!a!resident!working!in!the!Accident!and! The!answer!is!E.!


Emergency!Department.!!A!39TyearTold!English!
man!was!married!to!a!Chinese!Singaporean!is! !
brought!to!the!Emergency!Department!after!he!
attempted!to!hang!himself.!He!is!actively!suicidal! Explanation:!Hanging!is!considered!
and!has!marital!problems.!He!has!not!spoken!to!
his!wife!for!three!days.!The!hospital!does!not!have!
to!be!a!dangerous!suicide!method!
a!psychiatric!ward!nor!a!stayTin!psychiatric!team.! and!this!patient!has!high!suicide!
What!is!the!MOST!appropriate!management!plan?! risk.!As!he!is!not!forthcoming!and!
F. Admit Peter to the medical ward of your hospital. does!not!suffer!from!any!medical!or!
G. Discharge Peter from Emergency Department with
follow-up in the polyclinic to reduce the stigma. surgical!complication,!sending!him!
H. Discharge Peter from Emergency Department with to!IMH!for!assessment!under!the!
an early follow-up in the psychiatric department. Mental!Health!Act!is!the!safest!
I. Refer the couple for marital counselling by on-call
medical social worker.
option.!
J. Transfer Peter to IMH for assessment and consider
admission under Mental Health (Care and !
Treatment) Act.
Year:!2014!

59. A 40-year-old single man consulting you in a The!answer!is!E.!


general practice clinic reveals that since
adolescence he enjoys observing naked people !
and those involved in sexual activities as it
helps him achieve sexual arousal. The Explanation:!Voyeurism!refers!to!
psychiatric condition associated with this
behaviour is: the!presence!of!recurrent!and!
intense!sexual!arousal!from!
A. Exhibitionism observing!an!unsuspecting!person!
who!is!naked,!in!the!process!of!
B. Fetishism
disrobing!or!engaging!in!sexual!
C. Sadomasochism activity!over!a!total!duration!of!6!
months.!
D. Transvestism
!
E. Voyeurism.
Year:!2014!!

60. A 65-year-old retired man consulting you in a The!answer!is!E.!


general practice clinic reveals that he has

!
!
133
marital problems. His wife complains that he is !
very fuzzy about cooking. He does not eat
outside nowadays. If he goes to a high class Explanation:! ! He! exhibits!
restaurant, he expects that the steak must be
properly done and he is entitled to change the
narcissistic! personality! trait.! He!
steak if it is poorly done. Now he finds most demonstrates! of! sense! of!
steak restaurants in Singapore are not up to his entitlement!and!holds!belief!that!he!
standard. He claims it is better to cook by is! better! than! the! others! (e.g.! chef!
himself. If the food is not up to his standard, he
has no one to blame. He likes to go to high- from! all! steak! restaurants).! He!
class club in town. He claims to be a special preoccupies!with!unlimited!success.!
member in the club due to his unlimited success
and donation. Which of the following !
personality traits BEST describes this man?
Year:!2014!
F. Antisocial
G. Avoidant
H. Borderline
I. Histrionic
J. Narcissistic.

61. A! 30TyearTold! executive! complains! that! he! cannot! Answer is B.


fall! asleep! at! night.! ! Two! months! ago,! his! life! was!
interrupted! by! a! work! trip! to! the! United! States.!!
After! returning! to! Singapore,! he! can! only! falls!
asleep! at! 2! to! 3! a.m.! and! he! cannot! wake! up! on! Explanation: This patient exhibits
time! the! next! morning.! ! This! leads! to! negative!
circadian rhythm sleep disorder -
impact! on! his! work.! ! What! is! the! MOST! likely!
diagnosis?!! delayed sleep phase because he
F. Circadian rhythm sleep disorder advanced sleep can only sleep after 2 to 3 a.m.
phase The advanced phase sleep
G. Circadian rhythm sleep disorder delayed sleep
disorder is associated with early
phase
H. Excessive daytime sleepiness onset of sleep and early morning
I. Kleine-Levin syndrome wakening.
J. Random eye movement sleep disorder.

Year: 2014

Old age psychiatry !

62. A!65TyearTold!lady!with!bipolar!disorder!and! The!answer!is!E.!


severe!functional!decline!presents!with!threeTday!
history!of!acute!agitation,!suicidal!ideation!and! !
refusing!to!eat!and!drink.!Her!MiniTMental!Status!
Examination!(MMSE)!score!was!29/30.!auditory! Explanation:!ECT!seems!to!be!the!
hallucinations.!She!has!not!responded!to!sodium!
best!treatment!because!she!is!
valproate!and!olanzapine.!!The!MOST!appropriate!
treatment!at!this!stage!is:!!! severely!depressed!with!psychotic!
F. Acetylcholinesterase!inhibitor! features!and!suicidal!ideation.!Her!
G. Augmentation!with!antidepressant! MMSE!is!close!to!full!score!and!
H. Augmentation!with!anxiolytics!

!
!
134
I. Change!valproate!to!lithium! potential!cognitive!impairment!
J. Electroconvulsive!therapy.! should!not!be!a!relative!
contraindication!for!ECT.!On!the!
other!hand,!her!renal!impairment!is!
a!contraindication!for!lithium.!

Year:!2014!

63. You!are!public!health!doctor!and!work!in!the! The!answer!is!C.!


Ministry!of!Health.!The!Ministry!wants!to!
identify!protective!factors!against!the! !
development!of!Alzheimers!disease.!Which!of!
the!following!is!least!associated!with!the! Explanation:!
development!of!Alzheimers!disease?!!!
F. Consuming!red!meat!on!a!frequent!basis! Risk!factors!for!Alzheimers!disease!
G. Female!gender! include!old!age,!family!history,!
H. High!education!level!! ApoE4!genotype,!female!gender,!
I. High!homocysteine!level! hypertension,!diabetes,!never!
J. Never!married.! married,!low!education!level,!head!
trauma,!high!homocysteine!levels.!

Protective!factors!include!high!
levels!of!education,!APOE2,!
consuming!fish,!antiN
inflammatories,!statins$,$
bilingualism.!

Year:!2014!

64. You! are! a! member! of! the! Hospital! Quality! The!answer!is!D.!


Improvement! Committee.! The! hospital! wants! to!
improve! the! management! of! delirium.! Which! of! !
the! following! statements! regarding! delirium! is!
FALSE?! Explanation:!The!typical!course!of!
! delirium!last!for!10N12!days.!It!is!too!
early!to!expect!the!majority!of!delirious!
F. Around!30%!of!the!elderly!admitted!to!the!medical!
patients!recover!by!Day!3.!
ward!will!develop!delirium!during!hospitalisation.!
G. Delirium!is!often!missed!or!overlooked!by!health!care!
!
professionals!in!the!elderly.!
H. Delirium!occurs!in!10%!of!children!undergoing!day!
Year:!2014!
surgery.!
I. Delirium!usually!clears!within!3!days!of!correcting!
!
underlying!medical!cause.!

!
!
135
J. The!incidence!of!delirium!increases!with!age.! !
!

65. You are a geriatric doctor. The manager of a The!answer!is!A.!


nursing home wants to consult you the behavior
which is most likely correlated with delusions in !
Alzheimers disease. Your answer is:
Explanation:!Patients!with!severe!
F. Aggression and agitation Alzheimers!disease!often!suffer!
G. Disrobing (i.e.state of wearing of no clothes) from!delusion!of!theft!and!this!
H. Shadowing correlates!with!aggression!and!
I. Suicide
J. Wandering.
agitation!to!family!members!and!
domestic!helpers.!

Year:!2014!

66. You! are! a! paediatric! resident.! You! are! The!answer!is!B.!


concerned! about! the! mental! health! of! a! 50T
yearTold! mother! whose! 12TyearTold! daughter! !
died! of! leukaemia.! You! are! not! certain!
whether! she! suffers! from! normal! grief! or! Explanation:!Recurrent!suicidal!
pathological! grief.! Normal! grief! is! NOT! thoughts!are!considered!to!be!
characterized!by:!! pathological!grief.!
!
!
F. Denial!
G. Recurrent!suicidal!thoughts!! Year:!2014!
H. Searching!for!the!deceased!
I. Sensations!of!seeing!or!hearing!the!deceased! !
J. Transient!guilt.!
!

67. You are a geriatric resident reviewing a 75-year- The!answer!is!B.!


old woman with Alzheimers disease. Her family
is keen for her to take acetylcholinesterase !
inhibitor. Which of the following medical
conditions is MOST contraindicated? Explanation:!Acetylcholinesterase!
inhibitors!increase!cholinergic!
F. Anaemia functions!and!may!cause!spasm!in!
G. Asthma smooth!muscle!or!bronchospasm.!
H. Diabetes
I. Hyperthyroidism
As!a!result,!it!may!worsen!asthma.!
J. Rheumatoid arthritis.
!

!
!
136
Year:!2014!

Child and adolescent psychiatry !

68. Which of the following is the COMMONEST The!answer!is!C.!!


cardiac complication in a 14-year-old
adolescent suffering from anorexia nervosa !
on admission?
Explanation:!Bradycardia!and!
hypotension!are!the!most!common!
F. Atrial fibrillation and cardiac arrhythmia.
cardiac!complications!when!
G. Bradycardia and hypertension
H. Bradycardia and hypotension patients!suffering!from!anorexia!
I. Tachycardia and hypotension nervosa!are!admitted!to!the!ward.!
J. Tachycardia and hypertension.
!

Year:!2014!

Substance abuse !

69. A! 30TyearTold! man! admitted! to! the! medical! The!answer!is!E.!


ward!has!no!access!to!the!substance!which!he!
used! prior! to! admission.! He! complains! of! !
nausea! and! muscle! pain.! Physical!
examination!reveals!pupillary!dilation.!Which! Explanation:!Opiate!withdrawal!
of! the! following! substances! is! MOST! likely! to! leads!to!mood!changes,!
cause!the!above!withdrawal!symptoms?! gastrointestinal!disturbances!(e.g.!
! nausea,!vomiting),!muscular!aches,!
F. Alcohol! lacrimation!and!pupillary!dilation.!
G. Cocaine!
H. Midazolam! !
I. Nictoine!
J. Opiate. Year:!2014!

70. A female student has returned from Amsterdam The!answer!is!C.!


after an exchange programme and admits using
cocaine for 1 year. She developed psychotic !
symptoms. Which of the following is the MOST
IMPORTANT risk factor for the development of Explanation:!The!risk!factors!for!
psychosis while using cocaine?
psychosis!in!cocaine!users!include:!
male!gender,!intravenous!users,!
F. An!elevated!body!mass!index! first!time!users,!longer!duration!and!
G. Combination!with!benzodiazepine! greater!amount!of!cocaine!use,!low!
H. Being!a!first!time!user!of!cocaine!

!
!
137
I. Being!female! BMI.!
J. NonNintravenous!use.!
!

Year:!2014!

This!is!an!advanced!level!question.!

71. A!30TyearTold!man!who!is!dependent!on!alcohol! The!answer!is!D.!


enquires!of!his!family!physician!if!disulfiram!will!
help!him!to!reduce!his!alcohol!intake!while!he!still! !
drinks!in!a!controlled!manner.!!Which!of!the!
following!recommendations!is!CORRECT?! Explanation:!!This!patient!does!not!
! express!motivation!to!quit!drinking!
F. Disulfiram!is!recommended!because!it!has!more! or!remain!abstinent!with!alcohol.!He!
evidence!than!other!medications!in!helping!
wants!to!reduce!alcohol!intake!by!
patients!to!quit!alcohol.!
G. Disulfiram!is!recommended!because!it!will! controlled!drinking.!It!is!a!
discourage!him!from!drinking!alcohol.! dangerous!situation!if!he!drinks!and!
H. Disulfiram!is!recommended!because!he!has! takes!disulfiram!at!the!same!time.!
motivation!to!reduce!his!drinking.!
I. Disulfiram!is!not!recommended!because!it!may! !
cause!severe!side!effects!when!it!mixes!with!
alcohol.! !
J. Disulfiram!is!not!recommended!because!it!is!
indicated!for!opioid!dependence.! Year:!2014!
!

Psychopharmacology !

72. A!50TyearTold!man!with!a!major!depressive! The!answer!is!B.!


disorder!did!not!respond!well!to!an!adequate!trial!
of!sertraline!200mg!(a!selective!serotonin! !
reuptake!inhibitor).!!Neither!did!he!fully!respond!
to!a!subsequent!8Tweek!trial!of!venlafaxine!150mg! Explanation:!Option!B!may!lead!to!
(serotonin!noradrenaline!reuptake!inhibitor).!!
serotonin!syndrome.!
Which!of!the!following!is!NOT!RECOMMENDED!for!
this!man?!!!
! !
F. Augment!SSRI!with!lithium!(a!mood!stabilizer)! Year:!2014.!
G. Augment!SSRI!with!phenelzine!(a!monoaxmine!
oxidase!inhibitor)!
H. Consider!electroconvulsive!therapy!
I. Consider!psychotherapy!
J. Increase!the!dose!of!venlafaxine.!!

The answer is B.

73. Bupoprion differs from SSRI in which of the

!
!
138
following ways?

Explanation: Bupoprion has a lower


K. Bupoprion has a higher incidence of sexual incidence of sexual side effects (not
side effects and a lower incidence of
working on serotonin receptors) and a
sedation and weight gain.
L. Bupoprion has a lower incidence of sexual lower incidence of sedation and
side effects and a lower incidence of weight gain (as an energetic
sedation and weight gain. antidepressant).
M. Bupoprion has a lower incidence of sexual
side effects and higher incidence of !
sedation and weight gain
N. Bupoprion has lower incidence of sexual Year:!2014.!
side effects and weight gain and a higher
incidence of sedation.
O. Bupoprion has a higher incidence of sexual
side effects and sedation and a lower
incidence of weight gain.
74. Besides treatment resistant schizophrenia, The!answer!is!E.!
another appropriate indication for clozapine is?
!
F. Severe agranulocytosis Explanation:!Severe!tardive!
G. Severe embolism
H. Severe hypersalivation dyskinesia!is!an!indication!for!
I. Severe metabolic syndrome clozapine.!Option!A,!B,!C!and!D!are!
J. Severe tardive dyskinesia. side!effects!associated!with!
.
clozapine.!

Year:!2014.!

75. A!50TyearTold!cardiac!patient!suffers!from! The!answer!is!C.!


depression!and!the!psychiatrist!prescribed!
fluoxetine.!Which!of!the!following!statements! !
about!fluoxetine!is!TRUE?!
! Explanation:!Fluoxetine!has!been!
reported!to!produce!bleeding!and!
F. !Fluoxetine!decreases!warfarin!levels.! bruising!in!some!individuals.!SSRIs!
G.!!!!!Fluoxetine!increases!metabolism!of!warfarin.! as!a!class!are!believed!to!inhibit!
H. !!!!!Fluoxetine!increases!bruising!while!patient!is!
platelet!aggregation,!which!may!
taking!
!!!!!!!!!!!!!!!!warfarin.! underlie!this!effect.!The!ability!of!
SSRIs!to!reduce!platelet!aggregation!
I. He!has!no!increased!risk!of!gastrointestinal! may!be!an!important!intervention!
bleeding!with!fluoxetine.! for!patients!with!occlusive!coronary!
J. !!!!He!should!not!change!to!paroxetine!because! and!cerebrovascular!artery!disease!
paroxetine!decreases!anticoagulant!effect!of!
and!deserves!study.!
warfarin.!
!
!

!
!
139
! In!addition,!warfarin!is!metabolized!
via!CYP!3A4!and!fluoxetine!inhibits!
CYP!3A4!and!increases!the!levels!of!
warfarin.!

Year:!2014.!

!
This!is!an!advanced!level!question.!

76. Which of the following hypnotic agents causes The!answer!is!E.!


the LEAST daytime sedation is:
!
F. Diphenhydramine Explanation:!The!hypnotic!drug!
G. Diazepam
H. Lorazepam with!the!least!half!life!have!least!day!
I. Zopiclone time!sedation.!
J. Zolpidem.
!

Diphenhydramine!(halfNlife):!9N12!
hours!

Diazepam:!20N100!hours!

Lorazepam:!9N16!hours!

Zopiclone:!6!hours!

Zolpidem!2N3!hours.!

Year:!2014.!

This!is!an!advanced!level!question.!

77. Which of the following medications has the most The!answer!is!C.!


convincing evidence for reducing suicidal
behavior in bipolar patients? !

Explanation:!Lithium!reduces!
F. Clozapine
G. Lamotrigine suicidal!ideation!in!bipolar!patients.!
H. Lithium
I. Olanzapine !
J. Sodium valproate.

!
!
140
Year:!2014.!

78. An otherwise healthy 50-year-old male patient The!answer!is!A.!


receiving clozapine 300mg at night for treatment
resistant schizophrenia suddenly dies. The !
MOST likely cause of death is?
Explanation:!Myocarditis!occur!in!1!
A. Myocarditis
in!1300!patients!and!causes!death!
B. Neuroleptic malignant syndrome by!loss!of!right!ventricular!function.!

C. Pneumonia Year:!2014.!

D. Serotonin syndrome !

E. Tardive dyskinesia.

79. A 28-year-old woman has a borderline The!answer!is!C.!


personality disorder, primarily manifested by an
inability to control her urges to hurt herself. She !
frequently cuts her wrists when frustrated.
Which of the following medications has been Explanation:!Fluoxetine!is!the!first!
found to be MOST helpful and safest in curbing
her self-harm behaviour? line!treatment!for!impulsivity,!mood!
lability!and!rejection!sensitivity!in!
borderline!personality!disorder.!
F. Aripiprazole
G. Clonazepam
!
H. Fluoxetine
I. Lithium
J. Olanzapine. Year:!2014.!

Psychotherapy !

80. A 30-year-old woman suffers from obsessive The!answer!is!B.!


compulsive disorder and washes her hand 50
times a day. Which of the following techniques !
is the MOST essential in the psychological
treatment of her compulsive hand washing Explanation:!Exposure!and!
behaviour?
response!prevention!is!most!
important!behavioural!treatment!
F. Cognitive restructuring for!OCD.!Systemic!desensitization!is!
G. Exposure and response prevention
H. Social skill training
indicated!for!phobia.!
I. Systemic desensitization
J. Thought stopping. !

!
!
141
Year:!2014.!

81. A!25TyearTold!woman!with!a!panic!disorder!seeks! The!answer!is!E.!


an! opinion! whether! pharmacotherapy! or!
psychotherapy! is! the! best! treatment! option! for! !
her.! Which! of! the! following! statements! is!
CORRECT? Explanation:!ShortNterm!
F. Adding alprazolam on p.r.n. basis will make
exposure therapy ineffective. benzodiazepine!is!useful!and!does!
G. Adding alprazolam in the first week will lead to not!cause!dependence.!!SSRI!is!
dependence and does not improve patients useful!to!treat!panic!disorder.!
outcome.
Relaxation!therapy!can!be!part!of!
H. Antidepressants such as selective serotonin
reuptake inhibitors are not useful to treat panic the!behaviour!therapy!of!CBT.!
disorder.
I. Relaxation therapy is more effective than cognitive !
behaviour therapy in treating panic disorder.
J. Psychotherapy such as cognitive behaviour therapy Psychotherapy!requires!more!
requires more motivation than taking medication. motivation!from!patient!as!patient!
needs!to!cooperate!with!
psychologist!and!practise!various!
behavioural!techniques!and!
challenge!cognitive!errors.!!It!
requires!12N16!weekly!or!
fortnightly!sessions.!

Year:!2014.!

!!

82. A!70TyearTold!man!lodged!a!complaint!against!the! The!answer!is!B.!


hospital.!!His!wife!died!suddenly!one!year!ago.!!She!
suffered!from!diabetes!and!had!frequent!episodes! !
hypoglycaemia.!!The!husband!gave!her!higher!than!
prescribed!dose!of!hypoglycaemic!agent!because! Explanation:!!
he!thought!this!could!control!her!diabetes.!!His!
children!disagreed!with!his!management!but!he!
Displacement!refers!to!transferring!
refused!to!listen.!!The!coroners!report!stated!that!
his!wife!died!of!hypoglycaemia.!!The!70TyearTold! the!emotional!response!to!a!
man!is!angry!with!the!endocrinologist!and!thinks! particular!person!which!carries!less!
that!it!was!his!fault!to!prescribe!the!wrong! emotional!risk.!!
hypoglycamic!agent!leading!to!his!wifes!death.!!He!
demanded!a!written!apology!from!the! !
endocrinologist.!!Which!of!the!following!defence!
mechanisms!is!demonstrated!by!this!man?
In!this!case,!the!endocrinologist!
F. Altruism
G. Displacement bear!less!emotional!risk!than!his!
H. Reaction formation children!who!are!angry!with!him.!He!
I. Splitting does!not!want!to!blame!himself.!He!
J. Undoing.
displaced!his!guilt!of!causing!his!

!
!
142
wife!death!to!the!endocrinologist.!

Year:!2014.!

This!is!an!advanced!level!question.!

83. For post-traumatic stress disorder, which of the The!answer!is!C.!


following interventions is LEAST helpful to
patients? !

Explanation:!!Group!debriefing!may!
F. Cognitive behaviour therapy
be!harmful!based!on!recent!
G. Eye movement desensitization and reprocessing
H. Immediate group debriefing research!findings.!
I. Relaxation training
J. Supportive psychotherapy !

Year:!2014.!

84. You are a resident in plastic surgery. A 24- The!answer!is!A.!


year-old woman believes her nose is
!
malformed and always checks her nose in the
mirror. She was seen by multiple doctors and Explanation:!The!current!guidelines!
they confirmed that her nose is normal. recommend!offering!cognitive!
Which of the following psychological behaviour!therapy!(including!
treatment is the treatment of choice for this exposure!response!prevention)!to!
patient? people!suffering!from!body!
dysmorphic!disorder.!Exposure!and!
F. Exposure and response prevention response!prevention!is!part!of!the!
G. Family therapy cognitive!behaviour!therapy.!
H. Problem solving therapy
I. Psychodynamic psychotherapy !
J. Supportive psychotherapy.
Year:!2014!

Ethics and laws !

85. A! 40TyearTold! woman! complained! of! headache,! The!answer!is!C!


backache,!nausea,!numbness!and!visual!problems.!!
She! consulted! a! neurologist! whom! she! trusted! to! !
be! a! good! doctor.! ! The! neurologist! diagnosed! the!
patient! suffering! from! migraine! and! somatisation! Explanation:!Fidiciary!duty!refers!to!
disorder.! ! When! the! patient! requested! for!

!
!
143
Magnetic! Resonance! Imaging! (MRI),! the! the!duty!that!a!doctor!(i.e.!
neurologist! claimed! that! the! patient! had! neurologist)!must!act!in!the!
exaggerated! her! somatic! symptoms! and! insisted!
that! the! MRI! scan! should! be! reserved! for! more! patients!best!interest.!The!
severe!patients.!!Six!months!later,!the!patient!died! neurologist!is!in!a!legal!contract!
of!brain!tumour.!!In!addition!to!negligence,!which! with!the!patient!to!provide!best!
of!the!following!ethical!principles!was!violated!by! care.!A!fiduciary!duty!exists!when!
the!neurologist?!
! the!late!patient!places!confidence!in!
F. Autonomy! the!doctor!and!relied!upon!him!to!
G. Confidentiality! exercise!his!expertise!in!diagnosing!
H. Fiduciary!duty! and!managing!her!illness.!
I. NonNmaleficience!
J. Justice.! In!this!case,!the!neurologist!failed!to!
offer!earlier!MRI!scan!despite!
patients!request.!!He!wrongly!
attributed!the!neurological!
symptoms!to!somatisation!disorder!
without!thorough!investigation.!

Year:!2014.!

This!is!an!advanced!level!question.!

86. 40-year-old man arrested for shoplifting is The answer is C.


referred for a psychiatric assessment after he
tells the judge he suffers from kleptomania.
Which of the following criteria is MOST
important in establishing the diagnosis of Explanation:
kleptomania?

F. Acts reus. Option Act reus refers to a criminal act


G. Patient informs you that he was diagnosed with which involves bodily movement
kleptomania by a private psychiatrist before but whether voluntary or involuntary. In
cannot provide further clinical details.
this case, it refers to the act of stealing
H. Patient informs you that he cannot control his
impulse to steal and he does not want to steal. and its presence suggests that he
I. Patient informs you that he has started stealing indeed committed shoplifting.
since Primary 6. Similarly, Option D and E suggest that
J. Past history of criminal record of stealing. he had mostly likely committed a
crime as he has tendency to steal and
is not a honest person.

Option B is a common reason


provided by patients who committed

!
!
144
shopkifting but we need further clinical
details to establish the diagnosis of
kleptomania.

Year:!2014.!

87. A medical student has applied for a 3-month The!answer!is!A.!


research elective and he follows a professor to
conduct research. The professor asks him to !
conduct a cross-sectional questionnaire study
to assess mood symptoms of cancer patients. Explanation:!Autonomy!refers!to!the!
He is expected to recruit 300 patients in 3
months. This project was approved by the obligation! of! a! doctor! to! respect! a!
ethics committee. He attempts to obtain consent patients! rights! to! make! his! or! her!
from an inpatient who has received own! choice! in! treatment! or!
chemotherapy to participate in this study.
Patient refuses because he feels very lethargic. participation! in! research! in!
Thirty minutes later, he asks the patient to re- accordance!with!his!or!her!beliefs!or!
consider participating in this study again. The preferences.!The!patient!has!refused!
patient lodges a complaint to the professor
to! participate! and! the! decision!
about this medical student. Which ethical
principles has the medical student violated? should!be!respected!without!further!
asking.!

F. Autonomy! !
G. Beneficence!
H. Confidentiality! Year:!2014.!
I. NonNmaleficence!
J. Justice.! !

Liaison and neuropsychiatry !

88. A 50-year-old man who is dependent on alcohol is The!answer!is!A.!


admitted for observation after a fall when intoxicated.
The ward team is concerned about complications !
associated with alcohol withdrawal. Which of the
following statements is INCORRECT? Explanation:!Delirium!tremens!
F. Delirium tremens usually occur 6-12 hours after
the last drink. usually!occur!3N4!days!after!the!last!
G. Tremors usually occur 6-12 hours after the last drink.!
drink.
H. Anxiety usually occurs 12-18 hours after the last !
drink.
I. Convulsion usually occurs 12-18 hours after the last Year:!2014.!
drink.
J. Sleep disturbance usually occurs 12-18 hours after !
the last drink.

!
!
145
!

!
89. You are a resident in general surgery. A 50-
year-old man was admitted last night after he The!answer!is!D.!
cut his head, arms and hands during a
suicide attempt. He is married with a 10- !
year-old child. He works in the field of Explanation:!This!patient!suffers!
informational technology. According to his from!hypochondriasis!or!illness!
wife, his elder brother died of gastric cancer anxiety!disorder!because!he!worries!
6 months ago. He believed that he is (not!firmly!believes!as!in!delusion)!
genetically vulnerable for gastric cancer. He that!he!suffers!from!gastric!cancer!
saw multiple gastroenterologists and gastric but!not!confirmed!by!multiple!
surgeons. He went through multiple investigations.!
endoscopies and biopsies with normal
findings. Despite normal findings, he still !
worries that he has undiagnosed gastric
There!are!not!enough!symptoms!to!
cancer. He argued with his wife tonight and
support!the!diagnosis!of!
wanted to commit suicide. What of the
schizophrenia!and!abnormal!grief.!
following is the MOST appropriate
psychiatric diagnosis? !

F. Acute stress reaction/marital problems Year:!2014.!


G. Depression/abnormal grief
H. Generalised anxiety disorder / phobia
I. Hypochondriasis/illness anxiety
J. Schizophrenia/psychosis.

90. You have become a consultant physician. A The!answer!is!C.!


resident called you and informed you that the
parameter of a 20-year-old male psychiatric !
patient over the phone, His temperature is
Explanation:!This!young!man!suffers!
41 degree Celsius. His heart rate is 105 per
minutes and blood pressure fluctuates from from!neuroleptic!malignant!
120/70 to 180/100. His breathing is normal syndrome!as!evidenced!by!high!
and chest is clear. His abdomen is soft. His fever,!muscle!rigidity,!tachycardia,!
creatinine kinase is 800 IU/L (normal <100 labile!blood!pressure,!leucocytosis!
IU/L) and leukocyte count is 10 x109/L and!laboratory!evidence!of!muscle!
(normal range: 1.5-5 x109/L). His limbs are injury!(i.e.!increase!in!creatinine!
very rigid and he demonstrates difficulty in kinase!levels)!after!intravenous!
swallowing. He was given intravenous antipsychotic!injection!three!days!
injection of haloperidol 20mg three days ago.!
ago. What is the MOST likely diagnosis?
!
F. Catatonia
G. Meningitis

!
!
146
H. Neuroleptic malignant syndrome !
I. Serotonin syndrome
J. Status epilepticus. Year:!2014!

91. You are a doctor working in the polyclinic. A The!answer!is!D.!


40-year-old man suffers from bipolar
disorder and he takes lithium on a daily !
basis. His blood pressure is 170/100mmHg.
Explanation:!Thiazide!diuretics,!ACE!
Which of the following medications is the
BEST to lower his blood pressure? inhibitors!and!NSAIDS!increase!
lithium!level!and!may!cause!lithium!
F. Angiotensin-converting-enzyme inhibitor toxicity.!
G. Alpha agonist
!
H. Benzodiazapine
I. Beta blocker Option!B!and!C!do!not!reduce!blood!
J. Thiazide diuretic.
pressure.!

The!best!answer!is!Option!D.!

Year!2014.!!

This!is!an!advanced!level!question!

92. In medically ill patients with psychiatric illness, The answer is C.


which pharmacological properties of a
psychotropic drug is MOST appropriate?

Explanation: Narrow therapeutic


F. Long half-life psychotropic drug.
G. Psychotropic drugs with active metabolites. index like lithium is dangerous to
H. Psychotropic drug with wide therapeutic index. medically ill patients.
I. Psychotropic drug which inhibits metabolic
enzymes.
J. Psychotropic drug which induces metabolic
enzymes.

Year 2014

!
!
147
!

Paper 6 - Questions

Psychopathology

1. A 50-year-old woman claims that unfamiliar people whom she met on the street were
her husband. She believes that her husband put on a disguise. What is the diagnosis?

A. Capgras syndrome
B. Charles de Bonnet syndrome
C. DeClerambaults syndrome
D. Fregoli syndrome
E. Ganser syndrome.

2. A 50-year-old woman hears her late mothers voice when she falls asleep. She cannot
recall the content of her mothers speech. This phenomenon is known as:

A. Auditory illusion
B. Hypnopompic hallucination
C. Hypnagogic hallucination
D. Pseudohallucination
E. Third-person auditory hallucination.

3. You are seeing a 70-year-old man who suffers from Parkinsons disease. He
demonstrates a dulled emotional tone and seems to be indifferent to jokes. His family
describes him as being emotionally detached. Which of the following terms BEST
describes his psychopathology?

A. Alexithymia
B. Alogia
C. Anhedonia
D. Apathy
E. Attentional deficit.

4. A 50-year-old Singaporean Chinese man firmly believes that he needs to flee from
Singapore to Russia for his personal safety. He has developed this belief after reading a
newspaper article about Edward Snowden who released national security materials of
the United States (US). This man firmly believes that the newspaper mentions his name
and describes about his role in the leak of US national secrets. Which of the following
terms BEST describe his experience?

A. Delusion of erotomania
B. Delusion of grandiosity

!
!
148
C. Delusion of guilt
D. Delusion of jealousy
E. Delusion of reference.

Cognitive assessment

5. You are administering the Mini-Mental State Examination (MMSE) to a 70-year-old


woman who did not receive any formal education. She has difficulty to complete the full
version of MMSE. Which of the following tasks is MOST useful to assess her attention?

A. Constructing a double pentagon


B. Naming the months from December in backward direction
C. Orientation to time and place
D. Three-item registration and recall
E. Three-stage command.

6. The Montreal Cognitive Assessment (MOCA) is different from the Mini-Mental State
Examination. Which of the following neuroanatomical structures is specifically
assessed in the MOCA but not the MMSE?

A. Frontal lobe
B. Hippocampus
C. Occipital lobe
D. Parietal lobe
E. Temporal lobe.

Psychiatric epidemiology / Aetiology

7. After 65 years of age, the rate of Alzheimers disease changes approximately by how
many times every 5 years increase in age?
A. 0.5 time
B. 1 time
C. 2 times
D. 3 times
E. 4 times.

8. Lesions in which of the following neuroanatomical areas is the MOST vulnerable to


amnestic syndrome?

A. Basal ganglia
B. Cerebellum
C. Hippocampus
D. Nucleus accumbens
E. Raphe nucleus.

!
!
149
General adult psychiatry / Psychiatric diagnosis

9. Which of the following psychiatric diagnoses is MOST likely to contribute to inpatient


suicide?

A. Borderline personality disorder


B. Major depressive disorder
C. Generalised anxiety disorder
D. Hypochondriasis/ illness anxiety disorder
E. Schizophrenia.

10. Which of the following features is MOST characteristic of Cotards syndrome?

A. Delusional belief that familiar people have been replaced by their imposters
B. Delusional belief that the spouse is unfaithful
C. Delusional belief that strangers have taken on the psychological identity of a familiar person
D. Delusional belief that a person of higher status falls in love with the patient
E. Nihilistic and hypochondriacal delusions found in psychotic depression.

11. A 23-year-old man, recently promoted to a middle management position, presents to


your clinic with complaints of acute episodes of anxiety associated with heart
palpitations, dyspnoea, shaking, detachment from his body, and fear of losing control.
These experiences only occur during oral presentation in front of the upper
management. Which of the following diagnoses is MOST likely?

A. Generalized anxiety disorder


B. Panic disorder
C. Panic disorder with agoraphobia
D. Post-traumatic stress disorder
E. Social phobia.

12. A 40-year-old woman, Ms. Tan has witnessed a road traffic accident. Four hours ago,
an elderly man was knocked down by a taxi and suffered from severe head injury. Ms.
Tan was walking along the street and witnessed the whole accident. She did not suffer
from any injury. The police officer wants to interview Ms. Tan to obtain more
information about the accident. She cannot recall any details related to the accident.
Which of the following is the MOST appropriate explanation for her amnesia?

A. Acute and transient psychosis


B. Conversion disorder
C. Dissociative state
D. Factitious disorder
E. Underlying dementia.

13. You are a medical officer working in the Army Camp. A 19-year-old man is recently
enlisted for National Service and his parents inform you that he walks around his flat
during sleep. Which of the following recommendations is LEAST appropriate?

!
!
150
A. Safety measures
B. Sleep hygiene
C. Stay out of camp at night
D. Stimulant
E. Supportive psychotherapy.

14. A 25-year-old man believes that he suffers from adult ADHD. Which of the following
criteria is compulsory to establish such diagnosis?

A. Absence of inattention symptoms in adulthood


B. History of ADHD symptoms before the age of 12
C. History of conduct disorder
D. Presence of hyperactivity symptoms in adulthood
E. History of substance abuse.

15. All of the following findings are found in anorexia nervosa EXCEPT:

A. Decreased bone mineral density


B. Decreased growth hormone
C. Decreased gonadotropins
D. Decreased potassium
E. Decreased heart rate.

Old age psychiatry

16. You are a family doctor and a 70-year-old man consults you for annual medical
assessment to certify that he is fit to drive. Which of the following is LEAST relevant to
driving?

A. Assess his visual acuity by Snellens Chart


B. Assess his hearing by whispering to his ears
C. Inquire his driving practice and road safety records
D. Examine his elbow and knee joints
E. Measure his blood pressure.

17. Which of the following is TRUE about suicide in old people?


A. Old people with terminal illness have low suicide risk because they foresee that they will die
soon.
B. Old people often attempt suicide as a means to manipulate other people.
C. Old people frequently communicate their intent prior to committing suicide.
D. Old people generally attempt self-harm rather than suicide.
E. Suicide attempts are more lethal in old people as compared to young people.

18. Which of the following statements is FALSE about delirium in old people?

A. Could be an initial presentation of underlying dementia

!
!
151
B. Onset could be insidious
C. Rarely results in full resolution of symptoms in a short period of time
D. Un-medicated old people are at higher risk to develop delirium as compared to old people
treated by multiple medications.
E. Usually persists for weeks or months in old people hospitalized for medical or surgical
reasons.

19. The daughter of an 80-year-old woman with history of bipolar disorder calls you for
advice. Her mother assaulted one helper and the other helper is terrified. They tried to
stop her from leaving the house to buy things. Which of the following is MOST
appropriate management strategy?

A. Admission to the psychiatry ward


B. Employ a part-time nurse to look after her at home
C. Increase the dose of antipsychotics and review in 1 week
D. Increase the dose of benzodiazepine and review in1 week
E. Increase the dose of mood stabilizer and review in 1 week.

Child and adolescent psychiatry

20. A 4-year-old boys grandmother died four months ago. The grandmother was one of the
childs primary caregivers. The child still speaks to the grandmother. When he walks, he
still holds her hands up as though holding hands with his grandmother. Which of the
following is CORRECT?

A. This child fully understands the concept of death.


B. This child suffers from major depression.
C. This child is psychotic.
D. The child suffers from adjustment disorder with depressed mood.
E. This reaction is within normal limits for a child of this age.

21. Which of the following is the MOST established psychopharmacological treatment for
children and adolescents suffering from autism?

A. Actetylcholinesterase inhibitor
B. Antipsychotic drug
C. Benzodiazepine
D. Omega-3 supplement
E. Stimulant.

Substance abuse

22. You are a resident working at the Accident and Emergency Department. A 21-year-old
man is brought in by police and he is disorientated to time, place and person. He needs
to urinate very often and seems to have urinary incontinence. A package of capsules is
found in his pocket and suspected to be illicit drugs. Which of the following drugs is
MOST likely to cause the above symptoms?

!
!
152
A. Amphetamine
B. Cannabis
C. Cocaine
D. Ketamine
E. Phencyclidine.

23. Hypertension associated with cocaine misuse is BEST treated by:

A. AngiotensinNconvertingNenzyme!inhibitors
B. Beta-blockers
C. Benzodiazapine
D. Loop diuretics
E. Thiazide diuretics.

24. Which of the following addiction has the HIGHEST suicide risk?

A, Gambling

B. Internet

C. Sex

D. Steroid addiction in young athlete

E. Stimulant.

25. You are a general practitioner (GP). A 20-year-old university student wants you to prescribe
methylphenidate (ritalin). The student believes that ritalin can enhance his examination
performance and he has been obtaining Ritalin prescribed from another GP. He does not
have history of attention deficit and hyperactivity disorder (ADHD). Which of the following
advices is MOST APPROPRIATE?

A. Advise him to take methylphenidate on the days of examination and stops immediately after
the examination.
B. Advise him to stop taking methylphenidate because it is a controlled drug and not indicated in
his situation.
C. Methylphenidate can enhance his attention if he has family history of ADHD.
D. Methylphenidate can enhance his attention if he has past history of conduct disorder.
E. There is evidence to suggest that methylphenidate can enhance his attention and improve his
examination performance.

Psychopharmacology

26. A 30-year-old man suffers from autism and epilepsy. He has been very aggressive. The
psychiatrist prescribes sodium valproate 1300mg nocte to control his aggression and
epilepsy. His mother concludes the sodium valproate is not effective. The first step of

!
!
153
management should be:

A. Check the serum sodium valproate level


B. Increase the dose of sodium valproate
C. Increase the dose of sodium valproate and add an antipsychotic drug
D. Refer the patient to see a psychologist
E. Switch to another antipsychotic drug.
27. Which of the following side effects is more common in tadalafil as compared to other
phosphodiesterase type 5 inhibitors such as vardenafil and sildenafil?

A. Dyspepsia
B. Flushing
C. Headache
D. Impaired vision
E. Priapism.
28. A 30-year-old man suffers from schizophrenia. He presents a list of drugs which he took
in the past. Which of the following is NOT a second generation antipsychotic drug?
a. Clozapine
b. Flupenthixol
c. Olanzapine
d. Quetiapine
e. Risperidone.

29. A 30-year-old man suffers from insomnia and he sleeps well after taking diazepam. He
worries that he lacks a particular neurotransmitter in the brain and diazepam can
enhance the actions of this neurotransmitter. Which of the following neurotransmitters
is MOST relevant In this case?

A. Acetylcholine
B. Dopamine
C. -Aminobutyric acid
D. Noradrenaline
E. Serotonin.
30. Mrs. Tan is a 65-year-old woman who lost her husband three years ago. She is still
angry with him for not taking better care of his health. He was cremated and she keeps
his ashes in the urn. She is unsure what to do with the ashes. She coped well after her
husbands death, until her son moved to Australia a few months ago. She has long
standing headaches, but these have become worse over the past six months. She also
has new onset of somatic symptoms as well as anxiety symptoms which are
overwhelming and include a fear of dying. Her depression has been partially treated
with fluoxetine 40mg per day, but her anxiety symptoms persist and she feels restless.
Which of the following actions is LEAST appropriate?

A. Add alprazolam to her treatment for 2 weeks


B. Discontinue fluoxetine and start other antidepressant drug such as paroxetine
C. Increase the dose of fluoxetine to 80mg per day
D. Order a thyroid function test
E. Start psychotherapy with a focus on grief.

!
!
154
31. Which of the following antidepressants cause MOST SEDATION after potentiation with
benzodiazepine?

A. Agomelatine
B. Escitalopram
C. Fluvoxamine
D. Mirtazapine
E. Venlafaxine.
32. Which of the following factors limits the use of clozapine in treatment-resistant
schizophrenia?

A. A low incidence of seizures


B. The lack of efficacy against positive symptoms
C. The lack of efficacy against negative symptoms
D. The lack of efficacy against cognitive symptoms
E. The occurrence of agranulocytosis.

33. Which of the following medications is MOST likely to exacerbate psoriasis?

A. Amitriptyline
B. Haloperidol
C. Lithium
D. Risperidone
E. Sodium valproate.

34. Which of the following is CORRECT regarding repetitive transcranial magnetic


stimulation (rTMS)?

A. rTMS is administered under general anaesthesia.


B. rTMS is more efficacious than electroconvulsive therapy (ECT).
C. rTMS induces seizures with less energy as compared to ECT.
D. rTMS is a new imaging modality in psychiatry.
E. rTMS may be useful in the treatment of depression.

35. The MOST robust effect of lamotrigine is found in which of the following?

a. Bipolar depression

b. Manic episode

c. Mixed episodes

d. Hypomanic episode

e. Cyclothymia.

!
!
155
36. A 30-year-old woman complains of lack of sexual drive after taking paroxetine. She
wants to change her antidepressant. Which of the following antidepressant is the BEST
option to avoid sexual dysfunction?

A. Amitriptyline
B. Reboxetine
C. Mirtazapine
D. Duloxetine
E. Venlafaxine.
37. Agomelatine,!a!new!antidepressant!!is!BEST!given!in!which!of!the!following!parts!of!
the!day?
!

A. Early morning
B. Late morning
C. Early afternoon
D. Early evening
E. Past mid-night.

Psychotherapy!

38. A! 24TyearTold! depressed! woman! said,! People! in! my! office! try! to! make! my! life!
difficult! and! then! deem! themselves! as! ultimate! smart! to! know! about! me.! I! have!
resigned! as! I! am! uncomfortable! with! the! office! settings.! I! am! aware! that! I! am!
getting! verbally! aggressive! towards! other! people! whenever! they! provoke! me.! I!
want! to! see! a! psychotherapist! because! I! want! to! deal! with! these! people! before! I!
lose! my! selfTidentity.! Which! of! the! following! psychotherapies! is! MOST!
appropriate?!
!!

A. Cognitive!behaviour!therapy!
B. Grief!therapy!
C. Interpersonal!therapy!
D. Relaxation!exercise!
E. Supportive!psychotherapy!
39. You are a family doctor. A 30-year-old woman suffers from schizophrenia. Although she
is free from first rank symptoms, she is affected by weight gain, negative symptoms of
schizophrenia and no interest in sex. Her husband has high sexual drive and he is not
happy with their sexual life. He requests for anti-androgen to reduce his sexual drive.
What is BEST management strategy? !

A. Cognitive behaviour therapy


B. Eye movement desensitization and processing
C. Hypnotic therapy
D. Prescribe anti-androgen
E. Sensate focus therapy.
!

40. A 40-year-old man suffers from post-traumatic stress disorder (PTSD) and has history
of misusing cannabis. He claims that the government should legalise the sales of
cannabis because cannabis can reduce the PTSD symptoms and helps him to relax.

!
!
156
What is his defence mechanism?

A. Denial
B. Projection
C. Rationalisation
D. Repression
E. Sublimation.

Ethics and laws

41. The Mental Health (Care and Treatment) Act in Singapore can be applied at which of the
following hospital(s) for involuntary admission?

a. Changi General Hospital (CGH) only


b. Institute of Mental Health (IMH) only
c. National University Hospital (NUH) only
d. IMH, NUH and CGH
e. IMH, NUH, CGH, Singapore General Hospital (SGH) and Tan Tock Seng Hospital
(TTSH).

42. A 21-year-old woman suffering from body dysmorphic disorder consulted a doctor for
Botulinum toxin (botox) treatment. She was concerned of the smoothness of her face
and spent $10,000 for multiple botox injection. The botox injection caused paralysis of
facial muscles. Botox injection is not indicated for women younger than 30 years and
body dysmorphic disorder. Which of the following ethical principles was violated by
this doctor?

K. Autonomy!
L. Capacity!
M. Confidentiality!
N. NonNmaleficence!
O. Justice.!
43. A 25-year-old man was arrested for murder. According to the police, he shows no
remorse towards his act. He firmly believes that killing one person will not lead to death
sentence. He needs to kill several people to get death sentence. He always carries a
chopper and wants to attack innocent people. If he is released from the prison, he will
kill more people, including his parents. Which of the following diagnoses is MOST
appropriate?

A. Biopathy
B. Egopathy
C. Psychopathy
D. Sociopathy
E. Superegopathy.
Consultation liaison psychiatry

44. You are a resident working in the Children Emergency Department. A 5-year-old boy is
presented for recurrent urinary tract infection and confirmed by raised white blood cell
counts in urine. According to his mother, the boy likes to insert foreign body into his
penis via the urethra meatus. After taking a thorough history and conducting physical

!
!
157
examination, you cannot gather any evidence the boy could insert foreign body into his
penis. It is not anatomically and physiologically feasible for the boy to do so. You
cannot exclude the possibility that it could have been done by his mother. History also
reveals that the parents have marital problems and his father is suspected to visit
commercial sex workers frequently. She is very keen to admit her son for further
invasive investigations. What is the MOST likely diagnosis?

A. Conversion disorder induced by the mother


B. Hypochondriasis (or illness anxiety) induced by the mother
C. Malingering induced by the mother
D. Mnchausen syndrome by proxy induced by the mother
E. Somatisation disorder induced by the mother.
45. Which!of!the!following!questionnaires!would!be!MOST!useful!in!identifying!
symptoms!of!depression!and!anxiety!in!patients!suffering!from!chronic!medical!
illness?!!!
!

A. The!Beck!Anxiety!and!Depression!Inventory!
B. The!General!Health!Questionnaire!
C. The!Hamilton!Anxiety!and!Depression!Rating!Scale!
D. The!Hospital!Anxiety!and!Depression!Scale!
E. The!StateNTrait!Anxiety!Inventory.!

46. You are a resident working in the Accident and Emergency Department (AED). A 26-
year-old woman gave birth one month ago. She is a single mother and broke up with
her boyfriend. She has poor social support and no one helps her to look after the baby.
She has been depressed for 1 month and wanted to jump down from her flat two weeks
ago. She was referred to the womans mental health service. She was referred to a case
manager but no medication was prescribed. She breastfeeds her baby. Today, she
wants to jump again. The mother and her baby are bought to the AED by the case
manager. Which of the following is the BEST management?

A. Admit the mother and her baby to the hospital. The mother will go to the psychiatric ward and
the baby will go to paediatric ward. She will benefit from antidepressant and psychological
intervention.
B. Admit the mother and her baby to the hospital. The mother will go to the psychiatric ward and
the baby will go to paediatric ward. She should receive psychological intervention only
because antidepressants are dangerous for breastfeeding.
C. Discharge the mother and her baby from the AED because she suffers from adjustment
disorder. Her risk is low.
D. Discharge the mother and her baby from the AED because she receives good care from case
manager.
E. Send both mother and her baby to the Institute of Mental Health for compulsory admission.
47. What of the following psychiatric side effects is MOST common in Hepatitis C patients
receiving interferon treatment?

A. Anxiety
B. Cognitive impairment
C. Confusion
D. Depression
E. Hallucination.

!
!
158
48. The hospital wants to develop the best method to detect and identify delirium in
medically ill patients. Which of the following is the BEST strategy to detect and identify
delirium?

A. Administer Delirium Rating Scale to all patients older than 65 years to screen for delirium.
B. Looking for abnormalities on computerized topography (CT) brain scans for all patients.
C. Looking for abnormalities on magnetic resonance imaging (MRI) brain scans for suspected
patients.
D. Monitor signs of delirium on those patients with laboratory abnormalities.
E. Screen for history of delirium and monitor those patients with history of delirium only.
!

Rotation 2 - 2014

Psychopathology

7. A 50-year-old woman claims that unfamiliar The answer is D.


people whom she met on the street were her
husband. She believes that her husband put
on a disguise. What is the diagnosis? Explanation: Fregoli syndrome is a
delusional mis-indentification syndrome
when unfamiliar people are identified as
F. Capgras syndrome
G. Charles de Bonnet syndrome a familiar person.
H. DeClerambaults syndrome
I. Fregoli syndrome
J. Ganser syndrome.
Year: 2014

8. A 50-year-old woman hears her late mothers The answer is C.


voice when she falls asleep. She cannot recall
the content of her mothers speech. This
phenomenon is known as:
Explanation: Hypnogogic hallucination
occurs when a person falls asleep and
F. Auditory illusion conscious level fluctuates considerably in
G. Hypnopompic hallucination
different stages of sleep.
H. Hypnagogic hallucination
I. Pseudohallucination
J. Third-person auditory hallucination.
Year: 2014.

9. You are seeing a 70-year-old man who suffers The answer is D.


from Parkinsons disease. He demonstrates a
dulled emotional tone and seems to be
indifferent to jokes. His family describes him
as being emotionally detached. Which of the Explanation: Apathy refers to detachment
following terms BEST describes his from feeling or a dulled emotional tone.
psychopathology? The person conveys a sense of
indolence and indifference to what
F. Alexithymia normally causes excitement or interest.

!
!
159
G. Alogia Detachment is one of the aspects of
H. Anhedonia apathy.
I. Apathy
J. Attentional deficit.

Anhedonia is defined as the inability to


experience pleasure from activities
usually found enjoyable which is less
evident in this case.

Year: 2014.

10. A 50-year-old Singaporean Chinese man The answer is E.


firmly believes that he needs to flee from
Singapore to Russia for his personal safety.
He has developed this belief after reading a
newspaper article about Edward Snowden Explanation: This man firmly believes
who released national security materials of that the newspaper article referred to him
the United States (US). This man firmly and this is impossible because the
believes that the newspaper mentions his
person involved is a Caucasian with US
name and describes about his role in the leak
of US national secrets. Which of the following citizenship. This is known as delusion of
terms BEST describe his experience? reference.

F. Delusion of erotomania
G. Delusion of grandiosity Year: 2014.
H. Delusion of guilt
I. Delusion of jealousy
J. Delusion of reference.

Cognitive assessment

11. You are administering the Mini-Mental State The answer is B.


Examination (MMSE) to a 70-year-old woman
who did not receive any formal education. She
has difficulty to complete the full version of
MMSE. Which of the following tasks is MOST Explanation: Option B is the most useful
useful to assess her attention? task to assess her attention because she
may not be able to perform serial 7 due
to low level of education. Option A
assesses constructional apraxia. Option
F. Constructing a double pentagon C assesses orientation. Option D
G. Naming the months from December in backward assesses registration and short term
direction memory. Option E assess working
H. Orientation to time and place memory.
I. Three-item registration and recall
J. Three-stage command.

Year: 2014.

!
!
160
12. The Montreal Cognitive Assessment (MOCA) The answer is A.
is different from the Mini-Mental State
Examination. Which of the following
neuroanatomical structures is specifically
assessed in the MOCA but not the MMSE? Explanation: MOCA tests the frontal lobe
function by the trail making task. This
task assesses alternating sequencing
F. Frontal lobe
(number and alphabet) and rule out
G. Hippocampus
H. Occipital lobe perservation.
I. Parietal lobe
J. Temporal lobe.
Year: 2014.

This is an advanced-level question.

Psychiatric epidemiology / Aetiology

49. After 65 years of age, the rate of Alzheimers The answer is C.


disease changes approximately by how many
times every 5 years increase in age?
F. 0.5 time
G. 1 time Explanation: the rate of Alzheimers
H. 2 times disease doubles approximately every 5
I. 3 times years of age.
J. 4 times.

Year: 2014

50. Lesions in which of the following The answer is C.


neuroanatomical areas is the MOST
vulnerable to amnestic syndrome?

Explanation: Damage to hippocampus is


F. Basal ganglia most vulnerable to amnestic syndrome.
G. Cerebellum
H. Hippocampus
I. Nucleus accumbens
J. Raphe nucleus.

Year: 2014

General adult psychiatry / Psychiatric diagnosis

51. Which of the following psychiatric diagnoses The answer is B.


is MOST likely to contribute to inpatient
suicide?

Explanation: Major depressive disorder is


F. Borderline personality disorder most likely to contribute to inpatient

!
!
161
G. Major depressive disorder suicide among all the options.
H. Generalised anxiety disorder
I. Hypochondriasis/ illness anxiety disorder
J. Schizophrenia.
Year: 2014.

52. Which of the following features is MOST The answer is E.


characteristic of Cotards syndrome?

F. Delusional belief that familiar people have been Explanation: Cotards syndrome is a
replaced by their imposters nihilistic delusion in which a person
G. Delusional belief that the spouse is unfaithful believes that their possessions, friends,
H. Delusional belief that strangers have taken on the
or parts of their own body do not exist or
psychological identity of a familiar person
I. Delusional belief that a person of higher status are about to not exist.
falls in love with the patient
J. Nihilistic and hypochondriacal delusions found in
psychotic depression.
Year: 2014.

53. A 23-year-old man, recently promoted to a The answer is E.


middle management position, presents to
your clinic with complaints of acute episodes
of anxiety associated with heart palpitations,
dyspnoea, shaking, detachment from his Explanation: The diagnosis is social
body, and fear of losing control. These phobia but not panic disorder because he
experiences only occur during oral demonstrates those symptoms during the
presentation in front of the upper
presentation in front of the management.
management. Which of the following
diagnoses is MOST likely?

Year: 2014
F. Generalized anxiety disorder
G. Panic disorder
H. Panic disorder with agoraphobia
I. Post-traumatic stress disorder
J. Social phobia.

54. A 40-year-old woman, Ms. Tan has witnessed The answer is C.


a road traffic accident. Four hours ago, an
elderly man was knocked down by a taxi and
suffered from severe head injury. Ms. Tan was
walking along the street and witnessed the Explanation: This woman suffers from
whole accident. She did not suffer from any acute stress reaction after witnessing the
injury. The police officer wants to interview road traffic accident. She develops
Ms. Tan to obtain more information about the
dissociative amnesia and not able to
accident. She cannot recall any details related
to the accident. Which of the following is the provide details related to the accident.
MOST appropriate explanation for her
amnesia?

!
!
162
Year: 2014

F. Acute and transient psychosis


G. Conversion disorder
H. Dissociative state
I. Factitious disorder
J. Underlying dementia.

55. You are a medical officer working in the Army The answer is D.
Camp. A 19-year-old man is recently enlisted
for National Service and his parents inform
you that he walks around his flat during sleep.
Which of the following recommendations is Explanation: Stimulant treatment (e.g.
LEAST appropriate? modanfinil) is used to treat narcolepsy
but not sleep-walking.
F. Safety measures
G. Sleep hygiene
H. Stay out of camp at night Year: 2014
I. Stimulant
J. Supportive psychotherapy.

56. A 25-year-old man believes that he suffers The answer is B.


from adult ADHD. Which of the following
criteria is compulsory to establish such
diagnosis? Explanation: History of ADHD symptoms
in childhood is a compulsory diagnostic
F. Absence of inattention symptoms in adulthood criteria for diagnosing adult ADHD
G. History of ADHD symptoms before the age of 12
symptoms.
H. History of conduct disorder
I. Presence of hyperactivity symptoms in adulthood
J. History of substance abuse.
Year: 2014

57. All of the following findings are found in The answer is B.


anorexia nervosa EXCEPT:

F. Decreased bone mineral density


G. Decreased growth hormone Explanation: Growth hormone is
H. Decreased gonadotropins increased but not reduced in anorexia
I. Decreased potassium nervosa.
J. Decreased heart rate.

Year: 2014

Old age psychiatry

58. You are a family doctor and a 70-year-old man The answer is E.
consults you for annual medical assessment
to certify that he is fit to drive. Which of the
following is LEAST relevant to driving?
Explanation: Option A, B, C and D

!
!
163
assess driving and body systems that are
directly involved in driving. Blood
F. Assess his visual acuity by Snellens Chart pressure is not directly involved in driving
G. Assess his hearing by whispering to his ears
and less likely to influence driving
H. Inquire his driving practice and road safety
records performance.
I. Examine his elbow and knee joints
J. Measure his blood pressure.
Year: 2014

59. Which of the following is TRUE about suicide The answer is E.


in old people?
F. Old people with terminal illness have low suicide
risk because they foresee that they will die soon.
G. Old people often attempt suicide as a means to Explanation: Suicide in elderly is
manipulate other people. common and they are determined to die.
H. Old people frequently communicate their intent Doctors should take this seriously and
prior to committing suicide.
advise for hospitalization.
I. Old people generally attempt self-harm rather
than suicide.
J. Suicide attempts are more lethal in old people as
compared to young people.
Year: 2014

60. Which of the following statements is FALSE The answer is D.


about delirium in old people?

F. Could be an initial presentation of underlying Explanation: Old people who take


dementia multiple medications (or polypharmacy)
G. Onset could be insidious are at higher risk to develop delirium.
H. Rarely results in full resolution of symptoms in a
short period of time
I. Un-medicated old people are at higher risk to
develop delirium as compared to old people Year: 2014
treated by multiple medications.
J. Usually persists for weeks or months in old
people hospitalized for medical or surgical
reasons.

61. The daughter of an 80-year-old woman with The answer is A.


history of bipolar disorder calls you for
advice. Her mother assaulted one helper and
the other helper is terrified. They tried to stop Explanation: As part of the risk
her from leaving the house to buy things. management, this elderly lady should be
Which of the following is MOST appropriate
admitted to the psychiatry ward for
management strategy?
assessment. Besides further assessment
of bipolar disorder, the doctors need to
F. Admission to the psychiatry ward assess other medical causes of violent
G. Employ a part-time nurse to look after her at behaviour (e.g. delirium or alcohol
home withdrawal)
H. Increase the dose of antipsychotics and review in
1 week

!
!
164
I. Increase the dose of benzodiazepine and review
in1 week
J. Increase the dose of mood stabilizer and review Year: 2014
in 1 week.

Child and adolescent psychiatry

62. A 4-year-old boys grandmother died four The answer is E.


months ago. The grandmother was one of the
childs primary caregivers. The child still
speaks to the grandmother. When he walks,
he still holds her hands up as though holding Explanation: This is a normal grief
hands with his grandmother. Which of the reaction. Child at this age may not
following is CORRECT? understand the concept of death. He may
believe that his grandmother will return
F. This child fully understands the concept of death. one day.
G. This child suffers from major depression.
H. This child is psychotic.
I. The child suffers from adjustment disorder with
depressed mood. Year: 2014
J. This reaction is within normal limits for a child of
this age.
This is an advanced-level question

63. Which of the following is the MOST The answer is B.


established psychopharmacological treatment
for children and adolescents suffering from
autism?
Explanation: Antipsychotic drug e.g.
risperidone is the most established
F. Actetylcholinesterase inhibitor psychopharmacological drug to treat
G. Antipsychotic drug
young people with autism.
H. Benzodiazepine
I. Omega-3 supplement
J. Stimulant.
Year: 2014.

Substance abuse

64. You are a resident working at the Accident The answer is D.


and Emergency Department. A 21-year-old
man is brought in by police and he is
disorientated to time, place and person. He
needs to urinate very often and seems to have Explanation: Ketamine-induced
urinary incontinence. A package of capsules ulcerative cystitis which includes urge
is found in his pocket and suspected to be incontinence, decreased bladder
illicit drugs. Which of the following drugs is
compliance, decreased bladder volume,
MOST likely to cause the above symptoms?
detrusor overactivity, and painful
haematuria.
F. Amphetamine
G. Cannabis
H. Cocaine

!
!
165
I. Ketamine Year: 2014.
J. Phencyclidine.

65. Hypertension associated with cocaine misuse The answer is C.


is BEST treated by:

F. AngiotensinNconvertingNenzyme!inhibitors Explanation: Benzodiazapine is the


G. Beta-blockers safest and most appropriate option here.
H. Benzodiazapine The other option is alpha blocker which is
I. Loop diuretics not available. Beta-blockers will worsen
J. Thiazide diuretics.
hypertension by leaving alpha receptors
unopposed. As cocaine increases
noradrenaline levels, diuretics are less
relevant.

Year: 2014.

This is an advanced level question.

66. Which of the following addiction has the The answer is A.


HIGHEST suicide risk?

A, Gambling Explanation: The risk for suicide in


gambling is twice the risk compared to
B. Internet other forms of addiction as a result of
huge financial consequences and
C. Sex
unsettled debts. Other tell-tales signs of
D. Steroid addiction in young athlete suicide amog gamblers include insomnia,
poor appetite loss of interest to work and
E. Stimulant. somatic complaints.

Year: 2014.

67. You are a general practitioner (GP). A 20-year-old The answer is B.


university student wants you to prescribe
methylphenidate (ritalin). The student believes
that ritalin can enhance his examination
performance and he has been obtaining Ritalin Explanation: In Singapore, patients are
prescribed from another GP. He does not have more likely to misuse prescribed
history of attention deficit and hyperactivity medication than illicit substances due to
disorder (ADHD). Which of the following advices

!
!
166
is MOST APPROPRIATE? tight control of illicit drugs. This student
may have misused methylphenidate as
F. Advise him to take methylphenidate on the days stimulants and see different doctors to
of examination and stops immediately after the obtain methylphenidate. He is
examination.
recommended to stop taking
G. Advise him to stop taking methylphenidate
because it is a controlled drug and not indicated methylphenidate and there is no
in his situation. evidence to suggest that this will help his
H. Methylphenidate can enhance his attention if he examination performance.
has family history of ADHD.
I. Methylphenidate can enhance his attention if he
has past history of conduct disorder.
J. There is evidence to suggest that Year: 2014.
methylphenidate can enhance his attention and
improve his examination performance.

Psychopharmacology

68. A 30-year-old man suffers from autism and The answer is A.


epilepsy. He has been very aggressive. The
psychiatrist prescribes sodium valproate
1300mg nocte to control his aggression and
epilepsy. His mother concludes the sodium Explanation: The patient is on a relatively
valproate is not effective. The first step of high dose of sodium valproate. The first
management should be: step should check serum level of sodium
valproate because patient may not non-
F. Check the serum sodium valproate level
compliant to the medication.
G. Increase the dose of sodium valproate
H. Increase the dose of sodium valproate and add
an antipsychotic drug
I. Refer the patient to see a psychologist
Year: 2014.
J. Switch to another antipsychotic drug.

This is an advanced level question.

69. Which of the following side effects is more The answer is E.


common in tadalafil as compared to other
phosphodiesterase type 5 inhibitors such as
vardenafil and sildenafil?
Explanation: Tadalafil has the longest
half-life, 17.5 hours as compared to
F. Dyspepsia vardenafil (4-5 hours) and sildenafil (3-4
G. Flushing
hours).
H. Headache
I. Impaired vision
J. Priapism.
Year: 2014.

This is an advanced level question.

!
!
167
70. A 30-year-old man suffers from schizophrenia. The answer is B.
He presents a list of drugs which he took in
the past. Which of the following is NOT a
second generation antipsychotic drug?
f. Clozapine Explanation: Option B is a first-
g. Flupenthixol generation antipsychotic drug.
h. Olanzapine
i. Quetiapine
j. Risperidone.
Year: 2014

71. A 30-year-old man suffers from insomnia and The answer is C.


he sleeps well after taking diazepam. He
worries that he lacks a particular
neurotransmitter in the brain and diazepam
can enhance the actions of this Explanation: Benzodiazapine drugs bind
neurotransmitter. Which of the following to benzodiazepine receptors and this
neurotransmitters is MOST relevant In this enhances the binding of -Aminobutyric
case?
acid (GABA) to GABA receptors. This will
lead to influx of chloride ions and cause
F. Acetylcholine sedation.
G. Dopamine
H. -Aminobutyric acid
I. Noradrenaline
J. Serotonin. Year: 2014

72. Mrs. Tan is a 65-year-old woman who lost her The answer is C.
husband three years ago. She is still angry
with him for not taking better care of his
health. He was cremated and she keeps his
ashes in the urn. She is unsure what to do Explanation: increase fluoxetine to 80mg
with the ashes. She coped well after her may worsen anxiety and restlessness.
husbands death, until her son moved to
Australia a few months ago. She has long
standing headaches, but these have become
worse over the past six months. She also has Year: 2014
new onset of somatic symptoms as well as
anxiety symptoms which are overwhelming
and include a fear of dying. Her depression
has been partially treated with fluoxetine This is an advanced level question.
40mg per day, but her anxiety symptoms
persist and she feels restless. Which of the
following actions is LEAST appropriate?

F. Add alprazolam to her treatment for 2 weeks


G. Discontinue fluoxetine and start other
antidepressant drug such as paroxetine
H. Increase the dose of fluoxetine to 80mg per day
I. Order a thyroid function test
J. Start psychotherapy with a focus on grief.

!
!
168
73. Which of the following antidepressants cause The answer is D.
MOST SEDATION after potentiation with
benzodiazepine?
Explanation: Mirtazapine is the most
sedative antidepressant. The sedation
F. Agomelatine
effect will be worsen after potentiation
G. Escitalopram
H. Fluvoxamine with benzodiazepine.
I. Mirtazapine
J. Venlafaxine.

Year: 2014.

74. Which of the following factors limits the use The answer is E.
of clozapine in treatment-resistant
schizophrenia?

Explanation: 1 to 2% of patients using


F. A low incidence of seizures clozapine will develop agranulocytosis,
G. The lack of efficacy against positive symptoms and this is the main factor limiting its use;
H. The lack of efficacy against negative symptoms
risk increases with age, and is higher
I. The lack of efficacy against cognitive symptoms
J. The occurrence of agranulocytosis. among women; the risk is the highest in
the first six months of treatment (hence
the need for weekly blood tests in the
first six months, and then every two
weeks after that).

Year: 2014.

75. Which of the following medications is MOST The answer is C


likely to exacerbate psoriasis?

F. Amitriptyline Explanation: Lithium has higher risk to


G. Haloperidol worsen psoriasis as compared to other
H. Lithium psychotropic medications.
I. Risperidone
J. Sodium valproate.

Year: 2014.

This is an advanced level question.

76. Which of the following is CORRECT regarding The answer is E.


repetitive transcranial magnetic stimulation
(rTMS)?

!
!
169
Explanation: rTMS does not require
general anaesthesia. rTMS is an
F. rTMS is administered under general anaesthesia. alternative to ECT if patient has medical
G. rTMS is more efficacious than electroconvulsive
or surgical contraindicates to ECT. rTMS
therapy (ECT).
H. rTMS induces seizures with less energy as is not intended to induce seizures and it
compared to ECT. is not related to imaging.
I. rTMS is a new imaging modality in psychiatry.
J. rTMS may be useful in the treatment of
depression.

Year: 2014.

77. The MOST robust effect of lamotrigine is The answer is A.


found in which of the following?

a. Bipolar depression Explanation: Lamotrigine is effective in


treatment of bipolar depression with
b. Manic episode 200mg per day.

c. Mixed episodes

d. Hypomanic episode
Year: 2014.
e. Cyclothymia.

78. A 30-year-old woman complains of lack of The answer is C.


sexual drive after taking paroxetine. She
wants to change her antidepressant. Which of
the following antidepressant is the BEST
option to avoid sexual dysfunction? Explanation: The 5HT2A antagonism of
mirtazapine is associated with lesser
sexual side effects.
F. Amitriptyline
G. Reboxetine
H. Mirtazapine
I. Duloxetine
J. Venlafaxine.
Year: 2014.

79. Agomelatine,!a!new!antidepressant!!is!BEST! The!answer!is!D.!


given!in!which!of!the!following!parts!of!the!
day? !
!
Explanation:! Early! evening! is! the! best!
F. Early morning time! for! circadian! rhythms!
G. Late morning
resynchonizing! (1N2h! before! dark!
H. Early afternoon
I. Early evening

!
!
170
J. Past mid-night. phase).!

Year: 2014.

Psychotherapy! !

80. A! 24TyearTold! depressed! woman! said,! The!answer!is!C.!


People! in! my! office! try! to! make! my! life!
difficult! and! then! deem! themselves! as! !
ultimate! smart! to! know! about! me.! I! have!
resigned! as! I! am! uncomfortable! with! the! Explanation:! Interpersonal! therapy!
office!settings.!I!am!aware!that!I!am!getting! (IPT)! is! indicated! for! depressive!
verbally! aggressive! towards! other! people! disorder.! It! helps! the! patient! to!
whenever!they!provoke!me.!I!want!to!see!a! understand! the! communication!
psychotherapist! because! I! want! to! deal!
difficulties! (verbal! aggression).!
with! these! people! before! I! lose! my! selfT
identity.! Which! of! the! following! Psychotherapist!can!perform!roleNplay!
psychotherapies!is!MOST!appropriate?! to! help! patient! to! improve!
!! communcation.! IPT! also! helps! patient!
to! deal! with! the! loss! associated! with!
F. Cognitive!behaviour!therapy! her!selfNidentity!and!resignation.!
G. Grief!therapy!
H. Interpersonal!therapy! !
I. Relaxation!exercise!
J. Supportive!psychotherapy!
Year: 2014.

This!is!an!advanced!level!question.!

81. You are a family doctor. A 30-year-old woman The!answer!is!E.!


suffers from schizophrenia. Although she is
free from first rank symptoms, she is affected !
by weight gain, negative symptoms of
schizophrenia and no interest in sex. Her Explanation:!Sensate!focus!therapy!is!a!
husband has high sexual drive and he is not
happy with their sexual life. He requests for sex!therapy.!The!couple!focus!on!nonN
anti-androgen to reduce his sexual drive. vaginal!intercourse!and!master!other!
What is BEST management strategy? ! techniques!such!as!communication!
and!mutual!stimulation.!Prescription!
F. Cognitive behaviour therapy
G. Eye movement desensitization and of!antiNandrogen!is!associated!with!
processing side!effects!such!as!osteoporosis.!The!
H. Hypnotic therapy other!options!are!not!specific!for!
I. Prescribe anti-androgen
psychosexual!problems.!
J. Sensate focus therapy.

!
!
171
! !

Year:!2014!

82. A 40-year-old man suffers from post-traumatic The!answer!is!C.!


stress disorder (PTSD) and has history of
misusing cannabis. He claims that the !
government should legalise the sales of
cannabis because cannabis can reduce the
Explanation:!This!man!justifies!
PTSD symptoms and helps him to relax. What
is his defence mechanism? cannabis!misuse!with!an!explanation!
but!totally!forgets!the!potential!harms!
associated!with!cannabis!misuse.!
F. Denial
G. Projection
!
H. Rationalisation
I. Repression
J. Sublimation. Year:!2014!

Ethics and laws !

83. The Mental Health (Care and Treatment) Act in The!answer!is!B.!


Singapore can be applied at which of the
following hospital(s) for involuntary !
admission?
Explanation:!The!Mental!Health!Act!
f. Changi General Hospital (CGH) only
g. Institute of Mental Health (IMH) only
can!only!be!applied!at!IMH!for!
h. National University Hospital (NUH) only involuntary!admission.!!
i. IMH, NUH and CGH
j. IMH, NUH, CGH, Singapore General !
Hospital (SGH) and Tan Tock Seng
Hospital (TTSH). Year:!2014.!

84. A 21-year-old woman suffering from body The!answer!is!D.!


dysmorphic disorder consulted a doctor for
Botulinum toxin (botox) treatment. She was !
concerned of the smoothness of her face and
spent $10,000 for multiple botox injection. The Explanation:!NonNmaleficence!refers!to!
botox injection caused paralysis of facial
muscles. Botox injection is not indicated for the!obligation!of!a!doctor!to!avoid!
women younger than 30 years and body harm,!causing!muscular!atrophy!by!
dysmorphic disorder. Which of the following botox!injection!in!this!case.!
ethical principles was violated by this doctor?
!
P. Autonomy!
Year:!2014.!
Q. Capacity!
R. Confidentiality!
S. NonNmaleficence!
T. Justice.!

!
!
172
85. A 25-year-old man was arrested for murder. The!answer!is!C.!
According to the police, he shows no remorse
towards his act. He firmly believes that killing !
one person will not lead to death sentence. He
needs to kill several people to get death
Explanation:!In!forensic!psychiatry,!
sentence. He always carries a chopper and
wants to attack innocent people. If he is psychopath!represents!the!most!
released from the prison, he will kill more severe!form!of!antisocial!personality!
people, including his parents. Which of the disorder.!People!with!psychopathy!
following diagnoses is MOST appropriate?
demonstrate!extremely!low!level!of!
empathy!and!remorse.!They!have!high!
F. Biopathy chance!of!recidivism.!
G. Egopathy
H. Psychopathy !
I. Sociopathy
J. Superegopathy.
Year:!2014.!

Consultation liaison psychiatry !

86. You are a resident working in the Children The!answer!is!D.!


Emergency Department. A 5-year-old boy is
presented for recurrent urinary tract infection !
and confirmed by raised white blood cell
counts in urine. According to his mother, the Explanation:! Mnchausen syndrome by
boy likes to insert foreign body into his penis
via the urethra meatus. After taking a proxy occurs when an individual (the
thorough history and conducting physical mother) must have presented another
examination, you cannot gather any evidence individual (the boy) to others (AED
the boy could insert foreign body into his doctor) as medically ill (recurrent urinary
penis. It is not anatomically and tract infection) without the intention of
physiologically feasible for the boy to do so. obvious external rewards but invasive
You cannot exclude the possibility that it
investigation or hospitalizations.
could have been done by his mother. History
also reveals that the parents have marital Mnchausen syndrome by proxy is
problems and his father is suspected to visit considered to be a form of child abuse.
commercial sex workers frequently. She is She is displacing her anger (husband
very keen to admit her son for further invasive visiting commercial sex workers) towards
investigations. What is the MOST likely her son.
diagnosis?

F. Conversion disorder induced by the mother


G. Hypochondriasis (or illness anxiety) induced by Year:!2014.!
the mother
H. Malingering induced by the mother
I. Mnchausen syndrome by proxy induced by the
mother
J. Somatisation disorder induced by the mother.
87. Which!of!the!following!questionnaires! The!answer!is!D.!
would!be!MOST!useful!in!identifying!
symptoms!of!depression!and!anxiety!in! !
patients!suffering!from!chronic!medical!
illness?!!! Explanation:!The!Hospital!Anxiety!and!
! Depression!Scale!is!the!best!scale!

!
!
173
F. The!Beck!Anxiety!and!Depression!Inventory! because!the!items!are!specially!
G. The!General!Health!Questionnaire! constructed!to!enhance!sensitivity!in!
H. The!Hamilton!Anxiety!and!Depression!Rating! detecting!depression!and!anxiety!
Scale!
symptoms!in!medical!patients.!The!
I. The!Hospital!Anxiety!and!Depression!Scale!
J. The!StateNTrait!Anxiety!Inventory.! Beck!depression!and!anxiety!inventory!
and!Hamilton!Anxiety!and!Depression!
Rating!Scale!are!more!suitable!for!
psychiatric!patients.!

Year:!2014.!

88. You are a resident working in the Accident The!answer!is!A.!


and Emergency Department (AED). A 26-year-
old woman gave birth one month ago. She is a !
single mother and broke up with her
boyfriend. She has poor social support and
Explanation:! ! In! this! situation,! both!
no one helps her to look after the baby. She
has been depressed for 1 month and wanted mother! and! baby! should! be! admitted!
to jump down from her flat two weeks ago. to! the! hospital! because! no! one! looks!
She was referred to the womans mental after! the! baby! at! home.! IMH! does! not!
health service. She was referred to a case
manager but no medication was prescribed. have!facility!to!look!after!her!baby.!
She breastfeeds her baby. Today, she wants
to jump again. The mother and her baby are !
bought to the AED by the case manager.
Which of the following is the BEST The!psychiatrist!in!the!hospital!should!
management? consider! starting! antidepressant!
because!she!is!highly!suicidal!and!SSRI!
F. Admit the mother and her baby to the hospital.
The mother will go to the psychiatric ward and the like!sertraline!is!safe!for!breastfeeding.!
baby will go to paediatric ward. She will benefit
from antidepressant and psychological !
intervention.
G. Admit the mother and her baby to the hospital. Year:!2014.!
The mother will go to the psychiatric ward and the
baby will go to paediatric ward. She should
receive psychological intervention only because
antidepressants are dangerous for breastfeeding.
H. Discharge the mother and her baby from the AED
because she suffers from adjustment disorder.
Her risk is low.
I. Discharge the mother and her baby from the AED
because she receives good care from case
manager.
J. Send both mother and her baby to the Institute of
Mental Health for compulsory admission.
89. What of the following psychiatric side effects The answer is D.
is MOST common in Hepatitis C patients
receiving interferon treatment?

F. Anxiety Explanation: Interferon treatment is


G. Cognitive impairment

!
!
174
H. Confusion well known to be associated with
I. Depression depression.
J. Hallucination.

Year: 2014.

90. The hospital wants to develop the best The answer is A.


method to detect and identify delirium in
medically ill patients. Which of the following
is the BEST strategy to detect and identify
delirium? Explanation: Option A is the most cost-
effective strategy to detect and identify
delirium. Option D is not the best strategy
F. Administer Delirium Rating Scale to all patients because there are elderly patients who
older than 65 years to screen for delirium.
develop delirium without any laboratory
G. Looking for abnormalities on computerized
topography (CT) brain scans for all patients. abnormalities. Option E is not the best
H. Looking for abnormalities on magnetic resonance strategy because history of delirium may
imaging (MRI) brain scans for suspected patients. not predict future episodes of delirium.
I. Monitor signs of delirium on those patients with
laboratory abnormalities.
J. Screen for history of delirium and monitor those
patients with history of delirium only. Year: 2014.

Paper!7!N!Questions!

!
!
175
History, mental state exam, psychopathology and risk assessment!

1. You are a resident working in the accident and emergency department (AED).
A 30-year-old schizophrenia patient is brought in by his mother. He suddenly
heard a voice asking him to use a sword to stab himself and open up his
abdomen. His mother stopped him and bought him to the hospital. He does not
have antisocial behaviour. Which of the following BEST describes his current
risk at the AED?

A. High risk due to the possibility that he may attack other people.
B. High risk due to the possibility of damaging internal organs such as aorta and liver if
he stabs himself.
C. Low risk due to the fact that he has informed his mother and the chance to carry out
action is low.
D. Low risk due to the unlikeliness for him to harm himself as a result of disorganized
behaviour seen in schizophrenia patients.
E. Moderate risk due to possibility of superficial cut on abdomen.

2. You are a resident working in the accident and emergency department. A 30-year-old
bank officer is brought in by her supervisors after she has a failed suicide attempt in
the bank. She was recently transferred out from the head office to another branch.
According to her relatives, the patient claims that the Chief Executive Officer (CEO)
fell in love with her. She spent hours waiting for the CEO after work. She tried to stop
his car from leaving the car park. The CEO has never worked with her and does not
know her personally. Which of the following is the correct diagnosis?

A. Capgras!syndrome!
B. Charles!de!Bonnets!syndrome!
C. De!Clrambault's!syndrome!
D. Gansers!syndrome!
E. Othellos!syndrome!

3 Which of the following is NOT a thought disorder found in patients suffering from
schizophrenia?

A. Derailment
B. Echopraxia
C. Loosening of association
D. Knights move thinking
E. Word salad.

Cognitive assessment!

4 Which of the following is NOT a prominent cognitive feature of cortical dementia?

!
!
176
A. Problems with articulation
B. Problems with calculation
C. Problems with drawing a clock face
D. Problems with expression of emotion
E. Problems with recognition

5 A 20-year-old woman complains of low mood because she is bullied in her workplace.
She is a factory worker and her supervisor complains that she is very slow at work.
She stays with her family and is able to look after herself. Her mother describes the
patient as simple-minded. When she was young, she exhibited delay in understanding
and learning language. She kept failing the N level examination despite multiple
attempts. Which of the following BEST describes her current level of intelligence?

A. Normal intelligence
B. Mild intellectual disability
C. Moderate intellectual disability
D. Severe intellectual disability
E. Profound intellectual disability.

Psychiatric epidemiology

6 By 2020, which of the following illnesses is predicted to have higher global


disease burden than depressive disorder?

A. Anxiety disorder
B. Dementia
C. Diabetes
D. Ischaemic heart disease
E. Schizophrenia.

7 Which of the following mood disorders has the HIGHEST rate of co-occurrence with a
substance abuse disorder?

A. Adjustment disorder
B. Seasonal affective disorder
C. Cyclothymia
D. Bipolar disorder
E. Dysthymia.

Psychiatric aetiology, diagnosis and classification

8 Which of the following neuroanatomical areas is MOST implicated in

!
!
177
obsessive compulsive disorder?

A. Amygdala and prefrontal cortex


B. Frontal lobe and parietal lobe
C. Hippocampus and locus coeruleus
D. Nucleus accumbens and prefrontal cortex
E. Orbito-frontal cortex and basal ganglia

9 Which of the following is the MOST common cause of secondary hypersomnia?

A. Consumption of hydroxyzine
B. Dementia
C. Mixed anxiety and depression
D. Obstructive sleep apnea
E. Shift work.

10 You are a general practitioner. A 33-year-old man complains of stress due to


interpersonal problems and long term unemployment. He claims that he suffers from
depression but does not require antidepressant treatment. He has no history of self-
harm or criminal record. He has long-standing problems with his wife. His wife
mentions that he knows every detail about cooking and household chores. She finds
the 3- year marriage miserable and has gone through a very difficult period. His wife
realises that she will never satisfy his expectations. She describes the patient as
being too idealistic and extremely careful. When she cooks, the patient tries to belittle
her by watching her every single movement and interjecting a lot of advice to make a
perfect meal. He often scolds her for spending money and she is afraid of the patient.
She finds him very rigid with no sense of humour. Which of the following is the
correct diagnosis?

A. Avoidant personality disorder


B. Borderline personality disorder
C. Narcissistic personality disorder
D. Obsessive compulsive personality disorder
E. Paranoid personality disorder.

11 The mother of a schizophrenia patient wants to consult you the risk of his dizygotic
twin brother in developing schizophrenia. Your answer is:

A. 4%
B. 14%
C. 24%
D. 34%
E. 44%.
General adult psychiatry

12 You are a general practitioner. A 22-year-old patient comes to consult


you because she is stressed. Her father boarded an aeroplane which
was found missing and probably has crashed. She is not certain about

!
!
178
the status of her father and he is not contactable. At this moment, she is
MOST likely to develop which of the following psychiatric conditions?

A. Anticipatory grief
B. Compensation neurosis
C. Dissociative state
D. Psychosis
E. Hypochondriasis.

13 A 30-year-old schizophrenia patient drops out from rehabilitation. The


family member wants to find out the underlying reason. Which of the
following factors is LEAST likely to restrict rehabilitation potential?

A. Residual auditory hallucination


B. Intellectual disability
C. Lack of pleasure from social intervention and physical activity
D. Poor motivation
E. Restricted affect.
!

14 Which!of!the!following!statements!concerning!generalized!anxiety!disorder!is!
FALSE?!!
!

A. Alcohol!misuse!is!a!common!comorbidity.!!
B. Cognitive!behaviour!therapy!is!generally!helpful.!
C. Excessive!worries!and!anxiety!symptoms!are!episodic.!
D. ShortNterm!benzodiazepines!can!be!an!effective!treatment!
E. There!is!a!2!to!1!ratio!of!women!to!men!suffering!from!this!disorder.!

15 Which!of!the!following!is!TRUE!regarding!delusional!disorder?!
!!

A. Infrequent!hallucinations!totally!rule!out!the!diagnosis!of!delusional!disorder.!
B. Onset!is!in!adolescence.!!
C. Point!prevalence!is!3%.!
D. From!genetic!point!of!view,!there!is!frequent!crossover!to!other!psychotic!disorders!
such!as!schizophrenia.!
E. The!persecutory!subtype!is!the!most!common.!!
!

16 A 45-year-old man consults you because he is stressed over his daughters eating
disorder. His daughter suffers from anorexia nervosa. He is very concerned about her
condition. He wants to seek your reassurance. Which of the following indicates a
GOOD prognostic factor for his daughter?

!
!
179
A. Early age of onset
B. Past hospitalisation
C. Frequent self-induced vomiting rather than food restriction
D. Very low BMI
E. Perfectionistic personality.

17 You are a general practitioner. A 25-year-old man with history of Aspergers


syndrome. He has history of violent and compulsive behaviour. After recent increase
in dose of psychotropic medications, his father discovers that he masturbates less
often and seems to develop erectile dysfunction. Which of the following medications
is MOST likely to cause erectile dysfunction?

A. Alprazolam
B. Bupropion
C. Mirtazapine
D. Sertraline
E. Hydroxyzine
18 You are a general practitioner. A 30-year-old man comes to the clinic and claims that
he received a diagnosis of bipolar II disorder in the United States. Bipolar II disorder is
best described as:

A. Dysthymia and cyclothymia


B. Hypomanic episode and depressive episode
C. Manic episode only
D. Manic episode and depressive episode
E. Seasonal affective disorder

Old age psychiatry

19 Which of the following statements about depressive!disorder!in!old!people!is!TRUE?

A. Depressive disorder in elderly is accompanied by a much lower suicide risk than in younger
adults.
B. Depressive disorder in elderly is less likely to be associated with paranoia as compared to
younger adults.
C. Elderly suffering from depressive disorder takes shorter time to respond as compared to
younger adults
D. Mild depressive disorder is more prevalent in elderly women than men.
E. Prevalence of depressive disorder rises sharply with age in the community.

20 Which!of!the!following!is!MOST!commonly!seen!as!a!normal!change!associated!
with!aging?

A. Increase!in!acetylcholine!
B. Increase!in!paranoia!
C. Increase!in!brain!ventricle!size!!
D. Increased!electroencephalogram!(EEG)!frequency!
E. Increased!slowNwave!sleep.!

!
!
180
21 What!is!the!MOST!likely!diagnosis!in!a!75T!yearTold!man!with!declining!cognition,!
visual!hallucinations,!and!parkinsonism!symptoms?!!!
!

A. Alzheimers!disease!
B. Dementia!with!Lewy!Bodies!
C. LateNonset!schizophrenia!
D. FrontoNtemporal!lobe!dementia!
E. Vascular!dementia.!
!

22 Which of the following statements is false with regards to delirium in


the elderly?

A. 30% or more of the elderly admitted to medical ward will develop delirium
during hospitalisation.
B. Antipsychotic drug is the first-line psychiatric treatment for delirium.
C. Delirium is often overlooked by health care professionals in the elderly.
D. Delirium typically resolves within 3 days after correcting the underlying cause.
E. Incidence of delirium increases with age.

Child and adolescent psychiatry

23 If a young boy presents with intellectual disability, autistic features and multiple
maternal relatives are carriers of a genetic condition. Which of the following diagnosis
is MOST likely?

A. Angleman syndrome
B. Down syndrome
C. Prader Willi syndrome
D. Fragile X syndrome
E. Williams syndrome

Addiction and Substance abuse

24 You are an orthopaedic resident. A 22-year-old man suffers from cervical spondylosis
and results in severe neck pain. His pain does not respond to analgesics and
becomes very depressed and suicidal. His mother describes him as being obsessed
with online games. He is an active member of several online game groups. He spends
more around 20 hours per day in the internet caf and becomes very agitated when he
is not allowed to play online games. He has history of alcohol dependence. Which of
the following symptoms found in this patient is MOST important in establishing the
diagnosis of internet addiction?

!
!
181
A. Active member of the online group
B. Agitation when not allowed to play online game
C. Cervical spondylosis
D. Depression and suicidal thought
E. History of alcohol abuse.
25 Which of the following is NOT a factor which increases the risk of relapse to cocaine use in a 30-
year-old man who recently became abstinent from cocaine?

A. Feeling depressed, tired and the need to overeat.


B. Going to pubs where ice and special K are available.
C. Seeing a counsellor to talk about the previous use of cocaine.
D. Seeing a psychiatrist to complain about attention deficit and try to obtain methylphenidate.
E. Spending time with colleagues who use coke.

26 A 27-year-old woman presents with unplanned pregnancy. She smokes


cannabis 3-4 times per day. Her husband is concerned about the effects of
cannabis on her baby. During the interview, she avoids eye contact and
appears to be very anxious. She claims that she eats and sleeps well. She
thinks cannabis is safe for her foetus because she has a friend who delivered
a healthy baby despite continued use of cannabis during pregnancy. Which
Prochaskas and Diclementes stage of change BEST describes her current
status?

A. Action
B. Contemplation
C. Denial
D. Precontemplation
E. Maintenance.

Psychopharmacology

27 Which!of!the!following!neurochemicals!is!metabolized!by!monoamine!oxidase? !
!

A. Acetylcholine!
B. BrainNderived!neutrophic!factor!
C. Histamine!
D. Glutamate!
E. Serotonin.!

28 A 30-year-old man takes mirtazapine every night and experiences significant sedation
at 7.5mg/day. Which of the following neurotransmitter receptors are MOST likely to be
responsible for causing this side effect?

A. Adrenergic receptors
B. Dopamine receptors
C. Glutamate receptors
D. Histamine receptors
E. Nicotinic receptors.

!
!
182
29 Which of the following psychotropic medications is the MOST likely to
raise the prolactin levels in female psychiatric patients?

A. Aripiprazole
B. Clozapine
C. Lithium
D. Olanzapine
E. Risperidone.

30 Which of the following statements regarding quetiapine is LEAST CORRECT?

a. Quetiapine has anxiolytic effects.


b. Quetiapine has antidepressant effects
c. Quetiapine has antipsychotic effects.
d. Quetiapine has extrapyramidal effects
e. Quetiapine has sedative effects.

31 Which of the following is NOT a well-established side effect of lithium?

A. Hepatotoxicity
B. Hypothyroidism
C. Nephrotoxicity
D. Tremor
E. Weight Gain

32 A 30-year-old man suffers from bipolar disorder. He requests to be treated by


carbamazepine because it works on his friend. Which of the following genetic tests is
required by the Ministry of Health (MOH) before prescription of carbamazepine?

A. Dopamine transporter gene - D*1502


B. Human leukocyte antigen gene - B*1502
C. Integrin alpha M gene I*1502
D. Mannan-binding lectin gene M*1502
E. Serotonin transporter gene S*1502.

33 A 25-year-old woman is treated by bupropion for depressive disorder. She gives you a
list of symptoms and attributes to the side effects of bupropion. Which of the
following symptoms is LEAST likely to be side effect of bupropion?

A. Anxiety
B. Nausea
C. Seizure

!
!
183
D. Transient hallucination
E. Weight gain.

34 A 30-year-old schizophrenia patient receives olanzapine 20mg per day. He is a chronic


smoker and agrees to cease smoking in the past one month. After he quitted smoking,
he experiences more side effects associated with 20mg of olanzapine which include
significant sedation and weight gain. Which of the following is MOST likely
explanation?

A. He is not complaint to olanzapine.


B. He took an overdose of olanzapine.
C. After he stopped smoking, the hepatic metabolism of olanzapine reduces.
D. Nicotine reduces the risk of metabolic syndrome.
E. This is due to the effect of nicotine patch.

35 Which!of!the!following!is!the!mechanism!of!action!of!memantine,!a!medication!
used!to!slow!cognitive!decline!in!Alzheimers!dementia?

A. Cholinesterase!inhibitor!
B. Dopamine!receptor!blocker!
C. GammaNaminobutyric!acid!receptor!agonist!
D. NNmethylNDNaspartate!receptor!antagonist!
E. Serotonergic!reuptake!inhibitor.!

Psychotherapy

36 A 30-year-old woman with panic disorder does not respond to an initial treatment with
a selective serotonin reuptake inhibitor (SSRI). Which of the following treatment is
considered the best approach?

K. Benzodiazepine
L. Cognitive behaviour therapy
M. Hypnotherapy
N. Olanzapine
O. Psychodynamic psychotherapy.

37 You are a medical officer working in the army. A 22-year-old national serviceman
comes to see you because he is very angry with his male supervisor and wants to see
a psychologist. He also informs you that he is very angry with his own father. He
passed you an emotional 500-word open letter posted on his Facebook account. In his
letter, the patient mentions that his father has been abusive towards him. He finds his
male supervisor is as abusive as his father and the pattern keeps repeating itself.
Which of the following psychotherapy is MOST appropriate in this case?

A. Cognitive behaviour therapy


B. Problem solving therapy

!
!
184
C. Psychodynamic psychotherapy
D. Reminiscence therapy
E. Validation therapy.
38 A"20%year%old"woman"suffers"from"borderline"personality"disorder."She"works"as"a"
clerk." During" psychotherapy" session," she" discloses" that" she" really" hates" the"
psychologist"and"has"thoughts"of"killing"the"psychologist."Her"conviction"to"kill"the"
psychologist" is" 1" out" of" 10" (1" =very" unlikely," 10" =" very" likely)." Furthermore," she"
does"not"have"a"plan"or"not"sure"when"to"kill"the"psychologist."She"has"no"forensic"
history"and"no"history"of"violence."She"admits"the"idea"of"killing"the"psychologist"is"
her" own" thought." She" has" no" command" hallucination." " Which" of" the" following"
BEST"describes"this"phenomenon?
6

A. Chronic felling of emptiness


B. Displacement
C. Projective identification
D. Splitting
E. Transference6

39 A 30-year-old man suffers from disseminated gonococcal infection. The medical team
asks him about unprotected sexual activities. He claims he has one stable sexual
partner and practises safe sex all the time. The medical team thinks that his condition
could due to an underlying autoimmune cause because he has no risky behaviour.
Several days later, he finally admits that he has sexual intercourse with commercial
sex worker once per month. Sometimes, he does not use condom to protective
himself. What is the initial defence mechanism used by this patient?

A. Denial
B. Displacement
C. Projection
D. Projective identification
E. Undoing

Ethics and laws

40 You are a general practitioner. A 30-year-old female secretary complains of low mood,
poor appetite, poor sleep and suicidal thought. She is very depressed over the death
of her son who passed away 26 hours after birth. She has history of depression
without any psychotic feature. She underwent a traumatic delivery because an
obstetrician in private practice performed amniocentesis, induction of labour and
administered syntocinon 3 weeks before the expected date of delivery. The patient
later found out that the obstetrician induced labour because the doctor planned to go
for a holiday. Patient did not fully understand about the procedure and did not agree
with induction of labour which resulted in premature birth of her son. The obstetrician
claims that she does not have capacity to make decision due to history of depression.
She felt that the doctor performed the procedure against her will and the death of her
son was avoidable. Which ethical principle was violated by the obstetrician?

A. Autonomy

!
!
185
B. Beneficence
C. Confidentiality
D. Involuntary treatment
E. Justice.

41 You are a resident posted to the Accident and Emergency Department of


Institute of Mental Health (IMH). A 55-year-old man attended a party in a buffet
restaurant. He seldom drinks alcohol. In the party, he drank a few glasses of
hard liquor and could not find his way to the table. When a female waitress
tried to help him, he was irritable and physically assaulted her. The waitress
suffered from minor injury but the restaurant manager insisted to report to the
police. Subsequently, he was arrested and stayed in the police station for one
night. On the next day, he was sent to IMH for assessment. When you
interview him, he says, Oh gosh, I cant recall the event which occurred last
night. He does not have past psychiatric illness. Which of the following terms
BEST describes his behaviour leading to the arrest?

A. Actus reus
B. Jealousy
C. Automatism
D. Diminished responsibility
E. Mens rea.

Liaison Psychiatry

42 A 20-year-old man was given a high dose of intravenous haloperidol. He


develops high fever and you suspect that he may develop neuroleptic
malignant syndrome (NMS). All of the following clinical features suggest NMS
EXCEPT:

A. Autonomic instability
B. Diaphoresis
C. Decreased catecholamines in urine
D. Myoglobinuria
E. Rigidity.

43 An obstetrician refers a 30-year-old woman who presents with


psychosis after delivery to you. She wants to know whether this woman
suffers from postpartum psychosis. You want to find out from the
obstetrician the onset of psychotic symptoms after delivery. Which of
the following days of onset is MOST congruent with the diagnosis of
postpartum psychosis?

A. 1- 6 weeks after delivery


B. 7-12 weeks after delivery
C. 13 -18 weeks after delivery

!
!
186
D. 19 - 24 weeks after delivery
E. 25 31 weeks after delivery.
!

44 A 40-year-old man went to South Korea for skiing and suffered from head
injury. All of the following symptoms are commonly associated with sequelae of
head injury EXCEPT:

A. Light and noise sensitivity


B. Headache
C. Insomnia
D. Obsession
E. Word finding difficulty.
45 Which of the following is LEAST likely to be an area of concern in a 30-year-old
woman suffering from body dysmorphic disorder?

A. Breast
B. Nose
C. Genitalia
D. Hair
E. Skin.

46 You are a medical resident. A 40-year-old man is admitted to the medical ward
due to altered mental state and requires further medical investigations. Past
records show a history of substance abuse. He sleeps poorly and paces
around the unit, restless and grumpy on the next day. He appears to be
confused and exhibits tachycardia at 106 beats per minutes. His palms and
forehead are sweaty and his tongue is showing a course tremor. When asked,
the patient says he feels anxious. Which of the following is the CORRECT
diagnosis?

A. Alcohol withdrawal
B. Amphetamine withdrawal
C. Cannabis withdrawal
D. Nicotine withdrawal
E. Opioid withdrawal.
!

Paper!7!N!Questions!

History, mental state exam, psychopathology and risk !


assessment!

3. You are a resident working in the accident and The answer is B.


emergency department (AED). A 30-year-old
schizophrenia patient is brought in by his mother.
He suddenly heard a voice asking him to use a sword
to stab himself and open up his abdomen. His
mother stopped him and bought him to the hospital. Explanation: This patient is

!
!
187
He does not have antisocial behaviour. Which of the at high risk due to two
following BEST describes his current risk at the reasons: 1) command
AED? hallucination 2) the
possibility of damaging the
F. High risk due to the possibility that he may attack other aorta and liver. He may die
people. in 30 seconds if aorta is
G. High risk due to the possibility of damaging internal damaged.
organs such as aorta and liver if he stabs himself.
H. Low risk due to the fact that he has informed his mother
and the chance to carry out action is low.
I. Low risk due to the unlikeliness for him to harm Year: 2014
himself as a result of disorganized behaviour seen in
schizophrenia patients.
J. Moderate risk due to possibility of superficial cut on
abdomen.

4. You are a resident working in the accident and The!answer!is!C.!


emergency department. A 30-year-old bank officer is
brought in by her supervisors after she has a failed !
suicide attempt in the bank. She was recently
transferred out from the head office to another branch. Explanation:!The!patient!
According to her relatives, the patient claims that the
Chief Executive Officer (CEO) fell in love with her. She suffers!from!delusion!of!
spent hours waiting for the CEO after work. She tried to love!for!a!person!with!high!
stop his car from leaving the car park. The CEO has status!but!has!no!contact!
never worked with her and does not know her
personally. Which of the following is the correct with!the!patient.!
diagnosis?
!

F. Capgras!syndrome! Year: 2014


G. Charles!de!Bonnets!syndrome!
!
H. De!Clrambault's!syndrome!
I. Gansers!syndrome!
J. Othellos!syndrome!

47 Which of the following is NOT a thought disorder found in The answer is B.


patients suffering from schizophrenia?

F. Derailment Echopraxia is the motor symptom


G. Echopraxia analogous to echolalia.
H. Loosening of association Echopraxia refers to the imitation
I. Knights move thinking
of movements and gestures of the
J. Word salad.
person the patient is observing.

!
!
188
Year: 2013.!

Cognitive assessment! !

48 Which of the following is NOT a prominent cognitive The!answer!is!A.!


feature of cortical dementia?
!
F. Problems with articulation Explanation:!Articulation!
G. Problems with calculation
H. Problems with drawing a clock face difficulty!is!associated!with!
I. Problems with expression of emotion subcortical!dementia!which!
J. Problems with recognition is!characterised!by!
neurological!sign.!

Year: 2014

49 A 20-year-old woman complains of low mood because The!answer!is!B.!


she is bullied in her workplace. She is a factory worker
and her supervisor complains that she is very slow at !
work. She stays with her family and is able to look after
herself. Her mother describes the patient as simple- Explanation:!She!has!mild!
minded. When she was young, she exhibited delay in
understanding and learning language. She kept failing intellectual!disability!which!
the N level examination despite multiple attempts. is!characterised!by!ability!to!
Which of the following BEST describes her current level work!in!practical!
of intelligence?
occupations!and!ability!to!
F. Normal intelligence look!after!oneself.!
G. Mild intellectual disability
H. Moderate intellectual disability !
I. Severe intellectual disability
J. Profound intellectual disability. Her!slowness!at!work,!
repeated!failing!of!the!N!
level!examination!and!
delayed!in!language!
development!do!not!support!
normal!intelligence.!

Year: 2014

Psychiatric epidemiology !

!
!
189
50 By 2020, which of the following illnesses is The!answer!is!D.!
predicted to have higher global disease burden
than depressive disorder? !

Explanation:!Based!on!the!
F. Anxiety disorder prediction!of!WHO,!
G. Dementia ischaemic!heart!disease!will!
H. Diabetes
rank!No.1!while!depressive!
I. Ischaemic heart disease
J. Schizophrenia. disorder!will!rank!No.2!in!
global!disease!burden.!

Year: 2014

51 Which of the following mood disorders has the HIGHEST Answer!is!D.!


rate of co-occurrence with a substance abuse disorder?
!
F. Adjustment disorder Explanation:!Bipolar!
G. Seasonal affective disorder
H. Cyclothymia disorder!is!the!commonest!
I. Bipolar disorder mood!disorder!and!
J. Dysthymia. demonstrates!the!highest!
rate!of!coNoccurrence!with!a!
substance!abuse!disorder!
among!all!the!choices.!

Year: 2014

!!

Psychiatric aetiology, diagnosis and classification !

52 Which of the following neuroanatomical areas The answer is E.


is MOST implicated in obsessive compulsive
disorder?
Explanation: Orbito-frontal
cortex and basal ganglia are
F. Amygdala and prefrontal cortex most implicated in compulsive
G. Frontal lobe and parietal lobe behaviours associated with
H. Hippocampus and locus coeruleus OCD.
I. Nucleus accumbens and prefrontal cortex
J. Orbito-frontal cortex and basal ganglia !

!
!
190
Year: 2014

53 Which of the following is the MOST common cause of Answer!is!D.!


secondary hypersomnia?
!
F. Consumption of hydroxyzine Explanation:!Obstructive!
G. Dementia
H. Mixed anxiety and depression sleep!apnoea!is!the!most!
I. Obstructive sleep apnea common!cause!of!secondary!
J. Shift work. hypersomnia!

Year: 2014

54 You are a general practitioner. A 33-year-old man The!answer!is!D.!


complains of stress due to interpersonal problems and
long term unemployment. He claims that he suffers from !
depression but does not require antidepressant
treatment. He has no history of self-harm or criminal
Explanation:!This!patient!is!
record. He has long-standing problems with his wife. His
wife mentions that he knows every detail about cooking characterised!by!being!
and household chores. She finds the 3- year marriage perfectionistic,!rigid,!frugal!
miserable and has gone through a very difficult period. and!preoccupation!with!
His wife realises that she will never satisfy his
expectations. She describes the patient as being too minor!details.!!!
idealistic and extremely careful. When she cooks, the
patient tries to belittle her by watching her every single His!personality!disorder!
movement and interjecting a lot of advice to make a resulted!in!long!term!
perfect meal. He often scolds her for spending money unemployment,!
and she is afraid of the patient. She finds him very rigid
with no sense of humour. Which of the following is the interpersonal!problems!and!
correct diagnosis? marital!problems.!!

F. Avoidant personality disorder !


G. Borderline personality disorder
H. Narcissistic personality disorder Year: 2014
I. Obsessive compulsive personality disorder
J. Paranoid personality disorder.
!

55 The mother of a schizophrenia patient wants to consult The!answer!is!B.!


you the risk of his dizygotic twin brother in developing
schizophrenia. Your answer is: !

Explanation:!The!risk!for!
F. 4%
G. 14% another!dizygotic!twin!to!
H. 24% develop!schizophrenia!is!
I. 34%

!
!
191
J. 44%. 14%.!

Year: 2014

General adult psychiatry !

56 You are a general practitioner. A 22-year-old The!answer!is!A.!


patient comes to consult you because she is
stressed. Her father boarded an aeroplane !
which was found missing and probably has
Explanation:!As!the!plane!has!
crashed. She is not certain about the status of
her father and he is not contactable. At this crashed,!the!chance!of!
moment, she is MOST likely to develop which survival!for!her!father!is!low.!
of the following psychiatric conditions? She!will!experience!
anticipatory!grief!during!the!
period!of!uncertainty.!

F. Anticipatory grief !
G. Compensation neurosis
H. Dissociative state Year: 2014
I. Psychosis
J. Hypochondriasis. !

57 A 30-year-old schizophrenia patient drops out The!answer!is!A.!


from rehabilitation. The family member wants
to find out the underlying reason. Which of the !
following factors is LEAST likely to restrict
Explanation:!Auditory!
rehabilitation potential?
hallucination!is!positive!
symptom!and!responds!to!
F. Residual auditory hallucination
G. Intellectual disability antipsychotic!drug.!Other!
H. Lack of pleasure from social intervention and options!are!negative!
physical activity symptoms!of!schizophrenia!
I. Poor motivation and!intellectual!disability.!
J. Restricted affect. These!factors!are!less!likely!
! to!respond!to!antipsychotic!
treatment.!

Year:!2014!

58 Which!of!the!following!statements!concerning! The!answer!is!C.!
generalized!anxiety!disorder!is!FALSE?!!

!
!
192
! !

F. Alcohol!misuse!is!a!common!comorbidity.!! Explanation:!Option!C!refers!
G. Cognitive!behaviour!therapy!is!generally!helpful.! to!panic!disorder.!
H. Excessive!worries!and!anxiety!symptoms!are!episodic.!
I. ShortNterm!benzodiazepines!can!be!an!effective! !
treatment!
J. There!is!a!2!to!1!ratio!of!women!to!men!suffering!from! Year:!2014!
this!disorder.!

59 Which!of!the!following!is!TRUE!regarding!delusional! The!answer!is!E.!
disorder?!
!! !

F. Infrequent!hallucinations!totally!rule!out!the!diagnosis!of! Explanation:!Delusion!
delusional!disorder.! disorder!is!quite!rare,!from!
G. Onset!is!in!adolescence.!! 0.025!to!0.03%.!The!mean!
H. Point!prevalence!is!3%.! age!of!onset!is!40!years.!
I. From!genetic!point!of!view,!there!is!frequent!crossover!to!
Delusional!disorder!does!not!
other!psychotic!disorders!such!as!schizophrenia.!
J. The!persecutory!subtype!is!the!most!common.!! cross!over!in!family!studies!
! with!schizophrenia!or!mood!
disorder.!Olfactory!
hallucinations!in!a!somatic!
delusion!of!body!odour,!may!
occur.!

!
!

Year:!2014.!

60 A 45-year-old man consults you because he is stressed The!answer!is!A.!


over his daughters eating disorder. His daughter suffers
from anorexia nervosa. He is very concerned about her !
condition. He wants to seek your reassurance. Which of
the following indicates a GOOD prognostic factor for his Explanation:!Based!on!eating!
daughter?
disorder!research,!early!age!
of!onset!indicates!good!
F. Early age of onset prognostic!factor.!The!other!
G. Past hospitalisation
H. Frequent self-induced vomiting rather than food restriction
factors!are!poor!prognostic!
I. Very low BMI factors.!
J. Perfectionistic personality.
!

Year:!2014.!!

!
!
193
61 You are a general practitioner. A 25-year-old man with The!answer!is!D.!
history of Aspergers syndrome. He has history of
violent and compulsive behaviour. After recent increase !
in dose of psychotropic medications, his father
discovers that he masturbates less often and seems to
Explanation:!Sertraline!is!A!
develop erectile dysfunction. Which of the following
medications is MOST likely to cause erectile SSRI!and!most!likely!to!cause!
dysfunction? sexual!dysfunction!among!all!
the!choices.!
F. Alprazolam
!
G. Bupropion
H. Mirtazapine
I. Sertraline Year:!2014.!!
J. Hydroxyzine
!

62 You are a general practitioner. A 30-year-old man comes The answer is B.


to the clinic and claims that he received a diagnosis of
bipolar II disorder in the United States. Bipolar II
disorder is best described as:
Explanation: Bipolar I
F.Dysthymia and cyclothymia disorder is characterised
G. Hypomanic episode and depressive episode by at least one manic
H. Manic episode only episode. Bipolar II disorder
I. Manic episode and depressive episode is characterised by
J. Seasonal affective disorder
hypomanic episode and
depressive episode.

Year: 2013.

Old age psychiatry !

63 Which of the following statements about depressive! The!answer!is!D.!


disorder!in!old!people!is!TRUE?
!
F. Depressive disorder in elderly is accompanied by a much Explanation:!Depression!in!
lower suicide risk than in younger adults.
G. Depressive disorder in elderly is less likely to be associated elderly!is!accompanied!by!
with paranoia as compared to younger adults. higher!suicide!risk!and!more!
H. Elderly suffering from depressive disorder takes shorter time paranoia.!Elderly!takes!longer!
to respond as compared to younger adults
time!to!respond!as!they!cannot!
I. Mild depressive disorder is more prevalent in elderly women
than men. tolerate!medications!as!good!
J. Prevalence of depressive disorder rises sharply with age in as!young!adults.!Prevalence!of!
the community. depression!in!elderly!is!around!
3%!(10N15%!if!depressive!
symptoms!are!included).!Only!
10%!of!depression!emerges!in!

!
!
194
elderly!and!there!is!no!sharp!
rise!in!prevalence.!

Year:!2014.!!

64 Which!of!the!following!is!MOST!commonly!seen!as!a! The!answer!is!C.!
normal!change!associated!with!aging?
!

F. Increase!in!acetylcholine! Explanation:!Option!A,!D!and!E!
G. Increase!in!paranoia! should!be!reduced.!Increase!in!
H. Increase!in!brain!ventricle!size!! paranoia!occurs!in!late!onset!
I. Increased!electroencephalogram!(EEG)!frequency!
depression,!bipolar!disorder!
J. Increased!slowNwave!sleep.!
and!schizophrenia!but!not!part!
of!normal!aging.!

Year:!2014!

65 What!is!the!MOST!likely!diagnosis!in!a!75T!yearTold! The!answer!is!B.!
man!with!declining!cognition,!visual!hallucinations,!
and!parkinsonism!symptoms?!!! !
!
Explanation:!The!classical!
F. Alzheimers!disease! triad!of!dementia!with!Lewy!
G. Dementia!with!Lewy!Bodies! Bodies!is!memory!loss,!visual!
H. LateNonset!schizophrenia! hallucination!and!
I. FrontoNtemporal!lobe!dementia!
parkinsonism.!
J. Vascular!dementia.!
!
!

Year:!2014!

66 Which of the following statements is false with The!answer!is!D!


regards to delirium in the elderly?
!

F. 30% or more of the elderly admitted to medical Explanation:!Delirium!will!take!


ward will develop delirium during hospitalisation. long!time!to!resolve!and!not!
G. Antipsychotic drug is the first-line psychiatric typically!resolve!in!3!days.!
treatment for delirium.
H. Delirium is often overlooked by health care !
professionals in the elderly. Year:!2014!
I. Delirium typically resolves within 3 days after

!
!
195
correcting the underlying cause.
J. Incidence of delirium increases with age.

Child and adolescent psychiatry !

67 If a young boy presents with intellectual disability, The!answer!is!D.!


autistic features and multiple maternal relatives are
carriers of a genetic condition. Which of the following !
diagnosis is MOST likely?
Explanation:!Males!with!
F. Angleman syndrome intellectual!disability,!autistic!
G. Down syndrome features!and!have!maternal!
H. Prader Willi syndrome relatives!being!carrier!highly!
I. Fragile X syndrome
J. Williams syndrome
suggestive!of!fragile!X!
syndrome.!Option!A,!C,!and!E!
are!microdeletion!syndromes!
and!less!likely!to!run!in!
families.!

Year:!2014!

Addiction and Substance abuse !

68 You are an orthopaedic resident. A 22-year-old man The!answer!is!B.!


suffers from cervical spondylosis and results in severe
neck pain. His pain does not respond to analgesics and !
becomes very depressed and suicidal. His mother
describes him as being obsessed with online games. He Explanation:!This!patient!has!
is an active member of several online game groups. He
spends more around 20 hours per day in the internet psychological!withdrawal!(i.e.!
caf and becomes very agitated when he is not allowed agitation)!when!he!is!not!
to play online games. He has history of alcohol allowed!to!play!online!game.!
dependence. Which of the following symptoms found in
this patient is MOST important in establishing the This!is!common!in!other!forms!
diagnosis of internet addiction? of!substances!abuses!(e.g.!
opioid!dependence)!when!the!
subject!is!denied!access!to!the!
F. Active member of the online group
G. Agitation when not allowed to play online game substance.!
H. Cervical spondylosis
I. Depression and suicidal thought !
J. History of alcohol abuse.
Year:!2014!

!
!
196
!

69 Which of the following is NOT a factor which increases the risk The!answer!is!C.!
of relapse to cocaine use in a 30-year-old man who recently
became abstinent from cocaine? !
Explanation:!Seeing!a!
F. Feeling depressed, tired and the need to overeat. counsellor!to!talk!about!the!
G. Going to pubs where ice and special K are available. previous!use!does!not!increase!
H. Seeing a counsellor to talk about the previous use of cocaine. the!risk!of!relapse.!Option!A!
I. Seeing a psychiatrist to complain about attention deficit and try to
obtain methylphenidate. refers!to!crash!from!
J. Spending time with colleagues who use coke. stimulation.!Option!B,!C,!and!E!
will!increase!the!exposure!to!
stimulants.!

Year:!2014!

70 A 27-year-old woman presents with unplanned The!answer!is!D.!


pregnancy. She smokes cannabis 3-4 times per
day. Her husband is concerned about the effects of !
cannabis on her baby. During the interview, she
avoids eye contact and appears to be very anxious. Explanation:!She!has!no!
She claims that she eats and sleeps well. She motivation!to!change!with!
thinks cannabis is safe for her foetus because she impaired!insight!on!the!effect!
has a friend who delivered a healthy baby despite
of!cannabis!on!the!foetus.!This!
continued use of cannabis during pregnancy.
Which Prochaskas and Diclementes stage of stage!is!known!as!preN
change BEST describes her current status? contemplation.!

F. Action !
G. Contemplation
H. Denial Year:!2014!
I. Precontemplation
J. Maintenance. !

Psychopharmacology !

71 Which!of!the!following!neurochemicals!is! The!answer!is!E.!
metabolized!by!monoamine!oxidase? !
! !

F. Acetylcholine! !
G. BrainNderived!neutrophic!factor!
H. Histamine! Explanation:!This!refers!to!
I. Glutamate! pharmacodynamic!action!of!
J. Serotonin.!

!
!
197
MAOI.!

Year:!2014!

72 A 30-year-old man takes mirtazapine every night and The answer is D.


experiences significant sedation at 7.5mg/day. Which of
the following neurotransmitter receptors are MOST likely
to be responsible for causing this side effect?
Explanation: Besides the action
of mirtazapine on the serotonin
F. Adrenergic receptors receptors, its action on the
G. Dopamine receptors
histamine receptors cause
H. Glutamate receptors
I. Histamine receptors sedation and weght gain.
J. Nicotinic receptors.
!

Year:!2014!

73 Which of the following psychotropic The answer is E.


medications is the MOST likely to raise the
prolactin levels in female psychiatric patients?
Explanation: Risperidone is
most potent D2 receptor
F. Aripiprazole
blockage among all second
G. Clozapine
H. Lithium generation antipsychotic
I. Olanzapine drugs.
J. Risperidone.

Year:!2014

74 Which of the following statements regarding The answer is D.


quetiapine is LEAST CORRECT?

f. Quetiapine has anxiolytic effects. Explanation: Unlike the other


g. Quetiapine has antidepressant effects second generation
h. Quetiapine has antipsychotic effects. antipsychotic drug, the
i. Quetiapine has extrapyramidal effects incidence of extrapyramidal
j. Quetiapine has sedative effects.
effects with quetiapine is the
same as with placebo.

Year:!2014

!
!
198
75 Which of the following is NOT a well-established side The!answer!is!A.!
effect of lithium?
!
F. Hepatotoxicity Explanation:!Lithium!is!
G. Hypothyroidism
H. Nephrotoxicity excreted!by!kidney.!It!has!
I. Tremor minimal!effects!on!the!liver.!
J. Weight Gain
!

Year:!2014

76 A 30-year-old man suffers from bipolar disorder. He The!answer!is!B.!


requests to be treated by carbamazepine because it
works on his friend. Which of the following genetic tests !
is required by the Ministry of Health (MOH) before
prescription of carbamazepine?
Explanation:!!The!MOH!and!
Health!Science!Authority!of!
F. Dopamine transporter gene - D*1502 Singapore!recommends!doctor!
G. Human leukocyte antigen gene - B*1502 to!check!HLAB*=1502!
H. Integrin alpha M gene I*1502
I. Mannan-binding lectin gene M*1502
genotype!before!prescribing!
J. Serotonin transporter gene S*1502. carbamazepine.!Those!who!are!
positive!for!this!genotype!may!
have!high!risk!of!developing!
Steven!Johnson!syndrome.!

Year:!2014

77 A 25-year-old woman is treated by bupropion for The!answer!is!E.!


depressive disorder. She gives you a list of symptoms
and attributes to the side effects of bupropion. Which of !
the following symptoms is LEAST likely to be side effect
of bupropion?
Explanation:!Bupropion!is!
associated!no!weight!gain!and!
F. Anxiety mild!degree!of!weight!loss.!
G. Nausea
H. Seizure !
I. Transient hallucination
J. Weight gain. Year:!2014

!
!
199
78 A 30-year-old schizophrenia patient receives olanzapine The!answer!is!C.!
20mg per day. He is a chronic smoker and agrees to
cease smoking in the past one month. After he quitted !
smoking, he experiences more side effects associated
with 20mg of olanzapine which include significant
Explanation:!Nicotine!induces!
sedation and weight gain. Which of the following is
MOST likely explanation? the!metabolism!of!olanzapine!
via!cytochrome!P450!1A2!and!
reduces!its!serum!
F. He is not complaint to olanzapine.
G. He took an overdose of olanzapine.
concentration.!After!he!quits!
H. After he stopped smoking, the hepatic metabolism of smoking,!the!serum!
olanzapine reduces. concentration!of!olanzapine!
I. Nicotine reduces the risk of metabolic syndrome. has!increased!and!resulted!in!
J. This is due to the effect of nicotine patch.
more!side!effects.!

Year:!2014.!

79 Which!of!the!following!is!the!mechanism!of!action!of! The!answer!is!D.!
memantine,!a!medication!used!to!slow!cognitive!
decline!in!Alzheimers!dementia? !

Explanation:!Memantine!
F. Cholinesterase!inhibitor! works!on!the!NMDA!
G. Dopamine!receptor!blocker! receptors!and!reduces!the!
H. GammaNaminobutyric!acid!receptor!agonist!
neurotoxicity!caused!by!
I. NNmethylNDNaspartate!receptor!antagonist!
J. Serotonergic!reuptake!inhibitor.! glutamate.!

Year:!2014.!

Psychotherapy !

80 A 30-year-old woman with panic disorder does not The!answer!is!B.!


respond to an initial treatment with a selective serotonin
reuptake inhibitor (SSRI). Which of the following !
treatment is considered the best approach?
Explanation:!The!effect!of!CBT!
P. Benzodiazepine
Q. Cognitive behaviour therapy
is!as!efficacious!as!SSRI.!
R. Hypnotherapy
S. Olanzapine !
T. Psychodynamic psychotherapy.
!

!
!
200
Year:!2014.!

81 You are a medical officer working in the army. A 22-year- The!answer!is!C!


old national serviceman comes to see you because he is
very angry with his male supervisor and wants to see a !
psychologist. He also informs you that he is very angry
with his own father. He passed you an emotional 500- Explanation:!Psychodynamic!
word open letter posted on his Facebook account. In his
letter, the patient mentions that his father has been psychotherapy!is!most!
abusive towards him. He finds his male supervisor is as appropriate!in!this!case!
abusive as his father and the pattern keeps repeating because!this!patient!needs!to!
itself. Which of the following psychotherapy is MOST
appropriate in this case?
analyse!the!pattern!of!history!
repeating!itself.!He!may!learn!
not!to!project!his!father!onto!
F. Cognitive behaviour therapy
the!male!supervisor.!!
G. Problem solving therapy
H. Psychodynamic psychotherapy
I. Reminiscence therapy !
J. Validation therapy.
Year:!2014.!

82 A" 20%year%old" woman" suffers" from" borderline" The!answer!is!E.!


personality" disorder." She" works" as" a" clerk." During"
!
psychotherapy" session," she" discloses" that" she" really"
hates"the"psychologist"and"has"thoughts"of"killing"the" Explanation:!The!patient!
psychologist."Her"conviction"to"kill"the"psychologist"is" exhibits!intense!negative!
1" out" of" 10" (1" =very" unlikely," 10" =" very" likely)." transference!towards!the!
Furthermore," she" does" not" have" a" plan" or" not" sure" psychologist.!She!does!not!
when" to" kill" the" psychologist." She" has" no" forensic" have!the!real!intention!to!kill!
history" and" no" history" of" violence." She" admits" the" the!psychologist!but!she!
idea" of" killing" the" psychologist" is" her" own" thought." mentions!this!due!to!intense!
She" has" no" command" hallucination." " Which" of" the" negative!transference.!!
following"BEST"describes"this"phenomenon?
6 !

F. Chronic felling of emptiness Year:!2014.!


G. Displacement
H. Projective identification
I. Splitting
J. Transference6

83 A 30-year-old man suffers from disseminated The!answer!is!A.!


gonococcal infection. The medical team asks him about
unprotected sexual activities. He claims he has one !
stable sexual partner and practises safe sex all the time.
The medical team thinks that his condition could due to Explanation:!He!denies!the!
an underlying autoimmune cause because he has no
risky behaviour. Several days later, he finally admits that threatening!fact!that!he!has!
he has sexual intercourse with commercial sex worker been!practising!unprotected!
once per month. Sometimes, he does not use condom to sex!with!commercial!sex!

!
!
201
protective himself. What is the initial defence workers!and!result!in!
mechanism used by this patient? gonorrhoea!infection.!

F. Denial
!
G. Displacement
H. Projection !
I. Projective identification !
J. Undoing
Year:!2014!

Ethics and laws !

84 You are a general practitioner. A 30-year-old female The!answer!is!A.!


secretary complains of low mood, poor appetite, poor
sleep and suicidal thought. She is very depressed over !
the death of her son who passed away 26 hours after
birth. She has history of depression without any
Explanation:!Most!depressed!
psychotic feature. She underwent a traumatic delivery patients!have!the!capacity!to!
because an obstetrician in private practice performed give!consent.!Based!on!the!
amniocentesis, induction of labour and administered
history,!there!is!no!evidence!to!
syntocinon 3 weeks before the expected date of
delivery. The patient later found out that the obstetrician suggest!that!she!lacks!the!
induced labour because the doctor planned to go for a capacity!to!make!decision!!(e.g.!
holiday. Patient did not fully understand about the no!psychotic!features,!working!
procedure and did not agree with induction of labour
which resulted in premature birth of her son. The as!a!secretary).!The!
obstetrician claims that she does not have capacity to obstetrician!did!not!respect!
make decision due to history of depression. She felt that patients!autonomy!before!
the doctor performed the procedure against her will and
carrying!out!aminocentesis,!
the death of her son was avoidable. Which ethical
principle was violated by the obstetrician? induction!of!labour!and!
administering!syntocinon.

F. Autonomy
G. Beneficence
H. Confidentiality Year:!2014.!
I. Involuntary treatment
J. Justice.

85 You are a resident posted to the Accident and The!answer!is!C.!


Emergency Department of Institute of Mental
Health (IMH). A 55-year-old man attended a party in !
a buffet restaurant. He seldom drinks alcohol. In
the party, he drank a few glasses of hard liquor Explanation:!Automatism!is!a!
and could not find his way to the table. When a legal!term!and!the!person!
female waitress tried to help him, he was irritable commits!a!crime!without!his!or!
and physically assaulted her. The waitress
her!own!personal!control!or!
suffered from minor injury but the restaurant
manager insisted to report to the police. awareness!due!to!an!
Subsequently, he was arrested and stayed in the underlying!medical!condition!
police station for one night. On the next day, he or!substances.!Automatism!can!
was sent to IMH for assessment. When you be!caused!by!alcohol!

!
!
202
interview him, he says, Oh gosh, I cant recall the intoxication,!sleep!walking,!
event which occurred last night. He does not have epilepsy!and!hypoglycaemia.!!
past psychiatric illness. Which of the following
terms BEST describes his behaviour leading to the !
arrest?
Year:!2014.!
A. Actus reus
B. Jealousy
C. Automatism
D. Diminished responsibility
E. Mens rea.

Liaison Psychiatry !

86 A 20-year-old man was given a high dose of The answer is C.


intravenous haloperidol. He develops high fever
and you suspect that he may develop neuroleptic
malignant syndrome (NMS). All of the following
clinical features suggest NMS EXCEPT: Explanation: Patients
suffering from NMS usually
present with elevated CK,
F. Autonomic instability ALT, AST, LDH in the serum
G. Diaphoresis
and high levels of myoglobin
H. Decreased catecholamines in urine
I. Myoglobinuria and protein in the urine
J. Rigidity.

Year:!2014.

87 An obstetrician refers a 30-year-old woman The!answer!is!A.!


who presents with psychosis after delivery to
you. She wants to know whether this woman !
suffers from postpartum psychosis. You want
Explanation:!The!onset!is!
to find out from the obstetrician the onset of
psychotic symptoms after delivery. Which of usually!between!1!to!6!weeks!
the following days of onset is MOST congruent because!the!progresterone!
with the diagnosis of postpartum psychosis? levels!fall!but!dopamine!levels!
rise!in!the!days!after!child!
F. 1- 6 weeks after delivery birth.!
G. 7-12 weeks after delivery
!
H. 13 -18 weeks after delivery
I. 19 - 24 weeks after delivery Year:!2014.!
J. 25 31 weeks after delivery.
!

88 A 40-year-old man went to South Korea for skiing The!answer!is!D.!


and suffered from head injury. All of the following
symptoms are commonly associated with sequelae of

!
!
203
head injury EXCEPT: !

Explanation:!Obsession!is!not!a!
F. Light and noise sensitivity
G. Headache common!symptom!of!postN
H. Insomnia concussion!syndrome.!
I. Obsession
!
J. Word finding difficulty.
!

Year:!2014.!

89 Which of the following is LEAST likely to be an area of The!answer!is!C.!


concern in a 30-year-old woman suffering from body
dysmorphic disorder? !
F. Breast
Explanation:!Genitalia!is!least!
G. Nose
H. Genitalia likely!to!be!an!area!of!concern!
I. Hair among!the!other!options.!The!
J. Skin. common!areas!are!listed!as!
follows:!skin!(73%),!hair!
(56%),!nose!(37%),!weight!
(22%),!stomach!(22%),!
breasts/chest/nipples!(21%)!!

eyes!(20%)!thighs!(20%)!teeth!
(20%).!

Year:!2014.!

90 You are a medical resident. A 40-year-old man is The!answer!is!A.!


admitted to the medical ward due to altered mental
state and requires further medical investigations. !
Past records show a history of substance abuse.
He sleeps poorly and paces around the unit, Explanation:!The!patient!
restless and grumpy on the next day. He appears suffers!from!delirium!tremens,!
to be confused and exhibits tachycardia at 106 resulted!from!alcohol!
beats per minutes. His palms and forehead are
withdrawal.!
sweaty and his tongue is showing a course tremor.
When asked, the patient says he feels anxious.
!
Which of the following is the CORRECT diagnosis?
Year:!2014.!
F. Alcohol withdrawal
G. Amphetamine withdrawal
H. Cannabis withdrawal
I. Nicotine withdrawal
J. Opioid withdrawal.
!

!
!
204
!

!
!
205
Paper 8- Questions

History, mental state exam, psychopathology, diagnosis

1. You are a resident doctor working at the Institute of Mental Health (IMH). The police
officers bought a 39-year-old man charged with possessing a scheduled weapon
and was sent to the IMH for a psychiatric evaluation. Although he speaks English,
you cannot make out what he is saying because his speech becomes diffuse and
unfocused. Which of the following best describes his psychopathology?

A. Circumstantiality
B. Loosening of association
C. Thought insertion
D. Thought withdrawal
E. Tangentiality.

2. You are a resident working at the Accident and Emergency Department (AED). A
35-year-old man was sent to the AED for psychiatric assessment by an ambulance.
He was involved in a road traffic accident. He was a passenger in a taxi. He told the
taxi driver that he was God and threatened the driver to believe that he was the
saviour of the world. The driver found him very irritable and he kept hitting the head
of the driver. The driver stopped the taxi and tried to call the police. The patient
drove the taxi at high speed and crashed the taxi into a lamp post. He was not under
influence of alcohol at the time of accident. You are required to enter a diagnosis
into the AED computerised system. Based on the information provided, which of the
following psychiatric diagnosis is most relevant?

A. Antisocial personality disorder


B. Bipolar disorder
C. Conversion disorder
D. Dissociative disorder
E. Schizophrenia.

Cognitive assessment

3. A 30-year-old woman has received 6 sessions of electroconvulsive therapy (ECT).


After the ECT, she complains of cognitive impairment and attributes the following
symptoms as side effects of ECT. Which of the following is most likely caused by
ECT?

A. Cannot follow the instructions given by her supervisor after she returns to work
B. Cannot recall the password of her email account
C. Cannot remember details of her ex-marriage
D. Difficulty to perform two tasks at one time
E. Unable to calculate.

!
!
206
4. You are administering the Serial 7 test to a patient. His answers are listed as follows: 100
7 = 90; 90 7 = 77; 77 7 = 70; 70 7 = 63; 63 7 = 50. What is his score?

A. 0 out of 5
B. 1 out of 5
C. 2 out of 5
D. 3 out of 5
E. 4 out of 5.
Psychiatric epidemiology

5. Which of the following is the most common psychiatric co-morbidity of panic


disorder?

A. Alcohol misuse
B. Agoraphobia
C. Avoidant personality disorder
D. Benzodiazapine misuse
E. Generalised anxiety disorder.
Psychiatric aetiology, diagnosis and classification

6. The prevalence of borderline personality disorder in Singapore and worldwide


has increased in the past three decades. The most likely explanation for this
observation is:

A. Changes in diagnostic criteria


B. Increase in competition in schools
C. Increase in divorce rate
D. Increase in immigration and migration
E. Increasing urbanization of the population.

7. Which of the following is least likely to be a risk factor for delusional disorder?

K. Having a religion
L. Increased age
M. Immigration
N. Sensory impairment
O. Social isolation.

8. The genetic polymorphisms of which of the following neurotransmitter transporter


genes affect the risk of developing depressive disorder?

A. Dopamine

!
!
207
B. -Aminobutyric acid (GABA)
C. Glutamate
D. Norephinephrine
E. Serotonin.
9. A 40-year-old patient with schizophrenia presents to the emergency department with
confusion and agitation following an overdose of haloperidol. The patient has a
temperature of 41 degree Celsius, blood pressure of 160/100 mm Hg, and pulse of
120 beats per minute. Physical examination reveals rigidity, hot, wet skin, and
decreased bowel sounds. This patients symptoms, are most likely caused by
blocking of which of the following receptors?

A. Alpha1-adrenergic
B. Dopaminergic
C. Histaminergic
D. Muscurinic cholinergic
E. Nicotinic cholinergic.
General adult psychiatry

10. A 30-year-old man suffers from schizophrenia. He first exhibited first rank symptoms
at the age of 16 and the onset was insidious. He exhibits anhedonia although his
hallucinations are under control by antipsychotic drug. His uncle suffers from
depressive disorder. All of the following are poor prognostic factors except?

A. Anhedonia
B. Exhibition of first rank symptoms at the age of 16
C. Family history of depressive disorder
D. Male gender
E. Insidious onset.

11. You are the resident working in the Accident and Emergency Department. A
schizophrenia patient informs you that he has homicidal thought. Which of the
following sign or symptom is least important in predicting homicide?

A. Command hallucinations
B. Delusion of reference
C. History of previous violence
D. Irresistible urge to attack
E. Need to defend oneself as are result of persecutory delusion.

12. All of the following are poor prognostic signs for obsessive compulsive disorder
except:

A. Bizarre compulsions
B. Childhood onset
C. Episodic course
D. Comorbid major depression

!
!
208
E. Succumbing to obsessions and compulsions.

13. You are a general practitioner. A 35-year-old hotel butler came to see you because
his lawyer wants him to be seen by a psychiatrist. He was recently arrested for
installing pinhole camera in the hotel guestrooms. He secretly filmed couples having
sex in the hotel guestrooms. He has filmed more than 100 videos and enjoyed
watching them over and over again. He claims that he has the compulsion to collect
those videos and he feels relieved after the arrest. Which of the following is the most
likely diagnosis?

A. Erotomania
B. Kleptomania
C. Obsessive compulsive disorder
D. Sadomasochism
E. Voyeurism.

14. A 30-year-old woman always believes that she is socially inept and fears of negative
evaluation by other people. Her sisters describe her as timid and insecure. She is
single and stays with two elder sisters. She was admitted to the psychiatric ward due
to nervous breakdown after meeting new colleagues in her workplace. She refuses
to attend family meeting with her sisters because she feels that her sisters do not
like her although they appear to be very concerned about her condition. She is
isolated and has one best friend in her work place. Which of the following personality
best describes this patient?

A. Avoidant personality disorder


B. Borderline personality disorder
C. Histrionic personality disorder
D. Paranoid personality disorder
E. Schizoid personality disorder
15. Which of the following is not a common feature of serotonin syndrome?

A. Acidosis
B. Diaphoresis
C. Hyperreflexia
D. Hypothermia
E. Myoclonus.

!
!
209
16. You are a resident working in the accident and emergency department
(AED). A single mother was sent to the AED with her 2-year-old daughter.
She was arrested by police because she tried to assault her daughter and
stopped by passer-by. According to the informant, her partner refuses to
marry her due to her drug habit. She does not have past forensic history. She
appears to be agitated and restless. At one moment, she wants to stay in the
AED. At another moment, she wants to walk out of the AED. Physical
examination reveals nystagmus. Which of the following diagnosis is most
relevant?
A. Antisocial personality disorder
B. Cannabis intoxication
C. Delusion of jealousy
D. Paranoid schizophrenia
E. Phencyclidine intoxication
Old age psychiatry

17. You are about to start a selective serotonin reuptake inhibitor (SSRI) for an 80-year-
old man who suffers from depression. Which of the following pharmacokinetic
changes has the least effect on drug therapy in the elderly?

A. Absorption
B. Distribution
C. Excretion
D. Metabolism
E. Protein binding.

18. A 79-year-old man complains of extreme drowsiness when he goes to toilet at


midnight and he almost fell down on several occasions. He wants to avoid fall at
night. The following is the list of his medications. Which of the following medications
should be stopped?

A. Melatonin
B. Mirtazapine
C. Risperidone
D. Rivastigmine
E. Sodium valproate.

19. A 70-year-old woman suffering from bipolar disorder and her daughter came to see
you today. Her daughter read about the symptoms of bipolar disorder and found her
mother different from younger adults suffering from the same condition. Which of the
following clinical features is more likely to be found in this 70-year-old woman?

A. Demonstrates less paranoid ideas as compared to adult patients


B. Demonstrates more flight of ideas as compared to adult patients
C. More likely to be hyperactive as compared to adult patients
D. More likely to demonstrate mixed (depressive/manic) clinical presentations as

!
!
210
compared to adult patients
E. More spending as compared to adult patients.

20. Which of the following is most important risk factor for vascular dementia?

A. History of depression
B. History of poor academic performance
C. History of hypertension
D. High level of physical activity
E. High level of high density lipoproteins.

21. You are the visiting physician of a nursing home. The nursing home staffs feel very
helpless when handling aggression in dementia patients and they need to seek your
advice. Which of the following psychotropic medication have the most evidence in
managing aggression in patients suffering from dementia?

A. Antipsychotic drug
B. Beta-blocker
C. Benzodiazapine
D. Lithium
E. Stimulant.

Child and adolescent psychiatry

22. In children and adolescents, high stimulation-seeking behaviour is associated with


which of the following conditions?

A. Adolescent-onset schizophrenia
B. Autism
C. Conduct disorder
D. Obsessive-compulsive disorder
E. School refusal.

23. Children with autism are most comfortable in which of the following situations?

A. Attending a party where there are a lot of surprises and have a chance to meet
different people.
B. Attending drama classes and imagines oneself playing different roles in the drama.
C. Looking for toys in a multi-storey department store from one level to another level.
D. Travelling in a bus when it stops at every bus stop until it reaches the bus terminal.

!
!
211
E. Going to school where there are multiple classes to attend in a day and teaching
content varies from subject to subject.
24. Which of the following statements is most correct regarding the impact of divorce on
children?

A. Boys aged two to four are not significantly affected.


B. Children do best if they have no contact with the father for two years.
C. Most children have no significant problems after three to six months.
D. Socio-economic status is not associated with the degree of impairment.
E. To evaluate the impact, one must take into consideration the age and the
developmental level of the child.

25. A 15-year-old obese female is brought by her parents for an evaluation as part of the
admission process for a weight management program. The adolescent agrees with
the fact that she is overweight and she is keen to lose weight. Which of the following
is most indicative that she will be successful in the program?

A. Her current BMI is between 26 and 30.


B. Mother suffered from obesity and successfully lost weight
C. No family history of eating disorders
D. Motivation to take antidepressant
E. Willingness to change her eating habits.
Substance abuse

26. Methylene dimethylamphteamine (street name: ecstasy) usually cause people with
which of the following symptom?
a. Amotivation
b. Closeness to others
c. Depression
d. Introversion
e. Phobia.

27. A 30-year-old adult claims to suffer from adult onset attention deficit and
hyperactivity disorder (ADHD). He has been seeing different doctors to obtain
methylphenidate. He claims that he lost all methylphenidate in the bus few days ago
and his wife has noticed that he looks very tired. Which of the following is most
appropriate explanation for his tiredness?

A. He develops depression and psychomotor retardation


B. He exhibits secondary gain and seeking attention from his wife
C. He suffers from comorbid somatisation disorder in addition to ADHD
D. He suffers from psychological withdrawal symptoms from stimulant misuse
E. He suffers from stimulant intoxication.
28. The most important objective of psychological intervention in treating people
suffering from alcohol misuse is:

!
!
212
A. To conduct in a group therapy format
B. To apply cognitive behaviour therapy (CBT) as the first approach
C. To combine psychological intervention and pharmacotherapy (e.g. disulfiram)
D. To focus on achieving and maintaining abstinence or reducing alcohol use
E. Explore unresolved psychodynamic conflicts.

Psychopharmacology

29. Which of the following medications is not associated with weight gain?

A. Clozapine
B. Mirtazapine
C. Olanzapine
D. Sodium valproate
E. Topiramate.

30. A 60-year-old man is started on an antidepressant for treatment of major depressive


disorder. Three weeks later, he complains of blurred vision, dry mouth, constipation,
urinary retention and tachycardia. Which of the following medications is most likely
to have caused this presentation?

A. Agomelatine
B. Duloxetine
C. Fluoxetine
D. Imipramine
E. Mirtazapine

31. Which of the following medications is most likely to be associated with polycystic
ovarian syndrome in female patients?

A. Carbamazapine
B. Lamotrigine
C. Lithium
D. Sodium valproate
E. Topiramate.

32. Regarding an antidepressant called bupropion, which of the following statements is


false?

A. It causes insomnia.
B. It has a half-life of approximately 12 hours.
C. It is safe for patients suffering from bulimia nervosa.
D. It is an antidepressant used for depressed patients with cardiovascular disease and

!
!
213
habit of smoking.
E. It does not cause weight gain.

33. Mirtazapine is associated with lesser sexual side effects. Which of the following
pharmacodynamics actions explains this phenomenon?

A. Serotonin receptor 5HT1A agonism


B. Serotonin receptor 5HT2A antagonism
C. Serotonin receptor 5HT2C antagonism
D. Serotonin receptor 5HT3 antagonism
E. Histamine receptor antagonism.

34. Which of the following side effects is least likely to occur in patients taking
quetiapine?

A. Antihistamine effects
B. Orthostatic hypotension
C. Increase in liver transaminase
D. Increase in prolactin
E. Weight gain.

35. Which of the following is the best treatment option for severe depressive episode
with psychotic features?

A. Amitriptyline
B. Cognitive behaviour therapy
C. Lamotrigine
D. Methylphenidate
E. Electroconvulsive therapy.

36. A 35-year-old man suffers from schizophrenia symptoms and his GP has started him
a medication to treat his symptoms. He develops jaundices after taking the
medication. Which of the following medications is most likely to cause jaundice?

A. Clozapine
B. Chlorpromazine
C. Haloperidol
D. Risperidone
E. Sulpiride.

37. A 60-year-old man suffers from depressive disorder and you have increased the

!
!
214
dose of antidepressant. He is seeing you today. His blood pressure is 180/90. Which
of the following medications should be stopped?

A. Diazepam
B. Quetiapine
C. Fluvoxamine
D. Hydroxyzine
E. Venlafaxine.

38. A 25-year-old woman with bipolar disorder is pregnant for the first time. Throughout
the pregnancy, she has been maintained on a mood stabilizer. At the time of birth,
the baby is noted to have Ebsteins anamoly. Which of the following medications is
most responsible for this teratogenic effect?

A. Carbamazepine
B. Lamotrigine
C. Lithium
D. Sodium valproate
E. Topiramate
Psychotherapy and defence mechanisms

39. A 28-year-old woman comes to consult you because she is very upset after being
scolded by her manager. The chief executive officer (CEO) of her company has
raised the standard for annual performance. Her manager is very upset this
arrangement and scolds her for no reason. Which of the following is the defence
mechanism exhibits by the manager?

A. Altruism
B. Denial
C. Displacement
D. Repression
E. Reaction formation.

40. You are a doctor working in polyclinic. A 25-year-old man bought his 55-year-old
mother to see you. He is concerned that his mother suffers from depression but she
refuses to take antidepressant. You realise that the mother and son are enmeshed
as the son is unmarried and not able to work. He claims that he needs to look after
his mother although she has good past health. The mother and son hold very
negative view against the father who is the sole breadwinner of family. There is a
psychologist working in your polyclinic. Which of the following forms of psychological
treatment is most appropriate?

A. Couple therapy
B. Dialectical behaviour therapy
C. Problem solving psychotherapy
D. Supportive psychotherapy

!
!
215
E. Family therapy.

41. A national serviceman is seeing a male psychologist because he suffers from


adjustment disorder and he finds his army supervisor uncaring. Furthermore, he is
very angry with his own father. He recalls an incident in camp about getting angry
with his male supervisor. The psychologist asks a factual question about the
circumstance and rage in the patient. Then the patient complains that the
psychologist is uncaring and trying to find fault in him. The psychologist says, I
wonder if what youve feeling right now is just like the feeling you have in camp,
when you attributed the same uncaring attitude toward your father and supervisor in
camp. These repetitive patterns of misunderstanding seem to make you very upset
in different situations and affect your life. Which of the following most accurately
describes the psychotherapeutic technique?

A. Affirmation
B. Clarification
C. Empathic validation
D. Observation
E. Interpretation.

42. A medical student used to be very keen to become an orthopaedic surgeon. He has
assisted an orthopaedic surgeon to conduct research since the second year of
medical school. During the residency application, he tells other classmates that he
does not bother whether he is accepted by the orthopaedic residency. He also
encourages other classmates not to apply for orthopaedic surgery as this specialty is
not a good career choice. On the other hand, he is secretly asking his orthopaedic
mentor to prepare him for the orthopaedic residency intake interview. What is his
defence mechanism?

A. Altruism
B. Denial
C. Displacement
D. Repression
E. Reaction formation.
43. Which of the following is the most effective approach for a 30-year-old woman with
obsession about contamination and washes her hands every time she touches
something which she considers dirty?

A. Having the patient place her hands in a container of worms to create extreme phobia
and ability to overcome obsession
B. Having the patient snap her wrist with a rubber band when she thinks about
contamination and about to wash her hands
C. Having the patient touch a dirty object, then not allowing her to wash her hands for
several hours
D. Highlighting to the patient that her obsession is a cognitive error.
E. Providing the patient with message cards to remind herself that the hand washing is
unnecessary.
44. A 30-year-old army officer presents for treatment of post-traumatic stress disorder
after surviving an accident which his tank fell into the river from a bridge. He was

!
!
216
driving a tank on a bridge when it suddenly collapsed. Several army officers were
seriously injured in this accident. As part of the treatment, the psychologist asks the
patient to imagine that he is safely driving his tank over a bridge. Which of the
following best describes this therapeutic intervention?

A. Aversion therapy
B. Cognitive therapy
C. Exposure therapy
D. Interpersonal therapy
E. Supportive therapy.

Ethics, management dilemma and laws

45. You are a paediatric resident. A 14-year-old girl has stayed in the paediatric ward for
1 week after a suicide attempt by drug overdose. She is due for discharge tomorrow.
At 6 pm, she suddenly claims that her elder sister had physically abused her prior to
admission. Her medical record reveals that she suffers from conduct disorder. She
also made similar reports of physical abuse in the past. The police officer and social
worker found her claims inconclusive and did not investigate further. She is
ambivalent to make a report about the alleged abuse and she is not keen for
discharge. You try to contact the consultant in charge of this case but the consultant
is not contactable. Which of the following is most appropriate action?

A. Banning her parents from visiting her.


B. Call 995 to inform the police immediately
C. Inform the on-call team and request the on-call doctors to inform the police before
midnight
D. Refer the patient to gynaecologist for further assessment
E. Discuss with the consultant-in-charge and the social worker on the next day.

46. A 35-year-old man was hospitalised for treatment of severe depressive episode in
September. The inpatient team prescribed two antidepressants for the patient and
then referred the patient to polyclinic. The patient came to the polyclinic for review in
October and November respectively. The patient had a relapse of depression and
committed suicide in December. The computerised record shows that the polyclinic
doctors had forgotten to prescribe antidepressants since the first visit. Which of the
following best describes the above situation based on the Common Law?

A. Battery
B. Diminished responsibility
C. Intentional torts
D. Negligence
E. Tarasoffs rule

Liaison psychiatry and neuropsychiatry

47. You are a resident in obstetrics. Which of the following is the safest medication to be

!
!
217
given to a pregnant woman who develops psychosis during pregnancy?

A. Aripiprazole
B. Haloperidol
C. Olanzapine
D. Quetiapine
E. Risperidone
48. A 25-year-old man expresses concerns that he got infected with HIV after visiting
commercial sex workers last week. According to him, he only received hand-genital
stimulation and there was no vaginal intercourse. He also mentions that the
commercial sex worker kissed his body. He has sought his girl-friends forgiveness.
Today, he requests HIV test urgently. He consulted you one year ago for similar
concern after he kissed another lady without informing his girl-friend. What is the
most likely explanation of his behaviour?

A. Underlying depression
B. Underlying guilt
C. Underlying obsession
D. Underlying psychosis
E. Underlying somatic concern
49. You are orthopaedic resident. A 25-year-old construction worker had a fall, fractured
his spine and injured the brain. He develops pain in multiple sites. He has one
episode of epilepsy after the injury. He is depressed but does not have suicidal
thought. Electrocardiogram is normal. Which of the following medication is most
helpful in this situation?

A. Amitriptyline

B. Bupropion

C. Chlorpromazine

D. Duloxetine

D. Moclobemide

50. You are a medical resident and rotated to rheumatology. A 24-year-old


woman suffers from systemic lupus erythematosus (SLE). She suffers from
lupus nephritis and requires dialysis three times a week. She was very
concerned about her illness and felt hopeless. She took an overdose of her
lupus medications and admitted to the medical ward. You need to take a
history from her. Which of the following is not part of the neuropsychiatric
symptoms of SLE experienced by this patient?
A. Anxiety
B. Binge eating
C. Cognitive impairment

!
!
218
D. Epilepsy
E. Headache.
!

MCQ exam 2014/2015 Rotation 4

History, mental state exam, psychopathology,


diagnosis

51. You are a resident doctor working at the Institute The answer is B.
of Mental Health (IMH). The police officers
bought a 39-year-old man charged with
possessing a scheduled weapon and was sent to
the IMH for a psychiatric evaluation. Although he Explanation: Loosening of
speaks English, you cannot make out what he is association is defined as a
saying because his speech becomes diffuse and speech where topics seem to be
unfocused. Which of the following best describes disconnected and it is hard for
his psychopathology?
others to establish a logical link
between topics. The speech
F. Circumstantiality
G. Loosening of association becomes diffuse and unfocused.
H. Thought insertion
I. Thought withdrawal
J. Tangentiality.
Year: 2014.

52. You are a resident working at the Accident and The answer is B
Emergency Department (AED). A 35-year-old
man was sent to the AED for psychiatric
assessment by an ambulance. He was involved
in a road traffic accident. He was a passenger in Explanation: This man suffers
a taxi. He told the taxi driver that he was God and from bipolar disorder, manic
threatened the driver to believe that he was the episode as evidence based by
saviour of the world. The driver found him very grandiose delusion, irritability,
irritable and he kept hitting the head of the driver.
reckless and dangerous
The driver stopped the taxi and tried to call the
police. The patient drove the taxi at high speed behaviour.
and crashed the taxi into a lamp post. He was not
under influence of alcohol at the time of accident.
You are required to enter a diagnosis into the
Year: 2015.
AED computerised system. Based on the
information provided, which of the following
psychiatric diagnosis is most relevant?

F. Antisocial personality disorder


G. Bipolar disorder
H. Conversion disorder

!
!
219
I. Dissociative disorder
J. Schizophrenia.

Cognitive assessment

53. A 30-year-old woman has received 6 sessions of The answer is B.


electroconvulsive therapy (ECT). After the ECT,
she complains of cognitive impairment and
attributes the following symptoms as side effects
of ECT. Which of the following is most likely Explanation: Young patients
caused by ECT? usually develop retrograde
amnesia of recent events but not
anterograde amnesia (option A)
F. Cannot follow the instructions given by her
or life events (option C). ECT
supervisor after she returns to work
G. Cannot recall the password of her email account does not affect multi-tasking and
H. Cannot remember details of her ex-marriage calculation.
I. Difficulty to perform two tasks at one time
J. Unable to calculate.

Year: 2015.

54. You are administering the Serial 7 test to a patient. The!answer!is!C.!


His answers are listed as follows: 100 7 = 90; 90 7
= 77; 77 7 = 70; 70 7 = 63; 63 7 = 50. What is his !
score?
Explanation:!100!!7!=!90!(0!
F. 0 out of 5 point)!
G. 1 out of 5
H. 2 out of 5 90N7!=!77!(0!point)!
I. 3 out of 5
J. 4 out of 5. 77N7!=!70!(1!point)!

70N7!=63!(1!point)!

63N7!=!50!(0!point)!

Total:!2!points!out!of!5!

Year:!2014.!

Psychiatric epidemiology

55. Which of the following is the most common The answer is B.


psychiatric co-morbidity of panic disorder?

!
!
220
Explanation: Agoraphobia (40%)
F. Alcohol misuse > alcohol dependence (30%).
G. Agoraphobia
The other options are less likely
H. Avoidant personality disorder
I. Benzodiazapine misuse to be associated with panic
J. Generalised anxiety disorder. disorder.

Year: 2014.

Psychiatric aetiology, diagnosis and classification

The answer is C.

56. The prevalence of borderline personality


disorder in Singapore and worldwide has
increased in the past three decades. The most Explanation: increase in divorce
likely explanation for this observation is: rate is associated with trauma,
neglect and splitting in family.
These are important
F. Changes in diagnostic criteria
G. Increase in competition in schools predisposing factors for
H. Increase in divorce rate borderline personality disorder.
I. Increase in immigration and migration There is no major change in
J. Increasing urbanization of the population. diagnostic criteria. Some of the
factors may occur too late to
affect personality development
(e.g. migration). As Singapore is
a city state, the effect of
urbanization is minimal.

Year: 2014

57. Which of the following is least likely to be a risk The answer is A.


factor for delusional disorder?

P. Having a religion Explanation: Having a religion is


Q. Increased age not a risk factor for developing
R. Immigration delusional disorder. The risk of
S. Sensory impairment delusional disorder increases
T. Social isolation.
with age. Hearing loss is a risk
factor. Low socioeconomic status
and severe stress are also risk

!
!
221
factors.

Year: 2014

58. The genetic polymorphisms of which of the The answer is E.


following neurotransmitter transporter genes
affect the risk of developing depressive disorder?

F. Dopamine Explanation: Individuals with


G. -Aminobutyric acid (GABA) homozygous short alleles at the
H. Glutamate serotonin transporter gene are
I. Norephinephrine more likely to develop
J. Serotonin.
depression.

Year: 2014

59. A 40-year-old patient with schizophrenia presents The answer is B.


to the emergency department with confusion and
agitation following an overdose of haloperidol.
The patient has a temperature of 41 degree
Celsius, blood pressure of 160/100 mm Hg, and Explanation: This patient
pulse of 120 beats per minute. Physical develops neuroleptic malignant
examination reveals rigidity, hot, wet skin, and syndrome due to blockage of
decreased bowel sounds. This patients dopamine receptors by
symptoms, are most likely caused by blocking of
haloperidol.
which of the following receptors?

F. Alpha1-adrenergic
G. Dopaminergic Year: 2014
H. Histaminergic
I. Muscurinic cholinergic
J. Nicotinic cholinergic.
General adult psychiatry

60. A 30-year-old man suffers from schizophrenia. The answer is C.


He first exhibited first rank symptoms at the age
of 16 and the onset was insidious. He exhibits
anhedonia although his hallucinations are under
control by antipsychotic drug. His uncle suffers Explanation: Poor prognostic
from depressive disorder. All of the following are factors include young onset, no
poor prognostic factors except? precipitating factors, insidious
onset, poor premorbid

!
!
222
F. Anhedonia functioning, family history of
G. Exhibition of first rank symptoms at the age of 16 schizophrenia, negative
H. Family history of depressive disorder symptoms, poor support,
I. Male gender
neurological symptoms and poor
J. Insidious onset.
compliance.

Year: 2014

61. You are the resident working in the Accident and The answer is B.
Emergency Department. A schizophrenia patient
informs you that he has homicidal thought. Which
of the following sign or symptom is least
important in predicting homicide? Explanation: Delusion of
reference is not directly related
F. Command hallucinations to violence and carries lower risk
G. Delusion of reference of homicide as compared to
H. History of previous violence
other signs and symptoms.
I. Irresistible urge to attack
J. Need to defend oneself as are result of
persecutory delusion.
Year: 2014

The answer is C.

62. All of the following are poor prognostic signs for


obsessive compulsive disorder except:
Explanation:

F. Bizarre compulsions Poor prognosis:


G. Childhood onset
H. Episodic course 1. yielding to compulsions
I. Comorbid major depression (not resisting)
J. Succumbing to obsessions and compulsions. 2. childhood onset
3. bizarre compulsions
4. need for hospitalization
5. presence of overvalued
ideas (i.e. acceptance)
6. personality disorders
(especially schizotypal)
7. co-existing delusional
beliefs
8. co-existent major
depressive disorder

Year: 2014

63. You are a general practitioner. A 35-year-old The answer is E.


hotel butler came to see you because his lawyer

!
!
223
wants him to be seen by a psychiatrist. He was
recently arrested for installing pinhole camera in
the hotel guestrooms. He secretly filmed couples Explanation: Voyeurism is
having sex in the hotel guestrooms. He has defined as recurrent and intense
filmed more than 100 videos and enjoyed sexual arousal from observing
watching them over and over again. He claims other people being naked or
that he has the compulsion to collect those
engaging sexual activity.
videos and he feels relieved after the arrest.
Which of the following is the most likely
diagnosis?
Year: 2014
F. Erotomania
G. Kleptomania
H. Obsessive compulsive disorder
I. Sadomasochism
J. Voyeurism.

64. A 30-year-old woman always believes that she is The answer is A.


socially inept and fears of negative evaluation by
other people. Her sisters describe her as timid
and insecure. She is single and stays with two
elder sisters. She was admitted to the psychiatric Explanation: She suffers from
ward due to nervous breakdown after meeting avoidance personality disorder
new colleagues in her workplace. She refuses to as evidenced by pervasive
attend family meeting with her sisters because tension (e.g. nervous
she feels that her sisters do not like her although
breakdown), unwilling to be
they appear to be very concerned about her
condition. She is isolated and has one best friend involved unless being liked (e.g.
in her work place. Which of the following family meeting with her sister),
personality best describes this patient? fear of criticism, showing
restraint in intimate relationship
(e.g. being single) and inhibition
F. Avoidant personality disorder
of new interpersonal situations
G. Borderline personality disorder
H. Histrionic personality disorder (e.g. meeting new colleagues)
I. Paranoid personality disorder and belief of socially inept.
J. Schizoid personality disorder

Year: 2014

65. Which of the following is not a common feature of Answer: D


serotonin syndrome?

F. Acidosis
G. Diaphoresis Explanation: Serotonin syndrome
H. Hyperreflexia actually causes fever. Metabolic

!
!
224
I. Hypothermia acidosis in seen in 9% of
J. Myoclonus. patients.

Year: 2014

66. You are a resident working in the accident The answer is E.


and emergency department (AED). A single
mother was sent to the AED with her 2-year- Explanation: The immediate
old daughter. She was arrested by police diagnosis is PCP intoxication
because she tried to assault her daughter
as evidenced by
and stopped by passer-by. According to the
informant, her partner refuses to marry her unpredictability (staying in
due to her drug habit. She does not have AED versus walking out of
past forensic history. She appears to be AED), assaultativeness on her
agitated and restless. At one moment, she daughter, agitation and
wants to stay in the AED. At another nystagmus. Antisocial
moment, she wants to walk out of the AED. personality disorder requires
Physical examination reveals nystagmus.
further evidence as forensic
Which of the following diagnosis is most
relevant? history is negative and cannot
F. Antisocial personality disorder explain nystagmus.
G. Cannabis intoxication
H. Delusion of jealousy
I. Paranoid schizophrenia
J. Phencyclidine intoxication Year: 2014

Old age psychiatry

67. You are about to start a selective serotonin The answer is A.


reuptake inhibitor (SSRI) for an 80-year-old man
who suffers from depression. Which of the
following pharmacokinetic changes has the least Explanation: There is no
effect on drug therapy in the elderly? consistent age effect on
absorption of antidepressant in
F. Absorption elderly. For distribution, the
G. Distribution
elderly have more fat stores;
H. Excretion
I. Metabolism therefore they will have lower
J. Protein binding. plasma concentrations of lipid
soluble drugs, but will have
larger distribution volumes of
these drugs and therefore longer
half-lives. This may cause
problems for elderly taking
fluoxetine.

!
!
225
For metabolism, all psychotropic
medications metabolized in the
liver have their elimination half-
life increased two or three-fold in
the elderly.

For excretion, decreased GFR


and renal blood flow lead to
decreased clearance of lithium
and antidepressant metabolites.

For protein binding, the elderly


have lower levels of plasma
albumin due to malnutrition, and
therefore have an increased free
fraction of protein bound drugs.

Year: 2014

This is an advanced level


question.

68. A 79-year-old man complains of extreme The answer is B.


drowsiness when he goes to toilet at midnight
and he almost fell down on several occasions. He
wants to avoid fall at night. The following is the
list of his medications. Which of the following Explanation: Mirtazapine is the
medications should be stopped? most sedative medication among
all the options.

F. Melatonin
G. Mirtazapine
H. Risperidone Year: 2014
I. Rivastigmine
J. Sodium valproate.

The answer is D.

!
!
226
69. A 70-year-old woman suffering from bipolar
disorder and her daughter came to see you
today. Her daughter read about the symptoms of Explanation: Elderly with bipolar
bipolar disorder and found her mother different disorder are more likely to be
from younger adults suffering from the same paranoid and present with mixed
condition. Which of the following clinical features clinical presentations. Elderly are
is more likely to be found in this 70-year-old
less likely to develop flight of
woman?
ideas, hyperactivity and
overspending.
F. Demonstrates less paranoid ideas as compared
to adult patients
G. Demonstrates more flight of ideas as compared
to adult patients Year: 2014
H. More likely to be hyperactive as compared to
adult patients
I. More likely to demonstrate mixed
(depressive/manic) clinical presentations as
compared to adult patients
J. More spending as compared to adult patients.

70. Which of the following is most important risk The answer is C.


factor for vascular dementia?

F. History of depression Explanation: Among all the


G. History of poor academic performance options, hypertension is the most
H. History of hypertension significant risk factor for vascular
I. High level of physical activity dementia.
J. High level of high density lipoproteins.

Year: 2014

71. You are the visiting physician of a nursing home. The answer is A.
The nursing home staffs feel very helpless when
handling aggression in dementia patients and
they need to seek your advice. Which of the
following psychotropic medication have the most Explanation: Antipsychotic drug
evidence in managing aggression in patients (e.g. risperidone) has the most
suffering from dementia? evidence in improving
aggression in dementia patients
F. Antipsychotic drug among all options.
G. Beta-blocker
H. Benzodiazapine
I. Lithium
J. Stimulant. Year: 2014

!
!
227
Child and adolescent psychiatry

72. In children and adolescents, high stimulation- The answer is C.


seeking behaviour is associated with which of the
following conditions?

Explanation: Young people with


F. Adolescent-onset schizophrenia conduct disorder have to be
G. Autism involved in delinquent behaviour
H. Conduct disorder to get high stimulus.
I. Obsessive-compulsive disorder
J. School refusal.

Year: 2014

73. Children with autism are most comfortable in The answer is D.


which of the following situations?

F. Attending a party where there are a lot of Explanation: Travelling in a bus


surprises and have a chance to meet different because it has a predictable
people. route (i.e. the bus stops at every
G. Attending drama classes and imagines oneself bus stop and there is
playing different roles in the drama.
uncertainty).
H. Looking for toys in a multi-storey department
store from one level to another level.
I. Travelling in a bus when it stops at every bus
stop until it reaches the bus terminal. Year: 2014
J. Going to school where there are multiple classes
to attend in a day and teaching content varies
from subject to subject.
74. Which of the following statements is most correct The answer is E.
regarding the impact of divorce on children?

F. Boys aged two to four are not significantly


affected. Explanation: Options A and B
G. Children do best if they have no contact with the have no evidence base. For
father for two years. option C, most children take 3-5
H. Most children have no significant problems after years to recover. Socio-
three to six months. economic status affects the
I. Socio-economic status is not associated with the
outcome.
degree of impairment.
J. To evaluate the impact, one must take into
consideration the age and the developmental
level of the child. Year: 2014

75. A 15-year-old obese female is brought by her The answer is E.


parents for an evaluation as part of the admission

!
!
228
process for a weight management program. The
adolescent agrees with the fact that she is
overweight and she is keen to lose weight. Explanation: The patient must
Which of the following is most indicative that she agree to use conservative
will be successful in the program? methods including changing diet
habits, seeing a dietician and
F. Her current BMI is between 26 and 30.
going for exercise to lose weight.
G. Mother suffered from obesity and successfully
lost weight
H. No family history of eating disorders
I. Motivation to take antidepressant Year: 2014
J. Willingness to change her eating habits.
Substance abuse

76. Methylene dimethylamphteamine (street name: The answer is B.


ecstasy) usually cause people with which of the
following symptom?
f. Amotivation
g. Closeness to others Explanation: Ecstasy is also
h. Depression known as an empathogen and
i. Introversion makes the patient to feel close to
j. Phobia. others.

Year: 2014

77. A 30-year-old adult claims to suffer from adult The answer is D.


onset attention deficit and hyperactivity disorder
(ADHD). He has been seeing different doctors to
obtain methylphenidate. He claims that he lost all
methylphenidate in the bus few days ago and his Explanation: This patient is
wife has noticed that he looks very tired. Which doctor shopping and tries to
of the following is most appropriate explanation obtain methylphenidate because
for his tiredness? he is dependent on this
medication. He lost the
F. He develops depression and psychomotor medication few days ago and
retardation developed psychological
G. He exhibits secondary gain and seeking withdrawal symptoms including
attention from his wife depression, tiredness,
H. He suffers from comorbid somatisation oversleeping and overeating.
disorder in addition to ADHD
I. He suffers from psychological withdrawal
symptoms from stimulant misuse
J. He suffers from stimulant intoxication. Year: 2014

78. The most important objective of The answer is D.


psychological intervention in treating people
suffering from alcohol misuse is:

!
!
229
F. To conduct in a group therapy format Explanation: Achieving
G. To apply cognitive behaviour therapy (CBT) as abstinence or reduction of
the first approach alcohol use is most important
H. To combine psychological intervention and
psychotherapeutic objective.
pharmacotherapy (e.g. disulfiram)
I. To focus on achieving and maintaining
abstinence or reducing alcohol use
J. Explore unresolved psychodynamic conflicts. Year: 2014.

Psychopharmacology

79. Which of the following medications is not The answer is E.


associated with weight gain?

F. Clozapine Explanation: Topiramate is


G. Mirtazapine associated with weight loss and
H. Olanzapine not weight gain.
I. Sodium valproate
J. Topiramate.

Year: 2014

80. A 60-year-old man is started on an The answer is D.


antidepressant for treatment of major depressive
disorder. Three weeks later, he complains of
blurred vision, dry mouth, constipation, urinary Explanation: Imipramine is a
retention and tachycardia. Which of the following tricyclic antidepressant and
medications is most likely to have caused this associated with anticholinergic
presentation?
side effect.

F. Agomelatine
G. Duloxetine
H. Fluoxetine
I. Imipramine
Year: 2014
J. Mirtazapine

81. Which of the following medications is most likely The answer is D.


to be associated with polycystic ovarian
syndrome in female patients?

F. Carbamazapine Explanation: Sodium valproate is


G. Lamotrigine associated with polycystic ovary
H. Lithium syndrome in female patients.
I. Sodium valproate
J. Topiramate.

!
!
230
Year: 2014

82. Regarding an antidepressant called bupropion, The answer is C.


which of the following statements is false?

F. It causes insomnia. Explanation: bupoprion works on


G. It has a half-life of approximately 12 hours. noradrenaline and dopamine but
H. It is safe for patients suffering from bulimia not serotonin. It is not
nervosa. recommended for eating disorder
I. It is an antidepressant used for depressed
because of the risk of seizure.
patients with cardiovascular disease and habit of
smoking.
J. It does not cause weight gain.
Year: 2014

83. Mirtazapine is associated with lesser sexual side The answer is B.


effects. Which of the following
pharmacodynamics actions explains this
phenomenon?
Explanation: Mirtazapine
antagonises 5HT2A receptors and
F. Serotonin receptor 5HT1A agonism leads to anxiolytic effects,
G. Serotonin receptor 5HT2A antagonism restores sleep and less sexual
H. Serotonin receptor 5HT2C antagonism
side effects.
I. Serotonin receptor 5HT3 antagonism
J. Histamine receptor antagonism.

Year: 2014

84. Which of the following side effects is least likely The answer is D.
to occur in patients taking quetiapine?

F. Antihistamine effects Explanation: Quetiapine is


G. Orthostatic hypotension associated with antihistamine
H. Increase in liver transaminase effects, orthostatic hypotension,
I. Increase in prolactin increase in liver transaminase
J. Weight gain.
and weight gain.

Year: 2014

85. Which of the following is the best treatment Answer is E.


option for severe depressive episode with
psychotic features?

Explanation: ECT has the most


F. Amitriptyline evidence in treating psychotic

!
!
231
G. Cognitive behaviour therapy depression as compared to other
H. Lamotrigine treatment options.
I. Methylphenidate
J. Electroconvulsive therapy.

Year: 2014

86. A 35-year-old man suffers from schizophrenia The answer is B.


symptoms and his GP has started him a
medication to treat his symptoms. He develops
jaundices after taking the medication. Which of Explanation: Chlorpromazine is
the following medications is most likely to cause associated with cholestatic
jaundice? jaundice and is most likely to
cause jaundice among all
F. Clozapine options.
G. Chlorpromazine
H. Haloperidol Year: 2014
I. Risperidone
J. Sulpiride.

87. A 60-year-old man suffers from depressive The answer is E.


disorder and you have increased the dose of
antidepressant. He is seeing you today. His blood
pressure is 180/90. Which of the following
medications should be stopped? Explanation: Venlafaxine is
associated with dose-dependent
hypertension.
F. Diazepam
G. Quetiapine
H. Fluvoxamine
I. Hydroxyzine Year: 2014
J. Venlafaxine.

88. A 25-year-old woman with bipolar disorder is The answer is C


pregnant for the first time. Throughout the
pregnancy, she has been maintained on a mood
stabilizer. At the time of birth, the baby is noted
to have Ebsteins anamoly. Which of the Explanation: Ebsteins anomaly
following medications is most responsible for this is associated with lithium.
teratogenic effect?

F. Carbamazepine
G. Lamotrigine Year: 2014
H. Lithium
I. Sodium valproate
J. Topiramate

!
!
232
Psychotherapy and defence mechanisms

89. A 28-year-old woman comes to consult you The!answer!is!C.!


because she is very upset after being scolded by
her manager. The chief executive officer (CEO) !
of her company has raised the standard for
annual performance. Her manager is very upset Explanation:!The!manager!
this arrangement and scolds her for no reason. transferred!his!anger!to!his!
Which of the following is the defence mechanism subordinate!which!carries!less!
exhibits by the manager?
emotional!risk!as!compared!to!the!
CEO.!This!is!known!as!
F. Altruism displacement.!
G. Denial
H. Displacement !
I. Repression
J. Reaction formation. Year: 2014!

90. You are a doctor working in polyclinic. A 25-year- The answer is E.


old man bought his 55-year-old mother to see
you. He is concerned that his mother suffers from
depression but she refuses to take
antidepressant. You realise that the mother and Explanation: Family therapy such
son are enmeshed as the son is unmarried and as structural family therapy
not able to work. He claims that he needs to look focusing on boundary is helpful
after his mother although she has good past in this situation.
health. The mother and son hold very negative
view against the father who is the sole
breadwinner of family. There is a psychologist
working in your polyclinic. Which of the following Year: 2014
forms of psychological treatment is most
appropriate?

F. Couple therapy
G. Dialectical behaviour therapy
H. Problem solving psychotherapy
I. Supportive psychotherapy
J. Family therapy.

91. A national serviceman is seeing a male The answer is E.


psychologist because he suffers from adjustment
disorder and he finds his army supervisor
uncaring. Furthermore, he is very angry with his
own father. He recalls an incident in camp about Explanation: Interpretation is a
getting angry with his male supervisor. The statement made by the
psychologist asks a factual question about the psychologist to enhance patients
circumstance and rage in the patient. Then the understanding of his or her own
patient complains that the psychologist is
thoughts or behaviours.

!
!
233
uncaring and trying to find fault in him. The Affirmation refers to confirmation
psychologist says, I wonder if what youve of validity of previous judgement
feeling right now is just like the feeling you have or behaviour in a patient.
in camp, when you attributed the same uncaring
Empathetic validation refers to
attitude toward your father and supervisor in
camp. These repetitive patterns of the situation when the therapist
misunderstanding seem to make you very upset tries to put himself into the
in different situations and affect your life. Which patients shoes and tries to
of the following most accurately describes the understand patients inner state.
psychotherapeutic technique?

F. Affirmation
G. Clarification Year: 2014
H. Empathic validation
I. Observation
J. Interpretation.

92. A medical student used to be very keen to The answer is E.


become an orthopaedic surgeon. He has
assisted an orthopaedic surgeon to conduct
research since the second year of medical
school. During the residency application, he tells Explanation: His external attitude
other classmates that he does not bother whether and behaviours (not caring about
he is accepted by the orthopaedic residency. He orthopaedic surgery) is the
also encourages other classmates not to apply opposite of the internal intention
for orthopaedic surgery as this specialty is not a
(wanted to join orthopaedic
good career choice. On the other hand, he is
secretly asking his orthopaedic mentor to prepare surgery).
him for the orthopaedic residency intake
interview. What is his defence mechanism?
Year: 2014
F. Altruism
G. Denial
H. Displacement
I. Repression
J. Reaction formation.
93. Which of the following is the most effective The answer is C.
approach for a 30-year-old woman with
obsession about contamination and washes her
hands every time she touches something which
she considers dirty? Explanation: Option C is known
as exposure and response
F. Having the patient place her hands in a container prevention (ERP) and is the most
of worms to create extreme phobia and ability to effective approach among all
overcome obsession
options. Option D is not good
G. Having the patient snap her wrist with a rubber
band when she thinks about contamination and enough because the
about to wash her hands psychologist should guide the
H. Having the patient touch a dirty object, then not patient to further challenge the
allowing her to wash her hands for several hours obsessions rather than just

!
!
234
I. Highlighting to the patient that her obsession is a highlighting the obsessions.
cognitive error. Option B is thought stopped and
J. Providing the patient with message cards to is not as effective as ERP.
remind herself that the hand washing is
unnecessary.

Year: 2014

94. A 30-year-old army officer presents for treatment The answer is C.


of post-traumatic stress disorder after surviving
an accident which his tank fell into the river from
a bridge. He was driving a tank on a bridge when Explanation: The psychologist is
it suddenly collapsed. Several army officers were providing in-vivo exposure by
seriously injured in this accident. As part of the imaging the accident scene to
treatment, the psychologist asks the patient to
overcome fear and anxiety
imagine that he is safely driving his tank over a
bridge. Which of the following best describes this associated with flashback.
therapeutic intervention?

F. Aversion therapy
G. Cognitive therapy Year: 2014
H. Exposure therapy
I. Interpersonal therapy
J. Supportive therapy.

Ethics, management dilemma and laws

95. You are a paediatric resident. A 14-year-old girl The answer is E.


has stayed in the paediatric ward for 1 week after
a suicide attempt by drug overdose. She is due
for discharge tomorrow. At 6 pm, she suddenly
claims that her elder sister had physically abused Explanation: As this female
her prior to admission. Her medical record adolescent suffers from conduct
reveals that she suffers from conduct disorder. disorder, she has the tendency
She also made similar reports of physical abuse to tell lies to avoid discharge. As
in the past. The police officer and social worker
the patient is currently in the
found her claims inconclusive and did not
investigate further. She is ambivalent to make a ward and she is safe, there is no
report about the alleged abuse and she is not need to inform the police
keen for discharge. You try to contact the immediately without further
consultant in charge of this case but the clarification from the treatment
consultant is not contactable. Which of the team. Furthermore, the patient is
following is most appropriate action? ambivalent to make a report. The
best approach is to discuss with
F. Banning her parents from visiting her. the treatment team and they can
G. Call 995 to inform the police immediately clarify with her parents whether
H. Inform the on-call team and request the on-call the elder sister did physically
doctors to inform the police before midnight abuse her.
I. Refer the patient to gynaecologist for further
assessment

!
!
235
J. Discuss with the consultant-in-charge and the
social worker on the next day.
Year: 2014

96. A 35-year-old man was hospitalised for treatment The answer is D.


of severe depressive episode in September. The
inpatient team prescribed two antidepressants for
the patient and then referred the patient to
polyclinic. The patient came to the polyclinic for Explanation: The polyclinic
review in October and November respectively. doctors forgot to prescribe
The patient had a relapse of depression and antidepressants and resulted in
committed suicide in December. The relapse and suicide. Their act is
computerised record shows that the polyclinic
considered to be negligence /
doctors had forgotten to prescribe
antidepressants since the first visit. Which of the unintentional torts. Option A is
following best describes the above situation intentional torts and not relevant
based on the Common Law? for this case. Diminished
responsibility only applies in
murder cases. Tarasoffs rule
F. Battery
involves duty to warn when a
G. Diminished responsibility
H. Intentional torts patient expresses his/her wish to
I. Negligence harm another person.
J. Tarasoffs rule

Year: 2014

Liaison psychiatry and neuropsychiatry

97. You are a resident in obstetrics. Which of the The answer is B.


following is the safest medication to be given to a
pregnant woman who develops psychosis during
pregnancy?
Explanation: Haloperidol is the
oldest medication and is the
F. Aripiprazole safest among all options.
G. Haloperidol
H. Olanzapine
I. Quetiapine
J. Risperidone Year: 2014

98. A 25-year-old man expresses concerns that he The answer is B.


got infected with HIV after visiting commercial
sex workers last week. According to him, he only
received hand-genital stimulation and there was Explanation: It is common in
no vaginal intercourse. He also mentions that the clinical practice for patients
commercial sex worker kissed his body. He has

!
!
236
sought his girl-friends forgiveness. Today, he asking for urgent HIV test after
requests HIV test urgently. He consulted you one they engaged in activities which
year ago for similar concern after he kissed did not transmit HIV. The
another lady without informing his girl-friend.
patients request for such test as
What is the most likely explanation of his
behaviour? a result of guilt. It is unlikely to be
caused by obsession as the
patient requests for the HIV test
F. Underlying depression after sexual activity but not a
G. Underlying guilt thought. The patient does not
H. Underlying obsession have any somatic concern.
I. Underlying psychosis
J. Underlying somatic concern

Year: 2014

99. You are orthopaedic resident. A 25-year-old The answer is D.


construction worker had a fall, fractured his spine
and injured the brain. He develops pain in
multiple sites. He has one episode of epilepsy
after the injury. He is depressed but does not Explanation: Duloxetine is an
have suicidal thought. Electrocardiogram is antidepressant indicated for both
normal. Which of the following medication is most depression and pain. Bupropion,
helpful in this situation? amitriptyline and chlorpromazine
increases the risk of seizure.
A. Amitriptyline
There is no indication to
B. Bupropion prescribe moclobemide in this
case.
C. Chlorpromazine

D. Duloxetine
Year: 2014
D. Moclobemide

100. You are a medical resident and The!answer!is!B.!


rotated to rheumatology. A 24-year-old
woman suffers from systemic lupus !
erythematosus (SLE). She suffers from lupus
Explanation:!!Binge!eating!is!not!a!
nephritis and requires dialysis three times a
week. She was very concerned about her recognised!neuropsychiaric!
illness and felt hopeless. She took an symptom!of!SLE.!
overdose of her lupus medications and
!
admitted to the medical ward. You need to
take a history from her. Which of the Year: 2014!
following is not part of the neuropsychiatric
symptoms of SLE experienced by this
patient?

!
!
237
F. Anxiety
G. Binge eating
H. Cognitive impairment
I. Epilepsy
J. Headache.
!

SAQ 1
Question 1
A 50-year-old man is admitted to the ward. He complains that there are a lot of bugs on
his skin and he needs to wash hands excessively. His wife wants to consult you how to
differentiate an obsession from a delusion.

1. Define an obsession (2 marks). Examiners use only: / 2 marks

2. Define a delusion (2 marks). Examiners use only: / 2 marks

3. List TWO (2) features that differentiate an obsession from a delusion (4 marks).
Examiners use only: / 4 marks
4. List TWO (2) types of evidence-based and commonly used treatment for obsessive compulsive
disorder (2 marks).
Examiners use only: / 2 marks

Administrators use only (Total): Administrator 1: / 10 marks Administrator 2: /10 marks


Question 3
A 3-year-old boy has delayed language development. He is of normal height and weight, and has no
obvious physical abnormalities.

1. List THREE (3) main clinical features which you would look for to support the diagnosis of
autistic disorder (3 marks)
Examiners use only: / 3 marks

2. List TWO other causes for his poor language skills besides autism (2 marks).
Examiners use only: / 2 marks

!
!
238
3. Name THREE (3) investigations which you order to establish the underlying cause for language
delay. (3 marks)

Examiners use only: / 3 marks

4. Name TWO (2) non-pharmacological interventions which are useful for autism. (2 marks)

Examiners use only: / 2 marks

Administrators use only (Total): Administrator 1: / 10 marks Administrator 2: /10 marks


Question 4
You are a medical resident. Your hospital does not have a psychiatric ward and a 40-year-old woman
presenting with catatonia was admitted last night. No investigation and treatment was ordered.

1. List THREE (3) characteristic features of catatonia (3 marks)

Examiners use only: / 3 marks

2. Name TWO (2) psychiatric disorder commonly associated with catatonia.(2 marks)

Examiners use only: / 2 marks

3. At NAME TWO (2) medical disorders that are known to be associated with catatonia. (2 marks)

Examiners use only: / 2 marks

4. List ONE (1) most important investigation which you would order to identify the underlying
neurological cause for catatonia.(1 mark)

Examiners use only: / 1 mark

5. List ONE (1) pharmacological treatment for this patient. (1 mark)

Examiners use only: / 1 mark


6. Name ONE (1) non-pharmacological management order for this patient (1 mark)

Examiners use only: / 1 mark

Administrators use only (Total): Administrator 1: / 10 marks Administrator 2: /10 marks

!
!
239
Question 1
A 50-year-old man is admitted to the ward. He complains that there are a lot of bugs on
his skin and he needs to wash hands excessively. His wife wants to consult you how to
differentiate an obsession from a delusion.

5. Define an obsession (2 marks). Examiners use only: / 2 marks


Answer may vary. The examiner is advised to use his or her discretion to decide whether marks should be
awarded.
Obsession is a
recurrent and persistent thoughts, impulses, or images that are experienced as intrusive and
inappropriate
they cause anxiety and distress
they are not simple excessive worries about real-life problems
the person attempts to ignore or suppress the thoughts, impulses, or images, or to neutralize them
with some other thought or action
the person recognizes them as products of his/her own mind.

6. Define a delusion (2 marks). Examiners use only: / 2 marks


Answer may vary. The examiner is advised to use his or her discretion to decide whether marks should be
awarded.
Delusion is a
a false belief, based on an incorrect inference about external reality.
the belief cannot be corrected by reasoning.
the patient firmly believes that the false belief is true (close to 100% true).

!
!
240
7. List TWO (2) features that differentiate an obsession from a delusion (4 marks).
Examiners use only: / 4 marks
Answer may vary. The examiner is advised to use his or her discretion to decide whether marks should be
awarded.
Differences between obsessions and delusions:
obsessions are ego-dystonic (disharmony with ones own ego) while delusions are ego-syntonic (in
harmony with ones own ego) 2 marks or
obsessions involve intact reality testing, whereas in delusions reality testing is impaired (2 marks)
or
obsessions cause anxiety and distress, whereas patients with delusion are usually calm (2marks)
or
obsessions involve compulsive actions to neutralize them, whereas delusions do not (2 marks)
obsessions are not held with absolute certainty whereas delusions are (2 marks).
This is an advanced level question.
8. List TWO (2) types of evidence-based and commonly used treatment for obsessive compulsive
disorder (2 marks).
Examiners use only: / 2 marks

Selective!serotonin!reuptake!inhibitors!!or!name!of!SSRI!e.g.!fluoxetine!(1!mark);!antidepressant!
(0.5!mark);!other!(0!mark)!
Cognitive!behaviour!therapy!(1!mark);!Exposure!and!response!prevention!(0.5!mark);!Other!(0!
mark)!

Administrators use only (Total): Administrator 1: / 10 marks Administrator 2: /10 marks


Question 3
A 3-year-old boy has delayed language development. He is of normal height and weight, and has no
obvious physical abnormalities.

5. List THREE (3) main clinical features which you would look for to support the diagnosis of
autistic disorder (3 marks)
Examiners use only: / 3 marks
Answer may vary. The examiner is advised to use his or her discretion to decide whether marks should be
awarded.
1. Restricted interests or hobbies or repetitive behaiour.
2. Communication difficulties (both verbal language delay and non-vebal lack of eye contact, failure
to understand emotional cues)
3. Stereotyped movements or social interaction difficulty.

6. List TWO other causes for his poor language skills besides autism (2 marks).
Examiners use only: / 2 marks
Answer may vary. The examiner is advised to use his or her discretion to decide whether marks should be
awarded. Any two of the following:
1. Maltreatment or abuse
2. Mental retardation

!
!
241
3. Neurological injury
4. Hearing impairment
5. Specific language impairment (such as expressive language disorder)
6. Genetic disorders (Fragile X syndrome)

7. Name THREE (3) investigations which you order to establish the underlying cause for language
delay. (3 marks)

Examiners use only: / 3 marks


Answer may vary. The examiner is advised to use his or her discretion to decide whether marks should be
awarded. Any two of the following

1. Intelligence quotient (IQ) testing


2. Diagnostic interview for Autism
3. Electroencephalogram (EEG)
4. Hearing assessment
5. Speech assessment
6. Genetic testing (e.g. look for fragile X site or non tandem repeats in X chromosome.

8. Name TWO (2) non-pharmacological interventions which are useful for autism. (2 marks)

Examiners use only: / 2 marks


Answer may vary. The examiner is advised to use his or her discretion to decide whether marks should be
awarded. Any two of the following
1. Behaviour therapy (e.g. intensive behaviour intervention)
2. Training to use alternative communication
3. Visual strategies or visual support for communication
4. Social skill training
5. Special education
6. Speech therapy

Administrators use only (Total): Administrator 1: / 10 marks Administrator 2: /10 marks


Question 4
You are a medical resident. Your hospital does not have a psychiatric ward and a 40-year-old woman
presenting with catatonia was admitted last night. No investigation and treatment was ordered.

7. List THREE (3) characteristic features of catatonia (3 marks)

Examiners use only: / 3 marks


Any three of the following:

! Motoric immobility as evidence by cataplesy (or waxy flexibility) or stupor


! Excessive Motoric Activity (purposeless activity, not influenced by external stimuli)
! Ambitendency
! Automatic obedience
! Negativism (as evidenced by motiveless resistance to all instructions or maintenance of a rigid posture
against attempts at being moved)
! Mutisn
! Stereotyped movements, prominent mannerisms or prominent grimacing)
Echolalia
! Echopraxia.

!
!
242
8. Name TWO (2) psychiatric disorder commonly associated with catatonia.(2 marks)

Examiners use only: / 2 marks


Any of the following:

Schizophrenia
Bipolar disorder
Depressive disorder
Substance abuse with acute intoxication of recreation drugs.

9. At NAME TWO (2) medical disorders that are known to be associated with catatonia. (2 marks)

Examiners use only: / 2 marks


Any of the following:

Central nervous system infection: meningitis and encephalitis


Central nervous system tumour
Cerebrovascular accident
Lethal catatonia
Acute poisoning

10. List ONE (1) most important investigation which you would order to identify the underlying
neurological cause for catatonia.(1 mark)

Examiners use only: / 1 mark

Magnetic Resonance Imaging (MRI) brain scan (1 mark); Computerized Tomography (CT) brain scan
(1 mark); other answer (0 mark)

11. List ONE (1) pharmacological treatment for this patient. (1 mark)

Examiners use only: / 1 mark


Benzodiazepine or lorazepam (1 mark)

12. Name ONE (1) non-pharmacological management order for this patient (1 mark)

Examiners use only: / 1 mark

Any of the following:


1. Hydration
2. Nutrition
3. Early mobilization when condition is stable
4. Close observation and frequent monitoring for vital signs.
5. Transfer to ICU if patient deteriorates.

ECT = 0 mark as it involves general anaesthesia (pharmacological)

Administrators use only (Total): Administrator 1: / 10 marks Administrator 2: /10 marks

!
!
243
!

SAQ (Paper 2)
Question 1

You are a General Practitioner. A 30-year-old woman comes to your clinic and she complains of low
mood. You suspect that she suffers from depressive disorder. She read a lot of information on the internet
and has her personal views on treatment. She would like to discuss with you on various treatment options.
1. List Four (4) pieces of information which you would inquire from the history to support the
decision for immediately prescribing an antidepressant to this patient (4 marks).
2. The patient would like to know more about psychotherapy. Name TWO (2) types of
psychotherapy with MOST evidence to demonstrate effectiveness for her condition (2 marks).

!
!
244
1. Based on your answer in question 2, select ONE (1) type of psychotherapy and explain
how it works. (4 marks)

Question!2!

You are a general practitioner and looks after a 30-year-old woman who is 14-week pregnant.
She has been drinking alcohol and has poor compliance to antenatal follow-up. Besides alcohol,
she does not use other drug or substance. She feels that it is completely harmless to drink
alcohol during pregnancy. You need to offer psychoeducation to her.

1. What is the name of the syndrome which is of MOST concerned to her foetus? (2 mark)
2. List THREE (3) facial features associated with the syndrome described in Question 1. (3
marks)

3. If her baby is born with the syndrome described in question 1, list THREE (3)
psychiatric impairments which may occur during the childhood of her offspring (3
marks)

4. State TWO (2) specialities which you would refer this patient to consult for her current
problems. Please indicate the reasons for referral. (2 marks)

Specialities Reasons for referral

Question!3!

You!are!a!general!practitioner!(GP)!and!seeing!a!65TyearTold!man!who!complains!of!
memory!loss.!He!also!suffers!from!depression.!!His!daughter!has!read!information!from!
the!internet!that!he!is!at!risk!of!developing!Alzheimers!disease!due!to!his!age.!She!is!not!
certain!about!the!impact!of!depression!on!his!memory.!!

1.!Name!FOUR!(4)!differences!between!dementia!and!pseudodementia!(4!marks)!

!
!
245
! Dementia! Pseudodementia!

Example:! Caused!by!Alzheimers!disease! Caused!by!depression!

Aetiology!

1.! ! !

! !

! !

2.! ! !

! !

! !

3.! ! !

4.! ! !

!
3. You!have!assessed!the!patient!and!he!suffers!from!depression.!You!have!decided!
to!start!fluoxetine.!Name!ONE!(1)!common!electrolyte!abnormality!found!in!old!
people!taking!SSRI.!(2!marks)!

!
!
246
!

4. Four!months!later,!this!patient!has!tried!fluoxetine!and!it!does!not!work.!He!took!
benzodiazepine!from!another!GP!for!insomnia.!He!develops!suicidal!thought.!You!
have!referred!this!case!to!a!geriatric!psychiatrist.!The!geriatric!psychiatrist!
recommends!electroconvulsive!therapy!(ECT).!Based!on!the!clinical!information!
provided,!name!TWO!(2)!factors!which!may!increase!the!seizure!threshold!for!this!
patient!(2!marks)!during!the!ECT.!

!
1. His!daughter!is!concerned!about!memory!impairment!after!ECT.!State!TWO!(2)!
adjustments!which!can!apply!during!the!administration!of!ECT!to!reduce!the!risk!
of!cognitive!impairment.!

Question!4!

You are a specialist working in renal medicine. A 60-year-old woman suffers from chronic
renal failure and she requires life-long hemodialysis. She wants to discontinue dialysis. It is a
long weekend and the psychiatrist is not available in your hospital. You need to determine her
capacity to make a decision to discontinue dialysis.

1. List FOUR (4) questions which you would ask this patient to assess her capacity to
make a decision to discontinue dialysis (4 marks).

2. You are reviewing her past psychiatric record. State FOUR (4) psychiatric disorders
that may affect her capacity to give consent.(4 marks)

3. If this patient is free from psychiatric illness and has the capacity to make decision. You respect
her right to make a decision to discontinue dialysis. State the MOST RELEVANT ethical
principle which support your decision. (2 marks)

SAQ (Paper 2)

!
!
247
Question 1

You are a General Practitioner. A 30-year-old woman comes to your clinic and she complains of low
mood. You suspect that she suffers from depressive disorder. She read a lot of information on the internet
and has her personal views on treatment. She would like to discuss with you on various treatment options.
5. List Four (4) pieces of information which you would inquire from the history to support the
decision for immediately prescribing an antidepressant to this patient (4 marks).
Answer may vary. The examiner is advised to use his or her discretion to decide whether marks should be
awarded.
1. The severity of depression: her symptoms are severe (e.g. with the presence of suicidal thought and a
lot of biological symptoms).
2. Lack of precipitant in her depression: her depression seems to be endogeneous and it is not a reactive
depression.
3. Presence of family history of depression: suggest biological cause of depression.
4. Previous response to antidepressant or previous relapse after stopping antidepressant.

5. Severe impairment in functioning.


6. Failure of psychotherapy.
7. Past history of depression.
8. Patients preference
6. The patient would like to know more about psychotherapy. Name TWO (2) types of
psychotherapy with MOST evidence to demonstrate effectiveness for her condition (2 marks).
2. Cognitive behaviour therapy
3. Interpersonal therapy
Other types of psychotherapies (0 mark)

4. Based on your answer in question 2, select ONE (1) type of psychotherapy and explain
how it works. (4 marks)

Answer may vary. The examiner is advised to use his or her discretion to decide whether marks should be
awarded.

For students who chose to explain CBT:

The frequency of Cognitive behaviour therapy (CBT) is usually weekly or fortnightly. It requires 12 to 16
sessions. The cognitive therapy involves identifying negative automatic thoughts and use dysfunctional thought
diary to identify pattern between the time, events, negative thoughts and resulted emotions and behaviours. The
psychologist will read the diary and help patients to question the negative automatic thoughts. Behaviour
therapy involves activity scheduling (for those depressed patients with psychomotor retardation), relaxation
techniques (for those patients with mixed anxiety and depression).
For students who chose to explain IPT:
Interpersonal therapy (IPT) is held weekly or fortnightly. It involves 12 to 20 sessions. IPT is indicated for
depressed patients whom precipitating factor is interpersonal problems. The psychologist closely examines
interpersonal relationship and relates depression to the interpersonal event. The therapist helps the patient to
work on one of the interpersonal problem areas: 1) grief; 2) interpersonal role dispute; 3) role transition; 4)
interpersonal deficits. IPT helps the patient to develop new skills to deal with people and life situations
associated with depressive symptpms. IPT works with the patient to look at interpersonal relationship from
another angle to minimise impact on the mood. The therapist also uses role-play to improve communication
skills.

!
!
248
Question!2!

You are a general practitioner and looks after a 30-year-old woman who is 14-week pregnant.
She has been drinking alcohol and has poor compliance to antenatal follow-up. Besides alcohol,
she does not use other drug or substance. She feels that it is completely harmless to drink
alcohol during pregnancy. You need to offer psychoeducation to her.

5. What is the name of the syndrome which is of MOST concerned to her foetus? (2 mark)

Foetal alcohol syndrome

6. List THREE (3) facial features associated with the syndrome described in Question 1. (3
marks)

Answer may vary. The examiner is advised to use his or her discretion to decide whether marks should be
awarded.
Any three of the following:

Small head
Small eyes
Short nose
Ear lobe abnormality
Short palpebral fissures (measured from inner to outer canthus; reflect underlying brain growth)
Thin upper lip
Long and flat philtrum

7. If her baby is born with the syndrome described in question 1, list THREE (3)
psychiatric impairments which may occur during the childhood of her offspring (3
marks)

Answer may vary. The examiner is advised to use his or her discretion to decide whether marks should be
awarded.
Any three of the following:
intellectual impairment
attention deficit or hyperactivity
difficulty with reasoning
depression
anxiety
oppositional defiant or conduct disorder

8. State TWO (2) specialities which you would refer this patient to consult for her current
problems. Please indicate the reasons for referral. (2 marks)

Specialities Reasons for referral

!
!
249
Specialities Reasons for referral

Obstetrics and gynaecology (0.5 mark) Frequent ultrasound monitoring for foetal
abnormalities (0.5 mark)

Psychiatrist or addiction specialist (0.5 mark) For detoxification and treatment of alcohol
dependence (0.5 mark)

Other specialist: no mark.

For the reasons of referral, the answer may vary. The examiner is advised to use his or her discretion to decide
whether marks should be awarded.

This is an advanced-level question.

!
!
250
!

Question!3!

You!are!a!general!practitioner!(GP)!and!seeing!a!65TyearTold!man!who!complains!of!
memory!loss.!He!also!suffers!from!depression.!!His!daughter!has!read!information!from!
the!internet!that!he!is!at!risk!of!developing!Alzheimers!disease!due!to!his!age.!She!is!not!
certain!about!the!impact!of!depression!on!his!memory.!!

1.!Name!FOUR!(4)!differences!between!dementia!and!pseudodementia!(4!marks)!

! Dementia! Pseudodementia!

Example:! Caused!by!Alzheimers!disease! Caused!by!depression!

Aetiology!

1.! ! !

! !

! !

2.! ! !

! !

! !

3.! ! !

!
!
251
!

4.! ! !

Answer may vary. The examiner is advised to use his or her discretion to decide whether marks should be
awarded.
!

Any 3 of the following categories:

Dementia Pseudo-dementia

Onset Insidious onset (1 mark) Acute onset (1 mark)

Progression Slow progression (1 mark) Rapid progression (1 mark)

Disability Minimize disability (1 mark) Emphasize on disability (1 mark)

Extent of memory Recent memory loss more severe. Equal loss of recent and remote events.
loss (1 mark) (1 mark)

Consistency Consistent pattern of memory loss Variable pattern of memory loss (1


(1 mark) mark)

Insight/ performance Deny memory loss or confabulate Focus on cognitive loss(1 mark) or
during Mini Mental to fill up memory gap
State Examination Dont know answer (1 mark)

!
!
252
Minimize disability (1 mark)

Variation within a Sun-downer effect (worse in the Diurnal variation (worse in the
day evening) (1 mark) morning) (1 mark)

Personal history or No or less likely (1 mark) Yes or more likely (1 mark)


family history of
depression

!
7. You!have!assessed!the!patient!and!he!suffers!from!depression.!You!have!decided!
to!start!fluoxetine.!Name!ONE!(1)!common!electrolyte!abnormality!found!in!old!
people!taking!SSRI.!(2!marks)!

Hyponatremia.!

8. Four!months!later,!this!patient!has!tried!fluoxetine!and!it!does!not!work.!He!took!
benzodiazepine!from!another!GP!for!insomnia.!He!develops!suicidal!thought.!You!
have!referred!this!case!to!a!geriatric!psychiatrist.!The!geriatric!psychiatrist!
recommends!electroconvulsive!therapy!(ECT).!Based!on!the!clinical!information!
provided,!name!TWO!(2)!factors!which!may!increase!the!seizure!threshold!for!this!
patient!(2!marks)!during!the!ECT.!

Any!2!of!the!following:!

2. Old!age!
3. Male!gender!
4. Baldness!
5. Usage!of!benzodiazepine.!

This!is!an!advancedNlevel!question.!

6. His!daughter!is!concerned!about!memory!impairment!after!ECT.!State!TWO!(2)!
adjustments!which!can!apply!during!the!administration!of!ECT!to!reduce!the!risk!
of!cognitive!impairment.!

Any!2!of!the!following:!

1. Unilateral!ECT!
2. Use!lower!energy!level!
3. Reduce!frequency!of!ECT!(two!times!per!week!instead!of!three!times!per!week)!

This!is!an!advancedNlevel!question.!

!
!
253
!

Question!4!

You are a specialist working in renal medicine. A 60-year-old woman suffers from chronic
renal failure and she requires life-long hemodialysis. She wants to discontinue dialysis. It is a
long weekend and the psychiatrist is not available in your hospital. You need to determine her
capacity to make a decision to discontinue dialysis.

4. List FOUR (4) questions which you would ask this patient to assess her capacity to
make a decision to discontinue dialysis (4 marks).

Answer may vary. The examiner is advised to use his or her discretion to decide whether marks should be
awarded. Any three of the following:

1) What is the nature of your medical condition? / What has the treatment team told you about your condition?/
What do you believe is wrong with your health?

2) What is the purpose of the dialysis? / What have you been told about the recommended treatment?

3) Can you tell me the benefits of dialysis? / What have you been told about benefits of dialysis?/ What is the
treatment likely to do for you?

4) Can you tell me the risks/side effects of dialysis? / What have you been told about the risks or discomforts
associated with dialysis?

5) What happens if you do not get dialysis?

6) Are there alternative to dialysis? / What have you been told about alternatives to dialysis?

7) Why do you want to discontinue dialysis? / What have you been told about risks and benefits of no
treatment?/ How did you decide to discontinue dialysis?

5. You are reviewing her past psychiatric record. State FOUR (4) psychiatric disorders
that may affect her capacity to give consent.(4 marks)

Answer may vary. The examiner is advised to use his or her discretion to decide whether marks should be
awarded.

Any three of the following:

1. Dementia (any form: Alzheimers disease, vascular dementia)


2. Delirium or acute confusional state
3. Schizophrenia
4. Severe depressive disorder with psychotic features (1 mark); depression (0.5mark)
5. Acute psychosis
6. Delusional disorder
7. Bipolar disorder with psychotic features (1 mark); bipolar disorder/mania/hypomania (0.5 mark)

!
!
254
8. Mental retardation

6. If this patient is free from psychiatric illness and has the capacity to make decision. You respect
her right to make a decision to discontinue dialysis. State the MOST RELEVANT ethical
principle which support your decision. (2 marks)

Respect patients autonomy

SAQ!(Paper!3)!!

Question!1!(Total!=!10!marks)!

You are a general practitioner (GP). You are asked to see a 36-year-old woman for the treatment of major depression.
A trial of fluoxetine 40mg every morning by another GP was unsuccessful.

She was very upset with the remarks made by her colleagues. As a result, she quitted her job 6 months ago and
became a housewife. She feels bored as a housewife and has conflicts with her in-laws. She has become more
depressed in the past 6 months. Her husband is supportive and there is no marital conflict. She has a 10-year-old son
and he has good relationship with her. She does not need to worry about his studies as he has done well in school.
She has other chronic medical illnesses and she takes other medications. She forgot to bring the medication and
cannot remember the name of those medications.

She has no history of substance abuse or dependence. Her ECG is normal.

1. State THREE (2) MOST RELEVANT points of the history that you would like to clarify regarding previous trial
of fluoxetine? (2 marks) / 2 marks

2. State TWO (2) medications which are used to treat chronic medical illnesses and known to be associated
with depression? (2 marks) / 2 marks

3. Based on the patients background, state THREE (3) common medical conditions from different body
systems that is known to be associated with depression. (3 marks) / 3 marks

4. Based on the patients history, state ONE (1) MOST appropriate and evidence-based psychotherapy that is
suitable for this patient. (1 mark) / 1 mark

5. Based on the patients history, state how the psychotherapy recommended by you (in Question 4) works for
her. (2 marks) /2 marks

Question!2!(Total!=!10!marks)!

!
!
255
You are a general practitioner. A 25-year-old man suffering from schizoaffective disorder is bought to the clinic by her mother. He
was diagnosed to suffer from schizoaffective disorder 6 months ago. Her mother is concerned about his recent weight gain of 10 kg
in 3 months. According to her mother, his psychiatrist from a restructured hospital started a new antipsychotic and mood stabilizer
to control his psychiatric symptoms.

1. State TWO (2) antipsychotics which are MOST likely to cause weight gain in this patient. (2 marks) /2 marks

2. State TWO (2) mood stabilizers which are MOST likely to cause weight gain in this patient. (2 marks) /2 marks

3. State THREE (3) MOST relevant blood tests which you would like to order and abnormalities you would like to rule out for
this patient. (3 marks) / 3 marks

Investigations Abnormalities

/0.5 mark /0.5 mark

/ 0.5 mark / 0.5 mark

/ 0.5 mark / 0.5 mark

4. State ONE (1) MOST RELEVANT AND SPECIFIC symptom in schizophrenia which may contribute to obesity (2 marks). !

5. State ONE (1) antipsychotic that has the LOWEST RISK to cause weight gain. (1 mark)

Question 3

You are a paediatric resident working in the Child and Accident Emergency Department. You
are seeing an 11-year-old boy who was brought in by his foster parents. According to his
foster parents, he has a lot of behavioural problems at home and school. He cannot sit still to
do his homework at home. He also breaks school rules. Tonight, he had a fight with his foster
parents. This boy is a victim of child abuse.

1. State TWO (2) MOST RELEVANT diagnoses (2 marks) that this boy may suffer from and
indicate THREE (3) additional clinical features (6 marks) which you will look for to
support your diagnosis. (total 8 marks)

Diagnosis 3 additional clinical features per diagnosis

1. 1.

!
!
256
2.

3.

/1 mark /3 marks

2. 1.

2.

3.

/1 mark / 3 marks

2. If you have a chance to speak to the child protection officer, state TWO (2) MOST
relevant psychiatric disorders that his parents may suffer from. (2 marks)

/ 2 marks

Any two of the following:

Question!4!

You!are!a!geriatric!resident.!A!70NyearNold!man!presents!with!dementia.!His!son!wants!to!find!
out!more!about!the!treatment!for!dementia.!

1. State the name of TWO (2) acetylcholinesterase inhibitors available in Singapore. (2 marks)!
/ 2 marks

!
!
257
2. List TWO (2) types of dementia for which acetylcholinesterase inhibitors are indicated (2
marks) / 2 marks

3. List TWO (2) COMMON side effects of acetylcholinesterase inhibitors (2 marks)


/ 2 marks

4. List TWO (2) SPECIFIC contraindications to the use of acetylcholinesterase inhibitors (2


marks)
/ 2 marks

5. His son wants to know potential benefits that can be expected from treatment with
acetylcholinesterase inhibitors. List TWO (2) potential benefits (2 marks).
/ 2 marks

SAQ!(Paper!3)!!

Question!1!(Total!=!10!marks)!

You are a general practitioner (GP). You are asked to see a 36-year-old woman for the treatment of major depression.
A trial of fluoxetine 40mg every morning by another GP was unsuccessful.

She was very upset with the remarks made by her colleagues. As a result, she quitted her job 6 months ago and
became a housewife. She feels bored as a housewife and has conflicts with her in-laws. She has become more
depressed in the past 6 months. Her husband is supportive and there is no marital conflict. She has a 10-year-old son
and he has good relationship with her. She does not need to worry about his studies as he has done well in school.
She has other chronic medical illnesses and she takes other medications. She forgot to bring the medication and
cannot remember the name of those medications.

She has no history of substance abuse or dependence. Her ECG is normal.

6. State THREE (2) MOST RELEVANT points of the history that you would like to clarify regarding previous trial
of fluoxetine? (2 marks) / 2 marks

Any two of the following:

1) Her previous adherence or compliance to fluoxetine (1 mark) or


2) Side effects associated with fluoxetine (1 mark) or
3) The duration of taking fluoxetine (1 mark).

!
!
258
7. State TWO (2) medications which are used to treat chronic medical illnesses and known to be associated
with depression? (2 marks) / 2 marks

Any two of the following:

1) Corticosteroids!/!steroids/!prednisolone!(1!mark)!or!!
2) BetaNblockers!/!atenolol!/!propranolol!(1!mark)!or!!
3) Clonidine!(1!mark)!or!!
4) Metoclopramide!(1!mark)!or!!
5) Theophylline!(1!mark)!or!!
6) Calicium!channel!blockers:!Nifedipine!(1!mark).!

8. Based on the patients background, state THREE (3) common medical conditions from different body
systems that is known to be associated with depression. (3 marks) / 3 marks

Answers may vary. Examiners are advised to use his or her own discretion to decide whether marks should be
awarded. Any three of the following:
1) Cardiovascular disorders: hypertension (1 mark)
2) Endocrine disorders: diabetes, hypothyroidism or Cushing syndrome or Addisons disease (1 mark)
3) Neurological disorders: epilepsy (1 mark)
4) Gastrointestinal system: peptic ulcer or irritable bowel syndrome or inflammatory bowel diseases (1 mark)
5) Immune system: systemic lupus erythematosus (1 mark)

Medical disorders which are more common in older adults or elderly e.g. Parkinsons disease, cerebrovascular accidents,
chronic obstructive pulmonary disease = 0 mark. Myocardial infarction = 0 mark as ECG is normal. If all answers are from one
single system = 1 mark.

9. Based on the patients history, state ONE (1) MOST appropriate and evidence-based psychotherapy that is
suitable for this patient. (1 mark) / 1 mark

Interpersonal psychotherapy

Other psychotherapies = 0 mark.

10. Based on the patients history, state how the psychotherapy recommended by you (in Question 4) works for
her. (2 marks) /2 marks

Answers may vary. Examiners are advised to use his or her own discretion to decide whether marks should be
awarded. For students who did not state interpersonal psychotherapy in Q.4, they should only get a maximum of 1
mark for question 5.

Intepersonal psychotherapy can help the patient to analyze previous interpersonal problems in her work place.
Interpersonal psychotherapy involves role play and patient can improve her communication with her in-laws. The role
play can identify previous maladaptive communication patterns. Interpersonal psychotherapy can help her to deal
with the role transition from an administrative person to become a housewife.

Question!2!(Total!=!10!marks)!

You are a general practitioner. A 25-year-old man suffering from schizoaffective disorder is bought to the clinic by her mother. He
was diagnosed to suffer from schizoaffective disorder 6 months ago. Her mother is concerned about his recent weight gain of 10 kg
in 3 months. According to her mother, his psychiatrist from a restructured hospital started a new antipsychotic and mood stabilizer
to control his psychiatric symptoms.

6. State TWO (2) antipsychotics which are MOST likely to cause weight gain in this patient. (2 marks) /2 marks

Any two of the following:


Olanzapine (1 mark) or
Clozapine (1 mark) or
Quetiapine (1 mark) or
Risperidone (0.5 mark)
Other antipsychotics = 0 mark

7. State TWO (2) mood stabilizers which are MOST likely to cause weight gain in this patient. (2 marks) /2 marks

Any two of the following:


Lithium (1 mark) or

!
!
259
Sodium valproate (1 mark) or
Lamotrigine (1 mark) or
Other mood stabilizers: no mark

8. State THREE (3) MOST relevant blood tests which you would like to order and abnormalities you would like to rule out for
this patient. (3 marks) / 3 marks

Investigations Abnormalities

/0.5 mark /0.5 mark

/ 0.5 mark / 0.5 mark

/ 0.5 mark / 0.5 mark

Investigations Abnormalities
Fasting glucose (0.5 mark) Rule out diabetes (0.5 mark)

Fasting lipid / cholesterol / HDL / Rule out hyperlipidaemia (0.5 mark)


LDL (0.5 mark)

Liver function tests (0.5 mark) Rule out raised AST and ALT / fatty liver / liver impairment
(0.5 mark)

Thyroid function test Rule out hypothyroidism and associated weight gain

9. State ONE (1) MOST RELEVANT AND SPECIFIC symptom in schizophrenia which may contribute to obesity (2 marks). !
1. Avolition or ahehdonia or lack of interest to do exercise (2 marks)
2. Negative symptom (1 mark)
3. Other negative symptom e.g. apathy, alogia, attentional deficit (0 mark)

10. State ONE (1) antipsychotic that has the LOWEST RISK to cause weight gain. (1 mark)
Aripiprazole/ haloperidol (1 mark). Other antipsychotic = 0 mark

!
!
260
Question 3

You are a paediatric resident working in the Child and Accident Emergency Department. You
are seeing an 11-year-old boy who was brought in by his foster parents. According to his
foster parents, he has a lot of behavioural problems at home and school. He cannot sit still to
do his homework at home. He also breaks school rules. Tonight, he had a fight with his foster
parents. This boy is a victim of child abuse.

3. State TWO (2) MOST RELEVANT diagnoses (2 marks) that this boy may suffer from and
indicate THREE (3) additional clinical features (6 marks) which you will look for to
support your diagnosis. (total 8 marks)

Diagnosis 3 additional clinical features per diagnosis

1. 1.

2.

3.

/1 mark /3 marks

2. 1.

2.

3.

/1 mark / 3 marks

!
!
261
Diagnosis 3 additional clinical features per diagnosis

1. Attention deficit Any THREE of the following:


hyperactivity disorder (1
mark) Starts tasks or activities but not able to follow through and
Abbreviation = 0 mark finish.
Organization of tasks or activities is impaired.
Loses things necessary for tasks and activities such as
school assignments or stationary.
Instructions are not followed.
Distraction by external stimuli.
Careless mistakes
Forgetfulness in daily activities
Waiting for in lines or await turns in game cause
frustration.
On the move most of the time such as running and
climbing.
Restlessness and jitteriness.
Squirms on seat.
Impulsivness
o Talk excessively without appropriate response to social
constraints.
o Fidgets with hands and feet.
o Answers are blurted out before questions.
o Interruption of other peoples conversations.
o Loud noise in playing.

2. Conduct disorder Any THREE of the following:

Repetitive and persistent pattern of behaviour in which


either the basic rights of the others or major age
appropriate societal rules are violated
Often telling lies and breaking promises
Refusing adults requests or defying rules
Staying out after dark against parental prohibition
Blaming others for his mistakes
/1 mark Initiating fights with the others
Using weapons to harm the others
Exhibiting physical cruelty (also to animals)
Confronting victims during a crime
Forcing another person into sexual activity
Frequently bulling the others.
Truancy
Oppositional defiant disorder = 0 mark Involvement in gang.

/ 3 marks

!
!
262
4. If you have a chance to speak to the child protection officer, state TWO (2) MOST
relevant psychiatric disorders that his parents may suffer from. (2 marks)

/ 2 marks

Any two of the following:

1) Antisocial personality disorder (1 mark)


2) Substance abuse (1 mark)

Adult ADHD = 0 mark (It is unlikely MFS needs to foster out the child if parent has adult ADHD).

Question!4!

You!are!a!geriatric!resident.!A!70NyearNold!man!presents!with!dementia.!His!son!wants!to!find!
out!more!about!the!treatment!for!dementia.!

6. State the name of TWO (2) acetylcholinesterase inhibitors available in Singapore. (2 marks)!
/ 2 marks
Any two of the following:
Donepezil (1 mark) or
Rivastigmine (1 mark) or
Galantamine (1 mark).!

7. List TWO (2) types of dementia for which acetylcholinesterase inhibitors are indicated (2
marks) / 2 marks

Any two of the following:


Alzheimers disease (1 mark)
Lewy body dementia (1 mark)

8. List TWO (2) COMMON side effects of acetylcholinesterase inhibitors (2 marks)


/ 2 marks

!
!
263
Any!two!of!the!following:!
Diarrhoea!(excessive!cholinergic!effects)!1!mark!or!!
Difficulty!in!sleeping!(excessive!cholinergic!effects)!1!mark!or!!
Dizziness!1!mark!or!
Feeling!agitated!1!mark!or!
Headache!1!mark!or!
Loss!of!appetite!1!mark!or!
Muscle!cramps!(excessive!cholinergic!effects)!1!mark!or!
Tiredness!1!mark.!
Uncommon!side!effects!like!gastrointestinal!bleeding!or!bradycardia!(0!mark)!

9. List TWO (2) SPECIFIC contraindications to the use of acetylcholinesterase inhibitors (2


marks)
/ 2 marks
Any two of the following:
Severe dementia or Score of Mini Mental State Examination < 10 or 12 out of 30 (1 mark)
Bradycardia or taking beta blockers (1 mark)
Gastrointestinal bleeding or peptic ulcer (1 mark)

Allergy to acetylcholinesterase inhibitors: not specific enough (0 mark)

10. His son wants to know potential benefits that can be expected from treatment with
acetylcholinesterase inhibitors. List TWO (2) potential benefits (2 marks).
/ 2 marks

Answers may vary. Examiners are requested to use his or her own discretion to decide whether
marks should be awarded.

1. Reduce the rate of deterioration of memory loss or improve cognition (1 mark)


2. Delay nursing home placement (1 mark)
3. Improve behavioural and psychiatric disorders associated with dementia (1 mark)

!
!
264
SAQ (Paper 4)

Question 1

You are the resident working in the Accident and Emergency Medicine Department. A 50-year-
old man is admitted as a result of paracetamol overdose. He was treated in the Institute of
Mental Health for alcohol dependence 7 years ago.

1. State Four (4) clinical features which you would elicit in this man during history taking
to establish the diagnosis of alcohol dependence. (4 marks) Examiner: /4 marks

2. State TWO (2) medical or surgical disorders from two different body systems which will
result from prolonged alcohol usage. (2 marks) Examiner: /2 marks

Body system Specific medical or surgical disorder (not sign/symptom)


/1 mark

/1 mark

3. State TWO (2) physical signs which you will elicit from two different body systems to
support the diagnosis of alcohol dependence. (2 marks) Examiner: /2 marks

Body system Specific sign


/1 mark

/1 mark

4. Please explain the following laboratory finding (2 marks).

AST: 600 IU/L


ALT: 150 IU/L

!
!
265
Question 3

You are a general practitioner. A 33-year-old woman delivered a boy two


months ago. She is single and she broke up with her boyfriend recently. This is her first
pregnancy and it was unplanned. The infant was delivered by normal vaginal delivery. She is
unemployed and receives social assistance ($300 per month). She is not close to her family
members and stays alone in a 1-room HDB flat. She was seen by a psychiatrist at the Institute
of Mental Health 3 years ago and she did not return for further appointment. She complains of
low mood, poor sleep, poor appetite, poor energy and hopelessness for the past 6 weeks. You
suspect that she suffers from postnatal depression.

1. From her history, list FOUR (4) specific risk factors for postpartum depression. (4 marks)
Examiner: /4 marks

2. State TWO (2) MOST important psychotic features which you would elicit as part of the risk
assessment (2 marks) Examiner: /2 marks

1. She has an appointment to see a psychiatrist in one month. She cannot wait and wants to
seek your opinion. She is very keen to continue breastfeeding. If she wants to take an
antidepressant such as a selective serotonin reuptake inhibitor (SSRI) and continues
breastfeeding, state TWO (2) advices to minimize SSRI exposure to the infant. (2 marks)
Examiner: / 2 marks

4. If the patient does not want to take antidepressant, please recommend TWO (2) alternative
treatment options. (2 marks) Examiner: / 2 marks

Question 4

You are a general practitioner. A mother brings her 13-year-old girl to see you. She complains
of low mood, poor sleep, poor appetite and recurrent self-harm. She often cuts her wrist with a
pen knife. She has difficulty to cope with her study in Secondary 1. She often brings a pen
knife to school and cuts herself in the toilet. Her mother breaks into tear in the middle of the
interview because her husband is dependent on alcohol and has history of violence. The
patient is very close to her mother. As a result, the patient and her mother have no
communication with her father.

Question 1: Her mother worries about her personality development. State ONE (1) personality
disorder which she will MOST likely develop if the above pattern continues without any
intervention? (2 marks) Examiner: / 2 marks

Question 2: Based on your answer in Question 1, state TWO (2) MOST relevant comorbid
psychiatric disorders. (2 marks) Examiner: / 2 marks

!
!
266
Question 3 Her mother hopes that you can prescribe a psychotropic medication to
stabilize her. State ONE (1) psychotropic medication which is most suitable to her. (2 marks)
Examiner: / 2 marks

Class of medication Specific name of medication

/1 mark /1 mark

Question 4 Besides cognitive behaviour therapy, state TWO (2) MOST relevant
psychotherapy which will benefit this patient. (2 marks)
Examiner: /2 marks

Question 5 Her mother is concerned about her safety in the next few weeks while waiting for
the effect of the medication. List TWO (2) MOST relevant strategies to manage safety issues in
this patient. (2 marks) Examiner: /2 marks

!
!
267
SAQ (Paper 4)

Question 1

You are the resident working in the Accident and Emergency Medicine Department. A 50-year-
old man is admitted as a result of paracetamol overdose. He was treated in the Institute of
Mental Health for alcohol dependence 7 years ago.

5. State Four (4) clinical features which you would elicit in this man during history taking
to establish the diagnosis of alcohol dependence. (4 marks) Examiner: /4 marks

Any 4 of the following criteria for substance dependence: WASTED/ CAGE


o Withdrawal
o Amounts of alcohol intake (large amount for long periods)
o Social activities replaced by time spent acquiring substance
o Tolerance: higher amount of alcohol to achieve the same effect
o Efforts to cut down unsuccessful
o Despite adverse consequences, keeps drinking alcohol

o Feel Annoyed by criticisms of other people on drinking


o Feel Guilty about drinking
o Eye opening: drinking alcohol after wake-up in the morning

6. State TWO (2) medical or surgical disorders from two different body systems which will
result from prolonged alcohol usage. (2 marks) Examiner: /2 marks

Body system Specific medical or surgical disorder (not sign/symptom)


/1 mark

/1 mark

Any 2 of the following (The list is not completed and examiner will use his or her discretion to decide
whether marks should be awarded).

CNS complications: withdrawal fits/epilepsy, head injury as a result of fall, cerebellar damage
Gastrointestinal system: cancer in the oral cavity (e.g. tongue, throat), oesophagitis, gastritis,
gastric cancer.
Hepatobiliary system: pancreatic cancer, liver cirrhosis, alcohol hepatitis.
Peripheral nervous system: peripheral neuropathy
Rheumatology: gouty arthritis
Reproductive system: erectile dysfunction.

!
!
268
7. State TWO (2) physical signs which you will elicit from two different body systems to
support the diagnosis of alcohol dependence. (2 marks) Examiner: /2 marks

Body system Specific sign


/1 mark

/1 mark

Any 2 of the following (The list is not completed and examiner will use his or her discretion to decide
whether marks should be awarded).

Endocrine system: gynaecomastia


Hepatobiliary system: spider angiomata/navei; ascites; signs of liver cirrhosis: hard liver
Central nervous system: cerebellar ataxia (unsteady and wide-based gait)
Rheumatology: painful big toe
Peripheral nervous system: less of peripheral sensation to different stimuli (e.g. pin or cotton wool)

8. Please explain the following laboratory finding (2 marks).

AST: 600 IU/L


ALT: 150 IU/L

This patient suffers from alcohol hepatitis due to AST: ALT ratio > 2.

!
!
269
Question 3

You are a general practitioner. A 33-year-old woman delivered a boy two


months ago. She is single and she broke up with her boyfriend recently. This is her first
pregnancy and it was unplanned. The infant was delivered by normal vaginal delivery. She is
unemployed and receives social assistance ($300 per month). She is not close to her family
members and stays alone in a 1-room HDB flat. She was seen by a psychiatrist at the Institute
of Mental Health 3 years ago and she did not return for further appointment. She complains of
low mood, poor sleep, poor appetite, poor energy and hopelessness for the past 6 weeks. You
suspect that she suffers from postnatal depression.

3. From her history, list FOUR (4) specific risk factors for postpartum depression. (4 marks)
Examiner: /4 marks

Any three of the following:


1. Single mother (the mother is not that old in this case).
2. Unplanned pregnancy
3. Recent break up/Unemployment / financial difficulties / low socio-economic status.
4. Poor support from family
5. History of depression
6. Negative confinement
7. Past psychiatric history

4. State TWO (2) MOST important psychotic features which you would elicit as part of the risk
assessment (2 marks) Examiner: /2 marks

Answer may vary. Examiner will use his or her discretion to decide whether marks should be
awarded.
2. Command hallucination which asks her to harm herself and her infant.
3. Delusion which suggests that the infant is malformed/abnormal or possessed by demons and
she need to attack the infant.

Other mood congruent delusions e.g. delusion of guilt or nihilistic delusion 0.5 marks

4. She has an appointment to see a psychiatrist in one month. She cannot wait and wants to
seek your opinion. She is very keen to continue breastfeeding. If she wants to take an
antidepressant such as a selective serotonin reuptake inhibitor (SSRI) and continues

!
!
270
breastfeeding, state TWO (2) advices to minimize SSRI exposure to the infant. (2 marks)
Examiner: / 2 marks

Answer may vary. Examiner will use his or her discretion to decide whether marks should be
awarded. Any two of the following:

1. Take the lowest effective dose of a SSRI


2. Take SSRI once per day instead of twice per day.
3. Breastfeed or pump out breast milk immediately after or just before taking the SSRI
4. Discard the first round of breast milk which may contain SSRI.
5. Take SSRI just before the longest period of sleep of the infant.
6. Combination of breast milk and formula milk.
7. Use SSRI with short half live.

Other answers:

Name specific SSRI without reasoning (0 mark)


SSRIs are harmful to the infant and she must not take SSRI (0 mark)
She must give up breastfeeding (0 mark)

4. If the patient does not want to take antidepressant, please recommend TWO (2) alternative
treatment options. (2 marks) Examiner: / 2 marks

1. Psychotherapy e.g. cognitive behaviour therapy or other forms of psychotherapy.


2. Electro-convulsive therapy

Suggest other medications: 0 mark

Question 4

You are a general practitioner. A mother brings her 13-year-old girl to see you. She complains
of low mood, poor sleep, poor appetite and recurrent self-harm. She often cuts her wrist with a
pen knife. She has difficulty to cope with her study in Secondary 1. She often brings a pen
knife to school and cuts herself in the toilet. Her mother breaks into tear in the middle of the
interview because her husband is dependent on alcohol and has history of violence. The
patient is very close to her mother. As a result, the patient and her mother have no
communication with her father.

Question 1: Her mother worries about her personality development. State ONE (1) personality
disorder which she will MOST likely develop if the above pattern continues without any
intervention? (2 marks) Examiner: / 2 marks

Answer: Borderline personality disorder (2 marks). Other diagnosis: no mark.

Question 2: Based on your answer in Question 1, state TWO (2) MOST relevant comorbid
psychiatric disorders. (2 marks) Examiner: / 2 marks

Any two of the following:


1. Bulimia nervosa
2. Depressive disorder
3. Post-traumatic stress disorder

!
!
271
4. Substance abuse or dependence

Question 3 Her mother hopes that you can prescribe a psychotropic medication to
stabilize her. State ONE (1) psychotropic medication which is most suitable to her. (2 marks)
Examiner: / 2 marks

Class of medication Specific name of medication

/1 mark /1 mark

Class of medication Specific name of medication

Selective serotonin reuptake inhibitor Fluoxetine

/1 mark /1 mark

Other answer: 0 mark.

Question 4 Besides cognitive behaviour therapy, state TWO (2) MOST relevant
psychotherapy which will benefit this patient. (2 marks)
Examiner: /2 marks

Any two of the following:


1. Dialectical behaviour therapy
2. Metalization based therapy
3. Interpersonal psychotherapy
4. Family therapy.

Question 5 Her mother is concerned about her safety in the next few weeks while waiting for
the effect of the medication. List TWO (2) MOST relevant strategies to manage safety issues in
this patient. (2 marks) Examiner: /2 marks

Examiner has to use his or her own discretion to decide whether marks should be awarded. Any two
of the following:

1. Discuss need for her mother and teacher to monitor for changes in behaviour or self-harm at
home and in school.
2. Remove sharp objects, weapons or pen knives or unnecessary medications from home and
school.
3. Provide information and access to crisis services in the emergency department.
4. Her mother needs to supervise and administer antidepressant to patient.
5. Frequent and regular review by doctors to monitor response and side effects

!
!
272
!

SAQ 2014 EOPT First rotation

Question 1 (10 marks)


A 24-year-old female executive consults her family physician complaining that she has
faced problems at work for the past year. It started after a presentation when her slides
were mixed-up and she was at a loss of words. Since then, she feels tremulous and
stammers at presentations. She is increasingly fearful of doing presentations, worrying
that she may faint or vomit in front of her colleagues. Her fears have grown to the
extent that she has not gone to work for 2 weeks.

1. What is the most likely diagnosis? (1 mark)

2. List TWO (2) medications from different classes that can treat her symptoms? (4
marks)

Class of medications Name of the medication

!
!
273
( /1 mark) ( /1 mark)

( /1 mark) ( /1 mark)

3. She wants to receive psychotherapy. Please state the most evidence based
psychotherapy based on your diagnosis. (1 mark)

4. She wants to know more how psychotherapy works. Based on your answer in
Question 3 and her clinical history, list TWO (2) specific strategies which can
help her. (4 marks)
!

Question 2 (10 marks)

A 40-year-old woman with a bipolar disorder was stable on sodium valproate till a
year ago when she suffered three episodes of relapses. Her psychiatrist recommends
switching to lithium. She uses recreational drugs, sometimes forgets to take her
medication when she is clubbing till 5 am and has had unprotected sex many times in
the past three months. A friend asks her to borrow $10,000 to invest in a food and
beverage business.

Question 1 She wants to know the systemic side effects of lithium when the dose is within
therapeutic range. State FOUR (4) body systems affected by lithium and Name ONE (1) specific
side effect of lithium under each system. (4 marks)

Systems Specific side effects

( /0.5 mark) ( /0.5 mark)

!
!
274
( /0.5 mark) ( /0.5 mark)

( /0.5 mark) ( /0.5 mark)

( /0.5 mark) ( /0.5 mark)

Question 2 Based on her history and the possibility of starting lithium, List THREE (3) most
relevant laboratory investigations (3 marks)

( / 3 marks)

Question 3 Based on her current lifestyle, offer THREE (3) MOST relevant advices. (3
marks)

Question 3

You are an Emergency Room Resident assessing a 16-year-old girl brought by


ambulance after she was found lying in a corridor in a mall. The brief history given is
that she had left home 3 days ago after a break-up with her boyfriend. She had
overdosed with 50 tablets of paracetamol at the mall. She was found by passer-by and
then sent to the Accident and Emergency Department by ambulance.

!
!
275
Question 1 During your assessment, state FOUR (4) most relevant risk factors
suggesting high suicide risk which you would explore.
( / 4 marks)
Question 2 During your assessment, state TWO (2) most relevant protective factors
against suicide which you would explore.

( / 2 marks)

Question 3 State FOUR (4) most appropriate management orders for this patient in
the Accident and Emergency Department.

( / 4 marks)

Question 4 (10 marks)

A 70-year-old man was bought in by his family because he claims he has been seeing
ghosts for the past few months and is now hearing voices asking him to jump from his
9th floor flat. He has hypertension which is well-controlled and has been in good health.

Question 1 During the assessment, state THREE (3) risk factors for late onset
psychosis. (3 marks)

Question 2 State TWO (2) differences between psychotic features found in an old

!
!
276
person and psychotic features found in a young person suffering from schizophrenia (
/ 2 marks)

Question 3 Based on his history, state THREE (3) most likely psychiatric diagnoses
(3 marks)

Question 4 The family informs you that a family doctor started risperidone and he
developed severe rigidity and tremor. State ONE (1) antipsychotic drug which is MOST
suitable for him. (2 marks)
!

SAQ 2014 EOPT First rotation

Question 1 (10 marks)


A 24-year-old female executive consults her family physician complaining that she has
faced problems at work for the past year. It started after a presentation when her slides
were mixed-up and she was at a loss of words. Since then, she feels tremulous and
stammers at presentations. She is increasingly fearful of doing presentations, worrying
that she may faint or vomit in front of her colleagues. Her fears have grown to the
extent that she has not gone to work for 2 weeks.

!
!
277
5. What is the most likely diagnosis? (1 mark)

Social phobia (1 mark) or diagnosis: 0 mark including agoraphobia. ( /1 mark)

6. List TWO (2) medications from different classes that can treat her symptoms? (4
marks)

Class of medications Name of the medication

( /1 mark) ( /1 mark)

( /1 mark) ( /1 mark)

Class of medications Name of the medication


Selective serotonin reuptake inhibitor 1. Paroxetine(1 mark) or
(SSRI) (1 mark) 2. Fluvoxamine (1 mark) or
3. Fluoxetine (1 mark) or
4. Escitaloporam (1 mark) or
5. Sertraline (1 mark)
Beta-blocker (1 mark) 1. Propranolol (1 mark)

Benzodiazepine (1 mark) 1. Alprazolam (1 mark)


2. Clonazepam (1 mark)
3. Lorazepam (1 mark)

7. She wants to receive psychotherapy. Please state the most evidence based
psychotherapy based on your diagnosis. (1 mark)

Cognitive behaviour therapy; other psychotherapy (0 mark) ( /1 mark)

!
!
278
8. She wants to know more how psychotherapy works. Based on your answer in
Question 3 and her clinical history, list TWO (2) specific strategies which can
help her. (4 marks)
!

Other logical answers are acceptable. Examiner is advised to use his or her discretion to
decide whether marks should be awarded.

Any two of the following:

1. Provide cognitive explanation for social phobia (2 marks) or


2. Exposure to simulated situations e.g. presentation with the psychologist (2 marks) or
3. Cognitive restructuring to teach control of maladaptive thoughts (e.g. humiliation,
vomiting in front of others). (2 marks) or
4. Social skill training (2 marks)

( / 4 marks)
!

Question 2 (10 marks)

!
!
279
A 40-year-old woman with a bipolar disorder was stable on sodium valproate till a
year ago when she suffered three episodes of relapses. Her psychiatrist recommends
switching to lithium. She uses recreational drugs, sometimes forgets to take her
medication when she is clubbing till 5 am and has had unprotected sex many times in
the past three months. A friend asks her to borrow $10,000 to invest in a food and
beverage business.

Question 1 She wants to know the systemic side effects of lithium when the dose is within
therapeutic range. State FOUR (4) body systems affected by lithium and Name ONE (1) specific
side effect of lithium under each system. (4 marks)

Systems Specific side effects

( /0.5 mark) ( /0.5 mark)

( /0.5 mark) ( /0.5 mark)

( /0.5 mark) ( /0.5 mark)

( /0.5 mark) ( /0.5 mark)

Other logical answers are acceptable. Examiner is advised to use his or her discretion to
decide whether marks should be awarded. Signs of lithium toxicity e.g. ataxia, coarse tremor,
cardiac arrest, respiratory difficulty does not count.

Systems Specific side effects

!
!
280
Endocrine system Hypothyroidism

( / 0.5 mark) ( /1 mark)

Renal system Renal failure or

Polyuria

( /0.5 mark)

( /1 mark)

Dermatological system Psoriasis

( /0.5 mark) ( /1 mark)

Gastrointestinal system Metallic taste or nausea

( /0.5 mark) ( /1 mark)

Nervous system Tremor

( /0.5 mark) ( /1 mark)

Question 2 Based on her history and the possibility of starting lithium, List THREE (3) most
relevant laboratory investigations (3 marks)

!
!
281
( / 3 marks)

Any three of the following:


1. Renal function tests (1 mark) or
2. Thyroid function test (1 mark) or
3. Urine drug screen (1 mark) or
4. Urine pregnancy test (1 mark) or
5. HIV test / screen for sexually transmitted diseases (1 mark) or
6. Valproate level (1 mark)

Question 3 Based on her current lifestyle, offer THREE (3) MOST relevant advices. (3
marks)

( / 3 marks)

Any three of the following:


1. Regular sleep time and wake-up time (1 mark) or
2. Avoid unprotected sex (1 mark) or
3. Avoid overspending or borrowing money or foolhardy investment (1mark) or
4. Avoid alcohol or recreational drug (1 mark) or
5. Ensure compliance to mood stabilizer (1 mark)

Question 3

You are an Emergency Room Resident assessing a 16-year-old girl brought by


ambulance after she was found lying in a corridor in a mall. The brief history given is
that she had left home 3 days ago after a break-up with her boyfriend. She had
overdosed with 50 tablets of paracetamol at the mall. She was found by passer-by and
then sent to the Accident and Emergency Department by ambulance.

!
!
282
Question 1 During your assessment, state FOUR (4) most relevant risk factors
suggesting high suicide risk which you would explore.
Other logical answers are acceptable. Examiner is advised to use his or her discretion to
decide whether marks should be awarded.
1. Poor relationship with parents and family members (1 mark) or
2. Previous suicidal ideation or attempts (1 mark) or
3. Belief about lethality of paracetamol overdose (1 mark) or
4. Circumstances of suicide (e.g. suicide notes, isolation, avoidance of discover) (1 mark) or
5. Current plans for further suicidality (1 mark) or
6. Strong preoccupation with the relationship/ suicide as a mean to salvage the relationship
(1 mark) or
7. Presence or absence of alcohol intoxication (1 mark) or
8. Family history of psychiatric illness (1 mark) or
9. Current or past history of major depression or other psychiatric illness/or chronic medical
illness (1 mark).
( / 4 marks)
Question 2 During your assessment, state TWO (2) most relevant protective factors
against suicide which you would explore.

Other logical answers are acceptable. Examiner is advised to use his or her discretion to
decide whether marks should be awarded.
1. Religion (1 mark) or
2. Seeing a counsellor or psychologist (1 mark) or
3. Regret over suicide attempt (1 mark) or
4. Positive plan in the near future (e.g. return to study) (1 mark).

( / 2 marks)

Question 3 State FOUR (4) most appropriate management orders for this patient in
the Accident and Emergency Department.

Other logical answers are acceptable. Examiner is advised to use his or her discretion to
decide whether marks should be awarded.

1. Order investigations including Full blood count, liver function tests, renal function
tests, paracetamol levels, urine drug screen, electrocardiogram etc (1 mark) or
2. Activate charcoal to reduce absorption (1 mark) or
3. Suicide precaution (1 mark) or
4. Start N-acetylcystine (NAC) (1 mark) or
5. Inform family about her suicide attempt or to obtain collateral information (1 mark) or

!
!
283
6. Inform on-call psychiatrist for assessment (1 mark) or
7. Admit to the medical ward for further management (1 mark).

( / 4 marks)

!
!
284
Question 4 (10 marks)

A 70-year-old man was bought in by his family because he claims he has been seeing
ghosts for the past few months and is now hearing voices asking him to jump from his
9th floor flat. He has hypertension which is well-controlled and has been in good health.

Question 1 During the assessment, state THREE (3) risk factors for late onset
psychosis. (3 marks)

Other logical answers are acceptable. Examiner is advised to use his or her discretion to
decide whether marks should be awarded.
1. Sensory impairment (e.g. deafness) (1 mark) or
2. Social isolation (1 mark) or
3. Paranoid or schizoid or schizotypal personality trait (1 mark) or
4. Past psychiatric illness (e.g. history of schizophrenia/ severe depression/ bipolar
disorder) (1 mark) or
5. Past medical illness or underlying medical illness (e.g. stroke, head injury, brain
tumour) or medication which causes psychosis steroid induced psychosis/
Parkinsons disease (1 mark)

( / 3 marks)

Question 2 State TWO (2) differences between psychotic features found in an old
person and psychotic features found in a young person suffering from schizophrenia (
/ 2 marks)

Other logical answers are acceptable. Examiner is advised to use his or her discretion to
decide whether marks should be awarded.

1. Elderly exhibits less first rank symptoms / the content of psychotic symptoms (e.g.
auditory hallucination) is often less elaborated in elderly (1 mark) or
2. Elderly exhibits less negative symptoms as compared to young people suffering from
chronic schizophrenia (1 mark) or
3. Elderly are more likely to exhibit more paranoid ideas or delusions compared to
young people (1 mark).

Question 3 Based on his history, state THREE (3) most likely psychiatric diagnoses
(3 marks)

Any 3 of the following:


1. Late onset schizophrenia/ schizophrenia (1 mark) or
2. Late onset depressive disorder with psychotic features/severe depressive disorder with
psychotic features (1 mark) or

!
!
285
3. Late onset bipolar disorder / bipolar disorder with psychotic features (1 mark) or
4. Dementia with psychotic features (1 mark) or
5. Delirium or acute confusional state (1 mark)

Question 4 The family informs you that a family doctor started risperidone and he
developed severe rigidity and tremor. State ONE (1) antipsychotic drug which is MOST
suitable for him. (2 marks)

Any 1 of the following


Quetiapine (2 marks) or
Olanzapine (1 mark) or
Aripiprazole (2 marks)

( / 2 marks)
!

SAQ!!2014!Rotation!2!

Question 1

You are a general practitioner. Depression is a common mental health problem. You are about
to start a patient on an antidepressant.

1. List FOUR (4) factors related to the patients which you would consider in choosing an
antidepressant for a patient suffering from depression (4 marks). ( /4 marks)

2. Besides selective serotonin reuptake inhibitors (SSRI), name two other


antidepressants available in Singapore and fill the following table. (6 marks)
( /6 marks)

Name of antidepressants Other therapeutic effects (besides Specific side effects associated with
antidepressant effect and anxiety antidepressant (besides gastric
reduction effect) discomfort, headache, giddiness)

( /1 mark) ( /1 mark) ( /1 mark)

( /1 mark) ( /1 mark) ( /1 mark)

!
!
286
Question!2!

You!are!a!general!practitioner.!A!25NyearNold!man!presents!with!disorganized!thinking!and!
speech.!You!have!a!few!psychiatric!differential!diagnoses.!

1. State!and!define!ONE!(1)!common!type!of!disorganised!thinking!that!are!found!in!people!
without!any!mental!illness.(4!marks)!!!!!!!!!!!!!!!!!!!!!!!!!!!!!!!!!!!!!!!!!!!!!!!!!!!!!!!!!!!!!!!!!!!!!!(!!!!!!!!!!!/4!
marks)!
!
!
!
2. State!and!define!TWO(2)!common!types!of!disorganised!thinking!that!are!found!in!
people!suffering!from!schizophrenia.!(4!marks)!!!!!!!!!!!!!!!!!!!!!!!!!!!!!!!!!!!!!!!!!!!!!!!!!!!!!!!!!!!!!!!!!!(!!!!!!!!!!!
/4!marks)!
!
Type!of!disorganised!thinking! Definition!

! !
! !
(!!!!!!!/1!mark)! (!!!!!!!/1!mark)!

! !
!
(!!!!!!!/1!mark)! ! !

(!!!!!!!/1!mark)!

!
!
3. State!and!define!ONE!(1)!common!type!of!disorganised!thinking!that!is!found!in!people!
suffering!from!bipolar!disorder.!(2!marks)!!!!!!!!!!!!!!!!!!!!!!!!!!!!!!!!!!!!!!!!!!!!!!!!!!!!!!!!!!!!!!(!!!!!!!!!!!/2!
marks)!
!
Type!of!disorganised!thinking! Definition!

! !
! !
(!!!!!!!/1!mark)! (!!!!!!!/1!mark)!

Question 3

!
!
287
You are a paediatric resident. You are seeing an 11-year-old boy with repetitive eye blinking
and this has caused social dysfunction. He exhibits facial tics and gets frequent facial tics
when he is anxious. He sometimes cannot control his hand movements which affect his
writing.

1. You suspect that he suffers from Tourettes syndrome. State the most APPROPRIATE
question which you would ask to establish the diagnosis of Tourettes syndrome (2
marks) ( / 2 marks)

2. What distinguishes Tourettes disorder from a chronic motor or vocal tic disorder? (2
marks) ( /2 marks)

3. State ONE (1) psychiatric condition often comorbid with Tourettes syndrome. (2
marks) ( /2 marks)

4. State ONE (1) psychiatric medication that can be used to treat Tourettes syndrome. (2
marks) ( /2 marks)

5. State Two (2) psychological interventions that can be used to treat Tourettes
syndrome. (2 marks) ( /2 marks)

Question!4:!

You! are! a! general! practitioner! (GP).! ! The! son! of! a! 60NyearNold! man! is! concerned! because! his!
father!is!taking!a!medication!called,!Flunitrazepam!(Rohypnol)!for!2!years.!The!patient!obtained!
Rohypnol! from! another! GP! who! had! retired! 1! month! ago.! Rophypnol! is! a! very! potent!
benzodiazepine!and!is!a!derivative!of!nitrazepam.!Rophypnol!is!highly!addictive!and!is!listed!as!
a! controlled! drug! by! the! Central! Narcotics! Bureau.! ! His! son! has! noted! the! following!
symptoms:! anxiety,! aggression,! getting! high,! insomnia,! memory! loss,! slurred! speech,!
tremor!in!his!father.!The!patient!was!admitted!to!the!hospital!due!to!fits.!!
!
1. Based! on! the! symptoms! reported! by! his! son! (underlined),! classify! the! symptoms! as!
intoxication!or!withdrawal!symptoms!associated!with!benzodiazepine!(4!marks)!
(!!!!!!!!!!!!!!/!4!marks)!

Intoxication!symptoms! Withdrawal!symptoms!

1.! 1.!

!
!
288
! !

(!!!!!!!!!/!0.5!marks)! (!!!!!!!!!/!0.5!marks)!

2.! 2.!

! !

(!!!!!!!!!/!0.5!marks)! (!!!!!!!!!/!0.5!marks)!

3.! 3.!

! !

(!!!!!!!!!/!0.5!marks)! (!!!!!!!!!/!0.5!marks)!

4.! 4.!

! !

(!!!!!!!!!/!0.5!marks)! (!!!!!!!!!/!0.5!marks)!

2. State! the! MOST! APPROPRIATE! psychotropic! medication! to! treat! his! withdrawals!
(2!marks).!

3. After! stabilisation,! his! son! would! like! you! to! prescribe! a! nonTbenzodiazapine!
medication!to!help!his!father!to!sleep.!Name!ONE!sedative!antidepressant!and!ONE!
other!sedative!medication.!4!marks!! ! ! ! ! ! !(!!!!!!!!!!
/4!marks)!

Sedative!antidepressant

(!!!!!!!!!!/2!marks)

Other!sedative!medication!(nonT

!
!
289
benzodiazapine)

!!!!!!!!!!!!!!!!!!!!!!!!!!!!!!!!!!!!!!!!!!!!!!!!!!!!!!!!!!!!!!!!!!!!(!!!!!!!!!!/2!marks)

SAQ!!2014!Rotation!2!

Question 1

You are a general practitioner. Depression is a common mental health problem. You are about
to start a patient on an antidepressant.

3. List FOUR (4) factors related to the patients which you would consider in choosing an
antidepressant for a patient suffering from depression (4 marks). ( /4 marks)

Any four of the following:

Previous trials of antidepressants (1 mark) or


Drug-to-drug interaction (1 mark)
Previous response to antidepressants / efficacy(1 mark) or
Side effect profile/ tolerability/ safety profile of antidepressant (1 mark)
Psychiatric comorbidity (1 mark) or e.g. sleep difficulties, comorbid anxiety
Risk of suicide or overdose (1 mark) or

!
!
290
Underlying medical conditions (1 mark) or
Concurrent medications (1 mark) or
Symptom profile of depression (1 mark) or
Patients preference (1 mark) or
Age of patient (1 mark) or
Pregnancy of patient / breast feeding (1 mark) or
Frequency of dose (only fluoxetine can be given every other day) (1mark) or
Financial status or affordability by patients (1 mark).
Other answers from students: Liver function (more important than kidney function),
current body weight and BMI, occupation (with explanation e.g. drowsiness when driving)

4. Besides selective serotonin reuptake inhibitors (SSRI), name two other


antidepressants available in Singapore and fill the following table. (6 marks)
( /6 marks)

Name of antidepressants Other therapeutic effects (besides Specific side effects associated with
antidepressant effect and anxiety antidepressant (besides gastric
reduction effect) discomfort, headache, giddiness)

( /1 mark) ( /1 mark) ( /1 mark)

( /1 mark) ( /1 mark) ( /1 mark)

Examples: mirtazapine, venlafaxine, trazodone, agomelatine, amitriptyline etc. The following is an


example of acceptable answer. Examiner has to use his or her own discretion to decide whether
marks should be awarded. Spelling error of the name of the drug (deduct 0.5 marks).

Name of antidepressants Other therapeutic effects (besides Specific side effects associated with
antidepressant effect and anxiety antidepressant (besides gastric
reduction effect) discomfort, headache, giddiness)

Mirtazapine Useful for sedation and insomnia Weight gain

( /1 mark) ( /1 mark) ( /1 mark)

Amitriptyline or Neuropathic pain or enuresis Constipation


imipramine
( /1 mark) ( /1 mark)
( /1 mark)

Bupropion Smoking cessation Seizure

!
!
291
Question!2!

You!are!a!general!practitioner.!A!25NyearNold!man!presents!with!disorganized!thinking!and!
speech.!You!have!a!few!psychiatric!differential!diagnoses.!

4. State!and!define!ONE!(1)!common!type!of!disorganised!thinking!that!are!found!in!people!
without!any!mental!illness.(4!marks)!!!!!!!!!!!!!!!!!!!!!!!!!!!!!!!!!!!!!!!!!!!!!!!!!!!!!!!!!!!!!!!!!!!!!!(!!!!!!!!!!!/4!
marks)!
!
Type!of!disorganised!thinking! Definition!

! !
! !
(!!!!!!!/2!marks)! (!!!!!!!/2!marks)!

!
!
5. State!and!define!TWO(2)!common!types!of!disorganised!thinking!that!are!found!in!
people!suffering!from!schizophrenia.!(4!marks)!!!!!!!!!!!!!!!!!!!!!!!!!!!!!!!!!!!!!!!!!!!!!!!!!!!!!!!!!!!!!!!!!!(!!!!!!!!!!!
/4!marks)!
!
Type!of!disorganised!thinking! Definition!

! !
! !
(!!!!!!!/1!mark)! (!!!!!!!/1!mark)!

! !
!
(!!!!!!!/1!mark)! ! !

(!!!!!!!/1!mark)!

!
!
6. State!and!define!ONE!(1)!common!type!of!disorganised!thinking!that!is!found!in!people!
suffering!from!bipolar!disorder.!(2!marks)!!!!!!!!!!!!!!!!!!!!!!!!!!!!!!!!!!!!!!!!!!!!!!!!!!!!!!!!!!!!!!(!!!!!!!!!!!/2!
marks)!
!
Type!of!disorganised!thinking! Definition!

! !
! !

!
!
292
(!!!!!!!/1!mark)! (!!!!!!!/1!mark)!

Answers!to!question!2!

1. State!and!define!ONE!(1)!common!types!of!disorganised!thinking!that!are!found!in!
people!without!any!mental!illness.(4!marks)!!!!!!!!!!!!!!!!!!!!!!!!!!!!!!!!!!!!!!!!!!!!!!!!!!!!!!!!!!!!!!!!!!!!!!(!!!!!!!!!!!
/4!marks)!
!
Type of disorganised thinking Definition (Other definitions are acceptable. Examiners
will decide on the marks)

Circumstantiality Unwarranted detail, elaborate and irrelevant, but returns


to the original topic after a long time

(2 marks)
(2 marks)

2. State!and!define!THREE!(3)!specific!types!of!disorganised!thinking!that!are!found!in!
people!suffering!from!schizophrenia.!(6!marks)!!!!!!!!!!!!!!!!!!!!!!!!!!!!!!!!!!!!!!!!!!!!!!!!!!!!!!!!!!!!!!!!!!(!!!!!!!!!!!
/6!marks)!
!
Any!three!of!the!following:!
Type of disorganised thinking Definition

Thought Blocking (1 mark) Cessation or complete interruption in the flow of the stream
of thought (1 mark)

Loosening of association or Loss of logical connections between ideas (1 mark)


knights move thinking (1 mark)
Derailment (1 mark) Insertion of novel or irrelevant idea in mid-sentence (1

!
!
293
mark)

Thought insertion (1 mark) External thoughts are inserted into the patients mind which
does not belong to the patient. (1 mark)

Thought withdrawal (1 mark) Patients thoughts are being taken away by the others (1
mark)

Thought broadcasting (1 mark) Patients own thoughts are known to the others through
broadcasting like a radio or TV station. (1 mark)

3. State!and!define!ONE!(1)!specific!type!of!disorganised!thinking!that!is!found!in!people!
suffering!from!bipolar!disorder.!(2!marks)!!!!!!!!!!!!!!!!!!!!!!!!!!!!!!!!!!!!!!!!!!!!!!!!!!!!!!!!!!!!!!(!!!!!!!!!!!/2!
marks)!
!
Type!of!disorganised!thinking! Definition!

! !
Flight!of!ideas! A continuous speech which topics jump rapidly from one
(1!mark)! to another and there is a logical link between ideas.
! (!1!mark)!

Question 3

You are a paediatric resident. You are seeing an 11-year-old boy with repetitive eye blinking
and this has caused social dysfunction. He exhibits facial tics and gets frequent facial tics
when he is anxious. He sometimes cannot control his hand movements which affect his
writing.

6. You suspect that he suffers from Tourettes syndrome. State the most APPROPRIATE
question which you would ask to establish the diagnosis of Tourettes syndrome (2
marks) ( / 2 marks)

Answer: Does he have vocal tics? (2 marks)

!
!
294
7. What distinguishes Tourettes disorder from a chronic motor or vocal tic disorder? (2
marks) ( /2 marks)

Answer: Patients suffering from Tourettes disorder present with both motor and vocal tics.(2
marks)

8. State ONE (1) psychiatric condition often comorbid with Tourettes syndrome. (2
marks) ( /2 marks)

Answer: Obsessive compulsive disorder (2 marks)

9. State ONE (1) psychiatric medication that can be used to treat Tourettes syndrome. (2
marks) ( /2 marks)

Answer: Risperidone (2 marks), sulpiride (2 marks), haloperidol (2 marks)


Clonidine (0 mark) not psychiatric medication

10. State Two (2) psychological interventions that can be used to treat Tourettes
syndrome. (2 marks) ( /2 marks)

Any!two!of!the!following:!

Behaviour!therapy!or!behaviour!modification!(1!mark)!!
Relaxation!exercise!(1!mark)!
Social!skill!training!(1!mark)!
SelfNesteem!building!(1!mark)!
Remedial!academic!help!(1!mark)!
Mass!practice!(1!mark)!
Habit!rehearsal!(1!mark)!

Question!4:!

You! are! a! general! practitioner! (GP).! ! The! son! of! a! 60NyearNold! man! is! concerned! because! his!
father!is!taking!a!medication!called,!Flunitrazepam!(Rohypnol)!for!2!years.!The!patient!obtained!
Rohypnol! from! another! GP! who! had! retired! 1! month! ago.! Rophypnol! is! a! very! potent!
benzodiazepine!and!is!a!derivative!of!nitrazepam.!Rophypnol!is!highly!addictive!and!is!listed!as!
a! controlled! drug! by! the! Central! Narcotics! Bureau.! ! His! son! has! noted! the! following!
symptoms:! anxiety,! aggression,! getting! high,! insomnia,! memory! loss,! slurred! speech,!
tremor!in!his!father.!The!patient!was!admitted!to!the!hospital!due!to!fits.!!
!
4. Based! on! the! symptoms! reported! by! his! son! (underlined),! classify! the! symptoms! as!
intoxication!or!withdrawal!symptoms!associated!with!benzodiazepine!(4!marks)!
(!!!!!!!!!!!!!!/!4!marks)!

!
!
295
Intoxication!symptoms! Withdrawal!symptoms!

1.! 1.!

! !

(!!!!!!!!!/!0.5!marks)! (!!!!!!!!!/!0.5!marks)!

2.! 2.!

! !

(!!!!!!!!!/!0.5!marks)! (!!!!!!!!!/!0.5!marks)!

3.! 3.!

! !

(!!!!!!!!!/!0.5!marks)! (!!!!!!!!!/!0.5!marks)!

4.! 4.!

! !

(!!!!!!!!!/!0.5!marks)! (!!!!!!!!!/!0.5!marks)!

Answers:!

Intoxication!symptoms! Withdrawal!symptoms!

1.!Aggression!(0.5!marks)! 1.!Anxiety!(0.5!marks)!

! !

2.!Getting!high!(euphoria)!(0.5!marks)! 2.!Insomnia!(0.5!marks)!

3.!Memory!loss!(amnesia)!(0.5!marks)! 3.!Tremor!(0.5!marks)!

4.!Slurred!speech!(0.5!marks)! 4.!Fits!(withdrawal!fits)!(0.5!marks)!

!
!
296
!

5. State! the! MOST! APPROPRIATE! psychotropic! medication! to! treat! his! withdrawals!
(2!marks).!
(!!!!!!!!!!!!!!/!2!marks)!

Diazepam!(Valium)!is!the!only!acceptable!answer.!(2!marks)!

6. After! stabilisation,! his! son! would! like! you! to! prescribe! a! nonTbenzodiazapine!
medication!to!help!his!father!to!sleep.!Name!ONE!sedative!antidepressant!and!ONE!
other!sedative!medication.!4!marks!! ! ! ! ! ! !(!!!!!!!!!!
/4!marks)!

Sedative!antidepressant

(!!!!!!!!!!/2!marks)

Other!sedative!medication!(nonT
benzodiazapine)

!!!!!!!!!!!!!!!!!!!!!!!!!!!!!!!!!!!!!!!!!!!!!!!!!!!!!!!!!!!!!!!!!!!!(!!!!!!!!!!/2!marks)

Sedative!antidepressant Any one of the following:

Mirtazapine (2 marks)

Agomelatine (2 marks)

Amitriptyline (2 marks)

Fluvoxamine (2 marks)

Trazodone (2 marks)

Paroxetine (2 marks)

Sertraline (1 mark)

O t h e r a n t id e p r e s s a n t i f a p p r o p r i a t e : 1 m a r k

!
!
297
Fluoxetine/ bupoprion: 0 mark.

Other!sedative!medication!(nonT Any one of the following:


benzodiazapine)
Hydroxyzine (2 marks)

Anti-histamine (1 mark)

Melantonin (2 marks)

Antipsychotics e.g. quetiapine (2 marks),


c h l o r p r o m a z i n e !!(2!marks),!!!o l a n z a p i n e ( 2
marks)

Non-benzodiazapine hypnotics: Zopiclone (2


marks) / Zolpidem (2 marks)

!
!
298
!

EOPT Rotation 3

Question 1

You are a general practitioner. A 30-year-old man suffers from major


depressive disorder. You are about to prescribe a selective serotonin reuptake
inhibitor (SSRI). He read about the SSRI on internet. You need to address the
following concerns.

1. SSRI may cause erectile dysfunction. Can you name ONE (1) antidepressant that
has relatively less sexual side effect as compared to SSRI? (2 marks)

2. If he wants to continue SSRI, state TWO (2) management strategies to improve his
erectile function if he develops SSRI induced erectile dysfunction. (2 marks)
3) He understands that the combination of SSRI and other medication may lead to
high fever. What is the name of such condition? (1 mark)
4) State ONE (1) medication that may cause serotonin syndrome if he takes together
with SSRI (1 mark)

5) State FOUR (4) signs or symptoms associated with serotonin syndrome (4 marks)

Question 2

You are a resident working in the Accident and Emergency Department (AED).
A 25-year-old man is brought in by his mother to see you after he reveals to
her about his great plan to fly to Europe by changing the magnetic pole of the
earth. He is very talkative and appears to be very excited. He disturbs
other patients in the AED.

1) State THREE (3) MOST LIKELY psychiatry diagnoses that you would
consider. (3 marks)

!
!
299
2) You are unable to perform any blood test as he is very agitated. Name TWO
(2) MOST relevant psychotropic medications to sedate this patient at the
AED. (2 marks)

3) List TWO (2) most important laboratory investigations you would do before
beginning lithium (2 marks)

4) State THREE (3) common side effects associated with lithium. (3 marks)
!

Question!3!

You!are!a!resident!working!in!surgery.!A!45TyearTold!man!with!a!dual!diagnosis!of!
schizophrenia!and!alcohol!dependence!is!admitted!to!the!ward!due!to!upper!
gastrointestinal!bleeding.!The!onTcall!consultant!recommends!urgent!
oesophagogastroduodenoscopy!(OGD).!!

1. You!need!to!take!an!informed!consent!from!this!patient.!State!THREE!(3)!important!
pieces!information!you!must!inform!this!patient!before!seeking!his!consent.!(3!
marks)!
!

2. State!THREE!(3)!criteria!which!you!would!like!to!assess!in!order!to!certify!he!has!the!
capacity!to!take!consent.!(3!marks)!

3) After the OGD is done, his wife hopes that he can quit drinking. List Four (4) techniques of
motivational interviewing which you could use to maximize his motivation to quit drinking (4
marks)

!
!
300
EOPT Rotation 3

Question 1

You are a general practitioner. A 30-year-old man suffers from major


depressive disorder. You are about to prescribe a selective serotonin reuptake
inhibitor (SSRI). He read about the SSRI on internet. You need to address the
following concerns.

3. SSRI may cause erectile dysfunction. Can you name ONE (1) antidepressant that
has relatively less sexual side effect as compared to SSRI? (2 marks)

Mirtazapine (2 marks) or Bupropion (2 marks) or agomelatine (2 marks)

4. If he wants to continue SSRI, state TWO (2) management strategies to improve his
erectile function if he develops SSRI induced erectile dysfunction. (2 marks)
Any two of the following:
1) Reduce the dose or prescribe the lowest dose of SSRI (1 mark)
2) Drug holiday or stop SSRI during weekend or take a long acting SSRI on alternate
day (1 mark)
3) Add sildenafil or PDE5 inhibitor (1 mark)
4) Refer patient to sensate focus or sex therapy / vacuum pump (1 mark)
Not grip method for premature ejaculation
3) He understands that the combination of SSRI and other medication may lead to
high fever. What is the name of such condition? (1 mark)
Serotonin syndrome (1 mark) Not Neuroleptic malignant syndrome

!
!
301
4) State ONE (1) medication that may cause serotonin syndrome if he takes together
with SSRI (1 mark)
Monoamine oxidase inhibitor or irreversible monoamine oxidase inhibitor (1 mark),
moclobemide (0.5 mark) MAOI (0.5 mark) (Isocaroxazid, phenelzine irreversible MAOI
1 mark)

5) State FOUR (4) signs or symptoms associated with serotonin syndrome (4 marks)
Any 4 of the following:
Rigidity, myoclonus, uncontrollable shivering
Sweating or tremor or hyperthermia or coma
Hyperkinesia or bradykinesia
High blood pressure, labile blood pressure
hyper-reflexia, headache
dysphagia, dilated pupils
cardiovascular collapse
confusion (delirium)
decreased level of consciousness/mutism
disorientation
status epilepticus
ataxia

Question 2

You are a resident working in the Accident and Emergency Department (AED).
A 25-year-old man is brought in by his mother to see you after he reveals to
her about his great plan to fly to Europe by changing the magnetic pole of the
earth. He is very talkative and appears to be very excited. He disturbs
other patients in the AED.

5) State THREE (3) MOST LIKELY psychiatry diagnoses that you would
consider. (3 marks)

Bipolar disorder / Mania (1 mark)


Schizoaffective disorder (1 mark)
Substance abuse stimulant (1 mark)

6) You are unable to perform any blood test as he is very agitated. Name TWO
(2) MOST relevant psychotropic medications to sedate this patient at the
AED. (2 marks)

Antipsychotics: risperidone, olanzapine, haloperidol (1 mark)


Benzodiazapine: diazepam, lorazepam (1 mark)

7) List TWO (2) most important laboratory investigations you would do before
beginning lithium (2 marks)

!
!
302
Renal function test (1 mark)
Thyroid function test (1 mark)
!

8) State THREE (3) common side effects associated with lithium. (3 marks)
!
Any!one!of!the!following:!
Polyuria!(1!mark)!or!Nephreogenic!diabetes!insipidus!
Polydipsia!(1!mark)!or!
Weight!gain!(1!mark)!or!
Cognitive!impairment!(1!mark)!or!!
Fine!tremor!(1!mark)!or!
Metallic!taste!(1!mark)!or!
Hypothyroidism!(1!mark)!or!
Sexual!dysfunction!(1!mark)!or!!
Drowsiness!(1!mark)!or!!
Alopecia!(1!mark)!

Question!3!

You!are!a!resident!working!in!surgery.!A!45TyearTold!man!with!a!dual!diagnosis!of!
schizophrenia!and!alcohol!dependence!is!admitted!to!the!ward!due!to!upper!
gastrointestinal!bleeding.!The!onTcall!consultant!recommends!urgent!
oesophagogastroduodenoscopy!(OGD).!!

3. You!need!to!take!an!informed!consent!from!this!patient.!State!THREE!(3)!important!
pieces!information!you!must!inform!this!patient!before!seeking!his!consent.!(3!
marks)!
!
1. Explain the potential diagnosis of Mallory-Weiss syndrome which is associated with alcohol
misuse (1 mark) or
2. Explain purpose and nature of the diagnostic procedure about OGD (e.g. passing an
endoscope to examine the stomach and identify bleeding site, may take some tissues) (1
mark) or
3. Explain risks and benefits of OGD (e.g. small risk of perforation of feeding tube; benefit: can
assist diagnosis) (1 mark) or
4. Alternative investigation (e.g. monitor haemoglobin levels but bleeding may continue) ( 1
mark) or
5. Prognosis if without this investigation (e.g. continue bleeding) (1 mark)

!
!
303
4. State!THREE!(3)!criteria!which!you!would!like!to!assess!in!order!to!certify!he!has!the!
capacity!to!take!consent.!(3!marks)!
!
Any!3!of!the!following:!
1. He has a factual understanding of the information provided. ( 1 mark) or
2. He can appreciate the nature of the condition (1 mark)
3. He can appreciate consequences of accepting OGD (1 mark)
4. He can appreciate consequences of refusing OGD (1 mark)
5. He can manipulate information provided in a rational manner to reach a decision (1 mark).

3) After the OGD is done, his wife hopes that he can quit drinking. List Four (4) techniques of
motivational interviewing which you could use to maximize his motivation to quit drinking (4
marks)

4 Elements to the process of Motivational interviewing:

! Listen empathically (1 mark for any of the following)


1. Understand the patients unique perspective and experience
2. To put you and patient on equal ground
3. Recognition and justification and very important parts of the process
! Have patients voice their own reasons for change (1 mark for any of the following)
1. Articulate reasons for or not for changing
2. Look at what they find challenging
! Roll with resistance (1 mark)
1. Never put yourself in position of arguing, or of trying to persuade patient to take a difficult
position, or of telling the patient what must be done immediately
! Support Self-Efficacy (1 mark for any of the following)
1. Give patient hope, optimism, and provide tools to help patient succeed
2. Not to blame for failures

Short!answer!questions!(2014/2015!rotation!4)!

Question!1!(10!marks)!

You!are!a!general!practitioner.!A!65NyearNold!woman!has!taken!haloperidol!for!30!years!is!
concerned!about!a!side!effect!called!tardive!dyskinesia!(TD).!

1. State!FOUR!(4)!risk!factors!for!TD!(4!marks).!!!!!!!!!!!!!!!!!!!!!!!!!!!!!!!!!!!!!!!!!!!!!!!!!!!!!(!!!!!!!!!!!!/!4!
marks)!
!
!
2. You!are!about!to!perform!a!physical!examination.!State!TWO!(2)!anatomical!structures!
and!ONE!(1)!clinical!feature!under!each!anatomical!structure!which!you!would!look!for!
to!confirm!the!presence!of!TD!(4!marks).!!!!!!!!!!!!!!!!!!!!!!!!!!!!!!!!!!!!!!!!!!!!!!!!!!!!!!!!!!!!!!!!!(!!!!!!!!!!!!/!4!
marks)!
!
Anatomical!structure! Clinical!feature!

!
!
304
! !
! !
!!!!!!!!!!!!!!!!!!!!!!!!!!!!!!!!!!!!!!!! !
! !
!!!!!!!!!!!!!!!!!!!!!!!!!!!!!!!!!!!!!!(!!!!!!!!!/!1!mark)! (!!!!!!!!!/!1!mark)!

! !
! !
! !
! !
(!!!!!!!!!/!1!mark)! (!!!!!!!!!/!1!mark)!

!!

3. If she indeed suffers from TD, state TWO (2) management strategies (2
marks) ( / 2 marks)

Question 2 (10 marks)

A 68-year-old woman presents with poor sleep and decreased appetite for two months.
She has stopped reading because she cannot concentrate, and she is no longer seeing
friends that she is close to because she does not have the energy. She tells you that her
symptoms started after her son moved out of the house two months ago. Prior to
that, she was an energetic person who took care of her affairs and enjoyed life.

1. Besides major depressive disorder, state ONE (1) other psychiatric diagnosis based
on the above information (1 mark) ( / 1 mark)

!
!
305
2. Based on your diagnosis stated in Q.1, state THREE (3) non-pharmacological
interventions (3 marks) ( /3 marks)

3. State the term which describes the above phenomenon. (2 marks) ( /2 marks)

4. With the additional information, you suspect the patient may suffer from abnormal
grief reaction although she claims she has coped well with the death of her husband.
State FOUR (4) clinical features which are found in abnormal grief but not normal
grief (4 marks) ( /4 marks)
!

Question 3 (10 marks)

The public relation department of your hospital has received queries from a local newspaper. A
reporter wants to write about an article about alcohol misuse. You are invited to address the following
questions raised by the reporter.

1. How does long-term alcohol misuse damage our body systems? Please state FOUR (4)
physiological systems and name ONE (1) complication under each system (4 marks)
( / 4 marks)

Physiological system Complication

( / 0.5 mark) ( /0.5 mark)

( / 0.5 mark) ( /0.5 mark)

( / 0.5 mark) ( /0.5 mark)

( / 0.5 mark) ( /0.5 mark)

!
!
306
2. If an alcoholic stops drinking for 1 day, what would happen to him/her? State TWO (2) common
symptoms. (2 marks) ( / 2 marks)

3. If an alcoholic wants to join peer support group, name ONE (1) organisation which can support
the person (2 marks) ( /2
marks)

4. If an alcoholic wants to be admitted to a hospital to detoxify from alcohol, what are the
indications. State TWO (2) indications. (2 marks) ( / 2 marks)

Question 4

You are a medical resident. The accident and emergency department has sent a 55-year-old man to the
medical ward for further management due to change in mental state. When you assess him, he appears
to be manic.

1. State FIVE (5) neurological disorders which may cause mania. (5 marks) ( / 5marks)

2. State!TWO!(2)!medications!(not!illicit!drugs)!which!may!cause!mania!(2!marks)!!(!!!!!!!/2!
marks)!

3. When!you!are!clerking!the!patient,!he!suddenly!becomes!very!aggressive!and!tries!to!run!
away!from!the!ward.!You!have!tried!verbal!deNescalation!but!fails.!You!need!to!apply!
physical!restraint.!State!THREE!(3)!specific!instructions!to!nursing!staff!to!ensure!the!
physical!restraint!will!be!applied!successfully.!(3!marks)!!!!!!!!!!!!!!!!!!!!!!!!!!!!!!!!!!(!!!!!!!!!!!!!!/!3!
marks)!

Short!answer!questions!(2014/2015!rotation!4)!

Question!1!(10!marks)!

You!are!a!general!practitioner.!A!65NyearNold!woman!has!taken!haloperidol!for!30!years!is!
concerned!about!a!side!effect!called!tardive!dyskinesia!(TD).!

!
!
307
4. State!FOUR!(4)!risk!factors!for!TD!(4!marks).!!!!!!!!!!!!!!!!!!!!!!!!!!!!!!!!!!!!!!!!!!!!!!!!!!!!!(!!!!!!!!!!!!/!4!
marks)!
!
Any!3!of!the!following:!
> Old!age!(1!mark)!or!
> Female!gender!(1!mark)!or!
> Comorbid!affective!disorder!/!alcohol!abuse!(1!mark)!or!
> Comorbid!organic!brain!disorder!(1!mark)!or!
> Long!exposure!to!first!generation!antipsychotic!drug!/!potent!dopamine!receptor!
blockade!(1!mark)!or!!
> Precipitation!by!anticholinergic!drug!(1!mark)!or!
> High!antipsychotic!dose!/!rapid!increment!in!dose!(1!mark)!or!
> Previous!extra!pyramidal!side!effect!/!history!of!tremor!or!Parkinsons!disease!(1!
mark)!or!
> Genetic!predisposition!due!to!hypersensitive!dopamine!receptor!/!Family!history!of!
TD!(1!mark)!or!
> AfroNCaribbean!race!(!1!mark)!
!
5. You!are!about!to!perform!a!physical!examination.!State!TWO!(2)!anatomical!structures!
and!ONE!(1)!clinical!feature!under!each!anatomical!structure!which!you!would!look!for!
to!confirm!the!presence!of!TD!(4!marks).!!!!!!!!!!!!!!!!!!!!!!!!!!!!!!!!!!!!!!!!!!!!!!!!!!!!!!!!!!!!!!!!!(!!!!!!!!!!!!/!4!
marks)!
!
Anatomical!structure! Clinical!feature!

! !
! !
!!!!!!!!!!!!!!!!!!!!!!!!!!!!!!!!!!!!!!!! !
! !
!!!!!!!!!!!!!!!!!!!!!!!!!!!!!!!!!!!!!!(!!!!!!!!!/!1!mark)! (!!!!!!!!!/!1!mark)!

! !
! !
! !
! !
(!!!!!!!!!/!1!mark)! (!!!!!!!!!/!1!mark)!

!!
Any!two!of!the!following:!
Anatomical!structure! Clinical!feature!

Mouth!!!!!!!!!!!!!!!!!!!!!!!! Mouth!opening!

Eye!brow! Elevation!or!depression!

!
!
308
Eye!lids! Blepharospasm/sustained,!forced,!involuntary!closing!of!the!
eyelids!

Lips! pursuing/retraction!to!lateral!angle/!lip!smacking!

Tongue! Fly!catching,!tongue!protrusion!

Jaw! Abnormal!masticationNlike!movement!/!protrusion!and!
deviation!

Neck! Torticollis!

Upper!limb/!lower! Hypertonia/wrist!flexion/!pilling!rolling!tremor!
limb!

Hand! Choreiform!hand!pill!rolling!movements!

6. If she indeed suffers from TD, state TWO (2) management strategies (2
marks) ( / 2 marks)

Any two of the following:


1. Change!to!second!generation!antipsychotics!(1!mark)!or!
2. Add!vitamin!E!or!vitamin!B!(1!mark)!or!
3. Benzodiazapine!e.g.!clonazapine!(1!mark)!or!
4. Reduce!the!dose!of!haloperidol!to!minimum!effective!dose!in!long!run!(1!mark).!

Wrong!answers:!

There!is!not!enough!evidence!to!suggest!anticholinergic!as!good!treatment!for!TD.!

Drug!holiday!may!cause!relapse!in!schizophrenia.!

!
!
309
Question 2 (10 marks)

A 68-year-old woman presents with poor sleep and decreased appetite for two months.
She has stopped reading because she cannot concentrate, and she is no longer seeing
friends that she is close to because she does not have the energy. She tells you that her
symptoms started after her son moved out of the house two months ago. Prior to
that, she was an energetic person who took care of her affairs and enjoyed life.

5. Besides major depressive disorder, state ONE (1) other psychiatric diagnosis based
on the above information (1 mark) ( / 1 mark)

Adjustment disorder (1 mark), Other diagnosis: 0 mark

6. Based on your diagnosis stated in Q.1, state THREE (3) non-pharmacological


interventions (3 marks) ( /3 marks)

Any three of the following:


Supportive psychotherapy (1 mark) or

!
!
310
Problem solving therapy (1 mark) or
Cognitive behaviour therapy (1 mark) or
Interpersonal therapy (1 mark) or
Attending a day care centre (1 mark) or
Home visit by community psychiatric team (1 mark)

In the later part of interview, she mentions about the death of her husband three years ago.
She admits that she has kept everything in her late husbands room exactly the same since
his death.

7. State the term which describes the above phenomenon. (2 marks) ( /2 marks)

Mummification (2 marks), other term = 0 mark

8. With the additional information, you suspect the patient may suffer from abnormal
grief reaction although she claims she has coped well with the death of her husband.
State FOUR (4) clinical features which are found in abnormal grief but not normal
grief (4 marks) ( /4 marks)
Any 4 of the following:
> Prolonged!grief!with!duration!longer!than!6!months!(1!mark)!or!
> Inhibited!grief!at!the!time!of!death!(1!mark)!or!
> Delayed!grief!!(i.e.!no!grief!reaction!at!the!time!of!death!but!appeared!some!time!later)!(1!
mark)!!
> Strong!suicidal!ideation!to!join!the!deceased!(1!mark)!or!!
> Psychotic!experience!other!than!deceased!(1!mark)!or!
> Severe!psychomotor!retardation!(1!mark)!or!
> Severe!feeling!of!worthlessness!or!hopelessness!(1!mark)!

Question 3 (10 marks)

The public relation department of your hospital has received queries from a local newspaper. A
reporter wants to write about an article about alcohol misuse. You are invited to address the following
questions raised by the reporter.

5. How does long-term alcohol misuse damage our body systems? Please state FOUR (4)
physiological systems and name ONE (1) complication under each system (4 marks)
( / 4 marks)

Physiological system Complication

( / 0.5 mark) ( /0.5 mark)

!
!
311
( / 0.5 mark) ( /0.5 mark)

( / 0.5 mark) ( /0.5 mark)

( / 0.5 mark) ( /0.5 mark)

Physiological system Complication (The candidate needs to state one complication under
each physiological system)

Nervous system Blackouts/head injury, withdrawal fits/epilepsy, acute confusional


state, subdural haematoma, degeneration of cerebellum/ataxia,
peripheral neuropathy/loss of pain sensation, alcohol dementia,
Wernicke encephalopathy, Korsakoff psychosis, central pontine
myelinolysis, Marchiafava-Bignami syndrome, depression, anxiety,
morbid jealousy

Opthalmological Optic atrophy, retrobulbar neuropathy, loss of central vision,


system bilateral central scotoma

Gastrointestinal Cancer of mouth, larynx, pharynx, oesophagus, upper


system gastrointestinal bleeding, gastritis, malabsorption syndrome

Cardiovascular Hypertension, cardiomyopathy, atrial fibrillation


system
Respiratory system Increase in risk in respiratory infection (e.g. pneumonia,
tuberculosis)

Hepatobiliary system Cirrhosis, liver cancer, alcohol hepatitis, fatty liver, pancreatitis,
hepatic encephalopathy

Urogenital system Erectile dysfunction, sexual dysfunction, infertility, loss of


secondary sexual characteristics

Musculoskeletal Myopathy, gouty arthritis, palmar erythema.


system
Endocrine Cushing syndrome

!
!
312
Haematological Macrocytic anaemia
system

6. If an alcoholic stops drinking for 1 day, what would happen to him/her? State TWO (2) common
symptoms. (2 marks) ( / 2 marks)

Any two of the following:


> Tremor (1 mark) or
> Anxiety (1 mark) or
> Agitation (1 mark) or
> Muscle pain (1 mark) or
> Sweating (1 mark) or
> Nausea (1 mark)
> Sleep disturbance (1 mark)
> Withdrawal fits (1 mark)

7. If an alcoholic wants to join peer support group, name ONE (1) organisation which can support
the person (2 marks) ( /2
marks)

Alcoholic anonymous (2 marks)


12-step programme (1 mark)
Other answer: 0 mark

8. If an alcoholic wants to be admitted to a hospital to detoxify from alcohol, what are the
indications. State TWO (2) indications. (2 marks) ( / 2 marks)

Any two of the following:


> History of delirium tremens or withdrawal fits (1 mark) or
> Current moderate to severe symptoms of alcohol withdrawal (1 mark) or
> Significant risk of self-harm, suicide or harming other people (1 mark) or
> Unable to tolerate oral medication (e.g. naltrexone) (1 mark) or
> Multiple past inpatient detoxification (1 mark) or
> Acute psychosis (1 mark) or
> Severe cognitive impairment (e.g. Wernicke encephalopathy) 1 mark or
> Poor social support (1 mark) or
> Pregnancy (1 mark) or
> Comorbid severe mental illness (1 mark)

!
!
313
Question 4

You are a medical resident. The accident and emergency department has sent a 55-year-old man to the
medical ward for further management due to change in mental state. When you assess him, he appears
to be manic.

4. State FIVE (5) neurological disorders which may cause mania. (5 marks) ( / 5marks)

Any!5!of!the!following:!

o Multiple sclerosis (1 mark) or


o Subcortical dementia (e.g. Lewy body dementia) (1 mark) or
o Frontal lobe dementia (1 mark) or
o Temporal lobe epilepsy (1 mark) or
o Cerebrovascular accidents (1 mark) or
o Head injury (1 mark) or
o Encephalitis (virus e.g. HSV, HIV or parasite) (1 mark) or
o Huntingtons disease (1 mark) or
o Pseudobulbar palsy (1 mark) or
o Brain tumour (1 mark) or
o Neurosyphilis(1 mark)

5. State!TWO!(2)!medications!(not!illicit!drugs)!which!may!cause!mania!(2!marks)!!(!!!!!!!/2!
marks)!

Any!two!of!the!following:!

o Steroid / prednisolone
o Antidepressant
o Anticholinergic drug
o Levodopa / bomocrptine
o Bronchodilator

6. When!you!are!clerking!the!patient,!he!suddenly!becomes!very!aggressive!and!tries!to!run!
away!from!the!ward.!You!have!tried!verbal!deNescalation!but!fails.!You!need!to!apply!
physical!restraint.!State!THREE!(3)!specific!instructions!to!nursing!staff!to!ensure!the!
physical!restraint!will!be!applied!successfully.!(3!marks)!!!!!!!!!!!!!!!!!!!!!!!!!!!!!!!!!!(!!!!!!!!!!!!!!/!3!
marks)!

Any!three!of!the!following:!

1. Call!for!additional!manpower!(e.g.!call!security!guards!for!help,!at!least!4!additional!
staff)!(1!mark)!or!
2. Obtain!adequate!physical!restraints!(e.g.!1!body!jacket,!1!restraint!per!limb)!(1!mark)!or!

!
!
314
3. Standby!psychotropic!medication!(e.g.!rapid!dissolvable!antipsychotics!or!intravenous!
lorazepam!/!haloperidol)!(1!mark)!or!
4. The!doctor!will!ensure!the!airway!is!not!blocked.!Other!staffs!will!focus!on!the!body!and!
four!limbs.!(1!mark)!!

Sample!OSCE!

Introduction

OSCE Title: Oppositional Defiant Disorder

Construct: This is a station designed to test the candidates ability to take a history
of oppositional defiant disorder.

Objectives of the OSCE station:

To observe and assess candidates ability in:


1. Eliciting symptoms of oppositional defiant disorder
2. Can distinguish oppositional defiant disorder and conduct disorder
3. Expressing empathy and demonstrating good interview skills

Time: 8 minutes

Instructions to Candidates

Parents name: Ms. Tan or Mr. Tan

!
!
315
You are a general practitioner. Mr/Mrs Tan comes to see you. His/her 11-year-old
son exhibits difficult behaviour such as being disrespectful, spiteful and
argumentative at home and school. His behaviour starts to concern his parents and
teachers.

Tasks:
1. Take a history from the parent to establish a diagnosis.
2. Perform a risk assessment
3. At the end of your interview with the parent, please write down your diagnosis
and pass it to the examiner.

Time: 8 minutes

!
!
316
Answer sheet

Please write down the diagnosis of this patient and pass it to your examiner.

___________________________________________________________________

Instructions to Simulated Patients

Background
Your name is Ms. Tan. You are married and has a single child, an 11-year-old
son called Jason. You are currently staying in a 3-room HDB flat. You worked as
an administrative officer in a company and your spouse also works as an
administrative staff in another company.
You have a 11-month-old son (Jason). When yourself and your spouse go to
work, he is looking after by a domestic helper at home. He goes to school by
school bus. However, there were no problems in the pregnancy and giving birth.
He has normal development (e.g. normal in speaking, hearing and walking)
Jason has average results from Primary 1 to Primary 4. He did not have
behavioural problems in the past.
He is your first child. Yourself and your spouse cannot afford to have a second
child.
Yourself and your spouse have no past or family history of mental illness.
Yourself and your spouse do not have forensic history.

Symptoms
Main symptoms
Jason currently studies in Primary 5 and he finds the subjects in Primary 5 are
difficult to study. In the past one year, he has been disrespectful to you, your spouse,
domestic helper and his teachers. He is noted to be defiant in almost any situation
(at home, going out with family and school). He finds his temper is bad and worse
than other children of his age. He has been argumentative, spiteful and challenges
you and the teacher the reasons to do homework. He wanted to go for a buffet
dinner last weekend but the family wants to save money and ask him to wait until
next school holiday. He was very anger and throws things at home.

Your spouse is very exhausted and gives in and tells your son that, Yes, you do not
need to do your homework. Jason then argues, There is no need to study. This
type of behaviour, over and over, leaves you and your spouse feeling overwhelmed
and exhausted. Your domestic helper finds Jason very annoying and blames her for
his mistakes.

!
!
317
You met the teacher last week. In this academic year, the teacher finds Jason
struggling in school and keeping his old friends. Jason deliberately annoys his
classmates. The teacher finds him refuse to take responsibility and blame his
classmates for his mistakes. His academic performance was above average last
year and he got a borderline pass this year. His results are similar across subjects.
He does not have a favoured subject. He has no attention problems in school and he
can sit still to do his work. He have not broken any school rule and never being
suspended before.

He never abuses animals.

His mood is normal. He sleeps and eats well. No self-harm and no suicide.
He is not nervous. He always attends school and never skips school.
There is no history of truancy.
He does not have other unusual experiences such as hearing voices or seeing
things that are not there.
He does not use recreational drugs. He does not sniff glue.
His IQ should be normal. He studies in a normal neighbourhood school.
He does not have specific learning problem (e.g. he can read, write, spell and do
mathematics)
He has normal attention and can sit still to study at home and in school.
He has normal development and there is no delay in walking and speaking.
He likes to watch TV and plays computer games. He is not addicted to internet.

Your views about your parenting

You have poor relationship with your spouse.


You agree that your parenting is ineffective because yourself and your spouse
are inconsistent. You are very harsh but your spouse is always very lenient.
Both yourself and your spouse never punish Jason physically. Jason was never
abused.
You agree that Jason may be lack of supervision when you and your spouse are
working. The domestic helper spends a lot of time on cleaning the house and no
one supervises him.

Risks
Although his behaviour is difficult to manage, there are some good things
about Jason. Jason does not violate more serious rules like running away
from home or breaking school rules. He has never been physically aggressive
toward other people (parents, classmates) and animals. Jason never initiates
a fight. He does not bully other children. He lies about minor things but never
tells a big lie. He never steals and never damages others properties. He has
no trouble with the police.
He does not harm himself
He does not harm other people.

!
!
318
He seldom has accident. E.g. falling from height due to excessive climbing or
running too fast.

Other points:
You have not consulted any doctor or counsellor on this matter.
You want to seek advice from the GP.

Time: 8 minutes

!
!
319
Instructions to Examiners

Candidate is not expected to interact with the examiners in any way.!

Please keep to the exact time.

Things not to do: use/switch on mobile phone, yawn, teach or prompt.


If observers sit in, marking is not discussed with them. Observers record their own
assessment on separate sheet.

Marks are given depending on completeness or competency in each section


respectively. The global rating should be based on the total scores of
subsections. The total mark is 25. A candidate fails the OSCE station if he or
she scores less than 12.5.

!
!
320
EXAMINERS REPORT

Students Name: _________________ Matriculation No.: _____________ Examiners Name: ______________

Objectives very well Appropri patchy clearly not


(excellen ate needs to inadeq done
t) improve uate

1. Opens interview appropriately (greet, introduce, seek


permission, explains purpose) 2 1.5 1 0.5 0
Closes interview (summarizes, allows patient to clarify + to ask
questions, refers to future arrangements if any)
2. Creates safe environment, develops and maintains rapport,
controls interview appropriately (open ended questions,
respect, concern, sensitivity, empathy, responsive, normalizing 2 1.5 1 0.5 0
statements, avoids jargon)

3. Enquires about symptoms of Oppositional Defiant Disorder


a. Duration (1 year in this case)
b. Annoying and argue with other people
c. Blame others for mistake 4 3 2 1 0
d. Compliance refusal (e.g. refuse to study)
e. Defiant or Spiteful and Angry
f. Temper tantrums
4. Assess for symptoms for Conduct Disorder
a. Violates the rights of other people/animals: e.g.
bullying, fighting and torturing animals.
b. Vandalism and destroy properties/fire setting
4 3 2 1 0
c. Break rules at home or school
d. Forensic history: shoplifting, use of weapon, trouble
with police.
e. Truancy / Substance abuse / other relevant symptoms
5. Assess for risk factors and impact of for ODD
a. Inconsistent parenting
b. Lack of supervision
4 3 2 1 0
c. Impact on school results
d. Impact on interpersonal relationship
e. Parental relationship
6. Assess for risk issues
a. Harm to self accident, fall from height, self-harm. 3 2 1 0.5 0
b. Harm to others aggression to other people/animals
7. Other relevant information (e.g. Comorbidity: ADHD, IQ,
learning difficulty, family history of psychiatric illness/ forensic 3 2 1 0.5 0
history of parents, developmental history etc)
8. Correct diagnosis: Oppositional defiant disorder (3 marks); Any
3 0 0 0 0
other diagnoses (0 marks)
Subtotal score (For administrator to enter). Examiner can leave it
blank.
DECISION (circle one): Pass / Fail (For failure, the total score must be less than 12.5).
Signature of Examiner: ____________________________ Date: ______________________

!
!
321
Total score: Administrator 1:______________________ Administrator 2: _________________

EXAMINERS REPORT

Students Name: _________________ Matriculation No.: _____________ Examiners Name: ______________

Explain if student fails or is discussed in the examination board for borderline performance:
__________________________________________________________________________________________

__________________________________________________________________________________________
__________________________________________________________________________________________

__________________________________________________________________________________________

__________________________________________________________________________________________

!
!
322
!

Introduction

OSCE Title: Suicide risk assessment

Construct: This is a station designed to test the candidates ability to perform a


suicide risk assessment.

Objectives of the OSCE station:

To observe and assess candidates ability in:


4. To take a history of recent suicide attempt and assess suicide risk.
5. To assess depressive symptoms and other relevant history.
6. Recommend a management plan for suicide attempt.

Time: 8 minutes

Instructions to Candidates

Patients name: Ms. Gigi Lim

You are a resident working in the Accident and Emergency Department (AED). Ms.
Lim was bought to the AED by police because she tried to attempt suicide by
jumping from height. Your hospital is new and does not have a psychiatric ward.

Tasks:
4. Take a history from Ms Lim to assess her suicide attempt and current suicide
risk.
5. Assess depressive symptoms and obtain further relevant information.

!
!
323
6. At the end of your interview with Ms Lim, please select the MOST
APPROPRIATE management strategy and pass to the examiner.

Time: 8 minutes

Students sticker

Answer sheet

Please CIRCLE the MOST appropriate management strategy.

1. Discharge against medical advice.

2. Admit to the medical ward of your hospital.

3. Transfer the patient to the Institute of Mental Health.


!

Instructions to Simulated Patients

Background:
You are Ms. Gigi Lim, a 35-year-old woman tried to jump down from the roof
of her HDB flat tonight.
You are bought in by the police because someone spotted that you wanted to
jump down from a building and called the police who stopped you.

!
!
324
You are an unemployed, recently tested to be pregnant in urine pregnancy
test 2 weeks ago. This is an unplanned pregnancy. You do not use
contraception.
You are not married with your boyfriend. He is doing odd-job with unstable
income.

Suicide attempt
Why did you want to jump down tonight?
After you had argued with your boyfriend tonight, you wanted to jump from
height. You have thought about jumping for 2 weeks since you know the urine
result is positive.
Your relationship with him is not good in the past 1month. You have thought
of jumping down to end it all because he is not good to you and both of you
have no savings.
You want to get married with him as you are two-week pregnant. You
discovered the urine pregnancy test was positive 2 weeks ago. You have
known this boyfriend for 1 year but relationship has been unstable. He is not
keen to get married. He works as an odd job worker with unstable income.
You feel hopeless because there will not be enough money to support the
baby after the baby is born.
After you had argued with him, your boyfriend left the HDB flat.
As your boyfriend does not look after you and will leave you alone while you
are pregnant, you think there is no way out but to die.

The incident of jumping from height


You went up to the roof alone. You firmly believe that you would die by
jumping from the rooftop. You went there for a few times and planned for your
suicide if your boyfriend argues with you. Tonight, you are determined to die.
You went up alone and thought that no one would discover.
You drank two cans of beer to relax yourself so that it would be easier to jump
from height.
You did not do other things to harm yourself (e.g. overdose of Panadol or cut
yourself).
You sent a good bye sms message to your boyfriend and inform him that you
are going to die. You did not write a suicide letter.
You stayed at the rooftop for 30 minutes. When you are about to jump, two
police officers came up and pulled you down.
You were sent to the Accident and Emergency Department immediately.

Your current view of suicide:


You still want to die as your boyfriend did not respond to your sms. I do not
think he cares about me.
You do not feel sorry for the attempt.
You want to die with the foetus in your body.

!
!
325
If you are released from the AED, you will jump from height again and die with
your foetus to make your boy friend guilty.
You do not want to be admitted to any hospital and insist to go back on your
own.

Your mood and other relevant information:


In the past one month, due to frequent argument with your boy friend,
Your mood is low (Throughout the day, no variation in the day).
You lost your interest (e.g. shopping)/ not enjoy shopping nowadays
Your sleep is poor (Cannot fall asleep, wake up 5 times at night, wake up at 5
am cannot fall asleep).
Your appetite is poor.
Your energy level is low.
You have low sexual drive.
You cannot concentrate when you watch TV.
You feel hopeless.
You feel guilty towards your foetus because you have no money to raise your
future baby.
You feel very negative (hopeless) about the relationship with your boy friend
because he is an irresponsible person.
You have no one to talk to when you are depressed.
You have poor relationship with your family. You stay in your boy friends
HDB.
Your boy friend gives you $50 per week.

Say No to the following:


No past history of suicide
You did not overdose with medication tonight.
No stocking of medication or sharp objects at home.
You did not cut yourself with pen knife.
You have no religion.
You have not attempted suicide before.
You do not hear voices when stressed. No other false belief. (e.g. you
deserve severe punishment for minor mistake).
No family history of suicide.

You have not seen psychiatrist before.


No admission to psychiatric ward in the past.
No past medical problems (e.g. no thyroid problem, no chronic pain)
No family history of mental illness
No past history of mental illness
Not seen by psychiatrist before
Not on psychiatric medication
No history of alcohol or sleeping pill use.
Not seen by gynaecologist or obstetrician yet.

!
!
326
No other relationship in the past.
No past trauma
No emptiness
No fear of abandonment
No identity disturbance
No voices
No friend to talk to
Not in contact with family

Behaviour
Depressed; Still suicidal
If doctor says that he needs to transfer you to IMH, you do not look too happy
but you do not violently object.

Time: 8 minutes

Instructions to Examiners

Candidate is not expected to interact with the examiners in any way.!

!
!
327
Please keep to the exact time.

Things not to do: use/switch on mobile phone, yawn, teach or prompt.


If observers sit in, marking is not discussed with them. Observers record their own
assessment on separate sheet.

Marks are given depending on completeness or competency in each section


respectively. The global rating should be based on the total scores of
subsections. The total mark is 20. A candidate fails the OSCE station if he or
she scores less than 10.

!
!
328
EXAMINERS REPORT

Students Name: _________________ Matriculation No.: _____________ Examiners Name: ______________

Objectives very well Appropri patchy clearly not


(excellen ate needs to inadeq done
t) improve uate

9. Opens interview appropriately (greet, introduce, seek


permission, explains purpose) 2 1.5 1 0.5 0
Closes interview (summarizes, allows patient to clarify + to ask
questions, refers to future arrangements if any)
10. Creates safe environment, develops and maintains rapport,
controls interview appropriately (open ended questions,
respect, concern, sensitivity, empathy, responsive, normalizing 2 1.5 1 0.5 0
statements, avoids jargon)

11. Enquires about the suicide attempt


a. Precipitant (Why the patient attempted suicide tonight,
what is the participant?)
b. Preparation of suicide (Plan, avoid discovery, sms
suicide message) 4 3 2 1 0
c. The suicide method (e.g. jumping, other methods e.g.
overdose, cutting and use of alcohol during suicide)
d. The consequences of suicide attempt (e.g. how was
she discovered?)
12. Assess for symptoms of depression and other relevant history
a. Mood symptoms (e.g. low mood, loss of interest)
b. Biological symptoms of depression (e.g. sleep,
appetite)
c. Cognitive & other symptoms of depression (e.g. guilty, 4 3 2 1 0
psychosis)
d. Other relevant history (past history of
depression/suicide, family history of
depression/suicide).
13. Assess current suicide risk at the AED
a. Her views towards the suicide attempt? (Any remorse)
2 1.5 1 0.5 0
b. Her current suicide plan / view on treatment (any plan
for further attempt)
14. Asses for the cause of current suicide attempt/ongoing stress
a. Poor relationship with boy friend and financial problem 2 1.5 1 0.5 0
b. 2-week pregnancy
15. Other relevant information (e.g. medical history: chronic pain,
family history of depression, substance use (benzodiazepine 2 1.5 1 0.5 0
use, alcohol), religion, social support, antenatal care.
16. Correct management: Send to IMH (2 marks); Other options (0
2 0 0 0 0
mark)
Subtotal score (For administrator to enter). Examiner can leave it
blank.
DECISION (circle one): Pass / Fail (For failure, the total score must be less than 10).
Signature of Examiner: ____________________________ Date: ______________________

!
!
329
Total score: Administrator 1:______________________ Administrator 2: _________________

EXAMINERS REPORT

Students Name: _________________ Matriculation No.: _____________ Examiners Name: ______________

Explain if student fails or is discussed in the examination board for borderline performance:
__________________________________________________________________________________________
__________________________________________________________________________________________

__________________________________________________________________________________________

__________________________________________________________________________________________

__________________________________________________________________________________________
!

!
!
330
!

Introduction

OSCE Title: Cognitive Assessment

Construct: This is a station designed to test the candidates ability to perform a


cognitive assessment and elicit signs of post-concussion syndrome

Objectives of the OSCE station:

To observe and assess candidates ability in:


7. Interview and communication skills
8. Carry out a cognitive assessment, in particular frontal lobe assessment and Mini-
mental state examination (MMSE)

Time: 8 minutes

!
!
331
!

Instructions to Candidates

Patients Name: Mr. Tan

You are a General Practitioner. A 45-year-old man comes to see you today because
he injured his head after he was assaulted by another man in a pub.

Tasks:
1. Obtain the answer sheet (with your name label on it).
2. Take a brief history (less than 4 minutes) from the patient to assess post-
concussion syndrome, risk assessment and sequelae after head injury.
3. Perform cognitive tasks listed on the left hand side of the table found on your
answer sheet.
4. Fill the right hand side of the table and provide the scores for the patient and
indicate maximum score for each task.

You are NOT required to perform a physical examination.

Time: 8 minutes

!
!
332
!

Instructions to Simulated Patients

Background of patient

Your name is Mr. Tan, a 45-year-old married person, staying with your spouse and son.
You work as an account manager. You have university education.
You have NO personal or family history of mental illness or dementia.
You have good past health.
You are a non-smoker and a social drinker. You go to drink in a pub once a while.
One month ago, you went to a pub and drank with your friends. You had an argument
with another man in the pub. He used a baseball bat to hit the front part and both sides
of your head.
You had a black-out for 30 minutes and your scalp was bleeding from superficial wound.
Your friends sent you to the hospital immediately. You can remember events before and
after the fight.
You were treated at the Accident and Emergency Department. The doctor said the head
injury was mild. You were not required to stay in the hospital. The brain scan did not
show major internal bleeding in the head. No surgery was required. No bleeding from
ear, nose and throat. No bruising from eyes.
The man who attacked you left the pub and police could not catch him. Insurance
company thinks the injury is mild and refuses to compensate.

Problems after head injury

After head injury, you find your brain function not very good. You cannot concentrate at
accounting work and often make mistakes. Your supervisor asks you to take no pay
leave to consult a doctor. You cannot plan or make decision.
In your head, you experience headache over the frontal part of your head and giddiness.
You lost your balance a few times after getting up from bed and almost fell down.
For your eyes and ears, you are very sensitive and become irritable to light and noise.
You hear wee sound in your eyes (Tinnitus).
For your mental functions, you are slow in mental speed and cannot think as fast as the
past. You have noticed increased difficulty in finding the right words to express yourself.
You are also unable to play crossword puzzles which you used to enjoy as you are
unable to come up with the right words.
Your memory is failing you: misplacing things (wallet), forgetting recent conversations
you had with your family, forgetting what you just ate for meals.
For your mood, you are sad all the time but sometimes becomes angry for no reason,
poor appetite, poor sleep (4 hours at night, cannot fall asleep), low energy level, loss of
interest (e.g. tennis), loss of sexual drive, poor concentration and attention, feels
hopeless after the head injury. Your mood swings from normal mood to anger.
You do not have suicidal thought. You do not hear voices or see things that are not
there.
You feel nervous most of the time and easily startled. You worry that the head injury
caused a lot of problems in your head. You cannot sit still, need to pace around in the

!
!
333
!

house. No sweating, no diarrhea, no difficulty in breathing, no chest pain and no panic


attack.
You forget to switch fire and almost caught the whole house on fire. Luckily, no flooding
at home so far.
You can handle money and ATM machine.
Your wife and son are concerned as you become more irritable and angry. This affect
your relationship with them. They want you to see a doctor. You are so irritable and
argued with your friends. Now, you do not want to go out to meet people. You mainly
stay at home most of the time.

No nightmare, no flashbacks, no avoidance of pub, no reliving of experiences


No difficulty in breathing, no shortness of breath, no panic attack
No fear of losing control
No fear of dying/going to die
No fear to centre of attention, no fear of talking to other people.
No repetitive checking or hand-washing
No repetitive thoughts
No suicidal thought
Not hearing voices
Not an anxious or angry person In the past
You do not have a driving license and you do not drive.

No other treatment/ no other problems.


No past medical problems (e.g. no stroke), no surgery done, no head injury
No family history of mental illness
No past history of mental illness, no memory problems
Not seeing other doctors at this moment.
Not seen by psychiatrist or psychologist before
Not on psychiatric medication nor other medications.
No past hospitalisation
During memory testing

Be cooperative and helpful to the candidate.


You are able to repeat three words (out of 3 words) named by student.
e.g. Student told you apple, newspaper and pencil. You say apple, newspaper and
pencil.
Subtracting 7 from 100. You do this test fairly quickly. Able to say 93, but get all the
remaining 4 wrong. For example, 100-7 = 93; 93-7 = 90: 90-7 = 85; 85 7 = 80; 80 7 =
75.
If students asks you to spell the word WORLD backwards, say D and get all other
letters wrong.
The student ask you to say English words after an alphabet. For example, letter F. Say a
3 words Friend, Fly, face and then repeat the 3 words again slowly.
Unable to recall the three items which were told (e.g. apple, newspaper and pencil) and
you come up with another 3 items (orange, train and tea).

!
!
334
!

Time: 8 minutes

!
!
335
!

Instructions to Examiners

Candidate is not expected to interact with the examiners in any way.

Please keep to the exact time.

Things not to do: use/switch on mobile phone, yawn, teach or prompt.


If observers sit in, marking is not discussed with them. Observers record their own
assessment on separate sheet.

The global rating should be based on the total scores of subsections. The total
mark is 20. A candidate fails the OSCE station if he or she scores less than 10.

!
!
336
!

Students sticker

Answer sheet

1. Take a history (4 minutes) to assess post-concussion symptoms, sequelae after


head injury and risk assessments

2. Please perform the following cognitive assessments and fill the right side of the
table

Mental tasks Please indicate patients scores and


maximum scores for each task

Assess registration
Patients score:

Maximum score:

Assess attention and


concentration Patients score:

Maximum score:

Assess verbal fluency


with one alphabet in Number of words mentioned by patient:
one minute

Average number of words mention by a normal


person in one minute:

Assess short term


Patients score:

!
!
337
!

recall
Maximum score:

3. Please pass this form to the examiner.

EXAMINERS REPORT
Students are not allowed access to this document

Students Name: _________________ Matriculation No.: _____________ Examiners Name: ______________

Objectives very well Appropri patchy clearly not


(excellen ate needs to inadeq done
t) improve uate

Basic Communication Skills

17. Opens interview appropriately (greet, introduce, seek


permission, explains purpose), Creates safe environment 2 1.5 1 0.5 0
Closes interview (summarizes, allows patient to clarify + to ask
questions, refers to future arrangements if any)
Post-concussion symptoms

18. Take an appropriate history to assess background of head


injury (e.g. type of injury, treatment, duration of blackout, 2 1.5 1 0.5 0
amnesia before and after injury, duration of impairment.)
19. Assess neurological symptoms (e.g. headache, giddiness,
2 1.5 1 0.5 0
tinnitus)
20. Assess psychiatric symptoms (e.g. anxiety, depression,
2 1.5 1 0.5 0
irritability)
21. Assess cognitive symptoms (e.g. slow motor speed, poor
2 1.5 1 0.5 0
concentration, attention, memory loss )
22. Risk assessment (e.g. fall, forget to turn off stove/fire, flooding
at home) or background history: alcohol abuse, past medical 2 1.5 1 0.5 0
history/
Brief cognitive assessment

23. Competency to assess registration of 3 items and correct


scores: 2 1.5 1 0.5 0
Patients scores = 3 Maximum scores 3
24. Competency to perform serial 7 or WORLD backwards
2 2 1 0.5 0
Patients scores = 1 Maximum scores 3
25. Verbal fluency
2 1.5 1 0.5 0
Patients scores = 3 Average number of a normal person: 10-15

!
!
338
!

26. Competency to assess recall of 3 items


2 0 0 0 0
Patients scores = 0 ; Maximum scores 0
Subtotal score (For administrator to enter). Examiner can leave it
blank.

DECISION (circle one): Pass / Fail (For failure, the total score must be less than 12.5).
Signature of Examiner: ____________________________ Date: _____________________

Total score: Administrator 1:______________________ Administrator 2: _________________

EXAMINERS REPORT
Students are not allowed access to this document

Students Name: _________________ Matriculation No.: _____________ Examiners Name: ______________

Explain if student fails or is discussed in the examination board for borderline performance:
__________________________________________________________________________________________
__________________________________________________________________________________________

__________________________________________________________________________________________
__________________________________________________________________________________________

__________________________________________________________________________________________

!
!
339
!

Introduction

OSCE Title: Borderline personality disorder

Construct: This is a station designed to test the candidates ability to take a history
of borderline personality disorder.

Objectives of the OSCE station:

To observe and assess candidates ability in:


9. Eliciting symptoms of borderline personality disorder
10. Assess common psychiatric comorbidity associated with borderline personality
disorder
11. Establishing a diagnosis
12. Expressing empathy and demonstrating good interview skills

Time: 8 minutes

!
!
340
!

Instructions to Candidates

You are a paediatrician working in a private medical group. A mother bought her 15-
year-old daughter, Angela to see your paediatric colleague because of low body
weight and lethargy. Angela used to be a high academic achiever in school but she
was not able to sit for examination in November last year. Your colleague has
ordered a set of investigations but he needs to go off to attend some urgent matter.
He has asked you to review the laboratory results and electrocardiogram (ECG) on
his behalf.

Tasks:

After entering the room, please review your colleagues record, laboratory results
and ECG and identify abnormalities.

PLEASE DO NOT WRITE OR LEAVE ANY MARKS ON THE LABORATORY


RESULTS AND ECG. There is a calculator in the room. Please clear your
answer from the calculator after usage.

Please answer the questions stated on the answer sheet.

Please double check your name is correct before answering the questions.

Please pass your answer sheet to the circuit administrator at the end of the station.

You are required to stay in the room until the end of the 8th minute. You cannot
leave the room if you finish early.

Time: 8 minutes

!
!
341
!

Outpatient record
(To be placed on the desk of the candidates room)

Miss Angela Chan

S1234567X 15 Y 11 M Female Visit date: 8 January, 2015

Doctor: Dr. Albert Tan

History:
Angela was bought in by her mother due to low body weight
and lethargy.

Used to be a high achiever in exam.

Weight: 40 kg
Height: 170 cm

PE:
Her physical examination showed marked malnutrition
and paleness.

Plan:
Order full blood count, liver function test, renal function test,
thyroid function test and electrocardiogram.

To review results; then speak to patient and mother.

!
!
342
!

Laboratory results:
(To be placed on the desk of the candidates room)

Full Blood Count

Test Results Unit Reference interval

White Blood Cell 4.83 X 10 9/L 3.40 9.60

Red Blood Cell 2.60 X 10 9/L 4.30 5.70

Haemoglobin 10.0 g/dL 12.9 to 17.0

Mean corpuscular 113.5 fL 80.0 95.0


volume (MCV)

Mean corpsular 38.5 Pg 27.0 33.0


haemoglobin (MCH)

Platelets 140 X 10 9/L 132 - 372

Liver Function Test

Test Results Unit Reference interval

Albumin 29 g/L 38 48

Bilirubin, Total 20 umol/L 5 30

Bilirubin, Conj 4 Umol/L 05

Bilirubin, Unconj 6 Umol/L 5 25

Aspartate aminotransferase (AST) 30 U/L 10 50

Alanine aminotransferase (ALT) 35 U/L 10 70

Renal Function Test

!
!
343
!

Test Results Unit Reference interval

Sodium 143 mmol/L 135 145

Potassium 3.1 mmol/L 3.5 5.0

Urea 4.8 mmol/L 2.5 7.5

Creatinine 92 mmol/L 65 125

Thyroid Function Test

Test Results Unit Reference interval

Free thyroxine 15 pmol/L 10 23

Thyroid stimulating hormone (TSH) 3.50 mIU/L 0.45 4.50

Insert Students Sticker

1. Based on her height and weight, calculate her body mass index (BMI). (2 marks)
(Examiners Use only: /2 marks)

2. Please review the laboratory results and electrocardiogram. Identify THREE (3)
abnormalities. (6 marks) (Examiners Use only: /6 marks)

!
!
344
!

3. Based on the above information, state the MOST LIKELY psychiatric diagnosis. (2 marks)
(Examiners Use only: /2 marks)

(PLEASE TURN OVER)


4. Based on your psychiatric diagnosis, state FOUR (4) additional questions which you would
ask the patient to confirm your diagnosis (Examiners Use only: /4 marks)

5. Based on your psychiatric diagnosis, state FOUR (4) additional clinical signs which you
would look for during physical examination to confirm your diagnosis. (Examiners Use only:
/4 marks)

!
!
345
!

6. Based on your psychiatric diagnosis, state TWO (2) additional investigations which your
colleague forgot to order but relevant to this condition (2 marks) (Examiners Use only:
/2 marks)

Total marks: Marker: ______________ Verification by administrator: _____________

!
!
346
!

Instructions to Candidates

Patients name: Amy

You are a resident working at the Accident and Emergency Department (AED). Amy,
a 35-year-old woman comes to see you because she has cut her thighs. Your
colleague has put dressing on her thighs and the wounds are superficial.

Tasks:
7. Take a history from Amy to assess her personality and establish a diagnosis.
8. Assess common psychiatric comorbidity associated with her diagnosis.
9. At the end of your interview with Amy, please write down the most likely
psychiatric diagnosis and pass it to the examiner.

You are NOT required to examine the wound.

Time: 8 minutes

347!
!
!

Students sticker

Answer sheet

Please write down the most likely psychiatric diagnosis of this patient and
pass it to your examiner.

___________________________________________________________________

Instructions to Simulated Patients

348!
!
!

Background:
Your name is Amy, a 35-year-old woman. You live alone in a one-room HDB flat.
You are estranged from your family, being an only child whose father died of
cancer when you were 6 years old, and who has a very difficult relationship with
her mother. You have a history of physical abuse in childhood and her mother
was the perpetuator and she caned you very often for no reason. You are not in
contact with your mother. There is no history of sexual abuse.
You had a number of relatively short-lived jobs in the past as a caregiver for the
elderly in nursing home. You resigned from your last job after an argument with
the manager and have been unemployed for several years.
You consult the accident and emergency department today because you are very
stressed and cut your thigh.
You have no known medical illness. No known family history of mental illness.

Symptoms
Main symptoms:

The doctor may ask you a question how to describe yourself. You tell the doctor
that you dont know how to describe yourself. You are confused about who you
are. You are lost about yourself.

The doctor may ask you how your friends describe you. You tell the doctor that
you have a small number of close female friends, but find that people often let
you down so you do not have a wide social network. You feel that people in
general do not like you. It is very hard to maintain relationship with others.

The reason for cutting yourself is to re-experience the pain. The pain inflicted
by your mother during the childhood abuse. You are not suicidal at this moment.
You always cut your thighs with pen knife. You have no intention to kill yourself.

Your experience the following symptoms since young:


o Feel very empty inside yourself since young, cutting yourself since
adolescence
o Fear of being abandoned (see details below)
o Impulsive, like to cut your thighs, binge eating (i.e. eat a lot of cookies)
when stressed but does not vomit out the food. You dont regret about
cutting.
o For your mood, hard to say whether you are depression You have mood
swing or fluctuation: normal mood can become sudden anger and
irritability for no reason.
o You are confused about your own identity (e.g. you do not know who you
are).
o You had suicidal thoughts in the past (see details below).
o You saw angels when you were stressed. The angels appeared in the
corner of your flat. They just stood there but did not say anything. When
349!
!
!
doctors ask you to elaborate further about the angels. You cant. You do
not hear voices when nobody is around. You are not paranoid (i.e. does
not believe any one wants to harm you)
o You do tend to classify people as either all bad or all good. For example, in
your previous work place, the manager was the bad guy and other co-
workers were all good. No one was in between.

Relationship: You have a history of brief unstable relationships. You had 10 boy
friends in the past who were physically abusive. You have not been in a close
relationship for several years and no recent sexual activities..

Past suicide attempts: You are well apart from the lacerations, with no significant
medical history. You have scars on your thighs from past superficial lacerations. You
have had several past psychiatric admissions for self-harm or suicidality by drug
overdose across the last few years.
You use emergency A&E services at times. You have recently been admitted with
overdoses twice in the last month. Im just fed up and I cant go on.

Abandonment by your psychiatrist and psychologist: You are supposed to be


having follow-up from the psychiatrist at a general hospital. You are currently saying
that the psychiatrist has abandoned her and dont care. You are not able to talk to a
staff member at the psychiatric department who knew you, as the receptionist of the
clinic said that both your doctor and psychologist were away during Christmas.

Other problems:
You have nightmare, the images of childhood abuse popping up into your mind.
You have binge eating when under stress but no self-induced vomiting.
For your mood, very unpredictable and always have mood swings.
Your sleep, appetite and energy are normal.
You are not in any relationship at this moment and no sexual activity recently.

Say No to the following:


No body image problem. You never put yourself on diet or do excessive exercise.
You are not special when compared to other people.
You do not like to be centre of attention.
No past history of stealing, no problem with police.
You do not drink alcohol, smoke cigarettes or use sleeping pills or illicit drugs.
You are not on any psychotropic medications currently.
You have no memory problem. No past wandering from home.
You do not have other unusual experiences e.g. being controlled by other people
or your thoughts being interfered.
You do not feel low in mood. You do not have feelings of
hopelessness/worthlessness. You do not think that life is meaningless and have
no suicidal thoughts. In other words, your mood is unpredictable, your appetite is
ok (no problem), your sleep is ok (no problems).
You do not have much interest throughout your life.
There is no previous history of accident.

350!
!
!
No other medical problem. When you were admitted, the blood tests show normal
results.
No difficulty in breathing, no shortness of breath, no panic attack
No fear of losing control
No fear of dying/going to die
No fear of going out of the house. No fear of using MRT or bus.
No fear of going to a shopping mall or crowded place/enclosed space
No fear to centre of attention, no fear of talking to other people.
No fear of blood, height or spider.
No repetitive checking or hand-washing
No repetitive thoughts of contamination
Not hearing voices

No other treatment/ no other problems.


No past medical problems (e.g. no thyroid problem)
No family history of mental illness
Not seeing other doctors at this moment.
Not on psychiatric medication nor other medications.

Behaviour
You are cooperative during the interview.

Time: 8 minutes

Instructions to Examiners

Candidate is not expected to interact with the examiners in any way.!

Please keep to the exact time.

351!
!
!
Things not to do: use/switch on mobile phone, yawn, teach or prompt.
If observers sit in, marking is not discussed with them. Observers record their own
assessment on separate sheet.

Please use the calculator to add up the marks and write down the score at the
bottom of marking form.

Marks are given depending on completeness or competency in each section


respectively. The global rating should be based on the total scores of
subsections. The total mark is 20. A candidate fails the OSCE station if he or
she scores less than 10.

352!
!
!
EXAMINERS REPORT

Students Name: _________________ Matriculation No.: _____________ Examiners Name: ______________

Objectives very well Appropri patchy clearly not


(excellen ate needs to inadeq done
t) improve uate

27. Opens interview appropriately (greet, introduce, seek


permission, explains purpose) 2 1.5 1 0.5 0
Closes interview (summarizes, allows patient to clarify + to ask
questions, refers to future arrangements if any)
28. Open questions to allow patient to comment on her own
personality (1 mark) (e.g. How would you describe yourself as 2 1.5 1 0.5 0
a person?) and explore reason for cutting (1 mark)
Enquires about symptoms of borderline personality disorder

29. Chronic feeling of emptiness (1 mark) and fear of abandonment


2 1.5 1 0.5 0
(1 mark)
30. Unstable emotion (explore what constitutes mood swing,
2 1.5 1 0.5 0
normal mood to anger and irritability
31. Impulsive behaviour (e.g. past self harm-1 mark , binge eating /
2 1.5 1 0.5 0
stealing 1 mark)
32. Suicide risk assessment (e.g. current and past intention to die-
2 1.5 1 0.5 0
1 mark, explore past hospitalisation due to suicide 1 mark)
33. Other borderline symptoms e.g. past unstable relationship /
explore transient psychotic feature / identity disturbance 2 1.5 1 0.5 0
defence mechanism idealisation and devaluation or splitting
34. Assess for common comorbidity e.g. PTSD, depression,
2 1.5 1 0.5 0
bulimia, substance abuse, stealing)
35. Other relevant information (e.g. past trauma, childhood abuse,
past psychiatric history, family history social support etc) and 2 1.5 1 0.5 0
general performance of candidate
36. Correct diagnosis: Borderline personality disorder (2 marks for
full term and correct spelling); BPD abbreviation = 0 mark Any 2 0 0 0 0
other diagnoses: (0 mark)

DECISION (circle one): Pass / Fail (For failure, the total score must be less than 10).

Signature of Examiner: ____________________________ Date: ______________________

Total score: Examiners score:______________________ Verification by administrator : _________________

353!
!
!

EXAMINERS REPORT

Students Name: _________________ Matriculation No.: _____________ Examiners Name: ______________

Explain if student fails or is discussed in the examination board for borderline performance:
__________________________________________________________________________________________
__________________________________________________________________________________________

__________________________________________________________________________________________
__________________________________________________________________________________________

__________________________________________________________________________________________

354!
!
Mastering Psychiatry
A Core Textbook for Undergraduates
- Now with a companion guide to help you master your undergraduate exams!

The comprehensive textbook covers common psychiatric conditions


encountered in adults, children, adolescents and old people. The main textbook
provides core information you need for undergraduate examination and
future clinical practices.

This companion guide includes past exam papers to help you practice for the following
components in the actual exam:
Mul$ple'choice'ques$ons'
Short'Answer'ques$ons'
OSCE'sta$ons'
'
This'book'is'a'joint'eort'between'authors'from'Singapore'&'Authors'from'the'University'of'
Toronto.''

Melvyn'WB'Zhang'MBBS'(Singapore),'Diploma'(Cl'Psychiatry)''(RCP&S'Ireland),'MRPCPsych(UK)'
Resident'(Psychiatry),'Na$onal'Healthcare'Group'
'
Cyrus'SH'Ho'MBBS'(Singapore),'Diploma'(Cl'Psychiatry)''(RCP&S'Ireland),'MRCPsych'(UK)'
Registrar,'Na$onal'University'Healthcare'System'(NUHS)'
'
Roger'CM'Ho'MBBS'(Hong'Kong),'DPM'(Psych)'(Ireland)'Diploma'(Cl'Psychiatry)''(RCP&S'Ireland),'DIP''
(Psychotherapy)'(NUS),''MMed'(Psych)'(Singapore),'MRCPsych'(UK),'FRCPC'
Assistant'Professor'and'Consultant'Psychiatrist,'Department'of'Psychological'Medicine,'
Yong'Loo'Lin'School'of'Medicine,'Na$onal'University'of'Singapore'
'
'
'
Sanjeev'Sockalingam'MD,'FRCPC'
Deputy'Psychiatrist'ion'Chief'(TGH'&'PMH),'Director,'Con$nuing'Professional'and'Prac$ce'
Development'
Assistant'Professor,'Department'of'Psychiatry,'University'of'Toronto'
'
Raed'Hawa'MD,'FRCPC'
Associate'Professor,'Department'of'Psychiatry,'University'of'Toronto'
'
'
'
'
'
'
Our'online'companion:'www.masteringpsychiatry.com'
['Website'QR]''''''''''''''[Apple'Store]'''''''''''''[Android'Store]'''''''
'
'
ISBN'(Book):'978E981E07E0163E5'
ISBN'(DVD):''978E981E07E0164E2'
ISBN'(Ebook/App):''978-981-07-1493-2'
'

Vous aimerez peut-être aussi